Anda di halaman 1dari 437

www.pharmacyprep.

com Canadian Pharmacy Review

Pharmacy Prep

Evaluating Exam
Review and Guide
Q&A
MUST PASS July 2016

Misbah Biabani, Ph.D


Director
Toronto Institute of Pharmaceutical Sciences (TIPS) Inc.
Toronto, ON M2N K7

Pharmacy Prep
Professional Exams Preparation Center
4789 Yonge St. Suites # 417 Toronto, ON, M2N 5M5
WWW.PHARMACYPREP.COM
416-223-PREP (7737) / 647-221-0457
Toronto Institute of Pharmaceutical Sciences Inc.
© 2000 to 2016 TIPS Inc. All Rights Reserved.

Copyright © 2000 to 2016 Tips Inc 1


www.pharmacyprep.com Canadian Pharmacy Review

Disclaimer

Your use and review of this information constitutes acceptance of the following terms and
conditions:
The information contained in the notes intended as an educational aid only. It is not intended
as medical advice for individual conditions or treatment. It is not a substitute for a medical
exam, nor does it replace the need for services provided by medical professionals. Talk to your
doctor or pharmacist before taking any prescription or over the counter drugs (including any
herbal medicines or supplements) or following any treatment or regimen. Only your doctor or
pharmacist can provide you with advice on what is safe and effective for you. Pharmacy prep
make no representation or warranty as to the accuracy, reliability, timeliness, usefulness or
completeness of any of the information contained in the products. Additionally, Pharmacy prep
do not assume any responsibility or risk for your use of the pharmacy preparation manuals or
review classes.
In our teaching strategies, we utilize lecture-discussion, small group discussion,
demonstrations, audiovisuals, case studies, written projects, role play, gaming techniques, study
guides, selected reading assignments, computer assisted instruction (CAI), and interactive
video discs (IVD).
Our preparation classes and books does not intended as substitute for the advise of
NABPLEX®. Every effort has been made to ensure that the information provided herein is not
directly or indirectly obtained from PEBC® previous exams or copyright material. These
references are not intended to serve as content of exam nor should it be assumed that they are
the source of previous examination questions.
©2000 to 2016 TIPS Inc. All rights reserved.

Foreword by
Misbah Biabani, Ph.D
Coordinator/Director, Pharmacy Prep
Toronto Institute of Pharmaceutical Sciences (TIPS) Inc
4789 Yonge St. Suites 417
Toronto ON M2N 5M5, Canada

Copyright © 2000 to 2016 Tips Inc 2


www.pharmacyprep.com Canadian Pharmacy Review

Content
Abbreviations

Part I: Biomedical sciences


1. Human Anatomy
2. Gastrointestinal System
3. Nervous System
4. Cardiovascular System
5. Endocrine System
6. Renal System
7. Liver Function and Pathophysiology
8. Respiratory system
9. Urinary System
10. The Eye
11. Blood and anemia
12. Biochemistry
13. Clinical Biochemistry
14. Nutrition
15. Microbiology
16. Genetics and Molecular Biology
17. Pharmacogenomics
18. Immunology
19. Immunizations (vaccines)
20. Biotechnology
21. Toxicology
Part II: Pharmaceutical Sciences
22. Basic Calculations
23. Pharmacy Calculations: Dosage
24. Pharmacy Calculations: Dilutions
25. Pharmacokinetics
26. Rates and Orders of Reactions
27. Pharmacodynamics

Copyright © 2000 to 2016 Tips Inc 3


www.pharmacyprep.com Canadian Pharmacy Review

28. Basics of Medicinal Chemistry


29. Medicinal chemistry and Pharmacology of Drugs that act on autonomic nervous system
30. Medicinal chemistry and Pharmacology of Histamines, Serotonin, Prostaglandin and Non-
Steroidal anti-inflammatory Drugs
31. Medicinal chemistry and pharmacology of cardiovascular drugs
32. Medicinal chemistry and pharmacology of psychiatric & neurological diseases
33. Medicinal chemistry and pharmacology Endocrine drugs
34. Medicinal Chemistry and Pharmacology of Respiratory Drugs
35. Medicinal Chemistry and Pharmacology of Musculoskeletal drugs
36. Medicinal Chemistry and Pharmacology of Antimicrobial
37. Drug Metabolism
38. Biopharmaceutics
39. Physical Pharmacy
40. Pharmaceutical Excipients
41. Rheology
42. Pharmaceutical Dosage Forms
43. Drug delivery Systems
44. Sterile Preparations
45. Extemporaneous Pharmaceutical Preparations
46. Pharmaceutical Analysis
Part III: Social / Behavioural /Administrative Sciences
47. Bioethics and Professional ethics
48. Canadian Healthcare System
49. Canadian Pharmacy Regulations and administration
50. Social and Behavioural Aspects of Pharmacy Profession
51. Pharmacy Management
52. Pharmacoeconomics
53. Drug Information Resources
54. The new drug Approval Process
55. Basics of Clinical Research and Epidemiology
56. Biostatistics
57. Hospital Pharmacy

Copyright © 2000 to 2016 Tips Inc 4


www.pharmacyprep.com Canadian Pharmacy Review

Part IV: Pharmacy Practice/ Therapeutics and OTC Drugs


58. Generic and Brand Index
59. Prescription processing and Medication dispensing
60. Pharmaceutical Care and Drug Related Problems
61. Adverse Drug Reactions and Management
62. Drug Interactions
63. Therapeutic Drug Monitoring
64. Safety of medications in special populations
65. Identification and prevention of drug toxicity
66. Professional Pharmacy Communication Skills
67. Medication Errors
68. Pharmaceutical Preparation Storage Conditions
69. OTC and Prescription Drugs for Dermatological Disorders
70. OTC and Prescription Drugs for Ophthalmic, Ear and Mouth Disorders
71. OTC Drugs antihistamine, decongestants, antitussives, Expectorants
72. OTC Drugs for Nausea, Vomiting, Constipation, Diarrhea, and Hemorrhoids
73. OTC Drugs for Headache, Sports Injuries, Pressure Ulcers, Low Back Pain
74. Asthma and Chronic Obstructive Pulmonary Disease
75. Smoking cessation
76. Insomnia
77. Eating disorders
78. GERD, Ulcers, Inflammatory Bowel Disease and Irritable Bowel Syndrome
79. Diabetes Mellitus Type I and Type 2
80. Thyroid Disorders
81. Contraception
82. Reproductive, Gynaecologic and Genitourinary Disorders
83. Bone and Joint Complications
84. Osteoporosis
85. Hypertension
86. Coronary Artery Diseases
87. Stroke
88. Congestive Heart Failure
89. Cardiac Arrhythmias

Copyright © 2000 to 2016 Tips Inc 5


www.pharmacyprep.com Canadian Pharmacy Review

90. Peripheral vascular diseases


91. Anticoagulants
92. Anxiety Disorder
93. Depression
94. Psychosis and schizophrenia
95. Dementia
96. Seizures and epilepsy
97. Parkinson’s Disease
98. Anti-microbial agents
99. Anticancer drugs and chemotherapy
100. Natural Products and Pharmacognosy

Copyright © 2000 to 2016 Tips Inc 6


PharmacyPREP.com Human anatomy

PHARMACY PREP
HUMAN ANATOMY

1. What is an opposite of anterior view?


A. Ventral view
B. Posterior view
C. Lateral view
D. Medial view
E. Supination
Ans. B
Tips. Ventral (anterior) is opposite to Dorsal (posterior) and the lateral is opposite to medial.

2. Epithelial tissue is present in all of the following except?


A. sweat gland
B. milk gland
C. Endocrine gland
D. Blood vessel
Ans. D

2. All of the following have skeletal muscles, except?


A. arm
B. biceps
C. triceps
D. heart
E. Foot

Copyright © 2000-2015 TIPS Inc. Unauthorized reproduction of this manual is prohibited. This manual is 1-1
being used during review sessions conducted by PharmacyPrep.
PharmacyPREP.com Human anatomy

Ans. D

Copyright © 2000-2015 TIPS Inc. Unauthorized reproduction of this manual is prohibited. This manual is 1-2
being used during review sessions conducted by PharmacyPrep.
PharmacyPrep.Com Gastrointestinal System

PHARMACY PREP

GASTROINTESTINAL SYSTEM

1. A person with Vitamin B12 deficiency due to intrinsic factors. Should take?
A. Vitamin B 12 supplements
B. Vitamin B 12 IM inj.
C. Vitamin B 12 oral
D. Vitamin B 12 inj. and folic acid
Ans. B

2. PPI & H 2 RAs act on?


A. G cells
B. Gastrin
C. Parietal cell
D. Hydrochloric acid
Ans. C

3. Which of the following may increase excessive acid secretion in patient with?
A. high carbohydrate diet
B. high protein diet
C. High liquid diet
D. High salt diet
E. All of the above
Ans. b

4. What is the main cause of dyspepsia?


A. GERD
B. Peptic ulcers
C. Reflux esophagitis
D. Gastric cancer
Ans. B

5. What is not symptoms associated with irritable bowel syndrome?


A. chronic constipation
B. abdominal pain
C. Chronic diarrhea
D. GI bleeding
Ans. D

6. Epigastric pain is a symptom of Ulcer. (less likely GERD), GERD is heartburn, this could be
regurgitation of acid into esophagus to throat.

2-1
PharmacyPrep.Com Gastrointestinal System

A. Ulcer
B. GERD
C. Appendicitis
D. Angina
E. All of the above
Ans. A
Tips: Epigastric pain is a symptom of Ulcer. (less likely GERD), GERD is heartburn, this could be
regurgitation of acid into esophagus to throat.

7. Fistula or granulomas or skip patches occurs in?


A. Irritable bowel disease
B. Ulcerative colitis
C. Crohn's disease
D. Gastritis
E. GERD
Ans. C

8. Hyper acid secretion caused by?


A) Heart burn (reflux of gastric content into esophagus)
B) Peptic ulcers (H. pylori)
C) Ulcerative colitis (inflammatory)
D) Zollinger Ellison syndrome
E) GIT cancers
Ans. D
Tips. Zollinger-Ellison Syndrome occurs when gastrin secreted by non-beta cell tumors of
pancreas.

9. A regular customer of your pharmacy, is taking treatment of osteoarthritis. Recently got


gastritis? What is correct?
A) Patient is using acetaminophen
B) Patient is using corticosteroids
C) Patient is using NSAIDs
D) Patient is using methotrexate
E) Patient has heartburn
Ans. C

10. What is the most sensitive and specific means to diagnose the cause of dyspepsia?
A. Endoscopy
B. CT scan
C. Magnetic resonance imaging
D. Urea Breath Test
E. Esophagogastroduodenoscopy
Ans. A

2-2
PharmacyPrep.Com Gastrointestinal System

11. Which of the following bacteria can cause chronic peptic ulcer disease?
A. E. coli
B. H. pylori
C. S. aureus
D. S. pneumonia
E. Candida infections
Ans. B

12. What is the laboratory test confirm the peptic ulcer disease caused by H. pylori?
A. Endoscopy
B. CT scan
C. Magnetic resonance imaging
D. Urea Breath Test
E. Esophagogastroduodenoscopy
Ans. D

13. All GI conditions are associated with GI bleeding symptoms? except


A) GI cancer
B) Pancreatitis
C) Gastritis
D) Crohn's disease
E) Ulcerative colitis
Ans. b
Tips. Pancreatitis has no GI bleeding and pancreatitis symptoms are upper abdominal pain
that radiates into back. It may aggravated by eating, especially foods high in fat. Tender
abdomen, nausea and vomiting and increased heart rate.

14. What type of hernia is common in seniors or elderly?


A) heital hernia
B) sliding hernia
C) Para esophageal hernia
D) None of the above
Ans.A

15. Polypeptides and caffeine stimulates secretion of?


A. parietal cells
B. vitamin B 12
C. gastrin
D. chyme
E. amylase
Ans. C

16. When parietal cells are stimulated, they secrete?


A. Intrinsic factor and gastrin

2-3
PharmacyPrep.Com Gastrointestinal System

B. Intrinsic factor and HCl


C. Intrinsic factor and HCO 3
D. Intrinsic factor and mucus
Ans. B

17. A patient of your pharmacy, recently had gastrectomy and ileum was removed. Which of
the following can be deficient?
A. Iron deficient anemia
B. Calcium deficient due to deficiency of vitamin D
C. Vitamin B 12 deficiency due to deficiency of intrinsic factor
D. Vitamin K deficiency due to deficiency if GI bacteria
E. All of the above
Ans. C

2-4
PharmacyPrep.Com Nervous System

PHARMACY PREP

Nervous System

1. Which part of brain that controls important cognitive skills in humans, such as emotional
expression, problem solving, memory, language?
A. Frontal lobe B. Occipital lobe C. Temporal lobe D. Parietal lobe E. Brainstem
Ans. A

2.Involuntary functions are controlled by?


A. Cerebellum
B. Cerebrum
C. Cortex
D. Hypothalamus
Ans. B

3.Dopaminergic pathways are associated with?


A. occipital lobe
B. Middle lobe
C. Frontal lobe
D. Temporal lobe
Ans. C

4. Which part of brain that controls important cognitive skills in humans, such as emotional
expression, problem solving, memory, language?
A. Frontal lobe
B. Occipital lobe
C. Temporal lobe
D. Parietal lobe
E. Brainstem

Copyright © 2000-2015 TIPS Inc. Unauthorized reproduction of this manual is prohibited. This manual is 3-1
being used during review sessions conducted by Pharmacy Prep.
PharmacyPrep.Com Nervous System

5. Patient is diagnosed with Alzheimer's dementia. The Alzheimer's is associated with?


A. Temporal lobe
B. Frontal lobe
C. Occipital lobe
D. Brain stem
Ans. b

Copyright © 2000-2015 TIPS Inc. Unauthorized reproduction of this manual is prohibited. This manual is 3-2
being used during review sessions conducted by Pharmacy Prep.
PharmacyPrep.com Cardiovascular System

PHARMACY PREP

CARDIOVASCULAR SYSTEM

1. Which of the following cardiovascular disease is the most commonly associated with
atherosclerotic plaques?
A. Angina
B. Myocardial infarction
C. Deep vein thrombosis
D. Ischemic stroke
E. All of the above
Ans. e
Tips: if stable plaque than commonly cause angina, if plaque is ruptured than cause
inflammation can lead to MI.

2. What is the most common cause of arteriosclerosis is?


A) high LDL B) Smoking C) Hypertension D) emotional stress E) all of the above
Ans. A

3. Clarithromycin cause QT prolongation, it associated?


A. Atrial depolarization
B. ventricular depolarization
C. Atrial tachycardia
D. side effects of drugs
Ans. B

4. A patient with atrial fibrillation, have characteristic effect on ?


A. affect on QRS wave
B. affect on P wave
C. affect on T wave
D. affect on Q-T prolongation
Ans. B

Copyright © 2000-2016 TIPS Inc. Unauthorized reproduction of this manual is prohibited. This manual is
being used during review sessions conducted by PharmacyPrep.
PharmacyPrep.com Cardiovascular System

Copyright © 2000-2016 TIPS Inc. Unauthorized reproduction of this manual is prohibited. This manual is
being used during review sessions conducted by PharmacyPrep.
www.Pharmacyprep.com Endocrine System

PHARMACY PREP.

ENDOCRINE SYSTEM

1. Characteristic symptoms of hypothyroidism may include all of the following, EXCEPT:


A. Sensitivity to cold
B. Heat intolerance
C. Lethargy
D. Constipation
E. Weight gain
Ans. B

2. A diabetic patient mistakenly taken double dose of insulin. Which of the following is
NOT effect of excessive insulin administration?
A) Sweating
B. palpitation
C. Confusion
D. Diarrhea
E. Fatigue
Ans. D

3. Which of the following is NOT effect of excessive dose of levothyroxine?


A. diarrhea
B. Tachycardia
C. sensitive to heat
D. oily skin
E. hypertension
Ans. D

4. KCR is 49 yo women is using treatment of Levothyroxine (Synthroid) 75 mcg daily for


hypothyroidism. She complains palpitation, weight loss and sensitivity to heat. What is
appropriate?
A) refer doctor to increase dose of Synthroid 100 mcg
B) refer to doctor to decrease dose of Synthroid
C) hypothyroid symptoms so should refer to doctor
D) She may have some new problem so refer to doctor
E) None
Ans. B

5. MK is 30 year old female taking insulin for type 1 DM. Which of the following
conditions decrease requirement of insulin dose?
A) Pregnancy
B) Infection

Copyright protected by Tips Inc Protect copyright and prevent piracy 5-1
www.Pharmacyprep.com Endocrine System

C) Physical activities
D) Heavy meals
E) Acute illness
Ans . C

6. Which of the following stimulates secretion of calcitonin hormone from


thyroid gland?
A) hypocalcemia
B) hypercalcemia
C) Hypokalemia
D) Hyperkalemia
E) hypertension
Ans. B

7. Thyroid stimulating hormone (TSH) secretes from?


A. Anterior pituitary gland
B. Posterior pituitary gland
C. Thyroid gland
D. Parathyroid gland
E. None of the above
Ans. A

Copyright protected by Tips Inc Protect copyright and prevent piracy 5-2
Pharmacyprep.com

PHARMACY PREP

RENAL DISORDERS

1. Reabsorption of drugs in kidney does NOT depend on:


A. Flow rate
B. Tonicity
C. Tubular secretion
D. pH
E. Metabolism
Ans. C

2. Prerenal acute renal failure (ARF) is characterized as?


A) Inadequate blood circulation (perfusion) to the kidneys
B) Excessive blood perfusion to the kidney
C) Excessive drug elimination
D. Bladder cancer
E) All of the above
Ans. A

3. All of the following drug cause metabolic alkalosis, except?


A. ACIDzolamide
B. Thiazides
C. hydrochlorothiazide
D. Loop diuretics
E. Furosemide
Ans. A

4. A man with history of severe diarrhea, the loss of HCO 3 - from GI tract causes a
decrease in blood [HCO 3 -], and increase CO 2 his values are thus this man has? pH = 7.25,
pCO 2 = 24 mmHg, [HCO 3 -] = 10 mEq/L
A. Metabolic alkalosis
B. Metabolic acidosis
C. Respiratory acidosis
D. Respiratory alkalosis
E. Normal acid base status
Ans. B

5. What is incorrect about potassium levels?


A. Chronic renal disease may cause hyperkalemia
B. Adrenal cancer may cause hypokalemia
Copyright © 2000-2016 TIPS Inc. Unauthorized reproduction of this manual is prohibited. This
manual is being used during review sessions conducted by PharmacyPrep.
6-1
Pharmacyprep.com

C. ACE Inhibitors like captopril may cause hypokalemia


D. Spironolactone may cause hyperkalemia
E. Hydrochlorothiazide may cause hypokalemia
Ans. C

6. A regular customer of your pharmacy, age 55 yo, medication profile include Ramipril
10 mg, valsartan 5 mg, and recently doctor added spironolactone. Patient diet include
excessive intake of bananas. All of the following are pharmacist concern, except?
A. Ramipril
B. Valsartan
C. Spironolactone
D. Age 55 yo
E. Bananas
Ans. D

7) Which of the following is the least preferable combination antihypertensive in a


patient whose K+ levels are 5.5 mEq/L?
A) Ramipril/Hydrochlorothiazide
B) Valsartan/Amlodipine
C)Perindropril/spironolactone
D) Losartan/Hydrochlorothiazide
E) Amlodipine/Atorvastatin
Ans. C

8.Characteristics of nephrotic symptoms, include all except?


A) Proteinuria
B) Hypoalbuminemia
C) edema
D) Hyperalbuminemia
E) hyperlipidemia
Ans: D

9.A patient eGFR declined significantly and doctor suspect chronic kidney diseases. All of
the following are associated with chronic kidney disease, except?
A. Intravenous radiographic contrast
B. NSAIDs or COX-II inhibitors
C. Volume depletion
D. Strict blood pressure or blood glucose control
E. Aminoglycosides
Ans. D

Copyright © 2000-2016 TIPS Inc. Unauthorized reproduction of this manual is prohibited. This
manual is being used during review sessions conducted by PharmacyPrep.
6-2
Pharmacyprep.com

Copyright © 2000-2016 TIPS Inc. Unauthorized reproduction of this manual is prohibited. This
manual is being used during review sessions conducted by PharmacyPrep.
6-3
PharmacyPrep.Com Liver Function and Diseases

PHARMACY PREP

LIVER PHYSIOLOGY AND CHRONIC LIVER DISEASES

1. Which of the following is NOT a chronic hepatitis infection?


A. Hepatitis A
B. Hepatitis B
C. Hepatitis C
D. Hepatitis B and C
E. None of the above
Ans. A

2) Obstruction of bile secretion into duodenum from liver is referred as?


A. Cholestasis or cholestatitis
B. Cholecystitis
C) Cholelithiasis
D. Hepatic encephalopathy
E. Ascites
Ans. A
Tips. cholestasis is a reduction or stoppage of bile flow. The cause of cholestasis is acute
hepatitis, alcoholic liver disease, primary biliary cirrhosis, hormonal changes in
pregnancy, stones in bile duct, cancer of bile duct, pancreatitis.

3. All of the following are complications of cirrhosis, except?


A. ascites
B. liver cancer
C. hepatic encephalopathy's
D. edema
E. diabetes
ans. E

4. What are the common cause of liver cirrhosis?


A. chronic alcohol
B. hepatitis A
C. chronic alcohol and hepatitis B & C
D. Hepatitis B and C
E. hypertension
Ans. C

5. Liver cirrhosis is a type of end stage chronic liver disease (liver cirrhosis)? What clinical
laboratory tests is used to diagnose ?
A) ALT
B) AST

Copyright © 2000-2016 TIPS Inc. Unauthorized reproduction of this manual is prohibited. This 7-1
manual is being used during review sessions conducted by PharmacyPrep.
PharmacyPrep.Com Liver Function and Diseases

C) Bilirubin
D) Albumin
E) LDH
Ans. C
Tips. The liver cirrhosis diagnosis can be confirmed by a liver biopsy (removal of a tissue
sample for examination under microscope). CT scan and ultrasound may show that liver
is shrunken or abnormalities.
Alcoholic liver disease can result into liver cirrhosis. Trans aminases AST to ALT ratio
>2:1 strongly suggest alcohol abuse.

6. MK is a 50 year old man diagnosed for ascites. Which of the following are causes of
ascites, except
A) Tuberculosis
B) Abdominal surgeries
C) Congestive heart failure
D) Stroke
E) Liver cirrhosis
Ans. D
Tips. Stroke is cerebrovascular conditions and not associated with chronic liver disease
ascites.

7. Drugs that causes cholestatic jaundice side effects?


A) Cotrimoxazole
B) Erythromycin base
C) Erythromycin lactobacillus
D) Probiotics
E) Erythromycin estolate
Ans: E

8. Liver enzymes like cytochrome CYP450 cause drug metabolism in liver. What is NOT
related to drugs that are metabolized in liver?
A) metabolism effects by tissue binding
B) metabolism changes with liver blood circulation
C) metabolism changes with intrinsic activity
D) metabolism effected by chronic liver diseases
E) oxidative metabolism is catalyzed by cytochrome CYP450.
Ans: C

9. Hepatitis B is transmitted by all, except?


A) contact with blood and its products
B) sexual transmission
C) contact oral secretion
D) sharing needle
E) blood transfusion

Copyright © 2000-2016 TIPS Inc. Unauthorized reproduction of this manual is prohibited. This 7-2
manual is being used during review sessions conducted by PharmacyPrep.
PharmacyPrep.Com Liver Function and Diseases

Ans. C
Tips: oral secretion contact can cause hepatitis A.

10. Phase II metabolic reaction glucoronidation is effected by


Enterohepatic recirculation. Which of the following drugs are effected?
A) drugs that undergoes phase II metabolism
B) estrogen/progestins oral contraceptive pills
C) drugs with phase I metabolism
D) Drug with first pass metabolism
E) Lipid soluble drugs
Ans: A

11. What is the most common cause of alcoholic liver diseases?


A. Corticosteroids therapy
B. Acetaminophen toxicity
C. Alcohol abuse
D. Ascites
E. Wilson's disease
Ans. c

Copyright © 2000-2016 TIPS Inc. Unauthorized reproduction of this manual is prohibited. This 7-3
manual is being used during review sessions conducted by PharmacyPrep.
PharmacyPrep.Com Respiratory System

PHARMACY PREP.

RESPIRATORY COMPLICATIONS

1) Emphysema is?
A) COPD
B) Fibrosis
C) Bronchitis
D) Airway disease
E) None of the above
Ans. A

2. Dyspnea means
A. Painful muscle spasms
B. Pain in the heart
C. Pain in extremities
D. Painful breathing
E. Painful menstruation
Ans. D

3. Condition characterized by a irreversible form of airflow obstruction is


known as:
A. Aneurism
B. Emphysema
C. Embolism
D. Cirrhosis
E. Jaundice
Ans. D

4. Asthma is a condition of respiratory tract that may be aggravated by?


A) Allergens
B) Cold weather
C) Exercise
D) Emotional stress
E) All are correct
Ans. E

5. The MOST well known characteristic symptom of asthma include:


A. Wheezing
B. Mucosal edema
C. Cough
D. Chest tightness
E. Tachycardia

Copyright © 2000-2016 TIPS Inc. Unauthorized reproduction of this manual is prohibited. This 8-1
manual is being used during review sessions conducted by PharmacyPrep.
PharmacyPrep.Com Respiratory System

Ans. A

6. All are correct concerning the action of corticosteroids in asthma status, EXCEPT:
A. Suppress the inflammatory response
B. Decrease production of inflammatory mediators
C. Decrease airway responsiveness to inflammation
D. Relieve brochocontriction
E. Increase β-agonist receptors response
Ans. E

7. The capacity of inhalation is depends on the volume of?


a) alveoli
B) total lung capacity
C) airways
D) Bronchus
E) Bronchial cavity
Ans. B
Tips: Total lung capacity is the volume in the lungs after a maximal inspiration.
Total lung capacity includes tidal volume, inspiratory reserve volume, Expiratory volume
and residual volume.

8. Peak flow meter is test to determine asthma severity for patient at home. This test
measures?
A) The highest forced expiratory flow
B) Volume that has been exhaled at the end of the first second (FEV 1 ).
C) Total lung capacity
D) Expiration rate
E) Inspiration rate
Ans. A

9. Spirometer is device used in clinics to diagnose asthma? Spirometer measures?


A) The highest forced expiratory flow
B) Volume that has been exhaled at the end of the first second (FEV 1 ).
C) Total lung capacity
D) Expiration rate
E) Inspiration rate
Ans. b

10. Factors that decrease the respiration except:


A. Mucosal edema
B. Increase brachial secretion
C. Increase ventilation
D. Bronchospasm
Ans: C

Copyright © 2000-2016 TIPS Inc. Unauthorized reproduction of this manual is prohibited. This 8-2
manual is being used during review sessions conducted by PharmacyPrep.
PharmacyPrep.Com Respiratory System

Copyright © 2000-2016 TIPS Inc. Unauthorized reproduction of this manual is prohibited. This 8-3
manual is being used during review sessions conducted by PharmacyPrep.
www.pharmacyprep.com Urinary System

Pharmacy Prep

Urinary System

1. Which of the following is NOT a symptom of benign prostatic hyperplasia?


A. Frequent urine
B. Nocturia
C. Irritation
D. Jet urination
E. All of the above
Ans. D
Tips: in BPH normal stream of urination is not possible.

2. Which of the following is NOT a symptom of urinary tract infection?


A) Increase in urinary frequency
B) Burning sensations
C) Fever
D) Bactiurea
E) Weight loss
Ans. E

3. What is the most common type of UTI?


A. Complicated UTI
B. Uncomplicated UTI
C. Cystitis
D. Pylonephritis
E. Ureteritis
Ans. C
Tips. cystitis is bladder infection, and the most common.

4. What is the most common agent that cause UTI?


A. S.aureus
B. P. aeruginosa
C. E. coli
D. Syphilis
E. Gonorrhea
Ans. C

5. What is the drug of choice to treat cystitis?


A. Amoxicillin
B. Cefixime
C. Cotrimoxazole
D. Nitrofurontoin
Copyright © 2000-2012 TIPS Inc. Unauthorized reproduction of this manual is prohibited. This
manual is being used during review sessions conducted by PharmacyPrep.
9-1
www.pharmacyprep.com Urinary System

E. Trimethoprim
Ans.c

6. What is the distinguishing symptom of complicated UTI?


A. High fever
B. blood in urine
C. dysurea
D. frequent urination
Ans. b

7. What is the drug of choice to treat benign prostatic hyperplasia?


A. Doxazosin
B. Tadalafil
C. Finasteride
D. Saw palmetto
E. oxybutynin
Ans. c

8. All of the following used to treat BPH? except


A. Doxazosin
B. Tadalafil
C. Finasteride
D. Saw palmetto
E. oxybutynin
Ans. E

9. Which of the following is stress incontinence?


A) Relaxed pelvic floor
B) Urethral blockade
C) inability void urine
D) Urgency of urine
E) Urinary retention
Ans: A

10. What is the most common pathogen associated with cystitis?


A. S. aureus
B. Gonorrhea
C. Chlamydia
D. E. coli
E. Shigella sp
Ans. D

11. Which of the following medical conditions cause difficulty to urination in stream?
A. Urinary tract infections

Copyright © 2000-2012 TIPS Inc. Unauthorized reproduction of this manual is prohibited. This
manual is being used during review sessions conducted by PharmacyPrep.
9-2
www.pharmacyprep.com Urinary System

B. Prostatitis
C. Benign prostatic hyperplasia
D. Urinary incontinence
E. Cystitis
Ans: C

12. Hyperplasia is?


A. Increased in cell size
B. Increased in cell numbers
C. Decreased in cell size
D. Decreased in cell numbers
E. Increase cell size and numbers
Ans. B

13. Hypertrophy
A. Increased in cell size
B. Increased in cell numbers
C. Decreased in cell size
D. Decreased in cell numbers
E. Increase cell size and numbers
Ans. A

14. Which of the following cause tumors?


A. hypertrophy
B. hyperplasia
C. Atrophy
D. hyperacidity
Ans. b

15. KP is a 37 year old women. Presents to doctor with symptoms of frequent urination at
work place. Doctor has diagnosed with urinary incontinence. Her social history, denies
taking alcohol and smoking. She is mother of 2 children and gave a birth to her 3rd child
recently a month ago. What type of incontinence?
A) Urge incontinence
B) Overflow incontinence
C) Stress incontinence
D) Urinary tract infection

16. What drugs are least likely used in patient with urinary incontinence?
A. oxybutynin
B. diuretics
C. amitriptyline
D. All of the above
ans. B

Copyright © 2000-2012 TIPS Inc. Unauthorized reproduction of this manual is prohibited. This
manual is being used during review sessions conducted by PharmacyPrep.
9-3
www.pharmacyprep.com Urinary System

17. Urinary incontinence in children is defined as repeated daytime or night time voiding
urine into the bed or cloths. It is termed as?
A. Bed sores
B. Bed wetting
C. Wetting Phenomenon
D. All of the above
Ans. B

18. Enuresis is bed wetting is treated by?


A. Diuretics
B. Desmopressin
C. Fluoxetine
D. Diphenhydramine
Ans. B
Tips: Desmopressin is antidiuretic hormone analog.

19. Tamsulosin is used to treat benign prostatic hyperplasia symptoms. It blocks alpha 1a
receptors and facilitates urine flow?
A) coronary arteries
B) bladder arteries
C) prostate arteries
D) cerebral arteries
E) all of the above
Ans. B

20. Testosterone to dihydrotestosterone is catalyzed by 5-alpha reductase. The


dihydrotesterone cause prostate growth. What drugs inhibit 5-alpha reductase and
reduce prostate size?
A) Alpha blockers
B) tamsulosin
C) terazosin
D) androgen agonist
E) androgen antagonist
Ans. E
Tips. Finasteride is an androgen antagonist. The drug of choice for BPH. Finasteride and
dutasteride are androgen antagonist. act by inhibiting 5-alpha reductase.

21. Which of the following is least likely associated with UNCOMPLICATED urinary
tract infections?
A. dysurea
B. urinary urgency
C. fever
D. turbid urine
E. urinary frequency
Copyright © 2000-2012 TIPS Inc. Unauthorized reproduction of this manual is prohibited. This
manual is being used during review sessions conducted by PharmacyPrep.
9-4
www.pharmacyprep.com Urinary System

Ans. d
Ans.D

Copyright © 2000-2012 TIPS Inc. Unauthorized reproduction of this manual is prohibited. This
manual is being used during review sessions conducted by PharmacyPrep.
9-5
www.PharmacyPrep.com Human anatomy

PHARMACY PREP

The Eye

1. What is the other name of blind spot?


A. Optic disc
B. Retina
C. Cornea
D. Optic nerve
E. None of the above
Ans: A

2) What is the rate-limiting step in ophthalmic drops?


A. Optic disc
B. Retina
C. Cornea
D. Optic nerve
E. None of the above
Ans. C

Copyright © 2000-2012 TIPS Inc. Unauthorized reproduction of this manual is prohibited. This manual is 10-1
being used during review sessions conducted by PharmacyPrep.
www.PharmacyPrep.com Human anatomy

3. At night dark vision (dim light), what is the most sensitive


for motion, photoreceptors in retina?
A) Cones
B) Rods
C) Cones and rods
D) Optic nerve
E) Cornea
Ans. B

14. The major light absorbing pigment in retinal


photoreceptors is:
a. rhodopsin
b. melanin
c. glutamate
d. chlorophyll

Copyright © 2000-2012 TIPS Inc. Unauthorized reproduction of this manual is prohibited. This manual is 10-2
being used during review sessions conducted by PharmacyPrep.
www.PharmacyPrep.com Human anatomy

Ans: A

16) What is incorrect about sympathetic blockers, example


timolol in treatment of glaucoma?
A) Decrease glaucoma
B) Decrease IOP by inhibiting formation of aqueous humor.
C) Decrease IOP by increase outflow of aqueous humor.
D) First line therapy for glaucoma
E) Ophthalmic drops are available
Ans. C

Copyright © 2000-2012 TIPS Inc. Unauthorized reproduction of this manual is prohibited. This manual is 10-3
being used during review sessions conducted by PharmacyPrep.
www.PharmacyPrep.com Human anatomy

MP is a 55 year old male using Timolol 1 gtts bid ou. If patient


medication profile have the following drugs. Which of the
following medications are the pharmacist concern?
A) Hydrochlorothiazide
B) Salbutamol
C) Ramipril
D) Metformin
E) Levothyroxine
Ans. B

In the above prescription Timolol 1 gtt bid ou what is correct


direction?
A) instil one drop in both eyes
B) instill one drop two times daily in both eyes
C) Instill two drops once daily in both eyes
D) Instill one drop two times daily in right eye
E) Instill one drop two times daily in left eye

Copyright © 2000-2012 TIPS Inc. Unauthorized reproduction of this manual is prohibited. This manual is 10-4
being used during review sessions conducted by PharmacyPrep.
www.PharmacyPrep.com Human anatomy

Ans. B

A prescription of ciprofloxacin+dexamethasone otic prep?


What is correct statement?
A. instill in eye only
B. instill in ear only
C. can be instilled in eye and ear
D. taken orally
E. applied topically
Ans. B

What is the drug of choice to treat open angle


glaucoma?
A. Latanoprost
B. Latanoprost + timolol
C. Timolol
D. Ciprofloxacin ophthalmic
E. Dexamethasone
Ans. C

Copyright © 2000-2012 TIPS Inc. Unauthorized reproduction of this manual is prohibited. This manual is 10-5
being used during review sessions conducted by PharmacyPrep.
www.PharmacyPrep.com Human anatomy

A doctor prescribed timolol 1 gtt daily as directed.


What is correct?
A. Instill one drop in both eyes
B. Instill one drop in left eye
C. Instill one drop in right eye
D. Instill one drop in affected eye
E. Instill one drop in eye and ear
Ans. D

MK is a 55 year old asthma patient. Recently diagnosed for open angle


glaucoma. Doctor is considering to prescribe therapy for glaucoma. Which of
the following is least likely suitable?
A) latanoprost
B) Dorzolamide
C) Acetazolamide
D) Timolol
E) Pilocarpine
Ans. D

Copyright © 2000-2012 TIPS Inc. Unauthorized reproduction of this manual is prohibited. This manual is 10-6
being used during review sessions conducted by PharmacyPrep.
www.PharmacyPrep.com Human anatomy

17. What are the more sensitive photoreceptors in eye?


A. Cones
B. rods
C. rods and cones
D. cataract
E. pupil
Ans: B

All of the following drug have effect on eye, except?


a) amiodarone
B) hydrochloroquine
C) rifampin
D) ethambutol
E) isotretinoin
ans: B

JP is a 50 year old asthma patient and also receiving treatment for


hypertension and using metoprolol 50 mg daily. Recently diagnosed for

Copyright © 2000-2012 TIPS Inc. Unauthorized reproduction of this manual is prohibited. This manual is 10-7
being used during review sessions conducted by PharmacyPrep.
www.PharmacyPrep.com Human anatomy

glaucoma. Doctor considering to prescribe latanoprost to treat glaucoma. Why


doctor want to choose prostaglandin analog?
A) patient has asthma and also using metoprolol
B) patient has hypertension also using metoprolol
C) Patient has asthma and hypertension
D) Patient is 50 year old, asthma and hypertension
E) Patient has open angle glaucoma
Ans.

Which of the following is associated with aqueous humor?


A. retina
B. lens
C. celiary muscle
D. sweat glands
E. tear duct
Ans. C

Q. What dosage form of drug is preferred post cataract


management?
A. oral
B. Injection
C. Topical
D. Sublingual
E. OTIC

Copyright © 2000-2012 TIPS Inc. Unauthorized reproduction of this manual is prohibited. This manual is 10-8
being used during review sessions conducted by PharmacyPrep.
www.PharmacyPrep.com Human anatomy

Ans. c

An optometrist prescribed ciprofloxacin drops 2 gtt OD for


10 days. What is correct?
A. Instill 2 drops in both eye for 10 days
B. Instill 2 drops in affected eye for 10 days
C. Take ciprofloxacin 500 mg daily for 10 days
D. Instill 2 drops in right eye for 10 days
Ans. D

A patient currently using dexamethasone eye drops daily


for 3 to 4 wks after eye surgery. Another eye doctor
prescribed tobramycin/dexamethasone 2 gtts for 10 days.
What is pharmacist concern?
A. tobramycin
B. dexamethasone
C. duration of therapy
D. prescription from other doctor
Ans. b

Q. What is the drug of choice for dental prophylaxis for a pt.


having root canal treatment?
A. amoxicillin
Copyright © 2000-2012 TIPS Inc. Unauthorized reproduction of this manual is prohibited. This manual is 10-9
being used during review sessions conducted by PharmacyPrep.
www.PharmacyPrep.com Human anatomy

B. clindamycin
C. azithromycin
D. clarithromycin
E. cephalexin
ans. a

What is the drug of choice for dental prophylaxis for a pt.


having root canal treatment. Pt. allergic to penicillin?
A. amoxicillin
B. clindamycin
C. cloxacillin
D. ciprofloxacin
E. Cephalexin
ans. B

Copyright © 2000-2012 TIPS Inc. Unauthorized reproduction of this manual is prohibited. This manual is 10-10
being used during review sessions conducted by PharmacyPrep.
PharmacyPrep.Com Blood and Anemia

PHARMACY PREP

BLOOD AND ANEMIA

1. Microcytic anemia is characterized as?


A. Hypochromic anemia
B. Vitamin B 12 deficient anemia
C. Folate deficient anemia
D. Sickle cell anemia
E. Decreased total iron binding capacity
Ans: A

18) In microcytic anemia (iron deficient), all of the following decrease, except?
A) Mean cell volume (MCV)
B) Hematocrit (Hct)
C) Hemoglobin
D) Serum ferritin
E) Total iron binding capacity (TIBC)
Ans. E
Tips. In microcytic anemia total iron binding capacity increase. Hematocrit is actual volume of
RBCs in a unit volume of whole blood).

2. Megalobastic anemia is characterized as?


A. Iron deficient anemia
B. Vitamin B 12 or folic acid deficient anemia
C. Folate deficient anemia
D. Sickle cell anemia
E. Pernicious anemia
Ans: B

8. Which of the following is screening test is used to confirm iron deficient anemia?
A. MCV
B. Serum folate levels
C. Serum ferritin levels
D. RBC levels
E. Transferrin level
Ans. C
Tips. The screening test for iron deficient anemia is a ferritin level. Iron deficient anemia is
typically defined as ferritin level <30 ng/mL. However infections, inflammation and obesity
can influence ferritin level, in these situations definition changes to <50 ng/mL.

3. Hematocrit is?

Copyright © 2000-2016 TIPS Inc. Unauthorized reproduction of this manual is prohibited. This manual is 11-1
being used during review sessions conducted by PharmacyPrep.
PharmacyPrep.Com Blood and Anemia

A. Circulating iron storage


B. Measures the total iron binding capacity
C. Measures the total free iron
D. Mean volume of iron
E. It is transferrins
Ans: A

4. Which of the following iron supplements have the highest elemental iron?
A. Ferrous sulfate
B. Ferrous gluconate
C. Ferrous fumarate
D. Combination of iron supplements
E. All of the above
Ans.C
Tips. Ferrous fumarate 33%, Ferrous sulfate 20%, Ferrous gluconate 12%.

5. A 59 yo lady comes to pharmacy to buy some iron supplements. She currently using
ibuprofen 400 mg qid for her joint pains. She feels like she is very week and fainting. She
observed occult blood in stool, however she is NOT sure? What is appropriate to do?
A. Recommend ferrous fumerate because high elemental iron
B. Recommend ferrous glucose because it is light on stomach
C. Refer her to doctor to determine cause of bleeding
D. Ask her to stop taking ibuprofen
E. Ask her to take low dose of ibuprofen
Ans: C

6. Which of the following is the common deficiency in pregnancy?


A) Iron
B) Vitamin B 12
C) Folic acid
D) Multivitamin
E) Dimenhydrinate
Ans. A

7. Choose the incorrect statement?


A. Sickle cell anemia and haemolytic anemia are narmocytic anemia.
B. Coombs tests differentiates sickle cell anemia, and haemolytic anemia.
C. Penicillin's and cephalosporin's, methyldopa give false positive Coombs tests.
D. Patient with deficiency of G6PD enzyme may get haemolytic anemia side effect if using
sulfa drugs.
E. Serum TSH increases in hypothyroidism
Ans. E

8. What it the dose of folic acid supplement patient taking carbamazepine?

Copyright © 2000-2016 TIPS Inc. Unauthorized reproduction of this manual is prohibited. This manual is 11-2
being used during review sessions conducted by PharmacyPrep.
PharmacyPrep.Com Blood and Anemia

A. 1 mg B. 100 mg C. 200 mg D. 5 mg E. 300 mg


Ans. D

9. Blood hemoglobin transport oxygen in systemic circulation. The form of iron present in
hemoglobin is?
A. Ferrous Fe3+
B. Ferric Fe2+
C. oxidized form
D. oxyhemoglobin
E. Methemoglobin
Ans. B

Copyright © 2000-2016 TIPS Inc. Unauthorized reproduction of this manual is prohibited. This manual is 11-3
being used during review sessions conducted by PharmacyPrep.
PharmacyPREP.COM Biochemistry

PHARMACY PREP

BIOCHEMISTRY

1.What is correct regarding the formation of proteins?


A. Formed by condensation of peptide bond of amino acids.
B. Geometric sequence of amino acids
C. The formation occurs by complexation of amino acids
D. Proteins are formed from carbohydrates
E. None of the above is right
Ans. A

2. Which of the following is the end product of ANAEROBIC glycolysis?


A) Pyruvic acid
B) Urea
C) Uric acid
D) Lactic acid
E) Proteins
Ans. D

Glucose --> (aerobic) Pyruvate --------------------------> CO 2 + H20


l
l (anaerobic)
Lactate -----------------------> CO2 + H20

3. Linolenic acid is type of essential fatty acid?


15 12 9 1
COOH

Alpha-Linolenic Acid (omega-3)


A. omega 3 B. omega 6 C. Omega 9 D. EPA E) DHA

4. All of the following can cause denaturation of proteins, except?


A) acids
B) heating
C) bases
D) mechanical mixing
E) Buffers
Ans. E

5. Gluconeogenesis is?

Copyright 2000-2016 TIPS Inc 12-1


PharmacyPREP.COM Biochemistry

A) formation of glucose from glycogen


B) formation of glucose from proteins
C) formation of glucose from fats
D) formation of glucose from fats and proteins
E) formation of glucose from maltose
Ans. D

6. KP is a 35 year old women is using ferrous gluconate to treat anemia. What


decreases absorption of ferrous gluconate?
A) Food
B) Proton pump inhibitors
C) citrus juice
D) Apple juice
E) grapefruit juice
Ans: A

7. What is primary protein structure?


A. linear sequence of amino acids
B. Alpha helix
C. Beta pleated sheet
D. Protein consisting more than one protein chain
E. Three dimensional structure
Ans. A

8. The protein three dimensional structure and linked with disulphide bonds is?
A. primary
B. secondary
C. tertiary
D. quaternary
Ans. C

9.Which of the following protein structure describes more than one amino acid chain?
A. primary
B. secondary
C. tertiary
D. quaternary
Ans. D

10. What aminoacid is precursor for synthesis of –CH2- in tetrapyrrole structure of


porphyrin ring in hemoglobin.
A) alanin
B) Tyrosine
C) Glycine

Copyright 2000-2016 TIPS Inc 12-2


PharmacyPREP.COM Biochemistry

D) Phenyl alanin
E) Acetyl CoA
Ans. C

11. Heme containing enzymes are formed from protoporphyrin. What are the
examples of heme containing enzymes or proteins?
A) hemoglobin
B) myoglobin
C) cytochrome oxidase
D) all of the above
Ans. D

12. Which of the following is the rate limiting step in atherosclerosis formation?
A. HMG-COA to mevalonate formation
B. Mevalonate to cholesterol formation
C. Low density lipoprotein formation'
D. Lipid synthesis
Ans. A

Copyright 2000-2016 TIPS Inc 12-3


www.Pharmacyprep.com Nutrition

PHARMACY PREP
NUTRITION

1) Choose correct answer about linolenic acid and linoleic acid:


A) Linolenic acid is should be taken with diet.
B) Linolenic acid and linoleic acid have only difference of double bond in chemical
structure.
C) Linolenic acid is omega 3 essential fatty acid
D) Linoleic acid omega 6 essential fatty acid
E) All of the above
Ans. E
Tips. Linolenic is omega 3 essential fatty acids. Omega 3 has three double bonds at
position, 9, 12 and 15. Whereas omega 6 has two double bonds at position 9 and 12.

2. Rhodopsin is an eye pigmentation present in rods photoreceptor. Transform the


following conversion of 11 retinal to produce night vision?
A. 11-Cis-retinal to 11-trans retinal
B. 11-Trans retinal to 11-cis retinal
C. 13-Cis retinal to 13- trans retinal
D. 13 trans retinal to trans retinal
E. 12 cis retinoic acid to trans retinoic acid
Ans: A

3. Which of the following vitamin should you recommend for smokers?


A. Vitamin A
B. Vitamin B
C. Vitamin C
D. Vitamin D
E. Vitamin E
Ans. C

4 What vitamin overdose could cause toxicity?


A. Vitamin A
B. Vitamin B
C. Vitamin C
D. Vitamin D
E. Vitamin E
Ans. A

5. A breast fed infant should recieved which of the following vitamin drops?
A. Vitamin A drops
B.Vitamin B 12

Copyright © 2000-2012 TIPS Inc. Unauthorized reproduction of this manual is prohibited. This 14-1
manual is being used during review sessions conducted by PharmacyPrep.
www.Pharmacyprep.com Nutrition

C.Vitamin C
D.Vitamin D drops
E. Vitamin E drops
Ans. D

6. Age over 70 years and elderly commonly have deficiency of vitamin?


A. Vitamin B1
B.Vitamin B 12
C. Vitamin B 6
D. Vitamin D
E. Vitamin K
Ans. B

7. A person is allergic to gluten, which of the following should not recommend?


A. Milk
B. Rice
C. Wheat
D. Wine
E. Eggs
Ans. C

8. What is the closes option to breast milk for infant at age of 3 mo?
A. Formula milk
B. Cow based formula milk
C. Soy based formula milk
D. Lactose free formula milk
E. Cow milk
Ans. B

9.Inadequate intake of vitamin D over 50 years age can cause?


A. renal disease
B. osteoporosis
C. osteoarthritis
D. hypercalcemia
E. Hyper vitamin D syndrome
Ans. B

10. Nephrolithiasis caused by?


A. Deficiency of vitamin D
B. Deficiency of Vitamin A
C. Overdose of vitamin A
D. Overdose of vitamin B 12
E. Overdose of vitamin D
Ans. E

Copyright © 2000-2012 TIPS Inc. Unauthorized reproduction of this manual is prohibited. This 14-2
manual is being used during review sessions conducted by PharmacyPrep.
www.Pharmacyprep.com Nutrition

11. What is the best source vitamin D


A-Dairy product
B. Milk product
C. Dark green leafy vegetable
D. Sunlight
E. Sunscreen
Ans.D

Copyright © 2000-2012 TIPS Inc. Unauthorized reproduction of this manual is prohibited. This 14-3
manual is being used during review sessions conducted by PharmacyPrep.
PharmacyPrep.Com Microbiology

PHARMACY PREP
MICROBIOLOGY

1. Colon is large intestine which contains which of the following type of bacteria:
A. 95% to100% anaerobic
B. 95% to 100% aerobic
C. 30 to 50% anaerobic
D. 30 to 50% aerobic
E. There is no bacteria in lower gut
Ans . A

2. The most common causative organism of community acquired pneumonia (CAP) is:
A. S. pneumonia
B. M. pneumonia
C. H. influenza
D. S. aureus
E. E. coli
Ans. A

3. The following organism least likely causes pneumonia:


A. S. pneumonia
B. M. pneumonia
C. H. influenza
D. M. catharhalis
E. E. coli
Ans. E

4.Toxic shock syndrome (TSS) is caused by:


A) E. coli
B) Gonorrhea
C) S. aureus
D) Syphilis
E) Chlamydia
Ans. C

5. Which of the following NOT associated with Toxic shock syndrome?


A. Tampons
B. Condoms
C. Contraceptive sponges
D. Intra uterine devices (IUD)

Copyright protected by Tips Inc Protect copyright and prevent piracy 15-1
PharmacyPrep.Com Microbiology

E. None of the above


Ans: B

6. Annually flu season in Canada. Flu is caused by?


A. Influenza A only
B. Influenza B only
C. Influenza A and B
D. Influenza A and Flu vaccine
E. Flu vaccine only
Ans. C

7. Which of the following infection are commonly associated in


nosocomial infections?
I) S. aureus
II) P. aeruginosa
III) C. difficle
A. I only
B. III only
C. I and II only
D. II and III only
E. All are correct
Ans. E

8. What is true about cervical cancer?


I) caused by oral contraceptives pills
II) caused by papilloma virus
III) It is sexually transmitted infection
A. I only B. III only C. I and II only D. II and III only E. All are correct
Ans: D

9. MK is a 27 year old man presents with persistent ear pain for last 2 days and
associated with drainage. Pharmacist refer patient to physician because?
A. otitis externa caused by P. aeruginosa and S. aureus
B. Because it is associated with drainage
C. Because ruptured tympanic membrane
D. because ear pain with drainage could be perforation of tympanic membrane or
drainage from middle ear.
E. Because it can cause hearing loss
Ans.D

Copyright protected by Tips Inc Protect copyright and prevent piracy 15-2
PharmacyPrep.Com Microbiology

Copyright protected by Tips Inc Protect copyright and prevent piracy 15-3
www.Pharmacyprep.com Cell and Molecular Biology

PHARMACY PREP

CELL AND MOLECULAR BIOLOGY

1. What is difference in nucleotide and nucleoside?


A. phosphate B. Sugar C. base D. DNA E. RNA
Ans. A

2. Translation occurs:
A) Before transcription
B) After transcription
C) Same time as transcription
D) in mitochondria
E) in chromosomes
Ans. A

3. Hemophilia type A results from deficiency of


clotting factor?
A) factor 5 B) factor10 C) factor 8 D) factor 2 E) factor 9
Ans. C

4. A woman carrying hemophilia and has a child from a man with hemophilia. Which of
the following is NOT her child?
A. A girl with hemophilia
B. A boy with hemophilia
C. A girl having no gene of hemophilia
D. A girl having the gene of hemophilia
E. A boy having no hemophilia
Ans. C

m X Y
w
X xx xy
X xx xy
XX = Women with hemophilia
XY = Man with hemophilia
XY = Man with NO hemophilia
XX = Women with hemophilia carrier

5. What is cDNA?
A. DNA template produced by mRNA, is complementary DNA (cDNA)

16-1
www.Pharmacyprep.com Cell and Molecular Biology

B. specific set of DNA and RNA


C. specific set of DNA
D. Specific set of DNA bases
E. specific set of RNA bases
Ans. A

6. Which of the following is NOT a complimentary base pair?


A. G-C and A-T
B. C-G and A-T
C. G-C and T-A
D.T-G and G-C
E. A-U and G-C
Ans. D

7. Information transfer from DNA to M-RNA is referred as?


A. Transcription
B. Translation
C. DNA-gyrase
D. protein synthesis
E. DNA-Recombination
Ans. A

8. Centrosomes?
A) Centrosomes contains acid hydrolyzing enzyme
B) Centrosomes are main organizing microtubule centre
C) Centrosomes contain protein (rough) and lipid (smooth) synthesis
d) Centrosomes are primary site for biological protein synthesis.
E) Centrosomes are enzymes that produces and decomposes hydrogen peroxides.
peroxisomes breakdown long chain fatty acids
Ans. B
Tips. Centrosomes are organizing microtubule centre Lysosome contains acid
hydrolyzing enzyme. Endoplasmic reticulum contain protein (rough) and lipid (smooth)
synthesis. Ribosomes are primary site for biological protein synthesis. Peroxisomes
contains enzymes that produces and decomposes hydrogen peroxides. Peroxisomes
breakdown long chain fatty acids.

9 Antisense RNA targets specific viral or microbial nucleic acid sequences that interferes
with normal replication and expression. The antisense technology targets?
A. Tcell
B. B Cell
C. DNA
D. mRNA
E. cDNA
Ans. D

16-2
www.Pharmacyprep.com Cell and Molecular Biology

10. Transduction is?


A. the process by which DNA is transferred from one bacterium to another by a virus
B. Microbial nucleic acid sequences that interferes with normal replication and
expression
C. woman carrying hemophilia and has a child from a man with hemophilia.
D. Information transfer from DNA to M-RNA is referred
Ans. A
Tips: Transduction is the process by which DNA is transferred from one bacterium to
another by a virus. It also refers to the process whereby foreign DNA is introduced into
another cell via a viral vector.

11. Antisense technology is?


A) Codons that stops proteins synthesis are used to stop formation of defected
proteins.
B) Antisense technology targets mRNA
C) Antisense technology target DNA synthesis
D) Antisense technology target translation in protein synthesis
E) A and B
Ans. A

12. Enzymes that catalyze coupling of two molecules are classified as


A. Hydralases
B. Ligases
C. Oxidoreductases
D. Transferases
E. Isomerases
Ans. B

16-3
PHARMACY PREP
PHARMACOGENETICS

1) What is definition of Pharmacogenetics?


A) The pharmacogenetics is integration of pharmacology and genetics.
B) The study of pharmacogenetics allows designing and developing drugs that are customized
to each person’s genetic mark up.
C) The pharmacogenetics also utilized to study cytochrome enzymes that are responsible for
drug interactions.
D) The pharmacogenetics identifies the genetic variation that cause drug cause differences in
drug response.
E) All of the above
Ans. E

2. What is SNP?
A. Single nucleotide polymorph
B. Single nucleotide pharmacogenetic
C. Single genetic markup
D. Single genetic variation
E. Single nucleotide product
Ans. A
Tips. The SNP is single nucleotide polymorphism (SNP). It occurs when one base pair of
nucleotide replaces another. A single base differences that exist between individual. This is
the most common genetic variation in DNA.

3. If you know person genetics mark up, and target a medicine is defined as?
A. pharmaceutics
B. biotechnology
C. Nanotechnology
D. Pharmacogenetics
E. Molecular biology
Ans. D

Copyright © 2000-2016 TIPS Inc. Unauthorized reproduction of this manual is prohibited. This manual is
being used during review sessions conducted by PharmacyPrep.
PharmacyPREP.Com Immunology

PHARMACY PREP.
IMMUNOLOGY and Immunizations

1. Which of the following is a pro-inflammatory cell responsible for initiation of an acute


inflammation?
A. Eosinophils
B. Platelets
C. Mast cells
D. Basophiles
E. Neutrophils
Ans. C

2 Poison ivy is what type of hypersensitive reactions?


A) Type I
B) Type II
C) Type III
D) Type IV
E) Type V
Ans. E

3. A 70 yo elderly patient is treated with antibiotics for 7 days. However, still have signs
of symptoms of infections such as fever. The patient neutrophil count is still increased
over 70%. What kind of infections patient have?
A. parasitic
B. viral
C. fungal
D. inflammation
E. bacterial
Ans. E

4. All of the following can transmit HIV, except?


A) Sexual contact
B) Maternal transmission
C) Drug abuse and sharing needle
D) cut and wounds contact
E) Orofecal
Ans. E

5. A HIV patient currently has CD 4 T cell count is 300. What treatment doctor may target
for prophylaxis?
A. M. Tuberculosis
B. Cytomegalvoirus
C. Pneumocytisis pneumonia jerovicii or carinii
D. Non-Hodgkins lymphoma

Copyright © 2000-2016 TIPS Inc. Unauthorized reproduction of this manual is prohibited. This 18-1
manual is being used during review sessions conducted by PharmacyPrep.
PharmacyPREP.Com Immunology

E. Kaposis sarcoma
Ans. A

6. What is true about immunoglobulin structure?


A. consist of protein and amino acids sequence
B. Consist of long chain amino acids and disulfide bond linkage
C. Consist of long chain and small chain protein linked with disulfide bonds bridge
D. Consist of specific amino acids selective sites of infections.
E. Humoral antibody structures
Ans. C

7. What is average life span of survival for HIV patient?


A. 10 yrs
B. 11 yrs
C. 1-2 yrs
D. 2-3 yrs
E. 20-30 yrs
Ans. D

8. A customer of your pharmacy travelling to mountain. She has history of anaphylaxis.


Which of the following product pharmacist should recommend to take?
A) Travelers diarrhea Dukoral vaccine
B) Hepatitis A vaccine
C) Flu vaccine
D) DEET mosquito repellant
E) Epipen
Ans.

9.All of the following are NOT recommended for flu vaccine, except?
A) Child under 6 months age
B) A person allergy to eggs
C) Person with currently acute bronchitis symptoms
D) Pregnant
E) Person with current symptoms of pneumonia
Ans. D

10. Which of the following populations is LEAST considered high risk for influenza so
that vaccination with influenza vaccine is not necessary?
A) Person age > 5 yrs to 50 yrs
B. Residents of long-term care facilities (e.g., nursing homes)
C. Patients with diabetes mellitus
D. Patients over the age of < 65 yo
E. Patients with COPD or asthma
Ans. A

Copyright © 2000-2016 TIPS Inc. Unauthorized reproduction of this manual is prohibited. This 18-2
manual is being used during review sessions conducted by PharmacyPrep.
PharmacyPREP.Com Immunology

11. Flu immunization season in Canada?


A) November to April
B) October to Mid November
C) October to April
D) October through December
E) October through April
Ans. B

Copyright © 2000-2016 TIPS Inc. Unauthorized reproduction of this manual is prohibited. This 18-3
manual is being used during review sessions conducted by PharmacyPrep.
PharmacyPrep.Com Biotechnology

PHARMACY PREP
BIOTECHNOLOGY

1. Herceptin (Trastuzumab), is
A. Treatment of HER2-overexpressing metastatic breast cancer
B. Tumor necrosis factor alpha inhibitor
C. Glycoprotein inhibitor
D. Interleukin
E. Interferon's
Ans: A

6) Filgrastim is a granulocytes CSF indicated in the treatment of?


A) renal, cardiac and hepatic graft rejection
B) neutropenia associated with cancer chemotherapy
C) thrombocytopenia associated with cancer chemotherapy
D) anemia associated with chronic renal diseases
E) chemotherapy induced anemia
Ans. b

7. Filgrastim is?
A. Colony stimulating factor
B. Erythropoietin's
C. Interferon's
D. Human growth hormones
E. Monoclonal antibodies
Ans: A

Copyright © 2000-2012 TIPS Inc. Unauthorized reproduction of this manual is prohibited. This 20-1
manual is being used during review sessions conducted by PharmacyPrep.
PharmacyPrep.Com Biotechnology

8) Operlaveukin is?
A) Interleukin 11
B) Colony stimulating factor
C) Interleukin 3
D) Interleukin 2
E) Epoeitin
Ans. A

9) Operlaveukin (IL-11) is indicated for?


A) Chemotherapy induced thrombocytopenia
B) Bone marrow suppression
C) Acute graft rejection
D) Neutropenia
E) Growth factor
Ans. A

10. Muromonab (OKT 3 ) is monoclonal antibody is used for treatment of ?


A) Renal, cardiac and hepatic graft rejection
B. Neutropenia associated with cancer chemotherapy
C. Thrombocytopenia associated with cancer chemotherapy
D. Anemia associated with chronic renal diseases
E. None of the above
Ans: A

HAMA is?
A) Human antigen and Mouse antibody
B) Human antimouse antibody

Copyright © 2000-2012 TIPS Inc. Unauthorized reproduction of this manual is prohibited. This 20-2
manual is being used during review sessions conducted by PharmacyPrep.
PharmacyPrep.Com Biotechnology

Murine antibody
C)
D) Humanized antibody
E) Human antibody
Ans. B

Trastuzumab is produced by?


A) Cell culture
B) Monoclonal antibody method
C) Chimeric monoclonal antibody
D) murine monoclonal antibody
E) Humanized monoclonal antibody
Ans. E

Tips. Transtuzumab is humanized MAB, which is produced


from trangenic mice.

Chimeric antibody chain is?


A) humanized chain linked to mouse chain.
B) Humanized chain linked to rat chain
C) Humanized chain binding with different antigen
D) One quarter the antibody is from one species origin where
as the other 3 quarters from other species
Ans.

Mouse Chimeric Humanized Human


100% 25% mouse 10% mouse 100%
mouse human
Example drugs produced drug produced by 12 drug produced
drugs produced by 5 chimeric Humanized by 4 human
by murine antibodies? antibody? antibody?
antibody? Rituximab, (transgenic) Adalimumab
Muromonab, abciximab, Trastuzumab,

Copyright © 2000-2012 TIPS Inc. Unauthorized reproduction of this manual is prohibited. This 20-3
manual is being used during review sessions conducted by PharmacyPrep.
PharmacyPrep.Com Biotechnology

capromab infliximab, omalizumab,


cetuximab, daclizumab
basiliximab.

Example of drug produced by 4 immunoadhesin (protein)? Etanercept, abatacept.

Copyright © 2000-2012 TIPS Inc. Unauthorized reproduction of this manual is prohibited. This 20-4
manual is being used during review sessions conducted by PharmacyPrep.
www.Pharmacyprep.com Toxicology

PHARMACY PREP
TOXICOLOGY

2. Which of the following is a benzodiazepinics antagonist:


A. Naloxone
B. Physostigmine
C. Flumazenil
D. Naltrexone
E. Penicillamine
Ans. c
Tips: Flumazenil is a benzodiazepinic antagonist therefore used as antidote in
benzodiazepinic overdoses.

4) Carbon monoxide can be highly toxic because it easily


binds to:
A. Hemoglobin
B. Myoglobin
C. Cytochrome oxidase
D. All are correct
Ans. D

29) Specific antidote for iron preparations overdose


include:
A. Dimercaprol
B. Deferoxamine
C. Penicillamine
D. Naloxone
E. Sucimmer

Ans. B

Copyright © 2000-2011 TIPS Inc. Unauthorized reproduction of this manual is prohibited. This 21-1
manual is being used during review sessions conducted by PharmacyPrep.
www.Pharmacyprep.com Toxicology

35) Which of the following is considered the warfarin


antidote?
A) Vitamin K
B. Vitamin D
C. Vitamin A
D. Vitamin B 12
E. Vitamin E
Ans. A

What is initial symptoms of acetaminophen overdose?


A. vomiting
B. severe nausea and vomiting
C. bleeding
D. Jaundice
E. liver failure
Ans. A
Tips: acetaminophen large doses can cause toxicity. Initially for 2 to 4 hr no symptoms.
Then toxic symptoms can occur in 4 stages:
Stage 1: vomiting can occur
Stage 2: after 24 hr nausea, vomiting can get severe
Stage 3: can cause toxicity, symptoms bleeding, jaundice.
Stage 4: can recover or if not recovered it can cause liver failure

42. Glutathione conjugation produce the product called?


A. Mercapturic acid
B. Sulfonation
C. Oxidation
D. Glutathione conjugation
E. Reduction
Ans. A

Copyright © 2000-2011 TIPS Inc. Unauthorized reproduction of this manual is prohibited. This 21-2
manual is being used during review sessions conducted by PharmacyPrep.
www.Pharmacyprep.com Toxicology

In hospital ward a nurse mistakenly administered high dose of


heparin dose. Which of the following is used to treatment of
overdose?
A. Vitamin K oral
B. Vitamin K injection
C. Protamine sulphate
D. Check INR
Ans. C

A grandmother administered 2 tbsp tid of amoxicillin 300


mg/5 ml instead of 2 tsp tid to 9 mo old child. What to
do?
A) refer to emergence
B) refer to doctor
C) wait and watch
D) Give antidote
E) Give oral rehydration solution
Ans. B

Copyright © 2000-2011 TIPS Inc. Unauthorized reproduction of this manual is prohibited. This 21-3
manual is being used during review sessions conducted by PharmacyPrep.
www.Pharmacyprep.com Toxicology

At your pharmacy dispensed wrong medication to patient by a


relief pharmacist. You as pharmacy manager, the next day you
realized the dispensing error happened. What is appropriate
action?
A. Inform the doctor
B. Inform college of pharmacy
C. Inform the head office of your pharmacy store
D. Contact patient immediately and inform about error and tell
bring medication back to pharmacy.
E. Call immediately to relief pharmacist and inform him that he
did dispensing error.
Ans. D

Patient brought wrong medication back to pharmacy. What is appropriate


to do?
A. Tell patient to make complain against pharmacist to college of
pharmacy
B. Change medication to correct medication and do not charge additional
prescription fee.
C. Ask patient if has taken medication and ask to continue if has no
problems
D. Change medication to correct medication and ask if patient if has taken
medication and or any symptoms and document.
E. Take back wrong medication from patient and ask to get a new
prescription.
Ans,D

What is appropriate action after any dispensing error?

Copyright © 2000-2011 TIPS Inc. Unauthorized reproduction of this manual is prohibited. This 21-4
manual is being used during review sessions conducted by PharmacyPrep.
www.Pharmacyprep.com Toxicology

A. Complain to college of pharmacy


B. document and discuss with pharmacy team member
about error and prevention strategies.
C. make incident report to ISMP
D. Tell pt. that we will refund prescription fee
Ans. b

47. B

BIBLIOGRAPHIC REFERENCE

1- COMPREHENSIVE PHARMACY REVIEW – Lippincott William & Wilkins – Fourth edition

2- CPS-COMPENDIUM OF PHARMACEUTICALS AND SPECIALITIES - Canadian Pharmacist


Association – 2001 edition.

3- MEDICAL DICTIONARY – Dorland’s illustrated – 27th edition.

4- PHARMACY PREP – Lectures series & study guide for Evaluating Examination-TIPS -
2003/2004

5- THERAPUTIC CHOICES – Canadian Pharmacist Association -Third edition

Copyright © 2000-2011 TIPS Inc. Unauthorized reproduction of this manual is prohibited. This 21-5
manual is being used during review sessions conducted by PharmacyPrep.
www.Pharmacyprep.com Toxicology

6- USP DI – Drug Information for the Health Care Professional–15th edition – Volume I.

Copyright © 2000-2011 TIPS Inc. Unauthorized reproduction of this manual is prohibited. This 21-6
manual is being used during review sessions conducted by PharmacyPrep.
www.pharmacyprep.com

PHARMACY PREP.

PHARMACOKINETICS

PHARMACOKINETIC FORMULAS
Vd = D/Cp
Cp = D/Vd
D = Vd x Cp
V d = CL T /K el
t 1/2 = 0.693 /K el
K el = 0.693/t 1/2

1) A dose of 240 mg was given to a patient, his total body clearance is 3.5 L/min and
the drug excreted unchanged in the urine is 80 mg. What is his non-renal clearance?
a) 240 ml/min
b) 2.3 L/min
c) 3.3 L/min
d) 1.16 L/min
e) 160 ml/min
Ans. B
Renal clearance is drug excreted unchanged or metabolized in urine.
CL T = CL R +CL NR
CL NR = CL T - CL R
240 mg................3.5 L
80 mg....................?
=1.16 L
= 3.5 - 1.16 = 2.3L

or

3.5/240 mg = ?/80 mg

2. A drug hepatically metabolized and renally eliminates. Patient receiving this


medication has chronic renal disease thus has low renal clearance. What is correct
about drug affect?

Copyright © 2000-2016 TIPS Inc. Unauthorized reproduction of this manual is prohibited. This 1
manual is being used during review sessions conducted by PharmacyPrep.
www.pharmacyprep.com

A) metabolites excrete in stools


B) metabolites accumulates in blood
C) metabolites half life decrease
D) hepatic metabolism decrease
E) hepatic metabolism saturates

Ans. B

3.A dose was given, the plasma concentration was 64 mg, t 1/2 =0.7 hrs. After 7 hours
from the initial dose, what is the drug's concentration?
a) 0.0mg
b) 1mg
c) 2mg
d) 3mg
e) 4mg
Ans. A

Tips. 64 --> 32 --> 16 -->8-->4 -->2-->1-->0.5 -->0.25 -->0.125-->0.06

or
C = C o . e-kt
K = 0.693/t 1/2
K = 0.693/0.7 = 0.999

C = 64 x e(-0.999x7) = 0.06

or

ln C = -kt + ln C o
How to use calculator (Sharp EL-510RN)

Enter 64

Multiply
Enter 2ndF
Enter ex
Enter (
Enter (-)
Enter
Enter )
Enter =

Copyright © 2000-2016 TIPS Inc. Unauthorized reproduction of this manual is prohibited. This 2
manual is being used during review sessions conducted by PharmacyPrep.
www.pharmacyprep.com

4. A 40 year old patient who weighs 70 kg, needs intravenous infusion of amoxicillin. The
desired C ss of drug is 15mg/dL. The physician ordered antibiotic infused for 10 hours.
Amoxicillin has t 1/2 = 1 hour and Vd = 9L.
What rate of iv is recommended for this patient? (no loading dose was given);
A. 135 mg/hr
B.936 mg/hr
C. 1000 mg/hr
D. 333 mg/hr
E. 400 mg/hr

Ans. B
Tips:
C ss = R___
K x Vd
R = Rate of infusion
K = first order elimination constant
V d = volume of distribution

R = C ss x k x V d

R = 15 0.693 x 9000 =
100 1 hour
= 935.54 mg/hr
5) If the rate of infusion of the drug is 500 mg q8h and clearance is 7.3 L/hr. Find steady
state concentration
a) 85.6 mg/L
b) 856 mg/L
c) 8.56 g/L
d) 8.56 mg/L
e) 68.4 mg/L

C ss = rate of infusion/clearance

Ans. D C ss = Rate of infusion = 500/8 = 8.56 mg/L


Clearance 7.3

Css = R/CL
CL = R/Css
R = Css x CL
CL = K x Vd A patient parameter include elimination rate contant 0.17 h-1 and
volume of distribution 11.8 L. What is clearance? 2.02 L/h

Copyright © 2000-2016 TIPS Inc. Unauthorized reproduction of this manual is prohibited. This 3
manual is being used during review sessions conducted by PharmacyPrep.
www.pharmacyprep.com

6. For an effective antimicrobial to treat UTI it must be all, except?


A) low level of protein binding
B) short biological half life thus can reach high concentration rapidly
C) high distribution in body
D) high distribution in urinary tract
E) highly active against gram negative bacteria

Ans. D

7. A drug was found to have zero order kinetic 100 mg after 6 days 99 mg. How
long it will take to eliminate the entire drug from the body?
a) 100 days
b) 200 days
c) 300 days
d) 600 days
e) 900 days
Ans. d

8. A patient was given 100 mg drug orally. The drug absorbs at rate of 10 mg/min and
follows zero order kinetics. After 5 min how much drug will be absorbed?
A. 10 mg
B. 50 mg
C. 75 mg
D. 100 mg
Ans. b

9. A patient was given 100 mg drug orally. The drug absorbs at rate of 10 mg/min and
follows zero order kinetics. How long it takes to absorb all the drug?
A. 10 min
B. 50 min
C. 75 min
D. 100 min
Ans. A

10. Ampicillin in 5% dextrose degrades by first order kinetic, at rate constant of 0.026
h. What is the shelf life of ampicillin?
a) 4 hr
b) 8hr
c) 2hr
d) 16hr
e) 12hr
Tips. T 90 = 0.105/K
T 90 = 0.105/0.026 = 4hrs

Copyright © 2000-2016 TIPS Inc. Unauthorized reproduction of this manual is prohibited. This 4
manual is being used during review sessions conducted by PharmacyPrep.
www.pharmacyprep.com

11. Approximately 50% of cloxacillin is excreted unchanged in the urine. If the normal
dosage schedule for cloxacillin is 125 mg q6h, a patient with renal function 20% of
normal should receive?
a) 25 mg q6h
b) 31.25 mg q6h
c) 62.5 mg q6h
d) 75 mg q6h
e) 125 mg q12h
Tips:d

Renal Non renal


Cloxacillin 50% 50%

62.5 mg 62.5 mg

12.5 mg 62.5 mg

or

50-20 = 30%
50% .............125
30%................?

12. Drug which dose is 1500 mg is given every 24 hours. The renal clearance of this
drug is 1.2 mg/dL. Calculate the clearance in ml/min.
a) 98.6 ml/min
b) 76.8 ml/min
c) 86.8 ml/min
d) 66.8 ml/min
e) 43.6 ml/min
Ans. C
Tips
1.2 mg...........100 mL
1500 mg..........?

= 125,000 mL for Q24h (1440 min)

125,000............1440 min
? mL.........................min
= 86.8 ml/min
or
1.2 mg --------100 ml 125000 mls--------1440 min (60 min x 24 hrs)

Copyright © 2000-2016 TIPS Inc. Unauthorized reproduction of this manual is prohibited. This 5
manual is being used during review sessions conducted by PharmacyPrep.
www.pharmacyprep.com

1500 mg---------X X---------------1 min


X = 125000 mls X = 86.8 ml/min

or

125000 ml/1440 min = 86.6 ml/min

13. If a drug is 50% metabolized and half life drugs is 1 hr . What is the percentage of
blood concentration after 4hrs?
a) 50%
b) 25%
c) 12.5%
d) 6.25%
e) 3.12%
Ans. D
Tips: 100 50% --> 25 -->12.5 --> 6.25%-->
or
Drug concentration = 100%
t 1/2 (decrease the drug by half) = 50%
2 t1/2 t1/2 t1/2 = 6.25%

14. A new antifungal was given at a dose of 5mg/Kg by a single intravenous bolus
injection to a 32 years old female who weighed 75Kg. The antifungal has an
elimination half life of 2 hours and apparent volume of distribution of 0.28L/Kg.
What is the initial plasma drug concentration in this patient?
a) 36 mg/L
b) 1.8 mg/L
c) 17.8 mg/L
d) 1.79 mg/L
e) 19 mg/L
Ans. C C p = D o C p = 5 mg/Kg = 17.8mg/L
Vd 0.28L/kg

15. Following the anterior information, calculate the predicted plasma concentration at
8 hours after the dose:
a) 15mg/L
b) 111mg/L
c) 11.1mg/L
d) 1.11mg/L
e) 2.64mg/L
Ans. D Cp initial = 17.9mg/L and elimination t 1/2 is 2hours
Cp = 17.9 after 2 hours = 8.95
Cp = 8.95 after 2 hours = 4.47
Cp = 4.47 after 2 hours = 2.23

Copyright © 2000-2016 TIPS Inc. Unauthorized reproduction of this manual is prohibited. This 6
manual is being used during review sessions conducted by PharmacyPrep.
www.pharmacyprep.com

Cp = 2.23 after 2 hours = 1.11 IN 8 HOURS

16. Following the same anterior information, calculate how much drug remains in the
patient body, 8 hours after the administrated dose:
a) 23mg
b) 112mg
c) 100mg
d) 15.3mg
e) 84.4mg

Ans. A D = 375mg after t/1/2 of 2 hours = 187.5mg


after t1/2 of 2 hours = 93.75..Continue the same as the anterior
question!!!
After 8 hours you will have 23mg

17. Following the anterior information, how long after the dose is exactly 75%
eliminated from the patient body?
a) 2 hours
b) 4 hours
c) 6 hours
d) 8 hours
e) 10 hours
Ans. B Initial dose = 375mg and elimination t1/2 is 2hours
Cp = 375mg after rs
a er rs
4 hours → 75%

Let us assume the following characteristics of the drug in a 70kg man.


-Therapeutic effective plasma concentration: 2.0mg/L
-Biologic half-life: 80minutes
-Apparent volume of distribution: 0.70Kg/L

18. What is the recommended rate of infusion for this drug?


a) 85mg/min
b) 58mg/min
c) 0.85mg/min
d) 8.5mg/min
e) 0.085mg/min
Ans- ss n
K = 0.693/80min = 0.0087
Vd = 0.70 x 70Kg = 49L

Copyright © 2000-2016 TIPS Inc. Unauthorized reproduction of this manual is prohibited. This 7
manual is being used during review sessions conducted by PharmacyPrep.
www.pharmacyprep.com

19. What is the total body clearance for this patient?


a) 33L/min
b) 43mls/min
c) 43L/min
d) 0.43L/min
e) 0.43mls/min
Ans. D CL T T = 0.0087 x 49 = 0.43L/min

20. Still following the information of the anterior questions, calculate the corresponding
loading dose:
a) 98mg
b) 160mg
c) 48mg
d) 24mg
e) 100mg
16- A LD = V ss

21. Drug A bind to 28% with protein, when given at dose 150 mg/day. If given to 300
mg/day, what % of drug A bind with protein. Mol.wt of protein is more than drug A.
A-56% B-28% C-14% D-80% E-60%
Ans-B

22. Two drugs that metabolized by the kidney and liver. Which one has first pass
metabolism?
drug x drug y
CL t 1100 460
CL r 110 20
Vd 3.3 2
ProteinBinding 85% 90%
A) Drug x = oral
B) drug y = oral
C) drug y = rectal
D. drug x = rectal
Ans. B

Copyright © 2000-2016 TIPS Inc. Unauthorized reproduction of this manual is prohibited. This 8
manual is being used during review sessions conducted by PharmacyPrep.
www.pharmacyprep.com

23. Mr. Florence Backs is a 72-year-old patient weighing 71.8 kg who is taking
aminophylline 250mg by i.v. at stat. and then was given immediately by i.v. drip at a rate
of 40mg/hour. Theophylline level at stat = 7.5mg/ml and V d of theophylline is 0.5L/kg.
What is the expected serum concentration after the initial dose?
a. 5.55mcg/ml
b. 13.1mcg/ml
c. 7.5mcg/ml
d. 6.8mcg/ml

Ans-B

What is the expected serum concentration after the initial dose?

Co = Dose x F x S
Vd
= 250 x 1 x 0.8 = 5.55 ug/ml
(0.5) (71.8 kg)

5.55 + 7.5 = 13.05 = 13.1 ug/ml

24. During gentamycin therapy which parameter would you measure?


a. Creatinine clearance for renal toxicity (nephrotoxicity)
b. serum creatinine, BUN, blood concentration
c. auditory function because of ototoxicity
d. all of the above

Ans: D

25. What is correct about drug excretion in to breast milk?


A. acidic drug excrete more than base drugs
B. base drug excrete more than acidic drugs
C. neutral drug excrete more than basic drugs
D. drug excretion in to breast milk does not depends drug pH
Ans. B

26. Absolute bioavailability is the result of comparing


a. oral dosage from and IV dosage form
b. oral bioavailability from and sublingual form
c. oral bioavailability from and IM dosage form
d. oral bioavailability from and rectal form
Ans: A

27. What is the F value for an experimental drug tablet based on the following data?

Copyright © 2000-2016 TIPS Inc. Unauthorized reproduction of this manual is prohibited. This 9
manual is being used during review sessions conducted by PharmacyPrep.
www.pharmacyprep.com

Drug Dose
Form Dose AUC (µg/ml/mL/h)
Tablet 100 mg po 20
Solution (control) 100 mg po 30
Injection (control) 50 mg IV push 40
a) 0.25
b) 0.38
c) 0.50
d) 0.66
e) 0.90
Ans. A

28. If an oral capsule formulation of the drug A products a serum concentration-time


curve having the same area under the curve as that produced by an equivalent dose of
drug A given IV, it can generally be concluded that

a) the IV route is preferred to the oral route


b) the capsule formulation is essentially completely absorbed
c) the drug is very rapidly absorbed
d) all oral dosage forms of drug A will be bioequivalent
e) there is not advantage to the IV route
Ans. b
Tips; if the AUC are same, it can be said that the total amount of drug delivered to the
body by each dose form is equal.
For oral dosage for 100% bioavailability does not mean that the drug has 100%
absorption.

Product Company Dosage Form Dose Administered Cumulative Urinary


Amount (mg)
A Parenteral Injection 10 mg IV 9.4
A Tablet 20 mg po 12.0
B Tablet 20 mg po 8.2
B Capsule 15 mg po 6.8

29. Using the above data the absolute bioavailability of Company B tablets is best
estimated to be

a) 25%
b) 40%
c) 44%
d) 68%

Copyright © 2000-2016 TIPS Inc. Unauthorized reproduction of this manual is prohibited. This 10
manual is being used during review sessions conducted by PharmacyPrep.
www.pharmacyprep.com

e) 87%
Ans. C
Tips: The best measure of absolute bioavailability is considered to be AUC of iv. Because
AUC iv is not available for this drug, the next best comparison will be the cumulative
drug amount found in the urine.
Because iv dose was 10 mg, whereas the oral tablet dose was 20mg, a correction factor
of 2 x is needed.
Cumulative amount is 20 mg injected.
9.4 x 2 = 18.8
8.2/18.8 = 0.44 or 44%

30. Patient on phenytoin therapy. Therapeutic level is 10 – 20 mg/L. Blood level of


phenytoin is within therapeutic range. But still patient shows some side effects of
phenytoin Following is blood phenytoin analysis?
13.6 mg/L morning
14.2 mg/L evening
What is reason increase level of phenytoin
A-Phenytoin toxicity
B-Phenytoin has very strong protein binding
C-Liberation of drug from protein binding
D-None of the above
Ans-C

31. If blood flow to liver decrease?


A-Affects intrinsic metabolism
B-Reduce protein binding of drug
C-Increase protein binding of drug
D-Reduce metabolism of drugs
E-Do not change metabolism of drug
Ans. D

32. A drug completely renally eliminates, it depends on all pharmacokinetic factors,


except?
A. drug half life
B. Volume of distribution
C. Hepatic clearance
D. Renal function
E. rate of infusion
Ans. C

33. Steady state concentration is factor of


a. AUC B. half life C. plasma protein binding D. dose
Ans. B

Copyright © 2000-2016 TIPS Inc. Unauthorized reproduction of this manual is prohibited. This 11
manual is being used during review sessions conducted by PharmacyPrep.
www.pharmacyprep.com

PHARMACY PREP.
RATES AND ORDERS OF REACTIONS

Plot of log of concentration against time produces a straight line with a:


A. Slope of –K/2.303
B. Slope of –K/t 1/2
C. Slope of –K o
D. slope of K/2.303
E. slope of K/0.693
Ans. A

2. The time it takes to infused drug to reach plasma steady state concentration (Css)
depends on:
I-Elimination half-life of drug.
II-Fraction of free excreted in urine
III-Plasma concentration curve
A. I only B. III only C. I and II only D. II and III only E. All of the above
Ans. a

3. A drug with zero order reaction eliminating, which is independent of initial


concentration, What is correct about slope?
A. k o
B. k/2.303
C. 0.693/k
D. 1/ka
E. None
Ans. A

4. Use of loading dose?


I-To achieve therapeutic concentration immediately
II-Loading dose = desired concentration x V d
III-Steady state will be achieved immediately and maintained
A. I only B. III only C. I and II only D. II and III only E. All of the above
Ans. E

5. Time to reach steady state is determined by the?


A) Elimination half life
B) Loading dose
C) Maintenance dose
D) The absorption rate
E) dose interval
ans. A

Copyright © 2000-2016 TIPS Inc. Unauthorized reproduction of this manual is prohibited. This 24
manual is being used during review sessions conducted by PharmacyPrep.
www.pharmacyprep.com

6.A drug completely renally eliminated. The patient has renal failure. This drug is hepatic
metabolized. What will happen to drug?
A. Drug metabolite accumulates in blood
B. Drug is completely in metabolized in liver
C. Drug is completely eliminate renally
D. Drug is completely protein bound
E. Drug have linear kinetic
Ans. A

7. What is correct about a drug with first order elimination?


A) drug elimination depends on initial concentration
B) drug elimination depends on time only
C) drug elimination depends and concentration and time
D) drug elimination depends renal and hepatic function
E) drug elimination depends on renal function only
Ans. B

8. What is correct about a drug with zero order elimination?


A) drug elimination depends on initial concentration
B) drug elimination depends on time only
C) drug elimination does NOT depends and concentration and time
D) drug elimination depends renal and hepatic function
E) drug elimination depends on renal function only
Ans. b

Copyright © 2000-2016 TIPS Inc. Unauthorized reproduction of this manual is prohibited. This 25
manual is being used during review sessions conducted by PharmacyPrep.
www.pharmacyprep.com

Copyright © 2000-2016 TIPS Inc. Unauthorized reproduction of this manual is prohibited. This 26
manual is being used during review sessions conducted by PharmacyPrep.
www.pharmacyprep.com

PHARMACY PREP
PHARMACODYNAMICS

1. Competitive (reversible) antagonism


Ag (agonist) + E + Ag*(antagonist) AgE + Ag*E
If we use more concentration of Ag what happen?
A. Concentration of AgE increase and Ag*E decrease
B. Concentration of Ag*E increase and AgE decrease
C. Concentration of AgE and Ag*E decrease
D. Concentration of AgE and Ag*E increase
E) No change in AgE and Ag*E rate
Ans. A

Competitive (equilibrium) Non competitive (irreversible)


Competitive inhibitors are molecules that Non competitive inhibitors are molecules
bind to the same site as the substrate that bind to some other site on the
preventing the substrate from binding as enzyme reducing its catalytic power.
they do so but are not changed by the
enzyme.
Parallel rightward shift of agonist dose Flattening of dose response curve
response curve shift.
Q. Intensity of response depends on Q. Response depends on only on the
concentration of both agonist and concentration of antagonist
antagonist.
The same maximal response is attained by Maximal response is suppressed
increasing the dose of agonist

2. The k m value of an enzyme is numerically equal to, in first order kinetics


A. Half the maximum velocity (V max ) expressed in moles/liter
B. Velocity of a reaction divided by substrate concentration
C. Substrate concentration in moles/liter necessary to achieve half the maximum
velocity of a reaction
D. Maximum velocity divided by half the substrate concentration in moles necessary to
achieve maximum velocity
E. Substrate concentration divided by velocity of a reaction
Ans. C
First order Km = 1/2 V max
Zero order Km = V max

27-1
Pharmacyprep.com Medicinal Chemistry

PHARMACY PREP.

MEDICAL CHEMISTRY

1. Which of the following is the molecular structure of isotretinoin, used in oral


treatment of acne?
A. 13-CIS Retinoic acid
B. 13-TRANS Retinoic acid
C. 13-CIS-TRANS Retinoic acid
D. Retinoic acid
E. None of the above

2. What functional group is responsible for sulfa allergy


A) Sulfonamide
B) Sulfide
C) Sulfhydril group
D) Proline ring
E) Carbamate group
Ans. A

3.Fexofenadine, how many optical isomers are possible?


O
H3C
H3C
OH
CH3 CH3
CH3
HO N
HO N
OH
OH

Terfinadine Fexofenadine

A) 1 B) 2 C-3 D) 4 E) None
Ans.B

4. If a drug chemical structure has 3 chiral carbons. How many optical isomers are
possible?
A) 2 B) 4 C) 8 D) 9 E) 12
Ans. C

2n = 22 =2x2=4
n= number of chiral centers
23 = 2 x 2 x 2 = 8

Copyright © 2000-2016 TIPS Inc. Unauthorized reproduction of this manual is prohibited. This 28-1
manual is being used during review sessions conducted by PharmacyPrep.
Pharmacyprep.com Medicinal Chemistry

Copyright © 2000-2016 TIPS Inc. Unauthorized reproduction of this manual is prohibited. This 28-2
manual is being used during review sessions conducted by PharmacyPrep.
PharmacyPREP.Com Medicinal Chemistry of autonomic nervous system

Pharmacy Prep

MEDICINAL CHEMISTRY OF AUTONOMIC NERVOUS SYSTEM

1.What is incorrect about structure activity of acetylcholine agonist?


A) acetylcholine dissociate into acetyl and choline by acetylcholinesterase enzyme
B) methanocoline has one extra methyl group than Ach
C) Carbacholine has one carbmate group
D) Bethanacholine has one carbamate and methyl group than Ach
E) acetylcholine is the major neurotransmitter of sympathetic system
Ans: E

2.What is incorrect in catecholamine biosynthesis?


A) tyrosine is the precursor
B) dopamine produce norepinephrine hydroxylation
C) epinephrine produce norepinephrine by methylation
D) levodopa catalyzed by dopadecarboxylase to dopamine
E) norepinephrine catalyzed by methyl transferase to epinephrine
Ans: C

3)The difference between dopamine and amphetamine?


A) one methyl group
B) one hydroxyl group
C) 2 hydroxyl group and 1 methyl group
D) 2 methyl group and 1 hydroxyl group
E) One methyl and one hydroxyl group

NH2

HO Dopamine

OH
OH H OH H H
NH2 CH3 N CH3 N CH3 N CH3

CH3 NH2 CH3 CH3 CH3

Amphetamine Dextroamphetamine Ephedrine


Pseudoephedrine Methamphetamine
A B C D E
4.Difference between pseudoephedrine and methyl amphetamine is?
A) one methyl group
B) one hydroxyl group
C) 2 hydroxyl group and 1 methyl group
D) 2 methyl group and 1 hydroxyl group

Copyright © 2000-201 TIPS Inc. Unauthorized reproduction of this manual is prohibited. This manual 24-1
is being used during review sessions conducted by PharmacyPrep.
PharmacyPREP.Com Medicinal Chemistry of autonomic nervous system

E) One methyl and one hydroxyl group


Ans. b

5.What chemical reaction converts pseudoephedrine to crystal meth?


A) Hydroxylation
B) dehydroxylation
C) Hydrolysis
D) methylation
E) Glucuronidation
Ans. B

6.Beta blockade effect can be reversed by?


A. metoprolol
B. salbutamol
C. prazosin
D. propranolol
E. pseudoephedrine
ans. B
Tips: beta blockade can be reversed by beta agonist by displacement from occupied
receptors if large amount of agonist is given because all beta blockers are competitive
antagonist.

Copyright © 2000-201 TIPS Inc. Unauthorized reproduction of this manual is prohibited. This manual 24-2
is being used during review sessions conducted by PharmacyPrep.
PharmacyPrep.Com Medicinal Chemistry of autocoids and analgesics

PHARMACY PREP

MEDICINAL CHEMISTRY OF AUTOCOIDS AND ANALGESICS

1 . The following groups are essential for antihistaminic pharmacological activity:


I-Aromatic or heteroaromatic rings in structure
II-Tertiary amines in structure
III-Chain of alkyl group in structure
A. I only
B. III only
C. I and II only
D. II and III only
E. I, II, III
Ans. E

2. Ecosonides are?
A. Leukotrienes
B. Prostaglandins
C. serotonins
D.Histamines
Ans. b

3.Ondansetron is?
A. Serotonin type 1 antagonist
B. Serotonin type 2 antagonist
C. Serotonin type 3 antagonist
D. Serotonin type 3 agonist
Ans. C

4.Which of the following is an end product of glutathione conjugation in acetaminophen


metabolism?
A. N-benzoquinineamine
B. Mercapturic acid
C. Glutathione
D. cysteine
E. Glycine
ans. B

5.Glutathione is?
A. cysteine, glycine and lysine
B. cysteine, glycine and tryptophan
C. cysteine, glycine and glutamate
D. Cysteine, glycine and isoleucine
Ans. C

Copyright © 2000-2016 TIPS Inc. Unauthorized reproduction of this manual is prohibited. This manual is being 25-1
used during review sessions conducted by PharmacyPrep.
PharmacyPrep.Com Medicinal Chemistry of autocoids and analgesics

6.) The following structure is prostaglandin analogue of?


O
9 COOH

11
HO
A) Prostaglandin E
B) Prostaglandin F
C) Prostaglandin G
D) Prostaglandin H
E) Prostaglandin I
Ans. A

7.Morphine analgesic action is due to mu receptors by hydrogen bonding of?


A) Endorphine peptides
B) Enkephalin peptides
C) Leukotrienes
D) prostaglandins
E) histamines
Ans. B

8.Morphine analgesic actions is mainly produced due to?


A. mu, kappa, and delta receptors
B. mu, and kappa agonist
C. mu agonist
D. mu, and delta agonist
E. kappa and delta agonist
ans. C

9.Naloxone is an opioid antagonist and blocks the actions of?


A. mu, kappa, and delta receptors
B. mu, and kappa receptors
C. mu receptors
D. mu, and delta receptors
E. kappa and delta receptors
Ans. C

10.Synthetic estrogen like diethyl stilbenes gives?


A) enantiomers
B) diastereomers
C) geometric isomers
D) optical isomers
E) confirmers
Ans. c

Copyright © 2000-2016 TIPS Inc. Unauthorized reproduction of this manual is prohibited. This manual is being 25-2
used during review sessions conducted by PharmacyPrep.
www.pharmacyprep.com

PHARMACY PREP.
MEDICINAL CHEMISTRY AND PHARMACOLOGY OF CARDIOVASCULAR SYSTEM

1. What is incorrect about site of action of diuretics?


A) carbonic anhydrase and osmotic diuretics like acetazolamide act on proximal tubule
B) loop diuretics act on loop of henle
C) Thiazides act on distal convoluted tubule
D) Potassium sparing act on collecting duct
E) Potassium sparing diuretic act on proximal tubule
Ans. E

2. Diuretics effect on acid and base balance of blood. What is incorrect?


A) thiazides and loop cause metabolic alkalosis
B) acetazolamide cause hyperchloremic metabolic acidosis
C) spironolactone cause intracellular alkalosis
D) acetazolamide decrease intraocular pressure
E) Furosemide may cause metabolic acidosis
Ans. E

3.) Diuretic characterized for e a an a


A. Acetazolamide
B. Spironolactone
C. Hydrochlorothiazide
D. Furosemide
E. Triamterene
Ans. D

4.. Imidazoline is a molecular structure that contains a heterocyclic ring and belongs the
molecular structure of:
A. Prazosin
B. Nifedipine
C. Atropine
D. Clonidine
E. Allopurinol
Ans. D

5. Which of the following cardiovascular medications chemically classified as glycoside?


A. Digoxin
B. Warfarin
C. Nitrates
D. Nitrites
E. Cholestyramine
Ans. A

Copyright © 2000-2012 TIPS Inc. Unauthorized reproduction of this manual is prohibited. This 31-1
manual is being used during review sessions conducted by PharmacyPrep.
www.pharmacyprep.com

6. Nitroglycerine is classified as?


A. Nitrites B. Nitrates C. Nitriles D. Nitrol E. None of the above
Ans. B

7) What functional group is responsible for metallic taste or taste changes in captopril?
A) Sulfonamide
B) Sulfide
C) Sulfhydril group
D) Proline ring
E) Carbamate group
Ans. A

8) Pyridine is a mono heterocyclic compound very important in the activity of many


eren r s e ec ar s r c re -dihydropyridine is found in:
A. Verapamil
B. Digoxin
C. Clonidine
D. Nifedipine
E. Diltiazem
Ans. D

9. Thrombolytics (fibrinolytic) agents are contraindicated in?


a. Myocardial infarction
B. Ischemic stroke
C. Bleeding from the GIT
D. Pulmonary embolism
E. Deep vein thrombosis
ans. C

10 ce a a e a car n c an rase re c an r c r a e a a e
diuretic have an identical molecular group very important in their structure knows as:
A. Sulfonamide group
B. Pyrimidine group
C. Purine group
D. Pyrrolidine group
E. Ether group
Ans. A

Copyright © 2000-2012 TIPS Inc. Unauthorized reproduction of this manual is prohibited. This 31-2
manual is being used during review sessions conducted by PharmacyPrep.
www.pharmacyprep.com

11. Which of the following statements regarding HEPARIN is/are right


I. It is a muc sacar e r an c ac resen na ra n an ss es es ec a n
lungs and liver.
II. It has anticoagulant proprieties that inhibit the conversion of prothrombim to
thrombin and fibrinogen to fibrin
III. It has antithrombin III as it co-factor
A. I only B. III only C. I and II only D. II and III only E. All are correct

12. What class of drug represents the following chemical structure?

O
HS - H2C
N
CH3
CO2H
A) Diuretics
B) Angiotensin converting enzyme (ACE) Inhibitors
C) Calcium channel blockers
D) Selective Serotonin Receptor Inhibitors
E) Angiotensin Receptor Blockers (ARBs)
Ans. B

13) What is incorrect about ACE Inhibitors?


A) sulfhydril group is present in captopril
B) captopril and fosinopril are NOT prodrugs because these do not require hydrolysis to
produce active products
C) it essential to have esterase for the activation of ACEi.
In re en r a n an ens n II
E) Angiotensin II is strong vasodilator
Ans. E
Tips: Angiotensin II is strong vasoconstrictor

14 sa an s s n ra r a rea er ens n recen


reports cough. What is incorrect?
A) recommend to see doctor to change to ARBs
B) recommend cough suppressant dextromethorphan
C) ARBs have no cough SEs
c s n ce ra n ns n
E) CCBs have no cough side effects
Ans: B

Copyright © 2000-2012 TIPS Inc. Unauthorized reproduction of this manual is prohibited. This 31-3
manual is being used during review sessions conducted by PharmacyPrep.
www.pharmacyprep.com

15) What is active pharmacophore in Angiotensin receptor blockers?


N
Cl
Imidazole ring (essential) N

Tetrazole ring OH

N N
N
N
H

Losartan
A) Tetrahydropyrazole ring
B) On position 2 ethyl alcohol on imidazole ring
C) Two ring structure
D) Alkyl long chain on imidazole ring
E) AT 1 receptors
Ans. B

16) HMG Co A reductase inhibitor like statins effective lowers LDL. What is incorrect?
-dihydroxycarboxylate is essential for SARs
B) Myopathy SE is e ass c a e s a ns s s n re -MM
C) Initial SEs include stomach upset like diarrhea
a ns ncrease er en es s n r s
E) Statins combined with niacin decrease the risk of myopathy
Ans: E

17 sa ear a e patient has been prescribed amlodipine 10 mg daily. After 2


ee s n a n s a en re r s re en n ee ea ac e s n r
an a a n ar ac s s s ec ese are s e e ec a ne c
of the following is not side effects of CCBs?
A) ankle edema
B) flushing
C) headache
D) palpitation
E) dry mouth
Ans. E

Copyright © 2000-2012 TIPS Inc. Unauthorized reproduction of this manual is prohibited. This 31-4
manual is being used during review sessions conducted by PharmacyPrep.
www.pharmacyprep.com

18) Which of the following drugs represent the structure below?

A. Leukotriene antagonists
B. Beta blockers
C. Coumarins
D. Calcium channel blockers
E. Benzodiazepines
Ans: D

19 n na rec a e e erc se r e na s ress r c ce


A. Propranolol
B. Nitroglycerine SL
C. Diltiazem
D. Nifedipine
Ans-B
Tips: nitroglycerine SL or spray

20) Clopidogrel is classified as an:


A) Anti-hypertensive.
B. Inhibitor of platelets' aggregation.
C. Anti-diabetic.
D. Anti-Parkinson’s.
Ans: B
Tips: Clopidogrel (Plavix) is a blood thinning agent. Agents of this class generally act by
inhibiting aggregation of platelets which is thought to be the primary mechanism of
blood clotting. It is indicated for prevention of heart stroke in patients with CHF.

21.Which of the following oral anticoagulant is safe in renal disease?


A. Apixaban
B. Heparin
C. Dalteparin
D. Warfarin
E. Dabigatran
Ans.D

Copyright © 2000-2012 TIPS Inc. Unauthorized reproduction of this manual is prohibited. This 31-5
manual is being used during review sessions conducted by PharmacyPrep.
www.pharmacyprep.com

22.Factor Xa selective inhibitor is?


A. Dabigatran
B. Warfarin
C. Tinazaparin
D. Apixaban
E.Heparin
Ans. D

23.Nitrates produce smooth muscle vascular relaxation by producing?


A. Nitric acid
B. Nitric oxide
C. Nitrates
D. Vasodilator
ans. B

Copyright © 2000-2012 TIPS Inc. Unauthorized reproduction of this manual is prohibited. This 31-6
manual is being used during review sessions conducted by PharmacyPrep.
PHARMACY PREP

MEDICINAL CHEMISTRY AND PHARMACOLOGY OF PYCHIATRIC AND NEUROLOGICAL


DISORDERS

1. What are the physiological function of serotonin?


A) mood
B) Sleep
C) platelet aggregation
D. weight
E. All of the above
Ans. E

2.What is NOT a side effect of SSRIs?


A) GI bleeding
B) insomnia
C) nausea and vomiting
D) sexual dysfunction
E) anticholinergic SEs
Ans: E

3.What is incorrect about SSRIs?


A) if combine with MAOi, can cause serotonin syndrome
B) fluoxetine has long half life, thus have no serotonin syndrome
C) Abrupt discontinuation of SSRIs can cause withdrawal symptoms
D) Do not combine SSRIs with other SSRIs
E) SSRIs require washout period
Ans. B

4. Which of the following SSRI least likely associated with withdrawal symptoms?
A. Trazodone B. Amitriptyline C. Phenelzine D. Fluoxetine E. Mirtazapine
Ans. d

5.Topiramate can be used for all, except?


A. seizures
B. migraine prophylaxis
C. weight loss
D. dementia
E. bipolar disorders
ans. D

Copyright © 2000-2016 TIPS Inc. Unauthorized reproduction of this manual is prohibited. This manual is 27-1
being used during review sessions conducted by PharmacyPrep.
6.MP is 20 yo man using lithium carbonate 400 mg QID for the treatment of manic depression.
Patient presents with tremors. All of the following drugs increase lithium toxicity by
increasing lithium serum concentration, except
A. Ramipril
B. Naproxen
C. Caffeine
D. Hydrochlorothiazide
Ans. C

7. Which of the following drug combination least likely cause serotonin syndrome?
A. Phenelzine and fluoxetine
B. Phenelzine and amitriptyline
C. Phenelzine and Tranylcypromine
d. Bupropion and trazodone
E. Selegiline and tramadol
Ans. e
Tips. bupropion and trazodone have weak affect on serotonins, thereby less likely serotonin
syndrome. The most likely are combination with MAOi like C.

8. Which of the following is least likely side effect of haloperidol?


A. drowsiness
B. loss of appetite
C. weight gain
D. dizziness/fainting
E. Parkinsonism
Ans. A

9. Haloperidol can cause hypotension. It is due to the?


A. Inhibition of D 2 receptor
B. Inhibition of M 1 receptor
C. Inhibition of H 1 receptor
D. Inhibition of alpha 1 receptor
E. Inhibition of alpha 2 receptor
Ans. D

Copyright © 2000-2016 TIPS Inc. Unauthorized reproduction of this manual is prohibited. This manual is 27-2
being used during review sessions conducted by PharmacyPrep.
10. Which of the following is correct about dibenzazepine drugs like clozapine and
quetiapine?
A. Act more effectively on D 2 receptors than 5HT 2
B. Acts more effective on 5HT 2 than D 2 receptors
C. Acts more effective on 5HT 2 and M 1
D. Acts more effectively on D 2 and 5HT 2 , H 1 , M 1 and alpha1
E. acts on D 2 receptors only
Ans. B

11. Tobacco smoking?


A. induce CYP3A4
B. induce CYP1A2
C. inhibit CYP3A4
D. inhibit CYP1A2
E. Substrate of CYP1A2
Ans.A

12. Tardive dyskinesia is?


A. Parkinson's disease symptom
B. Extra pyramidal symptoms
C. Side effect of anti-Parkinson's drugs
D. Side effect of levodopa
E. Early symptom of antipsychotic drugs
Ans. b

Copyright © 2000-2016 TIPS Inc. Unauthorized reproduction of this manual is prohibited. This manual is 27-3
being used during review sessions conducted by PharmacyPrep.
PharmacyPrep.Com Medicinal Chemistry and Pharmacology of Endocrine System

PHARMACY PREP
MEDICINAL CHEMISTRY AND PHARMACOLOGY OF ENDOCRINE SYSTEM

1.What drugs increase insulin secretion?


A. Acarbose B. Sitagliptine C. Nateglinide D. Liraglutide E. Metformin
Ans. C

2. All of the following drugs increase endogenous insulin secretion, except?


A. glyburide
B. Glicalizide
C. Metformin
D. Nateglinide
E. Repaglinide
Ans. C

3. What is the common side effect of acarbose?


A. Diarrhea
B. Constipation
C. GI bleeding
D. ulcerative colitis
Ans. A

4.Examples of ketones that appears in blood in DKA?


A) beta hydroxyl butyric acid
B) acetoacetic acid or acetoacetate
C) acetone
D) All of the above
Ans. D

28-1
Copyright © 2000-2016 TIPS Inc. Unauthorized reproduction of this manual is prohibited. This manual is
being used during review sessions conducted by PharmacyPrep.
PHARMACY PREP

MEDICINAL CHEMISTRY AND PHARMACOLOGY OF RESPIRATORY DRUGS

1. Drug that may cause bronchospasm?


A. Ipratropium
B. Tiotropium
C. Betaxalol
D. Salbutamol
E. Montelukast
Ans.C

2. An asthma patient using salbutamol PRN but her asthma is not controlled. What drug is
added next?
A. Salmeterol
B. Fluticasone inhaler
C. Prednisone oral
D. Montelukast
E. Theophylline
Ans. b

3. A patient using salbutamol for the past 6 months, but asthma is not controlled. Which drug is
added next?
a. Increase salbutamol dose
B. Inhaled corticosteroids
C. Formterol
D. Oral prednisone
Ans. b

4.In asthma patient concomitant anti-inflammatory therapy (inhaled corticosteroids) should be


part of regimen if?
A. Salbutamol needs to be used >3 times per week (not including its use to prevent exercise
induced asthma).

5. Which of the following is the least likely use of montelukast?


A. ASA induced asthma
B. Acute attack of asthma
C. Uncontrolled asthma
D. nocturnal asthma
Ans. B

29-1
Copyright © 2000-2016 TIPS Inc. Unauthorized reproduction of this manual is prohibited. This manual is being
used during review sessions conducted by PharmacyPrep.
6. COPD patient currently using salbutamol PRN, ipratropium PRN, tiotropium daily,
theophylline bid and Formoterol daily.
Which of the following drug is the pharmacist concern?
A) salbutamol B) ipratropium C) tiotropium D) formoterol
E) theophylline
ans. E
Tips. Theophylline can cause tachycardia, tremors, nervousness, insomnia, abdominal cramps,
Nausea and vomiting.

7.A 50 yr old COPD patient exacerbations, which of the following antibiotics is least likely used?
a. amoxicillin B. cotrimoxazole C. azithromycin D. doxycycline E. ciprofloxacin
Ans. E
Tips. ciprofloxacin is least likely used for respiratory tract infections. However respiratory tract
fluroquinolones levofloxacin and moxifloxacin can be used.

29-2
Copyright © 2000-2016 TIPS Inc. Unauthorized reproduction of this manual is prohibited. This manual is being
used during review sessions conducted by PharmacyPrep.
www.pharmacyprep.com Med Chem and Pharma of Musculoskeletal Drugs

PHARMACY PREP
MEDICINAL CHEMISTRY AND PHARMACOLOGY OF MUSCULOSKELETAL DRUGS

Bisphosphonates mechanism by permanently binding surfaces of mineralized bone?


A) Prevent osteoblast activity
B) Prevent osteoclast activity
C) Prevent bone minerilization
D) Prevent bone deminerilzation
E) Increase calcium deposits on bone
Ans. B
Tips: osteoclasts cause bone resorption thus preventing osteoclast increase bone
mineral density

Teriparatide increases cortical thickness and bone size. It is?


A. Anabolic agent
B. Bisphosphonates
C. SERM
D. Hormone replacement therapy
Ans. A

Which of the following prescription is the pharmacist concern?


A. Azithromycin 250 mg daily for 3 days
B. Alprazolam 0.5 mg SL as needed
C. Methotrexate 7.5 mg daily
D. Acetaminophen 650 mg Q4-6hrs
E. Fentanyl patch 25 mg for 3 days for pain

What is the drug of choice for rheumatoid arthritis in pregnancy?


A. Methotrexate
B. Hydroxychloroquine
C. Azathioprine
D. Leflunomide
E. Infliximab
Ans. B
Copyright © 2000-2011 TIPS Inc. Unauthorized reproduction of this manual is prohibited. This
manual is being used during review sessions conducted by PharmacyPrep.
35-1
www.pharmacyprep.com Med Chem and Pharma of Musculoskeletal Drugs

What analgesics drugs are NOT used to treat acute gout attacks?
A) allopurinol
B) acetaminophen
C) colchicine
D) dexamethasone
E) indomethacin
Ans. b

Prednisone used in treatment of gout arthritis (intra articular inj.) and severe asthma
(oral). What is correct?
A) gout patient may have more systemic SEs
B) asthma patient may have more systemic SEs of steroid
C) Asthma & gout patient have same SEs
D) Only gout patient have prednisone SEs
E) None of the above

What is NOT a side effect of alendronate if not taken properly?


A) esophagitis
B) heartburn
C) GI bleeding
D) Dysphagea
E) Colorectal cancer
Ans: E

All of the following drugs have uricosuric effect, except?


A. allopurinol
B. Fenofibrate
C. Losartan
D. Febuxostat
E. Colchicine
Ans. E
Q. Monoclonal antibodies consist of human and mouse proteins
are?
A. Humanized MAB
Copyright © 2000-2011 TIPS Inc. Unauthorized reproduction of this manual is prohibited. This
manual is being used during review sessions conducted by PharmacyPrep.
35-2
www.pharmacyprep.com Med Chem and Pharma of Musculoskeletal Drugs

B. Murine MAB
C. Mice MAB
D. Chimeric MAB
D. Human MAB
Ans.d

Q. Bisphosphonates are?
A. osteoclasts
B. Osteoblast
C. Pyrophosphate
D. Phosphoric acids
E. calcium phosphate
Ans. c

Copyright © 2000-2011 TIPS Inc. Unauthorized reproduction of this manual is prohibited. This
manual is being used during review sessions conducted by PharmacyPrep.
35-3
www.pharmacyprep.com

Pharmacy Prep
MEDICINAL CHEMISTRY AND PHARMACOLOGY OF ANTIMICROBIALS

MP is 49-year-old women using calcium carbonate 500 mg QD and bring a new prescription of
ciprofloxacin 500 mg BID for 3 days to treat UTI. What is appropriate?
a) Skip calcium supplements for 3 days and take ciprofloxacin
b) Take ciprofloxacin morning and supper time and take calcium 2 tab morning.
C) take ciprofloxacin morning 2 tab and calcium 2 tab at supper time
D) Take ciprofloxacin morning and supper and calcium 2 tab at noon.
Ans. D

Which of the following is NOT an antifungal?


A. Ketoconazole
B. Clotrimazole
C. Miconazole
D. Metronidazole
E. Fluconazole.
Ans. D

What is correct about ketoconazole?


A. It is strong inhibitor of CYP3A4
B. It is strong inhibitor of CYP3A4 and 2C9
C. It is strong inducer of CYP3A4 and 2C9
D. It is substrate of CYP3A4 and 2C9
E. It is inhibitor of CYP2D6
Ans. b

All of the following drug can cause serotonin syndrome when combined with MAOi, except?
A. St. John wart
B. Linezolid
C. Fluoxetine
D. Amitriptyline
E. Amoxicillin
Ans. E

Copyright © 2000-2016 TIPS Inc. Unauthorized reproduction of this manual is strictly prohibited and it is
illegal to reproduce without permission. This manual is being used during review sessions conducted by
PharmacyPrep.
36-1
www.pharmacyprep.com

PHARMACY PREP.
METABOLISM
Q. What is the common metabolic reaction of acetyl salicylic acid?
A. glucuronidation
b. glutathione conjugation
C. Glycine conjugation
D. Hydroxylation
E. oxidative deamination
Ans. C

Which of the following enzyme catalyzes glycine conjugation?


A) UDP glucoronyl transferase
B) Glutathione-S-transferase
C) Acetyl co-A
D) Cytochrome CYP 450
E) Esterase
Ans: C

In hepatic detoxification related to glutathione conjugation, which aminoacid related?


A) Glycine
B) Cysteine
C) Phenylephrine
D) Glutathione
E) Glutamic acid
Ans. B

If a patient taking simvastatin 40 mg daily, and brings a prescription of clarithromycin


for 5 days. What is pharmacist appropriate action?
A. dispense clarithromycin and counsel patient to space for 2 hr from simvastatin
B. dispense clarithromycin and counsel patient to stop taking simvastatin for 5 days
while taking clarithromycin
C. Do not dispense clarithromycin and call doctor to change to other antibiotic
D. Call doctor to change simvastatin to other atorvastatin
E. No worries just dispense clarithromycin because patient has right to take antibiotics
Ans. B

Copyright © 2000-2015 TIPS Inc. Unauthorized reproduction of this manual is prohibited. This 37-1
manual is being used during review sessions conducted by PharmacyPrep.
www.pharmacyprep.com

PHARMACY PREP
BIOPHARMACEUTICS

13. if a medication if pH = Pka?


A-100% solute is ionized
B-40% solute is ionized
C-50% solute is ionized (1/2)
D-99% solute is ionized
E-95% solute is ionized
Ans. C

14. A drug degradation at room temperature is determined by?


A. Noye-Whittney equation
B. Arrhenius equation
C. Hasselbach-Handerson equation
D. Ficks law
E. Laplaws Law
Ans. B

16. Which of the following forms buffer?


A. weak acid and salt of acid
B. Weak base and salt of acid
C. Weak acid and salt of base
D. Strong acid and salt of weak base
E. Strong base and salt of weak acid
Ans. A

What is correct statement?


Compare Cmax and Tmax
Drug A Drug B
Dose 100 100
Cmax 80 50
Tmax 1 6

A. Drug A is immediate release and Drug B is Sustain release.


B. Drug A is controlled or sustained release
C. Drug B is immediate release
D. Drug A and B is controlled relase

Copyright © 2000-2012 TIPS Inc. Unauthorized reproduction of this manual is prohibited. This 38-1
manual is being used during review sessions conducted by PharmacyPrep.
www.pharmacyprep.com

Ans. A

Rx for Dimicron MR 30 mg. A pharmacist dispensed gliclazide MR


30 mg. What is concern?
A. Need prescriber approval
B. Need to inform patient about change
C. Cannot be changed because they are not equal
D. This is therapeutic interchange
E. This is Therapeutic alternate
Ans. B

Q. What type drug absorption across placenta in pregnancy?


A. Passive diffusion
B. Active facilitated transport
C. Facilitated diffusion
D. Endocytosis
E. Exocytosis
Ans. A

3. Determine the F value for a capsule AUC 20 mg/dL/hour with 100 mg dose when i.v
of same drug AUC is 25 mg/dL/hour with 100 mg dose.
A. 20%
B. 60%
C. 40%
D. 80%

Copyright © 2000-2012 TIPS Inc. Unauthorized reproduction of this manual is prohibited. This 38-2
manual is being used during review sessions conducted by PharmacyPrep.
www.pharmacyprep.com

E. 100%
Ans. D
AUC po x 100 = 80%
AUC iv

4. Determine the F value for a capsule AUC 20 mg/dL/hour with 50 mg dose when i.v of
same drug AUC is 25 mg/dL/hour with 100 mg dose.
A-20% B-60% C-40% D-80% E-160%
Ans. E
AUC po x 100
AUC iv
AUC po = 20 x 2 = 40 mg = 1.6 x 100 = 160%
25 mg

5. Two drugs to be pharmaceutical equivalent, the drugs must:


I-contain same exipient
II-Contain it salts
III-Same therapeutic moiety
A. I only B. III only C. I and II only D. II and III only E. All of the above
ans. d

3- What is the absolute bioavailability of tablet A?


TABLET A-----------------50mg---------------AUC 40
IV PUSH -------------------10mg---------------AUC 50
a) 80%
b) 0.16%
c) 16%
d) 8%
e) 25%

Ans- C We need to multiply the IV dose by 5 to find the same dosage so:
IV Push---------------50mg---------------AUC 250

Bioaval = AUC Tablet A = 40 = 0.16 X 100 = 16%


AUC IV Push 250

Physiochemical factors affecting the rate and extent of drug distribution includes?
a) plasma protein binding
b) types of surfactant in preparation

Copyright © 2000-2012 TIPS Inc. Unauthorized reproduction of this manual is prohibited. This 38-3
manual is being used during review sessions conducted by PharmacyPrep.
www.pharmacyprep.com

c) the partition coefficient the drug


d) percent of ionization
e) Intracellular binding
Ans. d
Area under the curve (AUC) of drug is double, the affect on clearance
A. Drug clearance is doubled
B. Drug clearance is increases
C. Drug clearance is decreases
D. Drug clearance does not change
Ans. C
Tips. AUC = (dose x F)/clearance
or
clearance = (dose x F)/AUC

Copyright © 2000-2012 TIPS Inc. Unauthorized reproduction of this manual is prohibited. This 38-4
manual is being used during review sessions conducted by PharmacyPrep.
PharmacyPREP.Com Physical Pharmacy

PHARMACY PREP.
PHYSICAL PHARMACY

1. Which of the following equations can be used to easily calculate the pH. of a buffer
solutions constituted of a weak acid and its salts.
A. Hixson-Crowell equation
B. Arrhenius equation
C. Stokes equations
D. Henderson-Hasselbalch equation
E. Noyes-Whitney equation
Ans. D
Tips: Henderson-Hasselbalch equations used to mathematically calculate the pH of a
buffer solution.

2. Arrhenius equation is used when we need calculate the:


A. pKa of an acid solution
B. Stability of a drug at room temperature
C. pH of a buffer solution
D. Rate of ionization of a drug
E. Solubility of a solution
Ans. B
Tips: Arrhenius equation is used to calculate the stability of a drug at room
temperature.

3. All are characteristics of a polymorph drug, EXCEPT:


A. It has a two different physical form
B. It melts in some determinate temperature, normally high temperatures (>30°C)
C. Theobroma oil is a classic example of polymorph drug
D. Polymorph drugs are widely used as suppository base
E. Polymorph drugs melt in very low temperature (<10°C)
Ans. E
Tips: Polymorph drug has two different physical form depending the currently
temperature that they are present, normally melting in high temperatures (>30C), and
keeping solid in low temperature (<10C). Theobroma oil or cocoa butter is a classic
example of polymorph drug

4. What is correct about buffer?


A. acid and base mixture
B. base and acid mixture
C. weak acid and salt of acid mixture
D. weak base and strong acid mixture

Copyright © 2000-2016 TIPS Inc. Unauthorized reproduction of this manual is prohibited. This 39-1
manual is being used during review sessions conducted by PharmacyPrep.
PharmacyPREP.Com Physical Pharmacy

E. strong base and weak base


Ans. C

5. Lyophylization method
A. sublimation of dry powder to vapor
B. drying wet drug to dry powder
C. powder sterilization methods
D. gaseous sterilization
Ans. B

Copyright © 2000-2016 TIPS Inc. Unauthorized reproduction of this manual is prohibited. This 39-2
manual is being used during review sessions conducted by PharmacyPrep.
PharmacyPrep.Com Pharmaceutical Excipients

PHARMACY PREP
PHARMACEUTICAL EXCIPIENTS

1) Magnesium stearate is?


A) Lubricant B) Antiadherant C) Glidant D) Disintegrants E) Polishing agent
Ans. A

Q. Ascorbic acid?
I. Acidifying agent
II. water soluble antioxidant
III. Water soluble vitamin
A. I only B. III only C. I and II D. II and III E. I, II,II
Ans.E

Copyright © 2000-2013 TIPS Inc. Unauthorized reproduction of this manual is prohibited. This manual is
being used during review sessions conducted by PharmacyPrep.
40-1
www.pharmacyprep.com

PHARMACY PREP
RHEOLOGY

Gel to solution transformation. What is incorrect?


A. increase viscosity
B. decrease viscosity
C. Increase flow
D. Thixotropic system
Ans. A

Copyright © 2000-2012 TIPS Inc. Unauthorized reproduction of this manual is prohibited. This 41-1
manual is being used during review sessions conducted by PharmacyPrep.
www.pharmacyprep.com

PHARMACY PREP
PHARMACEUTICAL DOSAGE FORMS

Copyright © 2000-2012 TIPS Inc. Unauthorized reproduction of this manual is prohibited. This 42-1
manual is being used during review sessions conducted by PharmacyPrep.
www.pharmacyprep.com

PHARMACY PREP.
DRUG DELIVERY SYSTEMS
1. What is incorrect about transdermal drug delivery systems?
A. Drugs with long half life are preferred
B. Transdermal drugs after topical application on skin surface have systemic absorption
C. Transdermal drugs are absorbed by diffusion method
D. Systemic absorption is maintained throughout the dosing interval
E. Transdermal patches is essential to contact with skin
Ans. A

2. What is TRUE about enteric coating tablet dosage form?


A. It protects from the drug moisture, light or air
B. it mask the taste or odor of the drug
C. It improves the appearance of the tablet
D. It may affect the release rate of the drug
E. It minimize the contact between the drug and the gastric region and gastric content
Ans. E
Tips. Enteric coated tablet do not allow for dissolution in the acid pH environment of the
stomach. Instead in the it dissolved in less acidic pH like small intestine.

Copyright © 2000-2014 TIPS Inc. Unauthorized reproduction of this manual is prohibited. This 43-1
manual is being used during review sessions conducted by PharmacyPrep.
www.pharmacyprep.com

PHARMACY PREP.
PHARMACEUTICAL ANALYSIS

1. By addition of a solute, solubility of non-electrolytes is decreased. This is:


A. Common ion effect
B. Disintegration
C. Salting out
D. Salting in
E. None of the above

2. Which of the following compound is known as amphiprotic solvent?


A. Methanol
B. Water
C. Acetic acid
D. Ethanol
E. All are right

3. Molecular ion (M+) or molecular weight is detected by:


A. Colorimetry
B. Mass spectroscopy
C. Spectrophotometry
D. Spectrometry
E. Ultraviolet detectors

4. Determination of alcohol in blood in done by:


A. HPLC
B. TLC
C. Reverse phase HPLC
D. Gas chromatography & Mass spectroscopy (GC-MS)
E. Mass spectroscopy

5. Mass spectroscopy is used to:


A. Detect traces of unknown chemical structure
B. Structure elucidation of unknown chemical
C. Traces in environmental samples
D. Drug traces in blood samples
E. All of the above

6. All of the following detector are used in High Performance Liquid Chromatography
(HPLC): EXCEPT:
A. Diode Array Detector
B. UV detector
C. Refractive Index detector

Copyright © 2000-2016 TIPS Inc. Unauthorized reproduction of this manual is prohibited. This 39-1
manual is being used during review sessions conducted by PharmacyPrep.
www.pharmacyprep.com

D. Gamma Rays
E. Radioactive detector

7. Mechanism of action of Thin Layer Chromatography (TLC) is by:


A. Gas absorption
B. Column theory
C. Capillary action
D. High pressure extraction
E. Temperature range theory

8. Lyophilization (freeze dry method) technique used in:


I- Sterilization of liquids and solids
II- Sterilization of solids only
III- To dry heat-sensitive wet powders
A. I only B. III only C. I and II only D. II and III only E. All are correct

9. Sterilization of hormone is done by:


A. Dry heat
B. Lyophilization
C. Filtration
D. Radiation
E. Autoclave

10. Correct statements regarding ethylene oxide sterilization include:


I- A gas sterilization method that combines heat and moisture
II- Has a high penetration power to sealed containers
III- It is used in dry heat sterilization
A. I only B. III only C. I and II only D. II and III only E. All are correct

11. In the assay of amphetamines in plasma, it can be extracted by:


A. Ether
B. Isopropyl alcohol
C. Tetrahydrofuran
D. Ninhydrin
E. Amides

12. Reverse phase high performance liquid chromatography (RP-HPLC)


I- Mobile phase is aqueous solution
II- The particle size of stationary phase affects resolution
III- Utilize high pressure pump to increase efficiency in the resolution
A. I only B. III only C. I and II only D. II and III only E. All are correct
13. All of the following proprieties are classified as colligative proprieties, EXCEPT:
A. Elevation of boiling point
B. Osmotic pressure

Copyright © 2000-2016 TIPS Inc. Unauthorized reproduction of this manual is prohibited. This 39-2
manual is being used during review sessions conducted by PharmacyPrep.
www.pharmacyprep.com

C. Increase in conductivity
D. Lowering of freezing point
E. Magnitude of vapor pressure

14. The colligative proprieties of a solution are related to:


A. The total number of solute particles
B. pH of the solution
C. Number of ions
D. Ratio of number of ions and number of molecules
E. Number of non-ionized molecule

15. Colligative proprieties are important in determining:


A. pH B. Tonicity C. Solubility D. Stability E. Sterility

16. What is the plate’s function in thin layer chromatography (TLC) ?


A. Filter
B. Increase rate of resolution
C. Support mobile phase
D. Make resolution
E. Support stationary phase

17. At acid/base titration, which affect the reaction?


I- Change in the pH of the preparation
II- Addition of a specific buffer will increase the reaction
III-Addition of a specific buffer will decrease the reaction
A. I only B. III only C. I and II only D. II and III only E. All are correct

18. Atoms with the same number of protons are known as:
A. I only B. III only C. I and II only D. II and III only E. All are correct

19. Instrument used to measure specific gravity by floating in a liquid:


A. Picnometer
B. Hydrometer
C. Thermometer
D. Thermostat
E. Tensiometer

20. Instrument used to measure quantity of two liquids with equal volume:
A. Picnometer
B. Hydrometer
C. Thermometer
D. Thermostat
E. Tensiometer

Copyright © 2000-2016 TIPS Inc. Unauthorized reproduction of this manual is prohibited. This 39-3
manual is being used during review sessions conducted by PharmacyPrep.
www.pharmacyprep.com

21. Solvents can be classified as:


A. Amphiprotic
B. Protophilic
C. Aprotic
D. Protogenic
E. All are correct

22. Kind of solvent where acids and bases are produced, react and dissociates producing
such anion or cation species is called:
A. Amphiprotic solvent
B. Protophilic solvent
C. Aprotic solvent
D. Protogenic solvent
E. Inert solvent

23. Correct statements regarding TITRATION include:


I- Determination of a given component in a solution by addition of a liquid reagent of
known strength until a given endpoint is reached.
II- Non-aqueous titration can be used in the assay of some materials that cannot be
easily titrated in aqueous system.
III- Titration is a chromatographic method of substance separation.
A. I only B. III only C. I and II only D. II and III only E. All are correct

24. The end point of a titration analysis is characterized by:


A. Change in color B. Change in solubility C. Change in pH
D. Change in dispersability E. Change in particles aggregation

25. Considerations that should be taken in selecting a solvent for an assay analysis:
A. Nature and solubility of the sample under investigation
B. Side reactions should not occur between the sample in analysis and the solvent
C. The solvent should be pure, low toxic and inexpensive
D. Solvent should have a high dielectric constant
E. If the sample is a weak acid the solvent should also be a weak acid

26. Solvent used in the assay of levodopa and methyldopa includes:


A. Acetic acid B. Pyridine C. Chloroform
D. Formic acid E. Dimethyl formamide

27. Dimethyl formamide is a solvent used in the assay of which of the following drugs?
A. Adrenaline B. Sulfonamides C. Cimetidine
D. Chlortalidone E. Levodopa

Copyright © 2000-2016 TIPS Inc. Unauthorized reproduction of this manual is prohibited. This 39-4
manual is being used during review sessions conducted by PharmacyPrep.
www.pharmacyprep.com

28. Which of the following spectroscopic method is used in detection of nanogram


amount of unknown substance?
A. Microspectroscopy B. Nanospectroscopy C. Spectroscopy
D. Microscopy E. None of the above

29. Gel electrophoresis is based on separation of;


A. Micro molecules B. Electric charges C. Protein mixture separation
D. electrolysis E. Molecular weight

30. Gel electrophoresis is used for separation of?


A. large molecules like proteins B. small organic molecules
C. Organic chemicals D. Chemical structure elucidation

31. Method that used to detect in organic impurities:


A. Flame photometry B. Mass spectroscopy
C. Nuclear Magnetic Resonance D. Infra red spectroscopy

32. For reversed phase high performance liquid chromatography (RP-HPLC)?


A. Stationary phase is hydrophobic
B. Mobile phase is polar like water as solvent
C. Polar is separated first
D. used for lipid soluble substances
E. Organic solvents are used as mobile phase

Copyright © 2000-2016 TIPS Inc. Unauthorized reproduction of this manual is prohibited. This 39-5
manual is being used during review sessions conducted by PharmacyPrep.
www.pharmacyprep.com

PHARMACEUTICAL ANALYSIS
ANSWERS
1. Ans: C
Tips: Salting out is characterized when the addition of a solute in a solution cause the
decrease of solution solubility, when the addition of a solute cause the increase of
solubility is called salting in.

2. Ans: E
Tips: Amphiprotic is a type of solvent where acids and bases are produced. It reacts and
dissociates producing such anion and/or cation species.

3. Ans: B
Tips: M+ molecular ion is detected by mass spectroscopy that determines the molecular
weight of the sample.

4. Ans: D
Tips: Gas chromatography is a chromatographic method used to separate volatile
organic compounds as alcohol, hydrocarbons and aerosols.

5. Ans: D
Tips: Mass spectroscopy may be used as adjunct in gas chromatography.

6. Ans: D
Tips: X-rays are not used as detectors in HPLC

7. Ans: C
Tips: TLC-Thin Layer Chromatography is a very fast and simple chromatographic method
that separates organic compounds, progress of organic reactions, and tests the purity of
products by capillary theory.

8. Ans: B
Tips: Freeze dryer is used for the conversion of wet powders to dry powders.

9. Ans: D
Tips: Radiation is the most appropriated sterilization method for hormones; the only
exception is testosterone that is normally sterilized by dry heat sterilization.

10. Ans: C
Tips: Ethylene oxide sterilization is a very effective technique that combines heat and
moisture having a very high penetration power to sealed containers.

11. Ans: A
Tips: Amphetamines can be extracted from plasma by ether.

Copyright © 2000-2016 TIPS Inc. Unauthorized reproduction of this manual is prohibited. This 39-6
manual is being used during review sessions conducted by PharmacyPrep.
www.pharmacyprep.com

12. Ans: E
Tips: HPLC- High Performance Liquid Chromatography is a method that utilizes high-
pressure pumps to increase efficiency in the separation. The mobile phase is normally
an aqueous solution and the particles size of the stationary phase definitely alters the
effect of resolution.

13. Ans: C
Tips: Colligative properties in physical chemistry, depending on the number of molecules
present in a given space, rather than on their size, molecular weight or chemical
constitution. The colligative properties of solutions are osmotic pressure, boiling point
elevation, freezing point depression, and vapor pressure lowering.

14. Ans: A
Tips: Colligative properties in physical chemistry of a solution, depending on the number
of molecules present in a given space, rather than on their size, molecular weight or
chemical constitution.

15. Ans: B
Tips: 15- Colligative proprieties are important in determining the tonicity of the
solution. The colligative properties of solutions are osmotic pressure, boiling point
elevation, freezing point depression, and vapor pressure lowering

16. Ans: C
Tips: The only function of plates in TLC is support the mobile phase that moves by
capillary mechanism through the stationary phase (plates).

17. Ans: A
Tips: Buffers are added to the solutions exactly to avoid the change in the solution’s pH,
therefore if added to the solution will not affect the reaction.

18. Ans: B
Tips: In physical chemistry, isotopes are atoms with the same number of protons.
Isotones with the same number of neutrons and isobaros with the same number of
mass.

19. Ans: B
Tips: Hydrometer is an instrument used to measure specific gravity by floating in a
liquid: Hydrometer (SWIM in water)

20. Ans: A
Tips: Picnometer is an instrument used to measure quantity of two liquids with equal
volume

21. Ans: E

Copyright © 2000-2016 TIPS Inc. Unauthorized reproduction of this manual is prohibited. This 39-7
manual is being used during review sessions conducted by PharmacyPrep.
www.pharmacyprep.com

Tips: Solvents play an important rule in determining acid/base properties in a solution


and may be classified depending of their chemical structure and activity as Solvents can
be classified as amphiprotic, protophilic, aprotic and protogenic.

22. Ans: A
Tips: Amphiprotic solvent as water, acetic acid, methanol and ethanol are kind of
solvents where acids and bases are produced, react and dissociates producing such
anion or cation species.

23. Ans: C
Tips: Titration is a determination of a given component in a solution by addition of a
liquid reagent of known strength until a given endpoint is reached. Non-aqueous
titration can be used in the assay of some materials that cannot be easily titrated in
aqueous system.

24. Ans: A
Tips: The end point of a titration analysis is characterized by change in color.
Commonly used indicators include triphenyl methane group corants and azo group
corants.

25. Ans: E (QUES. Need to fix)


Tips: In order to chose the proper solvent to be used in an analysis we need to consider
some important points as the nature and solubility of the sample under investigation,
and make sure that no any side reaction between the sample in analysis and the solvent
will happen. The solvent should be pure, low toxic, inexpensive and have a high
dielectric constant.

26. Ans: D
Tips: Formic acid is a solvent used in non-aqueous titration in the assay of levodopa and
methyldopa

27. Ans: B
Tips: Dimethyl formamide is a solvent used in non-aqueous titration in the assay of
sulfonamides, allopurinol and acetazolamide.

28. Ans: C
Tips: mass spectroscopy methods

29. Ans: B

30. Ans: A

31. Ans: A
32. Ans: B

Copyright © 2000-2016 TIPS Inc. Unauthorized reproduction of this manual is prohibited. This 39-8
manual is being used during review sessions conducted by PharmacyPrep.
www.pharmacyprep.com

BIBLIOGRAPHIC REFERENCE

1- COMPREHENSIVE PHARMACY REVIEW – Lippincott William & Wilkins – Fourth edition

2- CPS-COMPENDIUM OF PHARMACEUTICALS AND SPECIALITIES - Canadian Pharmacist


Association – 2001 edition.

3- MEDICAL DICTIONARY – Dorland’s illustrated – 27th edition.

4- PHARMACY PREP – Lectures series & study guide for Evaluating Examination-TIPS -
2003/2004

5- THERAPUTIC CHOICES – Canadian Pharmacist Association -Third edition

6- USP DI – Drug Information for the Health Care Professional–15th edition – Volume I.

Copyright © 2000-2016 TIPS Inc. Unauthorized reproduction of this manual is prohibited. This 39-9
manual is being used during review sessions conducted by PharmacyPrep.
www.pharmacyprep.com

Copyright © 2000-2016 TIPS Inc. Unauthorized reproduction of this manual is prohibited. This 39-10
manual is being used during review sessions conducted by PharmacyPrep.
PharmacyPrep.Com Canadian Healthcare System

PHARMACY PREP

CANADIAN HEALTHCARE SYSTEM

1) What is not included in provincial drug benefit program?


A) Seniors over 65 years medical coverage
B) Hospitalization expenses
C) immigrants
D) Inmate
E) Children immunization

2. A pharmacist is?
A. primary health care member
B. secondary health care member
C. tertiary health care member
D. Community health Centre
E. Public health care system
Ans. A

Copyright © 2000-2016 TIPS Inc. Unauthorized reproduction of this manual is prohibited. This 40-1
manual is being used during review sessions conducted by PharmacyPrep.
www.PharmacyPrep.Com Pharmacy Practice Regulations

PHARMACY PREP

PHARMACY PRACTICE REGULATIONS

1. All of the following are regulated as schedule 2 drug in harmonized drug schedule,
except?
A. Lactulose B. Vitamin B 12 inj. C. Dukoral D. Mupirocin E. Ferrous gluconate
Ans.d

2. A customer of your pharmacy comes for early refill of lorazepam. What is


appropriate action?
A. Early refill of benzodiazepine are not allowed
B. Early refill of benzodiazepine are not allowed if prescriber specified repeat intervals.
C. Ask patient the reason for early refill
D. Do not dispense early refill because patient may abuse
E. Call prescriber to get early refill permission
Ans.c

3. Which of the following drug comply with the requirements of the Narcotic
Regulations?
A. Alprazolam
B. Tylenol # 1
C. Ketamine
D. Amphetamine
E. Methylphenidate
Ans. C

4. Which of the following drugs prescription refill/repeats are NOT allowed?


A. Benzodiazepine
B. Methylphenidate
C. Narcotic drugs
D. Hydrochlorothiazide/Ramipril
E. Lorazepam
Ans. C

5. What practitioners may legally cannot prescribe narcotics


A. Dentist
B. Veterinarian
C. Physician
D. Optometrist
E. Midwife
Ans. D

Copyright © 2000 - 2016 TIPS Inc. Unauthorized reproduction of this manual is prohibited. This 41-1
manual is being used during review sessions conducted by PharmacyPrep.
www.PharmacyPrep.Com Pharmacy Practice Regulations

6.A women, she recently moved into your pharmacy area from other neighborhood. She
comes with request to transfer her son prescription of methylphenidate from her
previous pharmacy. What is appropriate action?
A. Call other pharmacy get prescription transferred
B. Ask her to get a new prescription because it cannot be transferred
C. Give her part fill of methylphenidate
D. Tell her to call other pharmacy to request for Rx transfer
E. Talk to her and ask the reason for moving in this area
Ans. B

Copyright © 2000 - 2016 TIPS Inc. Unauthorized reproduction of this manual is prohibited. This 41-2
manual is being used during review sessions conducted by PharmacyPrep.
PharmacyPrep.Com Social, Behavioral Aspects of Pharmacy Profession

PHARMACY PREP

SOCIAL / BEHAVIORAL ASPECTS OF PHARMACY PROFESSION

1. A staff pharmacist reports about a pharmacy manager billing, excessive fee to long term
care. What is termed as?
A. conflict of interest
B. integrity
C. whistle blowing
D. revenge
Ans. c

2. A pharmacy owned by two partners. One of the partner billing excessively to long term
care. The partner was caught by college of wrong doing. The other partner claim he is not
aware of the fraud?
A. two partner are responsible for crime
B. the person who committed crime is responsible
C. two of them are not responsible
D. college of pharmacy is responsible

Copyright © 2000 - 2016 TIPS Inc. Unauthorized reproduction of this manual is prohibited. This manual is 42-1
being used during review sessions conducted by PharmacyPrep.
www.pharmacyprep.com

PHARMACY PREP

PHARMACY MANAGEMENT

1. Pharmacy ABC has sales of prescription medication for the year 2013 to 2014. In
order to find out profit of pharmacy ABC, which of the following financial statement
may be useful:
I-Balance sheet II-Retain earning statement III-Income statement
A. I only B. III only C. I and II only D. II and III only E. I, II, and III

2. All of the following include in income statement, EXCEPT:


A. Sales B. Profit C. Cost of goods sold D. Account payable E. Gross margin

3. Financial statement that is total income minus expenses is:


A. Income statement B. Profit statement C. Retained statement D. Net income
E. Income sheet

4) Cost of goods for a drug is $10 and mark up is 30%. What is retail price? Prescription
fee is $10. What is the total amount customer pay?
A) $13 B) $10 C) $23 D) $22 E) $33

5. A financial statement that is used to determine the total value of Rxs drug sales for a
pharmacy during the course of a year is known as:
A. Income statement B. Profit statement C. Retained statement D. Net
income E. Balance sheet

6. Delegation includes:
I) Delegating a project to technician
II) Supervising technician in project
III) Monitoring completion of project
A. I only B. III only C. I and II only D. II and III only E. I, II, III

7. Negligence may be described as:


A. Refusing to fill prescription for control drug substance
B. Refusing to fill narcotic drug prescription
C. Unable to meet the standard of the average pharmacist
D. Refuse to disclose patient information to unidentified person
E. Avoiding a patient who is difficult to handle and argumentative

8. The manager of the pharmacy made a list of work that should be done by his
employees in order of priority, this is called
A. Delegation B. Job analysis C. Job description D. Negligence E. None of the
above

Copyright © 2000-2016 TIPS Inc. Unauthorized reproduction of this manual is prohibited. This 43-1
manual is being used during review sessions conducted by PharmacyPrep.
www.pharmacyprep.com

9. How can you provide the best counseling to your patient


I- The pharmacy assistant gives an information sheet to the patient
II- The pharmacist explain how to use the drug
III- The pharmacist discuss with patient side effects, therapy and listening the
expectations of patient
A. I only B. III only C. I and II only D. II and III only E. I, II, and III

10. A senior pharmacy technician has a worked in a community pharmacy for 10 years.
He is well trained and welcomes new learning opportunities. His manager now wanting
to delegate a new technical project to him, should:
I) Discuss suggested steps to accomplish the project
II) Negotiate a time schedule for completion of the project
III) Provide support and follow up as required
A. I only B. III only C. I and II only D. II and III only E. I, II, and III

11. Which of the following is the most important component of business plan:
A. Location analysis B. Market area analysis C. Site analysis D. Product
analysis E-None of the above

12. What is POS inventory management:


A. Point of science B. Point of society C. Point of sale D. Point of submit
E. All of the above

13. Computer software and hardware program very useful in pharmacies that combines
and track all the business information as product information, cash register, inventory,
stocks and so on.
A. Pharmacare system B. Health Watch system C. Point of sale (POS)
D. Universal Pharmacy system E. Business update computer system

14. What is DIN:


A. Drug Identification Notice B. Notice of compliance C. Drug identification
number D. Drug information number E. Drug Identity Notice

15. Which of the following is not a type of ownership?


A. Incorporation
B. Sole proprietorship
C. Leasing
D. Franchise
E. Cooperative businesses

16. Which of the following is not a type of pharmacy?


A. Banner pharmacy
B. Franchise pharmacy

Copyright © 2000-2016 TIPS Inc. Unauthorized reproduction of this manual is prohibited. This 43-2
manual is being used during review sessions conducted by PharmacyPrep.
www.pharmacyprep.com

C. Central fill pharmacy


D. Mail order pharmacy
E. Rental pharmacy license

17. Total sales of prescription drugs are $210,000 and cost of good sold is 180,000.
What is gross profit?
A. 71% B. 14% C. 50% D. 20% E. 25%

18. Net income is?


A. Total income from sales
B. Gross income + total expenses
C. Gross income - total expenses
D. Gross income + total expenses + current assets
E. Gross income + total expenses + current assets + Fixed assets

19. Which of the following business is easy to begin?


A. Incorporation
B. sole proprietorship
C. leasing
D. Franchise
E. Partnership

20. Which of the following is the most important in pharmacy location analysis:
A. Trading area
B. Market area analysis
C. Site consideration
D. Region
E. Geographical area

21. Which of the following is the most important in pharmacy site consideration:
A. Sales per square feet
B. Convenience and distance
C. Parking space
D. Physical characteristics of space
E. Use of ratios

22. What is the test marketing in pharmacy:


A. through new papers
B. through community papers
C. through TV
D. Door to door flyers
E. Distribution of flyer in mall

Copyright © 2000-2016 TIPS Inc. Unauthorized reproduction of this manual is prohibited. This 43-3
manual is being used during review sessions conducted by PharmacyPrep.
www.pharmacyprep.com

23. Pharmacy ABC have sales of prescription medication for the year 2004 to 2005. In
order to find out profit of pharmacy ABC, which of the following financial statement
may be useful:
I-Balance sheet
II-Retain earning statement
III-income statement or profit & loss statement
A. I only B. III only C. I and II only D. II and III only E. I, II, and III

24. All of the following include as part of income statement, EXCEPT:


A. sales B. profit C. cost of goods sold D. account payable E. gross margin

25. Which of the following is not covered in total assets:


I-Pharmacy furniture
II-Account receivable
III-Account payable
A. I only B. III only C. I and II only D. II and III only E. I, II, and III

26. The style of management in which owner of pharmacy store emphasizes the
development of detailed policies and written procedure for employees to observe at all
times is referred as:
A. bureaucratic work procedure are described in details policies and written
procedure.
B. autocratic Implementing personal opinion without consultations.
C. assertive
D. consultative with consultation
E. benevolent generous and kind

27. The inventory turnover rate is a measure of a pharmacy’s


A) liquidity B. solvency C. Efficacy D. Profitability

28) The cost of goods sold is $150,000, at start of year $25,000 at the end of
year 50,000 the inventory capital. What is the turnover rate?
A) 4 B) 3 C) 2 D) 1 E) 5

29. The previous turnover rate was 6 and this year cost of good sold is $150,000, at start
of year $25,000 at the end of year 50,000 the inventory capital. What is the difference
of turn over rate this year from previous year?
A. 4 B) 3 C) 2 D. 1 E. 5

30. What is the normal turnover rate of retail pharmacy?


A. 3 B. 7 C. 5 D. 1

31. Turnover rate (TOR) is indicator of:


A. Profit B. Efficiency C. Loss D. Inventory status

Copyright © 2000-2016 TIPS Inc. Unauthorized reproduction of this manual is prohibited. This 43-4
manual is being used during review sessions conducted by PharmacyPrep.
www.pharmacyprep.com

E. Merchandising problems

32. In a market there is only one seller and there are many buyers. This competition
described as?
A. Monopoly B. Monopolistic competition C. Oligopoly
D. Oligopolistic competition

33. All of the following are essential of business, except?


A. Promotion B. Place C. Product D. Price E. Transportation

34. If you open a pharmacy in multicultural area, what you should do increase your
sales?
A. you have signs of multicultural
B. Hire technician from same cultures
C. Develop literature with different language
D. Offer help in different languages

35. Which of the following structure of pharmacies is difficult open and operate?
A. Sole ownership B. Franchise C. Banner pharmacy D. Partnership

36. If pharmacy manager for 16 year he is doing all the job by himself, without taking
any idea and skills from other, what is that called?
A. Beaurocratic B-Autocratic C-Independent D-Assertive E-Submissive

ABC pharmacy has following figures appearing on its balance sheet.


Cost of goods sold $500,000
Beginning inventory $200,000
Ending inventory $220,000
Total current assets $150,000
Total fixed assets $40,000
Total liabilities $75,000
37. Inventory turnover rate for ABC pharmacy is
A. 4.3 B. 2.3 C. 3.5 D. 6.0 E. 8.0

38. Net worth of ABC pharmacy is


A. $195,000 B. $225,000 C. $115,000 D. $75,000 E. $110,000

39. Liquidity generally expresses a pharmacy’s abilities to meet its


A. Assets
B. Current liabilities
C. Inventory
D. Prepaid expense
E. Desired inventory levels

Copyright © 2000-2016 TIPS Inc. Unauthorized reproduction of this manual is prohibited. This 43-5
manual is being used during review sessions conducted by PharmacyPrep.
www.pharmacyprep.com

40. All of the following are important fundamental principles of marketing EXCEPT:
A. Product B. Price C. Profession D. Promotion E. Place

41. If a pharmacy has an inventory turnover rate of 4 that means that the “days of
inventory on hand” is:
a) 73 b) 102 c) 24 d) 91 E. 265

42. Drug identification number is given to each drug that is sold in Canada. This is
assigned by the?
A. Pharmacy Manager
B. Manufacturer
C. Health Canada
D. Pharmacy owner
E. None of the above

43. Which is true about associate franchise, except?


A. Pharmacist own business but not physical assets.
B Central inventory management.
C. Guaranteed minimum annual salary and share profitability.
D. Initial capital investment or capital risk.
E. Franchise company supports all areas of operation

44. When pharmacy manager describes the position in pharmacy. The qualifications
required and the duties required in the position are called?
A. Job analysis
B. Job posting
C. Position allocation
D. Job description
E. Job finding

45. Inventory efficiency is measured by?


A. Net sales B. Liquidity C. Solvency D. Turnover rate ratios E. Gross margin

46. A person brings prescription for 6 months of medical supplies, before he travels.
However, his insurance covers only 3 months. What is appropriate?
A. Insurance may cover for 3 months, rest a customer should pay
B. Call insurance, to verify if customer is covered for vacation supply
C. Do not dispense for 6 months
D. None of the above

47. A retail store general manager, what should we do all except?


A. reading resume
B. interviewing
C. Selecting candidate

Copyright © 2000-2016 TIPS Inc. Unauthorized reproduction of this manual is prohibited. This 43-6
manual is being used during review sessions conducted by PharmacyPrep.
www.pharmacyprep.com

D. placing advertisement
E. responsible for narcotics

48. Canada Health Acts includes all, EXCEPT


A. Accessibility
B. Affordability
C. Universality of population covered
D. Public administration on non-profit basis
E. Comprehensiveness of insured service covered

49. What is merchandising?


A. Visual display of item in store
B. Point of sale system for tracking inventory
C. Point of sale system to tracking sales
D. Inventory management
E. Recall of products

50. It is one type of cost sharing plan in which patient has to pay a specified amount
during a specific period of time. Before benefits are paid by third party.
A. Co-payment B. Co-insurance C. Deductible D. Discount E. Bankruptcy

51. Banner franchise pharmacies include all, EXCEPT


A. Capital investment require because you own physical assets.
B. No fixed salary
C. Franchising company provide name and marketing
D. No capital investment required
E. No central purchase require

52. The characteristics of a chain pharmacies include all, EXCEPT


A. A chain is not necessarily a member of the Canadian Association of chain drug
stores.
B. An individual or corporation must own at least 3 stores to be considered a chain.
C. Head office directs all marketing and merchandising.
D. They employ pharmacy managers who are salaried employees of head office.
E. None of the above

53. A business plan requires all the following information, EXCEPT


A. Type of business
B. Location
C. Organizational structure
D. Competition strategy
E. 10 years plan

54. All the following is/are true with respect to the balance sheet, EXCEPT

Copyright © 2000-2016 TIPS Inc. Unauthorized reproduction of this manual is prohibited. This 43-7
manual is being used during review sessions conducted by PharmacyPrep.
www.pharmacyprep.com

A. It is prepared at the end of each fiscal year


B. It is governed by assets, liabilities, and net worth
C. Not all business must have a balance sheet
D. It shows the financial position at a given date
E. None of the above

Copyright © 2000-2016 TIPS Inc. Unauthorized reproduction of this manual is prohibited. This 43-8
manual is being used during review sessions conducted by PharmacyPrep.
www.pharmacyprep.com

PHARMACY PRACTICE MANAGEMENT


ANSWERS:

1. Ans: B

2. Ans: D

3. Ans: D
Tips: Net income is the financial statement that is total income minus expenses.

4. Ans: C
Sales price = cost + [cost x %markup]
= $10 + [$10 x 0.3] = $13
= $13 + $10 = $23

5. Ans: A
Tips: Income statement is the financial statement used to determine the total value of
RXs drug sales for a pharmacy during the course of a year.

6. Ans: E

7. Ans: C

8. Ans: C
Tips: Job description is a description of all the work that should be done by the
employers in order of priority. The manager normally does it.

9. Ans: D
Tips: The pharmacist should always interact with the patient providing clear verbal
information such as explaining how to take the medication, side effects and therapy,
and completing the concealing with extra writing or sheet information.

10. Ans: E
Tips: Delegation of a job should include the suggested steps to accomplish the project; a
negotiation of time schedule for completion of the project and the person doing the
delegation should provide support and follow up as required.

11. Ans: B

12. Ans: C

13. Ans: C

Copyright © 2000-2016 TIPS Inc. Unauthorized reproduction of this manual is prohibited. This 43-9
manual is being used during review sessions conducted by PharmacyPrep.
www.pharmacyprep.com

Tips: POS-Point of sale is a computer software and hardware program very useful in
pharmacies that combines and track all the business information such as product
information, cash register, inventory, stocks and so on.

14. Ans: C

15. Ans: C
Tips: Leasing (Renting) is not a form of enterprise.

16. Ans: E

17. Ans: B
Tips: [Sales - COGS/Sales] x 100
COGS = Cost of goods sold
Total sales $210,00 is 100%
Cost of goods sold is 180,000 is 85.5%
100%-85.5% = 14.5%

18. Ans: C

19. Ans: C
Tips: Leasing (Renting) is not a form of enterprise

20. Ans: B

21. Ans: E
Tips: use of ratios is frequently used rule of thumb in site considerations, this include
sales per square feet, number of prescription per patient, and average price for
prescription.

22. Ans: D

23. Ans: B

24. Ans: D

25. Ans: B

26. Ans: A

27. Ans: C

28. Ans: A
Tips: Turnover rate = COGS/average inventory

Copyright © 2000-2016 TIPS Inc. Unauthorized reproduction of this manual is prohibited. This 43-10
manual is being used during review sessions conducted by PharmacyPrep.
www.pharmacyprep.com

Average inventory = (beginning inventory + closing inventory)/2


$150000/[$50,000 + $25000]/2 = 4

29. Ans: C
Tips: Average inventory = (beginning inventory + closing inventory)/2
$150000/[$50,000 + $25000]/2 = 4
this year TOR is 4, so previous year was 6.

30. Ans: C

31. Ans: B

32. Ans: A

33. Ans: E
Tips: there are 4 Ps; Place, Product, Price and promotion

34. Ans: C

35. Ans: A

36. Ans: C

37. Ans: B
Average inventory = $200,000+$220,000/2 = $210,000
Turnover rate = 500,000/210,000 =2.3

38. Ans: C
Tips: Net worth = Total assets – total liabilities
($150,000+$40,000) - $75,000 = $115,000

39. Ans: B

40. Ans: C
Tips: There are 4 “P” of marketing = Product, Price, Promotion and place.

41. Ans: D

42. Ans: C

43. Ans: D

44. Ans: D

Copyright © 2000-2016 TIPS Inc. Unauthorized reproduction of this manual is prohibited. This 43-11
manual is being used during review sessions conducted by PharmacyPrep.
www.pharmacyprep.com

45. Ans: D

46. Ans: B

47. Ans: E

48. Ans: B

49. Ans: A
Tips: Merchandising is visual display of products.

50. Ans: C

Deductible Co-payment Co-insurance


Pay onetime in the Pay each time for every Each time you share the
beginning of every year. prescription expenses

51. Ans: D
Associate franchise Banner Franchise
No capital investment Capital investment
Central distribution No central distribution
Example: Shopper drug mart Example: IDA

52. Ans: B
Tips: An individual or corporation must own 5 or more stores to be considered a chain.

53. Ans: E
Tips: 3-5 year plan is required.

54. Ans: C
Tips: At year end, every business must have a balance sheet prepared.

Copyright © 2000-2016 TIPS Inc. Unauthorized reproduction of this manual is prohibited. This 43-12
manual is being used during review sessions conducted by PharmacyPrep.
Pharmacyprep.com

PHARMACY PREP
PHARMACOECONOMICS

1. In the pharmacoeconomics considerations, the following aspects is the most


important:
A) Pharmacist
B) Hospital
C) Society
D) Doctor
E) Patient

2. Pharmacoeconomics means:
A. Analyses of cost and benefit of the drug therapy to the healthcare system and society
B. Cost of pharmaceutical dispensing fee
C. Cost of products sold in a pharmacy
D. Analyses of cost and benefit for the pharmacy business
E. Prices determined by the provincial authorities to be followed by the pharmacy

3. What is QALY?
A. Quantity Adjusted Life Year
B. Quality Adjusted Life Year
C. Quantity of Available for Long Year
D.Health related quantity of life
E. Quality of as per life year

4. What is HRQOL?
A. Health Related Quality Of Life
B. Health Related Quantity Of Life
C. Health Related Quantity Of Year
D. None of the above

5. Cost utility analysis (CUA) outcomes are measured in?


A. Quality unit
B. Cost unit
C. Natural unit
D. Quantity
E. Dollars

6. Pharmacoeconomic studies are used to analyze cost of drugs to:


A. Healthcare system
B. Society
C. Pharmacist
D. Pharmaceutical manufacturer

Copyright © 2000-2016 TIPS Inc. Unauthorized reproduction of this manual is prohibited. This 44-1
manual is being used during review sessions conducted by PharmacyPrep.
Pharmacyprep.com

7. Pharmacy manager wants to increase salary of staff pharmacy, do you consider all
except?
A. other pharmacist available to do his job
B. budget
C. pharmacist asking a raise
D. consider his experience
E. Life allowance

8. Cost Utility Analysis measures in terms of:


A. HRQL
B. QALY
C. Dollar
D. Financial consequences

9. Health related quality of life (HRQL) is an example of


A. Clinical outcome
B. Structural outcome
C. Economic outcome
D. Humanistic outcome

10. Cost utility analysis (CUA) outcome is measured in?


A. Quality unit
B. Cost unit
C. Natural unit
D. Quantity
E. Dollars

11. Pharmacoeconomic studies are used to analyze cost of drugs to:


A. Healthcare system
B. Society
C. Pharmacist
D. Pharmaceutical manufacturer
E. Physician

12. Pharmacy manager wants to increase salary of staff pharmacy, do you consider all
except?
A. other pharmacist available to do his job
B. budget
C. pharmacist asking a raise
D. consider his experience
E. Life allowance

Copyright © 2000-2016 TIPS Inc. Unauthorized reproduction of this manual is prohibited. This 44-2
manual is being used during review sessions conducted by PharmacyPrep.
Pharmacyprep.com

13. What is SF-36


A. Short-Form 36
B. Slow release 36
C. General health status
D. Short functions 36
E. Patient non clinical information

14. If two drugs have same effectiveness than selection is based on?
A. Cost effective analysis (CEA)
B. Cost utility analysis (CUA)
C. Cost minimization analysis (CMA)
D. Cost Benefit analysis (CBA)
E. Professional judgment of prescriber

15. Which attempts to define exactly how much good is achieved for each dollar spent?
A. Cost effective analysis (CEA)
B. Cost utility analysis (CUA)
C. Cost minimization analysis (CMA)
D. Cost Benefit analysis (CBA)
E. All of the above

16. Clinical parameters and cost effectiveness of two drugs can be compared by?
A. Cost effective analysis (CEA)
B. Cost utility analysis (CUA)
C. Cost minimization analysis (CMA)
D. Cost Benefit analysis (CBA)
E. All of the above

17) Review example of cost benefit analysis (CBA)


Drug A Drug B
Cost
drug $1000 $1100
Administration $100 $10
Monitoring $50 $0
Side effects $75 $0
Total cost $1225 $1110

Benefit 750 1200


Days at work ($)
Extra months of life ($)
C/B = <1 is beneficial
C/B = 1 no benefit or no loss
C/B = >1 is loss

Copyright © 2000-2016 TIPS Inc. Unauthorized reproduction of this manual is prohibited. This 44-3
manual is being used during review sessions conducted by PharmacyPrep.
Pharmacyprep.com

18. Review example of Cost effective analysis (CEA)

Drug A Drug B
Cost
dose $300 $300
Hospitalization $300 $300
Stay require
Total cost $600 $600

Outcome
Extra years of life 2 6
Cost effectiveness ratio: 600/2 600/6

$300 $100

19. Review example cost minimization analysis (CMA)


A B
Cost
Drug $250 $430
Administration $75 $0
Monitoring $100 $10
Side effects $100 $15

Subtotal cost: $525 $455

Antibiotic 80% 80%


effectiveness
Result = Cost Drug A>Cost of Drug B
Note: usually used for calculating different methods surgical techniques.

20. Review example cost utility analysis


A B
Cost
Drug $250 $430
Administration $75 $0
Monitoring $100 $10
Side effects $100 $15
Total cost $525 $455
Utilities
Extra years of life 2 3
Quality of life 0.33 0.25

Copyright © 2000-2016 TIPS Inc. Unauthorized reproduction of this manual is prohibited. This 44-4
manual is being used during review sessions conducted by PharmacyPrep.
Pharmacyprep.com

QALY 0.5 0.40


Cost utility ratio $525/0.5 $455/0.4
$1050 $1137
QALY = Quality Adjusted Life Years

Copyright © 2000-2016 TIPS Inc. Unauthorized reproduction of this manual is prohibited. This 44-5
manual is being used during review sessions conducted by PharmacyPrep.
Pharmacyprep.com

PHARMACOECONOMICS
ANSWERS:

1. Ans: C

2. Ans: A
Tips: Pharmacoeconomics is the description and analysis of the costs of drug therapy to
health care system and society.
3. Ans: B

4. Ans: A

5. Ans: A

6. Ans: B

7. Ans: E

8. Ans: B

9. Ans: D

10. Ans: A

11. Ans: B

12. Ans: E

13. Ans: A
Tips: Short form 36 (SF-36) most frequently used in pharmacoeconomic general health
status instrument measuring 8 dimensions: physical functions, social functions, role
emotional, role physical, bodily pain, mental health, general health, and vitality.
Cost

14. Ans: C

15. Ans: A

16. A

Copyright © 2000-2016 TIPS Inc. Unauthorized reproduction of this manual is prohibited. This 44-6
manual is being used during review sessions conducted by PharmacyPrep.
PHARMACY PREP
DRUG DISCOVERY AND DEVELOPMENT

1. What is correct about phase I clinical trials?


A. Healthy human volunteers are tested
B. Disease human volunteers are tested
C. Larger population than Phase II is tested
D. Drugs are approved for sale after phase III clinical trials.

2. All of the following are true about Phase III human clinical trial, EXCEPT?
A. Healthy human volunteers are tested
B. Disease human volunteers are tested
C. Larger population than Phase II is tested
D. Drugs are approved for sale after phase III clinical trials.

3. Medications that are prescribed and sold over the counter in Canada are approved by?
A. Canadian Health Act
B. NAPRA
C. Health Canada
D. Federal Government of Canada
E. Provincial governments

4. Healthy volunteers are tested in?


A. Pre clinical studies
B. Phase I clinical studies
C. Phase II clinical studies
D. Phase III clinical studies
E. Phase IV clinical studies

5. Which of the following is decisive phase in new drug approval


A. Pre clinical studies
B. Phase I clinical studies
C. Phase II clinical studies
D. Phase III clinical studies
E. Phase IV clinical studies

6. Who approves and authorizes the sale of medications in Canada?


A. Therapeutic Product Directorate of Health Canada
B. Food & Drug and Administration
C. Pharmacy manager
D. Pharmaceutical Manufacturer
E. National association of pharmacy regulatory authority (NAPRA)

Copyright © 2000-2016 TIPS Inc. Unauthorized reproduction of this manual is prohibited. This manual is 45-1
being used during review sessions conducted by PharmacyPrep.
7. Who sets prescription drug prices in Canada?
A. Health Canada
B. Patented medication review board (PMPRB)
C. Pharmacy manager
D. Provincial college of pharmacy
E. Pharmaceutical Manufacturer

8. Who sets OTC drug prices?


A. Health Canada
B. Patented medicine review board (PMPRB)
C. Pharmacy manager/owner
D. Provincial college of pharmacy
E. Pharmaceutical Manufacturer

Copyright © 2000-2016 TIPS Inc. Unauthorized reproduction of this manual is prohibited. This manual is 45-2
being used during review sessions conducted by PharmacyPrep.
DRUG DISCOVERY AND DEVELOPMENT
ANSWERS:

1.Ans: A

2. Ans: A

3. Ans: C

4. Ans: B

5. Ans: D

6. Ans: A

7. Ans: B

8. Ans: C

Copyright © 2000-2016 TIPS Inc. Unauthorized reproduction of this manual is prohibited. This manual is 45-3
being used during review sessions conducted by PharmacyPrep.
Copyright © 2000-2016 TIPS Inc. Unauthorized reproduction of this manual is prohibited. This manual is 45-4
being used during review sessions conducted by PharmacyPrep.
Pharmacyprep.com

PHARMACY PREP
EPIDEMIOLOGY AND CLINICAL STUDIES

1. Which of the following is an example of non interventional trials or observational


trials and follow up like a comparison of two patient with COPD, one smokes and the
other non smoker?
A. Cohort study B. Clinical trials C. interventional trials D. all of the above

2. What is included in "intent-to-treat analysis"?


A. Data from patients originally assigned
B. Data from patient participated from entire treatment
C. Data from patient discontinued treatment
D. Data from dropped out patient for non therapeutic reason
E. All of the above
Ans. E

3. How to test effectiveness for treating pneumonia?


A. randomized placebo trials
B. randomized comparative trial
C. cohort.
D. Case control studies
Ans. B

Copyright © 2000-2016 TIPS Inc. Unauthorized reproduction of this manual is prohibited. This
manual is being used during review sessions conducted by PharmacyPrep.
46-1
www.pharmacyprep.com

PHARMACY PREP.

BIOSTATISTICS CALCULATIONS

1.What is incorrect about the relation of confidence interval and sample size
A. There is a relationship between sample size and the width of the confidence interval.
B. The general rule is that the width of the confidence interval will be proportional
to 1/ n, so to halve the interval you need 4 times the sample size.
C. Larger samples give more precise estimates with lower standard error and tighter
confidence intervals.
D. As the sample size increases, the interval and its width decrease, thus providing a
more precise estimate of the population value.
E. The larger the sample size the larger the confidence interval
Ans. E
Tips. The larger the sample size the

2. A study experiment event rate or treatment reduced 10% risk and control reduced
7.5% risk. What is absolute risk reduction?
A. 7.5% B. 2.5% C. 10% D. 15%
Ans. B
ARR = EER-CER or 10-7.5 = 2.5%
NNT = 1/ARR = (1/2.5) x 100 = 40
NNT are always rounded up to the nearest whole number.

3. If a drug reduces the risk of a bad outcome from 50 per cent to 30 per cent. The ARR
is?
A. 50%
B. 30%
C. 20%
D.10%
Ans.

What is NNT?
1/0.2 = 5

4) The results of the trial of vasodilators in patient with congestive heart failure. The
mortality in the ACEi arm was 33% and in hydralazine plus nitrate group it was 39%.
What is absolute risk difference?
A) 0.86 B) 6% C) 14% D) 29% E) 3.3%
Ans. B

Copyright © 2000-2016 TIPS Inc. Unauthorized reproduction of this manual is prohibited. This 47-1
manual is being used during review sessions conducted by PharmacyPrep.
www.pharmacyprep.com

5.In the above case how many patient do you need to treat to benefit one case (NNT)?
A. 10
B. 5
C. 6
D.17
Ans. D

6.A new drug absolute risk reduction is 4% for preventing stroke. Means?
A. Four patient in 100 actually benefit from new drug
B. Four patient of all study group benefit from new drug
c. New drugs four times better than old drug
D. New drugs is 4% better than old drug
ans. a

7.The NNT for the above case is 25, means?


A. 25 patient had to receive the new drug in order to one patient to benefit
B. New drug benefit 25 people patient in 100 patients
C. 25 patient had to receive the new drug in order to benefit 100 patients.
D. 100 patient had to receive the new drug in order to benefit 25 patients
Ans. A

8) What is relative risk reduction?


A) 0.86 B) 5% C) 14% D) 29% E) 3.3%
Ans. C
Tips:( 39-33/39 )x 100 = 14%

Copyright © 2000-2016 TIPS Inc. Unauthorized reproduction of this manual is prohibited. This 47-2
manual is being used during review sessions conducted by PharmacyPrep.
Pharmacy Prep

HOSPITAL PHARMACY

1. Director of pharmacy reports to?


A. CEO
B. president of hospital
C. board of trustees
D. pharmacy manager
E. vice presidents
Ans. e

2. What is not included in unit dose system?


A. package containing the amount of drug for one dose
B. Oral medications contain 24 hr dosages
C. contain individual drawer or tray for each patient
D. nurses prepare unit dosages
Ans. D

3. Which of the following is NOT responsibility of P&T committee


A. Formulating policies regarding selection and therapeutic use of drugs
B. Educates professional staff such as physician, pharmacist and nurse about matters related to
patient quality and safety of drugs.
C. Develops list of formulary
D. Develops generic and therapeutic equivalent list of drugs
E. Recommends therapies to patients
Ans. E

4. Listing of drug by their generic name followed by strength, form, toxicology, use &
recommended quantity dispensed
B. Prepared locally by its own clinical staff or P&T committee
C. Each hospital has their own formulary
D. Formulary is federal and all hospital in Canada has the same formulary
Ans. d

5. Which of the following is least likely done by the regulated pharmacy technician?
A. sterile preparation of IV
B. Checking formulary stability reconstituted IV preps
C. Preparing TPN
D. Final check before delivery to wards
E. Counseling to patients
Ans. E
Copyright © 2000-2016 TIPS Inc. Unauthorized reproduction of this manual is prohibited. This manual is being
used during review sessions conducted by PharmacyPrep.
48-1
6. A hospital doctor ordered a drug that is not present in hospital formulary list. What is
appropriate action?
A. Dispense equivalent drug present in formulary
B. Check in non formulary list or order from community pharmacy
C. Call doctor to change the medication from list of formulary
D. Contact other hospital pharmacy to get the medication.
Ans. b

7. Hospital doctor prescribed amoxicillin 500 mg for 8 hourly administration. What is correct?
A. unit dose is dispensed with one dose of amoxicillin 500 mg one capsule
B. unit dose is dispensed with three capsule of amoxicillin 500 mg for an 8 hourly administration
C. unit dose is dispensed with nine capsule of amoxicillin 500 mg for 8 hourly for 3 days
administration
D. Call the doctor to clarify
Ans. b

8. Dispensing errors in hospitals can be minimized by


A. Automation
B. Computer generated prescription and eMAR
C. Continue education of hospital staff.
D. All of the above
Ans. D

Copyright © 2000-2016 TIPS Inc. Unauthorized reproduction of this manual is prohibited. This manual is being
used during review sessions conducted by PharmacyPrep.
48-2
www.Pharmacyprep.com Pharmacy Calculations

PHARMACY PREP.

Basic CALCULATIONS
1.A child weighs 22 lbs, and doctor wants to give a drug 1 mg/kg/day bid, how much you
should give per day?
a. 10 mg/day B. 5 mg/day C. 15 mg/day D. 20 mg/day
Ans. a

2.A child weighs 22 lbs, and doctor wants to give 1 mg/kg/day bid drug, how much you
should give per dose?
a. 10 mg/dose B. 5 mg/dose C. 15 mg/dose D. 20 mg/dose
Ans. b

3.A patient weight 180 lbs has admitted to emergence for congestive heart failure and
severe edema. Patient was give furosemide iv infusion for the past 24 hours. After
discharge the patient weight was 173 lbs. How many kg patient weight is lost?
A. 7 kg B.3.2 kg C. 2.2kg D. 172 kg

180 lbs-173 lbs = 7 lbs


7/2.2 = 3.2 kg

4.A prescription order of cisplatin 750 mg/m2/day, IV tid. The patient weighs 75 kg and
is 162 cm tall. Calculate the dosage to administer?
BSA in m2 = c
=

= 3.37
= 1.84 m2

= 750 mg/m2 m2 = 1380 mg per day


1380/3 = 460 mg TID
How to use Calculator
SHARP EL-510R
2ndF
press square root.
3.37

Copyright © 2000-2016 TIPS Inc. Unauthorized reproduction of this manual is prohibited. This 49-1
manual is being used during review sessions conducted by PharmacyPrep.
www.Pharmacyprep.com Pharmacy Calculations

5.Patient age 60 yo, weight 78 kg and height 184 cm. Doctor prescribed 5-flurouracil
300 mg/m2 se ac a e e se ra a s n body surface area?
BSA = c
1.99 mg/m2
er a

6.A chemotherapy protocol for a pt. body weight 68 kg and height 1.7 m. Chemo drug
leucovorin 400 mg/m2 is recommended what is daily dose?
c
= 1.79 m2

= 716 mg per day


For leucovorin, in day 2 cycle each dose of medication is reduced by 50%?
716/2
= 358 mg

7.You are to administer 5 ml of ferrous gluconate that must be diluted in water to


protect the client’s teeth and gastrointestinal track. The directions in your drug
reference state that each ml of this medication must be diluted in 20 ml of water. Once
diluted, the total volume to be administered is
A. 100 ml
B.105 ml
C. 125 ml
D. 120 ml
Ans. B
Note.
Diluted in, added to, added in may increase total volume or weight. Dilute up to,
quantity sufficient or qs, mark up to the volume should not increase volume.

8. Syrup is an 85% w/v solution of sucrose in water. It has a density of 1.313 g/ml. How
an ra s a er s e se a e s r
A. 85 g B. 106 g C. 125 g D. 164 g E. 58 g

Copyright © 2000-2016 TIPS Inc. Unauthorized reproduction of this manual is prohibited. This 49-2
manual is being used during review sessions conducted by PharmacyPrep.
www.Pharmacyprep.com Pharmacy Calculations

100 ........... 85
125.............?
= 106 g
164-106 = 58 g

9.Ampicillin 1 g drug. Add 6.8 ml sterile water to provide final concentration volume 8
mL. What is concentration of the reconstituted solution in the vial in mg/mL?
A. 8 mg/mL
B. 125 mg/mL
C. 100 mg/mL
D. 150 mg/mL

Unknown Known

x mg/mL 1000 mg/8mL

x mg = (1mL x 1000 mg)/8 mL

125 mg/mL

10.Given ampicillin 1 g vial. If the final concentration has been calculated to 125 mg/1
mL. What is the volume taken if dose of the ampicillin is 500 mg?
A. 8 mL
B. 4 mL
C. 2 mL
D.1 mL
Ans. B
125 mg...........1 mL
500 mg.............?

x/500 mg 1 ml/125 mg

= 4mL

11.A sterile methylprednisone powder vial contain 125 mg drug. Reconstituted by adding
1.8 mL of sterile water to final volume 2 mL. What is final concentration of reconstituted
solution in the per mg/mL?
A. 125 mg/mL
B. 62.5 mg/mL
C. 31.5 mg/mL
D.16.5 mg/mL
ans. B

Copyright © 2000-2016 TIPS Inc. Unauthorized reproduction of this manual is prohibited. This 49-3
manual is being used during review sessions conducted by PharmacyPrep.
www.Pharmacyprep.com Pharmacy Calculations

2 mL ..................125 mg (known)
1 mL .................? (unknown)

uknown known
x mg/ 1mL 125mg/2mL

x mg = (1 mL x 125 mg) / 2 mL

= 62.5 mg/mL

12.An order for product preparation to maintain alkaline urine.


na c ncen ra n c ra e
citric acid 6% w/w
Sterile water for irrigation added up final volume 1000 mL
How many grams of sodium citrate powder needed?
A. 100 g
B. 10 g
C. 60 g
D. 50 g

Xg/1000 mL 1g/100 mL

Xg = (1000 mL x 1g)/100 mL
= 10 g

13.Product order to prepare 100 ml of hydrocortisone oral suspension with final


concentration 1 mg/mL. Available hydrocortisone 20 mg tablets. How many tablets
needed to prepare 100 mL?
A. 20 tablets
B. 10 tablets
C. 5 tablets
D. 1 tablet
Ans. C

Copyright © 2000-2016 TIPS Inc. Unauthorized reproduction of this manual is prohibited. This 49-4
manual is being used during review sessions conducted by PharmacyPrep.
www.Pharmacyprep.com Pharmacy Calculations

Copyright © 2000-2016 TIPS Inc. Unauthorized reproduction of this manual is prohibited. This 49-5
manual is being used during review sessions conducted by PharmacyPrep.
www.Pharmacyprep.com Dose Calculations

Dose Calculations

1) How many tablets are needed for following prescription?


Provera 5 mg tab QD
16 –25 days for 3 months
One tablet containing 5 mg
a. 50 tablets
b. 60 tablets
c. 80 tablets
d. 75 tablets
e. 30 tablets
Ans. e

2) A physician wants to switch a terminally-ill patient from slow-release morphine


sulphate tablets, 15 mg twice daily, to a liquid morphine sulfate dosage form because
the patient has difficulty in swallowing tablets. If a morphine sulfate solution containing
5 mg per mL is prescribed q4h, what volume should be dispensed for a 20-day supply to
provide the same pain relief as the tablet regimen?
a) 20 mL
b) 60 mL
c) 80 mL
d) 100 mL
e) 120 mL
Ans. e
Working:
5 mg/1ml x 6 times daily = 6 ml
6 ml x 20 = 120 ml

or

i. 15 mg BID = 30 mg daily
ii. For 20 days: 30 mg x 20 = 600 mg are dispensed
iii. Solution contains 5 mg/ml. 600 mg are contained in 600/5 = 120 mL

Ans: E

Copyright © 2000-2016 TIPS Inc. Unauthorized reproduction of this manual is prohibited. This 50-1
manual is being used during review sessions conducted by PharmacyPrep.
www.Pharmacyprep.com Dose Calculations

3. A patient is currently taking 220 mg anhydrous zinc sulfate. To receive the equivalent
amount of elemental zinc, how many milligrams of zinc sulfate heptahydrate (.7H 2 O)
would be patient need to take? (MW: zinc 65, ZnSO 4 161, H 2 O 18).
a) 123 mg
b) 220 mg
c) 300 mg
d) 392 mg
e) 545 mg

ZnSO4 = 161
ZnSO4 heptahydrate= (7x18 =126)
161+126 = 287

161..............220 mg
287................?
(220 x 287)/161
= 392

OR

Working:
i. 220 mg ZnSO 4 provides 220/161 = 1.366 mmol Zn (and 1.33 mmol
SO 4 , but this is not relevant)
ii. Each mmol of ZnSO 4 . 7H 2 O also provides 1 mmol Zinc
iii. Calculate the molecular weight of ZnSO 4 . 7H 2 O as follows:

ZnSO 4 = 161
7H2O = (7 X 18) = 126
ZnSO 4 7H2O = 161 + 126 = 287

iv. Therefore, 1.366 mmol of ZnSO 4 . 7H2O (which provides 1.366 mmol
Zinc) are provided by 1.366 x 287 = 392 mg

Ans: D

4) A physician has prescribed 60 g of 0.01% fluocinolone acetonide cream. You have


available the commercially prepared cream containing 0.2% fluocinolone acetonine
(Synalar cream) and cream base (Dermabase ) for dilution. To prepare the Rx you
would require:
a) 1.2 g Synalar and 58.8 g Dermabase
b) 3 g Synalar and 60 g Dermabase
c) 0.3 g Synalar and 59.7 g Dermabase
d) 3 g Synalar and 57 g Dermabase
C1. V1 = C2. V2
0.01% x 60 g = 0.2% . V 2

Copyright © 2000-2016 TIPS Inc. Unauthorized reproduction of this manual is prohibited. This 50-2
manual is being used during review sessions conducted by PharmacyPrep.
www.Pharmacyprep.com Dose Calculations

X = (60 g x 0.01)/ 0.2 = 3 g Synalar


Order is for 60 g - 3 g = 57 g dermabase

Working:
i. 60 g of 0.01% fluocinolone acetonide (required in the Rx) contains
0.01/100 x 60 = 0.006 g fluocinolone acetonide)
ii. You have 0.2% fluocinolone acetonide (0.2 g per 100 g = 0.002 g
fluocinolone per gramme of cream)
iii. Therefore, for the Rx you need 0.006 / 0.002 = 3 g Synalar cream

Ans: D
B is wrong, because 3 g Synalar and 60 g of Dermabase will give 63 g of total cream,
with a concentration of 0.0095% (0.006 / 63 x 100)

5. In the International System of Units (SI), most drugs levels are to be reported in the
units of micromoles/litre (µmols/litre). The therapeutic concentration range for
theophylline (MW 180) is 10-20 µg/mL. the SI equivalent would be:
a) 0.055- 0.11 µmol/L
b) 27-55 µmol/L
c) 55-110 µmol/L
d) 110-220 µmol/L
e) 180-360 µmol/L
Working:
i. Careful!!! Theophylline therapeutic concentration is given in
µmol/mL, while answers are given in µmol/L.
ii. Convert therapeutic concentration to µg/L: 10-20 µmol/mL =
10,000 – 20,000 µg/L
iii. Lower parameter: 10 µg/mol = 10,000 µg/L = 10,000/180 = 55
µmol/L
iv. Upper parameter: 20 µg/mL = 20,000 µg/L = 20,000/180 =
111µmol/L
Ans-C

6. Prednisolone each tablet containing 10mg. Taper 1/2 tablet every week .How tablets
are needed?

1st week = 7 tab


2nd week = 3.5 tab
10.5 tablet

7. A family of 1 adults (300 mg/wk) and 8-year-old child (75mg/wk), 14 year old
(225mg/wk) teenager, going for 2 weeks trip to malaria infections area. You have 300

Copyright © 2000-2016 TIPS Inc. Unauthorized reproduction of this manual is prohibited. This 50-3
manual is being used during review sessions conducted by PharmacyPrep.
www.Pharmacyprep.com Dose Calculations

mg base drug. How many tablets will you give for whole family? (Prophylaxis require 4
weeks before trip, during trip and 2weeks after trip)
A-17 tablets
B-18 tablets
C-16 tablets
D-19 tablets
Ans. C
Total = 8 week therapy
Adult = 8 tablet
14 yo = 6 tablet (225 mg/wk)
8 yo = 2 tablet (75 mg/wk)
Total tablet = 16 tablets

8) A parenteral solutions used in hospital pharmacy. If 250 g of parenteral solution


dissolved in 1000 mL of glycerin (density of glycerin is 1.25 g/mL), the concentration of
parenteral solution is:
A) 26% w/w
B) 24% w/w
C) 16 % w/w
D) 20%w/w

TIPS:
D = M/V
Concentration = Solute x 100
Solute + Solvent

= 250 g x 100
250g+1250 g

= 16% w/w

M=DxV
M = 1.25 x 1000 mL = 1250 g
Then
250 g+1250 g = 1500 g (total volume)
(250g/250 g+1250 g) x100 = 16%

List of drugs require tapering.


Prednisone/prednisolone
Benzodiazepine
SSRI
Beta blockers

Copyright © 2000-2016 TIPS Inc. Unauthorized reproduction of this manual is prohibited. This 50-4
manual is being used during review sessions conducted by PharmacyPrep.
www.Pharmacyprep.com Dose Calculations

17. Prednisolone each tablet containing 5mg. Start 35 mg and then taper by 5mg every 1
day. How many tablets are needed?
A-20 tab
B-56 tab
C-14 tab
D-28 tab
Tips: 7+6+5+4+3+2+1 = 28 tab
For above questions, how many days patient takes prednisone? 7days

17. Prednisolone each tablet containing 5mg. Start 35 mg and then taper by 5mg every 2
day. How many tablets are needed?
A-20 tab
B-56 tab
C-14 tab
D-28 tab
7+7+6+6+5+5+4+4+3+3+2+2+1+1 = 56 tab = 14 days
For the above questions. Patient take for how many days? 14 days

18) If three tablets contain 975 mg of ASA, how many grams of ASA would be contained
in 12 tablets?
A) 4g
B)325g
C) 3.9g
D) 39g
E) 390g

19) How many milliliters of a heparin sodium injection containing 200,000 heparin units
in 10 mL should be used to obtain 5,000 heparin units?
A-0.50mL
B-0.25mL
C-0.10mL
D-0.75mL
E-250mL

200,000 ---- 10 ml
5000 ---- ?

5000 x 10 / 200,000 = 0.25 ml

20) A physician asks the pharmacist to compound codeine sulfate tablet triturates
containing 10 mg of codeine. The patient need a 5-day supplies and may take 1-2 q4-6h
PRN How much codeine is needed to fill this prescription?

Copyright © 2000-2016 TIPS Inc. Unauthorized reproduction of this manual is prohibited. This 50-5
manual is being used during review sessions conducted by PharmacyPrep.
www.Pharmacyprep.com Dose Calculations

A. 300 mg
B.400 mg
C-600 mg
D-1200 mg
Ans: c

PRN = as needed or on demand


12 tab per day
for 5 days = 60 tab
each tab contain 10 mg
so
10 mg x 60 tab = 600 mg

23) Prednisolone each tablet containing 5 mg. Start 10 mg and then taper by 1/2 tab
every week. How many tablets are needed?
A-21 tab
B-56 tab
C-14 tab
D-28 tab
E-35 tab
Ans.
Tips: 14 tab+10.5tab+7tab+3.5 tab = 35 tab

2 tab daily x 7 d = 14 tab


1.5 tab daily x 7 d = 10.5 tab
1 tab daily x 7 d = 7 tab
1/2 tab daily x 7 d = 3.5 tab

5 tab x 7 days

How many weeks therapy?

26) Dexamethasone 8 mg bid x 7 days then taper by 1 mg twice day every 2nd day until
finish? Available strength 4 mg and 0.5 mg. Calculate how many tablets of both
strengths you will dispense?
A) 112 tablets of 4 mg or 800 tablets of 0.5 mg
B) 126 tablets of 4 mg or 1008 tablets of 0.5 mg
C) 56 tablets of 4 mg and 96 of 0.5 mg
D) 136 tablets of 4 mg or 990 tablets of 0.5 mg
E) Give both 44 of 4 mg and 96 of 0.5 mg tablets

Copyright © 2000-2016 TIPS Inc. Unauthorized reproduction of this manual is prohibited. This 50-6
manual is being used during review sessions conducted by PharmacyPrep.
www.Pharmacyprep.com Dose Calculations

days 4 mg TAB 0.5 mg TAB


8 mg Bid 7d 28 0
7 mg bid 2d 4 24
6 mg bid 2d 4 16
5 mg bid 2d 4 8
4 mg bid 2d 4 0
3 mg bid 2d 0 24
2 mg bid 2d 0 16
1 mg bid 2d 0 8
Total 44 96

1st week 112 mg


2 days = 28 mg
2 days = 24 mg
2 day = 20 mg
2 day = 16 mg
2 days = 12 mg
2 day = 8 mg
2 day = 4 mg
Total = 224 mg for 21 days

7 d x 16 mg = 112 mg = 28 tab of 4 mg
2 d x 7 mg bid =14 mg for 2 d 28 mg = 4 of 4 mg & 24 of 0.5 mg
2 d x 6 mg bid 24 mg = 4 of 4 mg and 16 of 0.5 mg
2 d x 5mg bid= 20 mg = 4 of 4 mg and 8 of 0.5 mg
2 d x 4mg bid =16 mg = 4 of 4 mg
2 d x 3 mg bid = 12 mg = 24 of 0.5 mg
2 d x 2 mg bid = 8 mg = 16 of 0.5 mg
2 d x 1 mg bid = 4 mg = 8 of 0.5 mg
Ans.
Tips. If you give both 44 of 4 mg and 96 of 0.5 mg
Total 224 mg
Total therapy 21 days

Copyright © 2000-2016 TIPS Inc. Unauthorized reproduction of this manual is prohibited. This 50-7
manual is being used during review sessions conducted by PharmacyPrep.
Pharmacyprep.com Dilutions and Concentrations

Pharmacy prep

DILUTIONS AND CONCENTRATIONS


Q& A

1) A prescription for hydrocrotisone cream 0.1%. Pharmacy has 0.25% available in 30 g tube.
How many grams diluents base (vanishing cream) should be added?

A) 30 g B) 45 g C) 50 g D) 75 g E) 25 g

V2 = (C1 x V1)/C2

V2 = (0.25% x 30 g)/0.1%

V2 = 75 g

but the added tube contains 30 g

so the used base = 75 g - 30 g = 45 g

2) Dexamethasone is available as 4 mg/mL preparation. in infant is to receive 0.35 mg. Prepare


a dilution so that the final concentration is 1 mg/mL. How much diluents will you need if the
original product is in a 1mL vial and you use the full vial?

A) 4 ml B) 3ml C) 1ml D) 0.35ml

Ans: B

Step1: determine the volume of final product. Since dexamethasone is 4mg/mL, a 1 ml vial have
4mg of drug

X ml/ 4 mg = 1 ml/1 mg = 4 mL

Step2: subtract the volume of concentrate from the total volume to determine the amount of
diluents needed.

4 ml-1ml = 3 mL

3.If a 600 ml of a 15% (v/v) solution of methyl salicylates in alcohol are diluted to 1500 ml what
will be the percentage strength.

51-1
Pharmacyprep.com Dilutions and Concentrations

C2 = (C1xV1)/V2

C2 = (15 x 600 ml)/1500 mL

C2 = 6%

4) A physician has prescribed 60 g of 0.01% fluocinolone acetonide cream. You have available
the commercially prepared cream containing 0.2% fluocinolone acetonine (Synalar cream) and
cream base (Dermabase ) for dilution. To prepare the Rx you would require:

a) 1.2 g Synalar and 58.8 g Dermabase

b) 3 g Synalar and 60 g Dermabase

c) 0.3 g Synalar and 59.7 g Dermabase

d) 3 g Synalar and 57 g Dermabase

C1. V1 = C2. V2

0.01% x 60 g = 0.2% . V2

X = (60 g x 0.01)/ 0.2 = 3 g Synalar

Order is for 60 g - 3 g = 57 g dermabase

5.Prescriber orders to prepare 1% hydrocortisone 60 g. Your pharmacy has stock of 0.5%


hydrocortisone and 2.5% hydrocortisone. How many each part should be taken?

0.5 1.5

2.5 0.5

2.5% of HC ...........( 0.5/2) x 60 = 15 g

0.5% of HC............(1.5/2)x60 = 45 g

51-2
Pharmacyprep.com Dilutions and Concentrations

6.Hydrocortisone 1% 60 g.

Your pharmacy has 2.5% hydrocortisone and petrolatum base. What fraction of each
hydrocortisone needed use to prepare above prescription

2.5% 1

1%

0% 1.5

2.5% HC = 24 g

petrolatum base = 36 g

7. How many milliequivalents of sodium is in 92 mg of NaCl salt? M.wt of Na = 23


mEq = [mg x valence]/(Mol.Wt)

mEq = 92/23

= 4 mEq

8) You are given ZnCl 2 0.7%, phenylephrine 0.1% and boric acid 1.1% with E values 0.16, 0.32
and 0.5 respectively. This solution will be:

A) Hypotonic
B) Hypertonic
C) Isotonic
D) Non isotonic

Solution.
[(0.7 x0.16] + [(0.1 x 0.32] + [ (1.1 x 0.5)]

= 0.112 + 0.032 + 0.55 = 0.694

= 0.9>0.69 = Hypotonic
How much NaCl required making isotonic solution?
0. 9 - 0.69 = 0. 21 or 2 mg

51-3
Pharmacyprep.com Dilutions and Concentrations

9) Zinc sulfate is a two-ion electrolyte, dissociating 70% in weak solutions. Calculate its
dissociation factor.
ZnSO 4 Zn + SO 4 + ZnSO 4

70+ 70 +30

170/100 = 1.7

51-4
www.pharmacyprep.com Sterile Preparations

PHARMACY PREP

Brand and Generic Name Indexes

Copyright © 2000-2015 TIPS Inc. Unauthorized reproduction of this manual is prohibited. This 62-1
manual is being used during review sessions conducted by PharmacyPrep.
www.Pharmacyprep.com

PHARMACY PREP

PRESCRIPTION PROCESSING AND DISPENSING

1.A 65 yo customer at your pharmacy using nitroglycerine patch daily for the prevention
of coronary artery disease. If patient a new prescription. Which of the following Rx is the
pharmacist concern?
A. Nitroglycerine 0.3 mg SL tab
B. Nitroglycerine 0.4 mg SL spray
C. Sildenafil 50 mg as needed
D. Tylenol # 3 for as needed for pain
E. Ritonavir soft gelatin cap 100 mg daily
Ans. c

2.A very frequent customer at your pharmacy, request for 15 days supply of Lipitor 10
mg tablets because customer has no prescription and no refills of Lipitor. In the next few
hours he has to take a flight for vacation. Currently all walk-in-clinics are closed and his
physician is not available for prescription.? According to extended scope of pharmacy
practice pharmacist dispensed 15 days of drugs? This is defined as?
A. Adaptation
B. Advancing a drug
C. Pharmaceutical opinion
D. Professional discretion
E. Professional judgement
Ans.b

3.MP came from the British Columbia, during the journey he lost his medications. He
approaches at your pharmacy with a rough list of medication, he was using. The
pharmacy and doctor clinic is currently closed due to long weekend in BC. What is
appropriate?
A. dispense the list of medication
B. refer him to walk in clinic
C. refuse to dispense any medications
D. write prescription
E. dispense OTC medications
Ans. b

4.A 75 yo patient has been discharged from the hospital. Presents at your pharmacy with
the following discharge report prescription. What is the pharmacist concern?
a. Metformin 1000 mg, PO, evening with dinner and metformin 1000 mg, po, morning
with breakfast
B. Levodopa/carbidopa 25/100 mg, po, four times per day
C. Digoxin 120 mg once daily in the morning

Copyright © 2000-2016 TIPS Inc. Unauthorized reproduction of this manual is prohibited. This 53-1
manual is being used during review sessions conducted by PharmacyPrep.
www.Pharmacyprep.com

D. Donepezil 5 mg, po, daily


E. Tramadol 50 mg as needed for pain
Ans. c

5.Which of the following prescription is written incorrectly?


A. Pantoprazole 40 mg, po, daily before breakfast
B. Spironolactone 10 mg, po, daily
C. Acetaminophen 650 mg, every 4 hours
D. Fentanyl patch 50 mcg/hour daily
E. Alprazolam 0.5 mg SL tab daily at bedtime
Ans. d

Copyright © 2000-2016 TIPS Inc. Unauthorized reproduction of this manual is prohibited. This 53-2
manual is being used during review sessions conducted by PharmacyPrep.
www.pharmacyprep.com

PHARMACY PREP

DRUGS IN SPECIAL POPULATIONS

1. Which of the following incorrect about elderly?


A. Increased pH of stomach secretion
B. Slow gastric emptying time
C. Reduced enzyme production
D. Increased excretion of renal elimination drugs
E. Decreased metabolism of drugs
Ans. D

2. Which of the following incorrect about pregnancy?


A. Increased pH of stomach secretion
B. Slows gastric emptying time
C. Increased enzyme production
D. Increased excretion of renal elimination drugs
E. Increased metabolism of drugs
Ans. A

3. which of the following is the most critical of organ development in pregnancy?


A. Biogenesis
B. Organogenesis (2 to 8 wks) or emryotoxic
C. Fetal Period (9wk to labor)
D. The first 15 -21 days (biogenesis)
E. Ninth week to birth
Ans. b
Tips.
Biogenesis - The first 15-21 days after fertilization. Cleavage, and germ layer formation.
Organogenesis 2-8 wks - The major organ start developing. (congenital defects)
Fetal period- at 9th wk, the embryo referred to as fetus.

4. when you considering medication therapy in pregnancy.


A. both mother and fetus must be considered
B. weigh risk and benefits
C. rule of thumb is lowest effective dose for shortest period
D. All of the above
ans.d

Copyright © 2000-2016 TIPS Inc. Unauthorized reproduction of this manual is prohibited. This manual is
being used during review sessions conducted by PharmacyPrep.
Pharmacyprep.com

PHARMACY PREP
CLINICAL TOXICOLOGY

1- An adult patient who ingested 30 acetaminophen tablets (325 mg/tab) 6 hours ago
should be treated with/by
A. EDTA infusion
B. Ipecac syrup
C. Activated charcoal
D. N-acetylcysteine
E- Probenecid

2. Glutathione conjugation are catalyzation is extremely important in preventing


toxicity from variety of drugs, toxic products are excreted as:
A) S-adenosyl transferase
B) Glutamic acid
C) Marcapturic acid
D) Mercaptopurines
E) Sulfadryl groups

3-A 33-year-old male receiving Acetaminophen 300 mg + codeine 30 mg + caffeine 15


mg (Tylenol # 3) for pain relief, which of the following laxative is appropriate choice for
constipation associated with codeine;
A-Sennakot
B-Bulk laxatives
C-Lactulose
D-Psyllium
E-Glycerine

4-A patient brings the prescription of amitriptyline 300 mg 3 month’s supply. Patient
medical history includes, major depression and in the past has 2 unsuccessful attempts
to suicide. What is the appropriate, a pharmacist should do?
A-Dispense 3 months supply of amitriptyline and caution about overdose symptoms.
B-Dispense in part fills or small quantities and ask patient pick up in parts every 15 days
C-Refuse the prescription and ask patient to see other doctor
D-Refer patient to other physician for mental assessment.

5-Overdose of acetaminophen is treated by?


A-N-acetylcysteine, administered within 1 hour
B-N-acetylcysteine, administered within 8 hours
C-N-acetylcysteine can be administered anytime after overdose
D-No antidote is available for acetaminophen overdose

Copyright © 2000-2016 TIPS Inc. Unauthorized reproduction of this manual is prohibited. This 55-1
manual is being used during review sessions conducted by PharmacyPrep.
Pharmacyprep.com

E-Sodium bicarbonate diuresis

6-What is daily maximum dose of acetaminophen?


A-10 g B-4 g C-7 g D-1 g E-8 g

7- A grand mother gave 2 tbsp (250 mg/5ml) of amoxicillin suspension instead of 2 tsp
to a child 9 mo age. What is appropriate to do?
A-Monitor diarrhea
B-Refer to doctor
C-Refer to emergency
D-Call poison control center
E-Wait and watch

8- A 5 yr child swallowed 5 g of nystatin cream, what to do?


A-Monitor diarrhea
B-Refer to doctor
C-Refer to emergency
D-Call poison control center
E-Wait and watch

9- A regular customer of your pharmacy brings a prescription of amitriptyline 300 mg


daily for 3 mo. However patient profile shows he had attempted suicide in the past but
was not successful. What is appropriate to do?
A) recommend not to overdose
B) recommend to take as doctor prescribed
C) give a part fill of monthly dose
D) talk to doctor to prescribe other antidepressant
E) refuse to dispense

10- A grand mother gave 2 tbsp of amoxicillin of 500 mg/5ml bid instead of 2 tsp to a 9
month old child. What to do?
A) refer to emergency
B) refer to doctor
C) wait and see
D) recommend to skip next dose
E) contact regional poison control centre

11) A 2 yr old child swallowed nystatin 10 g suspension, What to do?


A) refer to emergency
B) refer to doctor
C) wait and see
D) recommend to skip next dose
E) not a concerned

Copyright © 2000-2016 TIPS Inc. Unauthorized reproduction of this manual is prohibited. This 55-2
manual is being used during review sessions conducted by PharmacyPrep.
Pharmacyprep.com

12) A doctor comes to your pharmacy and write a prescription for oxycontin 100 tablets
for himself? What to do?
A)dispense 50 tablets only
B)dispense 200 tablets only
C)it is illegal dispense narcotics to doctor
D)It is unethical to dispense narcotics for themselves
E)Refer doctor to Centre of addiction and mental health (CAMH)

13) A regular customer of your pharmacy, using recently picked up prescription of


Tylenol # 3 (acetaminophen 300 mg + codeine 30 mg + caffeine 15 mg) 90 tablets. Now
wants to buy acetaminophen 500 mg box of 200 tablets. What is the pharmacist
concern?
I) overdose
II) constipation
III) hepatotoxicity
A) I only B) III only C) I and II D) II and III E) I, II, III

14- Pharmacy tech received a phone call from your pharmacy customer. She has
swallowed 10 tablets of acetaminophen 500 mg. What is appropriate recommendation?
A) Suggest to take lots of fluids
B) refer to emergence
C) ask to contact poison control centre
D) ask wait and see
E) give N acetyl cysteine

15- What is the initial symptoms of acetaminophen toxicity?


A) stomach upset B) hepatotoxicity C) headache D) confusion E) vomiting

A child admitted to hospital following ingestion of overdose of aspirin and some other
OTC sleeping pills, which contain antihistamines, and anticholinergic drugs. The child is
in coma and flushed and flaccid. Her pupils were fixed and dilated. The lab results
indicated her blood aspirin drug levels are 5.8 H is 6. Her
an symptoms
r ne
also include deep and rapid respiration.

16) The symptoms of deep and rapid respiration may lead to:
A-Hypertension
B-Respiratory acidosis
C-Respiratory alkalosis
D-Increase in temperature
E-Reduction in temperature
Ans-C
Tips:
Respiratory Acidosis. This occurs due inadequate ventilation of CO 2 by lungs.

Copyright © 2000-2016 TIPS Inc. Unauthorized reproduction of this manual is prohibited. This 55-3
manual is being used during review sessions conducted by PharmacyPrep.
Pharmacyprep.com

Predisposing factor for respiratory acidosis:, Asthma, Beta blockers, Sleep apnea, CNS
depressants, Pulmonary edema or embolism, Cardiac arrest
RESPIRATORY ALKALOSIS. Due to increase in excretion of CO 2 However this condition is
not very common.

17) As result of aspirin mechanism, the child would likely to progress?


A-Metabolic acidosis
B-Metabolic alkalosis
C-Renal failure
D-Hyperglycemia
E-None of the above
Ans-A

18) In order to increase the excretion of aspirin from body, which of the following
should be administered:
A-Probenecid
B-Ammonium chloride
C-charcoal
D-Ipecac
E-Sodium bicarbonate
Ans-E

ANSWERS:

1- D

2. C

3- A
Tips: Stimulants laxative are the drug of choice for the treatment of constipation
associated with opioids use.

4-B

5- B

6- B

7- B

8- E

Copyright © 2000-2016 TIPS Inc. Unauthorized reproduction of this manual is prohibited. This 55-4
manual is being used during review sessions conducted by PharmacyPrep.
Pharmacyprep.com

9- C

10- E
Tips: serious toxicity is unlikely following large doses of amoxicillin. Acute ingestion of
large doses (250 mg/kg) of amoxicillin may cause nausea, vomiting, diarrhea, abdominal
pain. Acute oliguric failure and hematuria may occur following large doses
Reference: CPS 2009

11- C
Tips: Nystatin suspension have no systemic absorption, so overdose monitor symptoms
so wait and see.

12- D

13- E

14- C

15- E

Copyright © 2000-2016 TIPS Inc. Unauthorized reproduction of this manual is prohibited. This 55-5
manual is being used during review sessions conducted by PharmacyPrep.
Pharmacyprep.com

PHARMACY PREP
PHARMACY COMMUNICATION SKILLS
1) Which of the following is the most effective communication skill?
I-Verbal skills
II-Nonverbal listening skills
III-Verbal skills only
A-I only
B-III only
C-I and II only
D-II and III only
E-All of the above

2-A pharmacist is required to give knowledge of drugs and supply the patient with
enough pieces of information to overcome the patient ignorance. This is done by:
a) Written information only
b) Verbal information only
c) Verbal and written information
d) Magazines and folders information
e) Video information

3-The most effective communication skills include?


A-Written only
B-Verbal and written
C-Verbal and non-verbal
D-Verbal only
E-Listening only

4-All of the following communication distractions, except:


A-Telephone
B-Language
C-Cell phone
D-Music
E-Television

5) A patient who does not speak English and you don't understand his language. How do
you resolve this problem?
A-Find a interpreter or translator
B-Patient problem do not worry
C-Pharmacist problem learn his language
D-Both pharmacist and technician problem
E) Use non verbal techniques

Copyright © 2000-2016 TIPS Inc. Unauthorized reproduction of this manual is prohibited. This 56-1
manual is being used during review sessions conducted by PharmacyPrep.
Pharmacyprep.com

6-A 90-year-old senior patient cannot hear, how to counsel?


A-Speak louder
B-Counsel in quite area of pharmacy
C-Give recorded tape of counseling
D-speak vivid and closer to customer
E-Don’t speak to customer

7) As a pharmacist you should have speaking skills. All the following are true except:
A) Speak in a monotonous voice pitch.
B) Speak with precise language.
C) Write when verbal is not sufficient.
D) Greet the patient.

8) All the non verbal skills to use during telephone conversations except:
A) Give your full attention to phone call.
B) Put the patient on hold if you are busy.
C) Be prepared.
D) Smile

9) In non verbal communication you should:


A) Use appropriate body language.
B) Look down repeatedly.
C) Tilt to one side.
D) Have the barriers between you and the patient like at desk or counter.

10) A patient coming to take OTC cough syrup and pharmacist noticed that patient took
same medication yesterday, what you should do?
a. Tell her no more medication available
b. Ask her what medical conditions she needs the medication
c. Give her the name of the addiction group
d. Call the police

11) If you open a pharmacy in multicultural area, what will you do increase sales?
A) Display signs of different culture
B) Hire staff of different cultures
C) Offer only one culture service
D) Refuse all the customer who disagree with opinion
E) Offer services in different languages

12) What is the best marketing in pharmacy?


A) Advertise in national TV
B) Advertise in national news paper
C) Advertise door to door in neighborhood

Copyright © 2000-2016 TIPS Inc. Unauthorized reproduction of this manual is prohibited. This 56-2
manual is being used during review sessions conducted by PharmacyPrep.
Pharmacyprep.com

D) Advertise in mall

13) What are the characteristics of culturally competent pharmacist?


A) Understanding other cultures and respecting sensitive issues.
B) Offer services to all customers of different belief system
C) Offer privacy and confidentiality to customers of their personal information
D) All of the above

14) Customer of your pharmacy with kids. Parents discipline their kids but they are
beating and spanking? What to do?
A) Talk to parents and kids
B) Call police and inform about it
C) Call children society to protect kids
D) Ignore and let them go

15) Adult women come to your pharmacy to buy analgesics for her wounds. She seems
to physically and sexually abused what to do?
A) Call police
B) Ask her to go emergence
C) Refer to doctor
D) Give her police phone number to contact
E) Ask her to contact sexually abused support groups

16) Which of the following is NOT an environmental barrier?


A) Background noise
B) Telephone rings
C) Height of pharmacy counter
D) Dealing with a depressed patient
E) Loud music

ANSWERS:
1) C

2- C

3- B

4- B
Tips:The language can be barrier. However it is not distraction.

5. A

Copyright © 2000-2016 TIPS Inc. Unauthorized reproduction of this manual is prohibited. This 56-3
manual is being used during review sessions conducted by PharmacyPrep.
Pharmacyprep.com

6- D

7. A
Tips: It is boring to speak in a monotonous language

8. B

9. A

10) B

11) E

12) C

13) D

14) A

15) E

16) D

Copyright © 2000-2016 TIPS Inc. Unauthorized reproduction of this manual is prohibited. This 56-4
manual is being used during review sessions conducted by PharmacyPrep.
www.pharmacyprep.com

Pharmacy PREP

ETHICAL STANDARDS

1. The ethical standard beneficence in pharmacy practice means:


A. Pharmacist act with justice
B. Pharmacist act with fairness
C. Pharmacist act with honesty and without deception
D. Pharmacist does well to patient
E. Pharmacist do prevent harm

2. The ethical standard nonmaleficence in pharmacy practice means:


A. Pharmacist act with justice
B. Pharmacist act with fairness
C. Pharmacist act with honesty and without deception
D. Pharmacist does well to patient
E. Pharmacist do prevent harm

3. The ethical standard veracity in pharmacy counselling means:


A. Pharmacist act with justice
B. Pharmacist act with fairness
C. Pharmacist act with honesty and without deception
D. Pharmacist does well to patient
E. Pharmacist do prevent harm

4. The ethical standard justice in pharmacy practice means:


A. Pharmacist act with equality
B. Pharmacist act with fairness
C. Pharmacist act with honesty and without deception
D. Pharmacist does well to patient
E. Pharmacist do prevent harm

5. Patient cannot afford to buy new medication that is not covered by his insurance
plan. Pharmacist called his doctor and asked him to change the medication into another
medication covered by the patient’s insurance plan. Which ethical principle did the
pharmacist follow?
A. Beneficence
B. Veracity
C. Justice
D. Nonmaleficence
E. Fidelity

Copyright © 2000-2016 TIPS Inc. Unauthorized reproduction of this manual is prohibited. This 57-1
manual is being used during review sessions conducted by PharmacyPrep.
www.pharmacyprep.com

6) A pharmacist stops counseling a patient who repeatedly takes the medication


incorrectly. Which ethical principle does pharmacist violate?
A) Justice
B) Beneficence
C) Nonmaleficence
D) Autonomy
E) Veracity

7. Pharmacy professional ethical standards include nonmaleficence, which means:


A. Pharmacist should act with honesty and veracity
B. Pharmacist should act with fairness
C. Pharmacist should prevent harm
D. Pharmacist should place the benefit of the patient above all else.
E. Pharmacist should respect patient choice.

8. A physician phones a pharmacist asking him not to tell the patient about the side
effects of a drug because if he does, the patient will not take it. The pharmacist
complies. Both the physician and pharmacist are seeking which ethic for the patient?
A. Nonmaleficence
B. Beneficence
C. Autonomy
D. Veracity
E. Justice

9. A physician phones a pharmacist asking him not to tell the patient about the side
effects of a drug because if he does, the patient will not take it. The pharmacist
complies. Both the physician and pharmacist have violated which ethic principle?
A. Nonmaleficence
B. Beneficence
C. Autonomy
D. Veracity
E. Justice

10. A pharmacist who refuses to counsel and AIDS patient violates which ethic?
A. Nonmaleficence
B. Beneficence
C. Autonomy
D. Veracity
E. Justice

Copyright © 2000-2016 TIPS Inc. Unauthorized reproduction of this manual is prohibited. This 57-2
manual is being used during review sessions conducted by PharmacyPrep.
www.pharmacyprep.com

11. A father of teenage girl approach to your pharmacy. Asks if her teenage 16 yo
daughter using oral contraceptives? What ethical dilemma of pharmacist?
A. confidentiality and veracity
B. confidentiality and autonomy
C. confidentiality and beneficence
D. confidentiality and fidelity
E. confidentiality and justice

12. A father of teenage girl approach to your pharmacy. Asks if her teenage 16 yo
daughter using oral contraceptives? What ethical dilemma of pharmacist?
A. Veracity and confidentiality
B. Beneficence and confidentiality
C. Justice and confidentiality
D. Nonmaleficence and confidentiality
E. Autonomy and confidentiality

13. When can you break confidentiality?


A. patient who has HIV with partner
B. Patient inmate who broke the law
C. Patient who has Alzheimer's with care giver
D. A patient could be harmful to themselves and community risk
Ans. D

14. What is the principle of confidentiality does not break if a pharmacist disclose the
following medical conditions to the society?
A. diabetes B. Harm full C. HIV D. high BP E. cancer
Ans. B

15. In which of the following situation you can break confidentiality of patient?
A. patient is suffering from HIV
B. patient is on parole
C. suspect patient can harm himself
D. patient is in jail
Ans. C

16. A 30 yo women comes to your pharmacy to buy oral contraceptives. While


conversation she says she found antiretroviral drugs in her husband bag. If her partner
has HIV. Your medication record show patient is HIV? What is appropriate to do?
I) Tell her to take HIV tests
II) Tell her to talk his doctor
III) Tell her that she got HIV infection from her partner
A. I only
B. III only
C. I and II only

Copyright © 2000-2016 TIPS Inc. Unauthorized reproduction of this manual is prohibited. This 57-3
manual is being used during review sessions conducted by PharmacyPrep.
www.pharmacyprep.com

D. II and III only


E. All are correct

17. Can you release information to a prescriber if you have firsthand knowledge from
the third party or DUR about double doctoring?
A. Patient information is confidential
B. Let third part inform to prescriber
C. Yes, you can inform to prescriber because it is in circle of care
D. Ask patient first, before informing doctor

18. Pam is working in rural pharmacy. She is friend with all colleagues, including Chris.
Chris had vision problems in the past, but underwent laser surgery and now is ok. He
does not work pharmacy anymore, but Pam is still has his profile in her computer.
Should she date him?

19. Patient is using Terbutaline inhaler a lot. The pharmacist failed to advise patient to
see his doctor to add corticosteroids therapy to manage his condition. What is violated?
A. Beneficence B. Nonmaleficence C. veracity D. autonomy E. Paternalism

20) A patient has diagnosed with terminal ill cancer. His doctor advised him about his
serious illness. What ethical principle is followed?
A. Beneficence
B. Nonmaleficence
C. veracity
D. autonomy
E. Paternalism

21. Which of the following is express consent?


I) given verbally
II) written
II) given electronically
A) I only B) III only C) I and II only D) II and III only E) All are correct

22. A pharmacist Joe is being restricted from practice because of history of substance
abuse. Based on what the college makes this decision?
A. Incompetent B. Incapacity C. Negligence D. Mis-conduct E. accountability

23) A returned medication is placed on shelf by the technician? what you have to as
staff pharmacist?
A) talk to technician
B) inform to manager
C) complain to college of pharmacist
D) Discuss in meeting

Copyright © 2000-2016 TIPS Inc. Unauthorized reproduction of this manual is prohibited. This 57-4
manual is being used during review sessions conducted by PharmacyPrep.
www.pharmacyprep.com

24) Which of the following functions cannot be delegated to pharmacy technician?


A) Preparing prescription
B) Checking expiry date
C) Demonstrating a device to customer
D) Selecting a over the counter drug to a patient
E) Providing a disease literature to customer

25) Pharmacist suspects a junior technician preparing sterile IV preparations. However


technician did not followed proper sterile prep proper technique. What is appropriate to
do?
A. Discard the sterile preps and redo new batch
B. Tell the tech to follow standard techniques
C. Inform the pharmacy manager
D. Document and ask technician to sterile prep training
E. None of the above

26. While preparing prefilled syringes, in laminar airflow hood. After packaging you find
some syringes leaking. What you have to do?
A. Call the company complain
B. Discard all the batch leaking syringes and document
C. Select the good ones and through the damage
D. Just discard one leaking syringe from the batch and rest can be used
E. Continue processing of this batch of syringe but investigate to prevent leakage again

27. A doctor is asking a patient to sign a consent form for a particular treatment. Which
of the following applies?
A. Inform consent
B. Implied consent
C. Express consent
D. All of the above
Ans. C

28) A patient comes to your pharmacy with a prescription of his wife who is not covered
under his prescription drug plan. He requests you to process her prescription under his
name. He will pay cash for the cost and later send the receipt to his insurance company
to process the claim?
A) it is unethical
B. It is illegal
C. It is ok
D. He can claim from insurance

Copyright © 2000-2016 TIPS Inc. Unauthorized reproduction of this manual is prohibited. This 57-5
manual is being used during review sessions conducted by PharmacyPrep.
www.pharmacyprep.com

29. Which of the following is incorrect combination?


A. Autonomy = Right of self determination
B. Beneficence = doing good
C. Nonmaleficence = Preventing harm
D. Justice = unequal distribution of benefits and burden
E. Veracity = Honesty without deception

30. This principle expresses the concept that pharmacist have a duty to be honest and
trustworthy in their dealings with people.
A. Principles of Ethics
B. Code of Professional Conduct
C. Principle of veracity ("truthfulness")
D. Principle of honesty

31. A physician write prescribed for own use or family member for emergency?
A. Unethical
B. Illegal
C. physician can prescribe can prescribe in emergency
D. physician to refer to emergency

32. A very frequent customer at your pharmacy, request for 15 days supply of Lipitor 10
mg tablets. Next few hours he has to take a flight for vacation. Currently all walk-in-
clinics are closed and his physician is not available for prescription. Customer has no
prescription and no refills of Lipitor? pharmacist dispensed meds. what ethical principles
were followed (uphold)?
A. Beneficence
B. Nonmaleficence
C. veracity
D. Justice
E. Autonomy

33) A terminally ill cancer patient. Doctor did NOT informs patient that patient is
seriously ill and may die in few weeks? What ethics violated?
A) Beneficence B) Veracity C) Justice D) Nonmaleficence E) Fidelity

34) A police officer comes to your pharmacy, wants a patient profile of medication, who
has been caught on street drugs and in traffic rules violation? What is appropriate
action?
A) Ask police ID and patient ID and give patient profile
B) Ask police ID and if he is real police officer give patient profile
C) Refuse to give any patient information to police
D) Ask police officer if he has any court letter (search warrant) or letter of authorization
from patient to release patient information.

Copyright © 2000-2016 TIPS Inc. Unauthorized reproduction of this manual is prohibited. This 57-6
manual is being used during review sessions conducted by PharmacyPrep.
www.pharmacyprep.com

35) A lawyer calls to your pharmacy wants to get medication information of a divorced
spouse. Children? What to do?
A. Talk to lawyers, client and provide information
B. Refuse to provide information
C. Children information only can be shared by parents
D. Lawyer should provide official letter of patient authorization from adults
Ans. d

36) A phone call from emergency medical services, wants to a patient profile of patient
who is found unconscious in his apartment? What is appropriate action?
A) Ask ID of EMS and refuse to give patient information
B) Never trust any phone calls ask EMS to come personally to pharmacy.
C) Verify and give patient profile to EMS, it is in best interest of patient.
D) Ask EMS to contact to doctor of a patient to get information
E) Ask EMS to contact family member of patient.

37. A frequent customer, on benzodiazepine for long time, his doctor away and there
are no walk in clinics open. He wants to get some pills advance? What you do?
A. talk to that person
B. ask patient to see other doctor to get prescription
C. advance benzodiazepine
D. Refer to pharmacist

38. In what situation pharmacist call police?


I) forged prescription
II) Break - in happened
III) narcotic theft
A. I only B. III only C. I and II only D. II and III only E. All are correct

39) A father of two adults children and wife customer of your pharmacy, wants you to
print receipts of their medications for filing tax returns.
A) Get the authorization from wife and adult children
B) Receipts only without specifying medication just total amount can be printed to
father
C) Refuse to provide receipts
D) Ask to contact revenue Canada
Ans. A

Copyright © 2000-2016 TIPS Inc. Unauthorized reproduction of this manual is prohibited. This 57-7
manual is being used during review sessions conducted by PharmacyPrep.
www.pharmacyprep.com

40. A physician called to pharmacy to get patient record of medications patient


investigational drugs receipt. Patient is under the care of the doctor. Pharmaceutical
company already paid to that doctor and as technician what you have to reply?
A. need patient authorization
B. If it is not legal way to give to doctor
C. print out investigational drug receipt to the doctor
D. tell doctor to contact with pharmaceutical company

41) Who sets the prices of patented medications?


A) Patented Medicine Price Review Board (PMPRB)
B) Health Canada
C) Manufacturer
D) Pharmacist associations

42. A patient consent may be essential in all, except?


A. To collect patient information
B. To release patient information to third parties
C. To publish clinical trial results of a anticancer drug
D. To prepare prescription by the technician of your pharmacy
E. None of the above

43. A 16 yr old girl brings a prescription of oral contraceptive pills. However, her parents
calls to your pharmacy and warns you to not dispense OCP to their daughter because
using contraception is against of their belief system. What parents actions are:
A. Justice B. Paternalism C. Fidelity D. Beneficence E. Veracity

44) A woman comes to your pharmacy, ask if her husband has HIV? As your pharmacy
has her husband profile. He is HIV positive? What to do?
A) Do check for yourself and speak to your husband
B)Tell her yes! he has HIV
C) Don't tell her the truth
D) Tell her not to worry, HIV is treatable
Ans. a

45) The pharmacist preserves the confidentiality of information about individual patient
acquired in the course of his or her professional practice, and does not divulge this
information except where authorized by the patient or required by law. Which of the
following federal regulation protects personal health information of patient?
A)Canadian health act
B)Therapeutic directorate, health Canada
C)Institute of safe medication practices
D) Personal Health Information Protection Act
E) All of the above

Copyright © 2000-2016 TIPS Inc. Unauthorized reproduction of this manual is prohibited. This 57-8
manual is being used during review sessions conducted by PharmacyPrep.
www.pharmacyprep.com

Ans: D

46. Health information can be shared between?


A. nurse
B. doctor
C. spouse
D. circle of care
Ans. d

ANSWERS:

1. D

2. E

3. C

4. B

5. A

6. C

7. C

8. B

9. D

10. E

11. A

12. A

13 A

14. C

15. A

16. C

Copyright © 2000-2016 TIPS Inc. Unauthorized reproduction of this manual is prohibited. This 57-9
manual is being used during review sessions conducted by PharmacyPrep.
www.pharmacyprep.com

17. C
Tips. Information can be shared in circle of care (if patient is under the care of a doctor,
pharmacist, nurse or caregivers).

18.

19. B
Tips. By not preventing harm. The nonmaleficence is violated

20. C

21. E
Tips. The consent can be obtained by three types express, implied, and informed.

22. B
Incompetent: lack of knowledge, skills, judgment
Incapacity: substance abuse or accidents
Negligence: working lower than standard
Misconduct: unethical
Accountability: legally and morally liable when carrying out duty
Advance directives = legal will

23. A

24. D
Tips. According to pharmacy technician standards of practice. A regulated technician or
pharmacy assistant cannot recommend prescription or non prescription therapies to
patient. Any clinical recommendations or counseling is CANNOT be performed by
pharmacy technicians.

25. A

26. B

27. C

28. B

29. D

30. C

31. C

Copyright © 2000-2016 TIPS Inc. Unauthorized reproduction of this manual is prohibited. This 57-10
manual is being used during review sessions conducted by PharmacyPrep.
www.pharmacyprep.com

32. A

33. B

34. D

35. C

36. C
Tips: It is circle of care, you can share information after confirmation.

37. B

38) E

39) B

40) C

41. A

42. D

43. B

44. A pharmacist believes, using Plan B is against his faith. If a customer wants to use
plan B. would you dispense?

45. A physician write prescription for own use or family member for emergency?

46. A very frequent customer of your pharmacy, requests for 15 days supply of Lipitor
10 mg tablets. Next few hours he has to take a flight for vacation. Currently all walk in
clinics are closed and his physician is not available for prescription. Customer has no
prescription and no refills of Lipitor? If you dispensed what ethics are followed?

47. A patient of your pharmacy approaches to refill salbutamol inhaler. Patient asthma
is getting worse and using more frequent salbutamol. However pharmacist did not
suggest patient to see the doctor to get steroid inhaler prescription. What ethical
principles have been violated?

48. There are two customers of pharmacy, one customer all family members come to
your pharmacy. Other customer family members go to other pharmacy. A pharmacist

Copyright © 2000-2016 TIPS Inc. Unauthorized reproduction of this manual is prohibited. This 57-11
manual is being used during review sessions conducted by PharmacyPrep.
www.pharmacyprep.com

takes NO copayment from a customer who comes with all family members, where as
customer comes as single persons pays co-payment, What ethical principle is violated?

49. A pharmacist refuses to dispense plan B, because of pharmacist beliefs are against of
using contraception?

50. A family two adults and one 16 yo daughter are customer of your pharmacy. one of
the family member comes to your pharmacy, ask for list medications their family
members using for the past 6 months. What to do?

A cancer patient has one month to live. Doctor did not inform the patient about cancer.
What ethical principles is broken?
A) Beneficence
B) Veracity
C) autonomy
D) Confidentiality
E) Paternalism

Patient brings a new prescription. Pharmacist suspect the signature of prescription is


forged? What is appropriate action?
A) Send back patient to doctor
B) Directly call the doctor to confirm
C) Check previous records of prescription to compare the signature.
D) Fax prescription back to physician

A Friday evening a neighboring storeowner brings a prescription of tadalafil 10 mg. Your


pharmacy is busy and people are lined up. He gets off the line and quietly ask
pharmacist that he is in hurry and catch up the traffic to reach up cottage for weekend.
What is appropriate to do?
A) Ask permission other people in line
B) Tell him to line up
C) Dispense him private counseling area not to repeat.
D) Tell him to go other pharmacy

Copyright © 2000-2016 TIPS Inc. Unauthorized reproduction of this manual is prohibited. This 57-12
manual is being used during review sessions conducted by PharmacyPrep.
www.pharmacyprep.com

Covenant relationship?

Failing to maintain a standards of practice of profession is?


a. illegal
B. unethical
C. professional misconduct
D. Pharmacy act

A patient approach at your pharmacy for OTC drugs. Patient expressed that
he did not like this medication. Pharmacy tech got upset and verbally abused
patient?
a. illegal
B. unethical
C. professional misconduct
D. Pharmacy act

Pharmacy dispensing fee is?


A. A federal law
B. NAPRA
C. By-law
D. Controlled Act

Copyright © 2000-2016 TIPS Inc. Unauthorized reproduction of this manual is prohibited. This 57-13
manual is being used during review sessions conducted by PharmacyPrep.
www.PharmacyPrep.Com Information resources

PHARMACY PREP
PHARMACY PRACTICE INFORMATION RESOURCES

1. The Canadian pharmaceutical reference CPS stands for:


A. Compendium of pharmaceutical Sciences
B. Compendium of Pharmaceutical Supplements
C. Compendium of Pharmaceutical Society
D. Compendium of Pharmaceutical Specialties
E. Compendium of Pharmaceutical Sources

2. A patient presents a prescription that requires the extemporaneous preparation of a


buffered eyedrop. Which of the following is the source for information on the correct
buffering of the eyedrop?
A. Facts and comparisons
B. The Merck manual
C. Pharmacy PREP index
D. Remington’s pharmaceutical sciences
E. Physicians desk reference

3. One of your regular patients comes to the pharmacy in search of Dogmatil® for her
brother. He has just arrived from Singapore for a visit, but his medication was lost with
his luggage. She says her brother needs the medication for a CNS condition and
shouldn’t go without it. She wants to know if he can get Dogmatil® here. Which of the
following appropriate reference:
A. Canadian hospital journal
B. American hospital journal
C. Compendium of pharmaceutical specialties
D. Martindale, The complete drug reference
E. National formulary

4. Cochrane collaboration databases or collaborative library is the reference source of?


A. Evidenced based medicine information
B. A Canadian pharmacy journal
C. Collaboration of medical professionals
D. A natural products database
E. The group of people with same characteristics
Ans. A

Copyright © 2018 TIPS Inc. Unauthorized reproduction of this manual is prohibited. This manual is 58-1
being used during review sessions conducted by PharmacyPrep.
www.PharmacyPrep.Com Information resources

5. Compendium of pharmaceutical specialties is useful resource for locating which of the


following information?
A. Approved indication for all prescription pharmaceuticals in world
B. Detailed monographs for prescription pharmaceuticals available in Canada
C. Clinical recommendations for dental prophylaxis of bacterial endocarditis
D. Canadian consensus guidelines on drug therapies used to treat asthma and COPD
E. Recommendation patient self selection products in Canadian pharmacies

6. What information is NOT present in compendium of pharmaceutical specialties?


A. Monograph of prescription medications in Canada
B. Clinical information of drug and food interactions
C. Immunization schedules
D. Recommendation of OTC drugs to treat headache
E. Antibiotics and antiviral monographs

7. What information is not obtained from patient self care reference?


A. Non pharmacology advise for foot care
B. Red flags that require referral to doctor
C. Non prescription of medication
D. Treatment of cold sores
E. Choosing a brand of calcium supplements

8. What information is not obtained from therapeutic choices?


A. treatment options for initial therapy
B. treatment options for alternate drug of choices
C. Laboratory investigations for diagnosis of disease
D. treatment options for self selection drugs
E. treatment options of pneumonia

9. What is suitable reference source to locate information about immunization


schedules in Canada?
A. Martindale B.USP-DI C. Handbook of immunization
D. Compendium of Pharmaceutical Specialties E. Vaccine manufacturer

10. If a patient comes and asking for drug information, you would do?
A. search for compendium of pharmaceutical specialties
B. Ask doctor
C. search on Internet
D. Call information center
E. Search Medline

11. Which of the following are the most credible information resources?
A. scientific journals containing clinical studies

Copyright © 2018 TIPS Inc. Unauthorized reproduction of this manual is prohibited. This manual is 58-2
being used during review sessions conducted by PharmacyPrep.
www.PharmacyPrep.Com Information resources

B. indexes
C. abstracts
D. text books
E. TV and radio, internet

12. What information is NOT present in CPS?


A. Prescription drugs available in Canada
B. Drugs in dentistry
C. Drugs in pregnancy
D. Self care for nausea and vomiting
E. Prescription drug monographs

13. What information is NOT present in patient self care?


A. Antibiotics
B. Mouth hygiene in halitosis
C. choosing a sunscreen
D. self care for nausea and vomiting
E. treatment of nausea and vomiting in pregnancy

14. What is reference is used to recommend to initial therapy for hypertension?


A. CPS
B. Patient self care
C. hypertension clinical guidelines
D. Therapeutic choices
E. USP-DI vol.1

15. Which of the following resources would provide the most up-to-date answer when
checking if a drug is marketed in Canada?
A. e-CPS
B. CPS
C. PubMed
D. Martindale’s
E. USP-DI vol.1

16. CPS is?


A. primary literature
B. secondary literature
C. tertiary literature
D. All of the above
ans. c

Copyright © 2018 TIPS Inc. Unauthorized reproduction of this manual is prohibited. This manual is 58-3
being used during review sessions conducted by PharmacyPrep.
www.PharmacyPrep.Com Information resources

17. A prescriber wants to know drug interaction of simvastatin with clarithromycin.


What is initial reference pharmacist should search?
A. primary research journal
B. Internet search
C. Compendium of pharmaceutical specialties
D. Consumer product monograph of simvastatin
E. Product package insert
Ans. c
Tips: Begin search from Tertiary or Secondary -----> Primary

18. What is not found in Compendium of pharmaceutical specialties?


A. iron supplements
B. Vaccine monograph
C. Antacids drug interactions with tetracycline
D. Calcium supplements interactions with levothyroxine
E-non prescription drugs and self care
Ans. e

19. What is true about off labeled drug? except


A. Can be found in Martindale
B. Can be found in Medline or Pubmed
C. can be prescribed by a physician based published scientific evidence
D. Pharmacist should refuse dispensing off label used drugs
E. Patient should be informed about off label use of drug
Ans. D

ANSWERS:
1. D

2. D

3. D

4. A

5. B

6. D

Copyright © 2018 TIPS Inc. Unauthorized reproduction of this manual is prohibited. This manual is 58-4
being used during review sessions conducted by PharmacyPrep.
www.PharmacyPrep.Com Information resources

7. E
Tips: brands of over the counter products can be found in compendium of self care
products (CPSP)

8. D

9. D

10. D

11. A

12. D

13. A

14. C

15. A

Copyright © 2018 TIPS Inc. Unauthorized reproduction of this manual is prohibited. This manual is 58-5
being used during review sessions conducted by PharmacyPrep.
PharmacyPrep.Com Medication Errors

PHARMACY PREP

MEDICATION ERRORS

1. Verbal prescriptions of drug "names sound like" drugs, can cause medication errors. What
should manager do to decrease the error EXCEPT:
A. Put them away from each other
B. Put label on vial (call for error)
C. Put in computer system alarm to work on dispensing
D. Technician ask patient for what disease being treated for
E. Make two persons double check before dispensing

2-Busy pharmacies a long line up in 15 customers, a person came he is insisting and fighting
with pharmacist, do dispense him first?
A-ignore that customer
B-dispense him first
C-don’t dispense
D-just talk to him and explain there are other before you.
E-Call police

3) A very angry customer comes to your pharmacy and complains that you have dispensed
wrong medication. What is initial appropriate action?
I-offer private counselling area
II-check or verify medication
III-if there is dispensing error, apologize and correct it
A) I only B) III only C) I and II D) II and III E) I, II, III

4) In case of recall products, what is first thing to do?


A-Call all patient still using it and inform about recall
B-Inform the manager follow procedure for recall product
C-Post a sign in the pharmacy regarding recall product.
D-Inform doctor to stop dispensing this product in the future
E-Discard all product in waste container

5) Angry person walks in your pharmacy and complains that you have dispensed wrong
medications? What is appropriate first step in resolving this problem?
A-Apologise patient for dispensing error
B-Calm down patient
C-Acknowledge his anger, ask and verify what is error
D-Ignore and let him vent his anger
E-Acknowledge his anger and refer to doctor

Copyright © 2000-2016 TIPS Inc. Unauthorized reproduction of this manual is prohibited. This manual is 59-1
being used during review sessions conducted by PharmacyPrep.
PharmacyPrep.Com Medication Errors

6) If dispensing error occur in pharmacy. Who are the responsible?


I-Doctor
II-Pharmacist
III-Pharmacy Technician
A) I only B) III only C) I and II D) II and III E) I, II, III

7) If prescription error occurred in pharmacy on phone. Who are the responsible?


I-Doctor
II-Pharmacist
III-Pharmacy Technician
A) I only B) III only C) I and II D) II and III E) I, II, III

8) Angry customer comes to your pharmacy. Says dispensing error occurred, which of the
following first step should be done?
A) Apologize customer
B) Calm down and verify prescription error
C) Calm down angry customer
D) Apologize and return new drug
E) Refuse to accept any error

9) Which of the following should first checked by the pharmacist to minimize the dispensing
error?
A) Patient name B) Dr name C) DIN D) Date of birth E) Drug name

10) A patient comes to your pharmacy and complains that he got 10 less tablets of oxycontin
for his prescription. What is the better strategy to minimize these errors.
A) always count tablets in front of patient
B) double or triple count in pharmacy before dispense
C) if some patient complains just give extra tablets
D) narcotics cannot be dispensed extra tablets
E) Check inventory and reassure patient we double check

11) Which of the following is the best method of minimizing calculations errors in
preparations?
A) Choose a person has multiple years of experience
B) Choose a inexperience a new pharmacy graduate
C) Ask a pharmacist or technician to double check the calculation before preparation
D)The calculations should be performed by calculators
E) None of the above

12) Which of the following medication error would not risk harm?
A)Wrong drug B) Wrong instruction C)minor spelling mistakes D)inappropriate auxiliary
labels E) expired medications

Copyright © 2000-2016 TIPS Inc. Unauthorized reproduction of this manual is prohibited. This manual is 59-2
being used during review sessions conducted by PharmacyPrep.
PharmacyPrep.Com Medication Errors

13) A pharmacy dispensing error dispensed Pradox (dabigatran) instead of Plavix (clopidogrel).
If patient used. Which of the following problem expected?
A) Increase risk myocardial infarction
B) Increase risk of platelet aggregation
C) Increase risk of bleeding
D) Increase risk of blood clot formation

14) If patient used Lasix (furosemide) instead of Losec (omeprazole). What problem is
expected?
A) hyperglycemia B) hyperthyroidism C)hypotension D)Hypercalcemia E)None

15) A pharmacy technician prepared a prescription of Pradox instead of Plavix. What is the
appropriate strategies to prevent this error.
I) Advise technician to change to Plavix (clopidogrel)
II) Educate technician on drug name sound similar
III) Read label to patient while dispensing
A) I only B) III only C) I and II D) II and III E) I, II, III

16) A pharmacy intern receive a phone call from doctor for a new prescription. Dr want to
prescribe Losec. However pharmacy intern documented as Lasix. What is appropriate to
minimize this error?
A) Only pharmacist should take phone call for a new prescription
B) Pharmacy intern should require more training on sound like name drugs
C) Ask doctor to give generic name for the sound like name drugs
D) Ask doctor to fax prescription for sound like name drugs
E) Ask doctor to written prescription for all new prescriptions

17) Which of the following is NOT dangerous drug abbreviation?


A) od
B) qid
C) daily
D) IU
E) qd

18) What is recommended to write instead of "IU"


A) iu
B) units
C) international units
D) IU
E) I.units

19) What is the best recommendation to write prednisone and prednisolone to avoid errors.
A) prednisone and prednisolone
B) predniSONE and predniSOLONE

Copyright © 2000-2016 TIPS Inc. Unauthorized reproduction of this manual is prohibited. This manual is 59-3
being used during review sessions conducted by PharmacyPrep.
PharmacyPrep.Com Medication Errors

C) predNISone and predNISOLone


D) PREDNISONE and prednisolone
E) predsnisone and PREDNISOLONE

20) What is correct tallman letters for diphenhydramine and dimenhydrinate


A) dimenHYDRINATE and diphenHYDRAMINE
B) diMENhydrinate and diPHENhydramine
C) DIMENHYDRINATE and diphenhydramine
D) dimenhydrinate and DIPHENHYDRAMINE

21) Which of the following abbreviation least likely cause error?


A) q.d B) QD C) qid D) q6h E) I.U

22)What agencies in Canada are associated in reporting medication incidents?


A) Institute of safe medication practices (ISMP)
B) Health Canada’s “Med effect” program
C) Canadian Coalition for Medication Incident Reporting
D) All of the above

23. The practice of writing part of a drugs name in upper letter to help distinguish?
A. Dispensing error of generic and brand name drugs
B. Packaging error of medication
C. Dose errors of drugs
D. To prevent incidents of Sound alike and look alike drugs
E. drugs name with same first and last letter

24. Health Canada MedEffect program is related to?


A. adverse drug reaction monitoring
B. advisories, warnings and recalls
C. Safer health for Canadians
D. To develop actions and strategies to prevent incidents
E. National System for incident reporting

Copyright © 2000-2016 TIPS Inc. Unauthorized reproduction of this manual is prohibited. This manual is 59-4
being used during review sessions conducted by PharmacyPrep.
PharmacyPrep.Com Medication Errors

Medication Errors
ANSWERS:

1. Ans: D

2- Ans: D

3) Ans: E

4) Ans: B

5) Ans: C

6) Ans: D
Tips: Vicarious liability of pharmacist of dispensing

7) Ans: C
Tips: pharmacist should contact doctor to correct prescription.

8) Ans: B

9) Ans: A

10) Ans: B

11) Ans: C

12) Ans: C

13) Ans: C
Tips: Pradox is a new anticoagulant dabigatran act as factor Xa inhibitor. However Plavix is
clopidogrel an antiplatelets drugs. So increase risk of bleeding

14) Ans: C
Tips: Lasix is furosemide can cause hypotension. Losec is omeprazole.
Immodium (Loperamide) and Motilium (domperidone)
Dimenhydrinate and diphenhydramine
Cyclosporine and Cyclophosphomide
Proscar and Prozac
Lamisil (terbinafine) and Lamictal (lamotrigine)

15) Ans: E

Copyright © 2000-2016 TIPS Inc. Unauthorized reproduction of this manual is prohibited. This manual is 59-5
being used during review sessions conducted by PharmacyPrep.
PharmacyPrep.Com Medication Errors

16) Ans: C

17) Ans: C

18) Ans: B

19) Ans: C
Tips: Tallman letter can be used to avoid error

20) Ans: B

21) Ans: D

22 Ans:) D

23. Ans: D

24. Ans: B

Copyright © 2000-2016 TIPS Inc. Unauthorized reproduction of this manual is prohibited. This manual is 59-6
being used during review sessions conducted by PharmacyPrep.
www.pharmacyprep.com Sterile Preparations

PHARMACY PREP

Health Promotion and Disease Prevention

1. Pharmacist offering immunization is categorized as?


A. Primary care
B. Secondary care
C. Tertiary care
D. Quaternary care
E. All of the above
Ans. A
Tips. Levels of health care delivery systems. It can be categorized as primary, secondary
and tertiary health care delivery systems.
Primary care: This is first contact a person makes a with the system when a person feels
the necessity of health care. This usually occurs through the family physician,
pharmacist, or nurse at medical centers.

Secondary care: This is specialized service from a specialist. This requires referral from
primary health care levels.

Tertiary care. This is specialized in diagnosing and highly technical care and treating
complicated or unusual health problems. This generally takes place in hospital setting
where generally diagnostic and complicated therapies can take place.

2. What is an example of activities are designed to completely prevent a disease which is


categorized as primary level of health promotion activities?
A. use of beta blockers to help remodel the heart in congestive heart failure
B. colonoscopy to detect small cancerous polyps
C. Immunization against pneumonia or influenza
D. Prophylaxis antibiotics for endocarditic
ans. c

3. Sunscreen protect?
A. UVA
B. UVB
C. UVC
D. All of the above
Ans. B
Tips. UVA aging, UVB burns, UVC is safe

Copyright © 2000-2016 TIPS Inc. Unauthorized reproduction of this manual is prohibited. This 60-1
manual is being used during review sessions conducted by PharmacyPrep.
www.pharmacyprep.com Sterile Preparations

Health Promotion and Disease Prevention


Answers

1. Ans. A

Copyright © 2000-2016 TIPS Inc. Unauthorized reproduction of this manual is prohibited. This 60-2
manual is being used during review sessions conducted by PharmacyPrep.
www.pharmacyprep.com Sterile Preparations

PHARMACY PREP
Collaboration and Teamwork

1. Pharmacist identify a potential drug related problem for patient and then provide the
prescriber with a clinical recommendation to resolve the problem, is described as?
A. Advancing
B. Adaptation
C. Pharmaceutical opinion
D. Beneficence
E. Regulations
Ans.c

2.Main goal of academic detailing is?


A. Advancing prescription
B. Providing pharmaceutical opinion
C. Enhancing prescription practices
D. Pharmaceutical care
E. Counseling patient
Ans. c

3.Unnecessary use or high dose of antibiotics cause?


A. Tolerance
B. Resistance
C. Dependency
D. Addiction
E. Ineffective therapy
Ans. b

4.A doctor switched patient to a new opioid therapy and started low dose. It is due to?
A. Tolerance
B. Resistance
C. Dependency
D. Addiction
E. Ineffective therapy
Ans. a

5. nurse practitioner can prescribe?


A. injections
B. narcotics
C. inhalational
D. all of the above
ans.d

Copyright © 2000-2016 TIPS Inc. Unauthorized reproduction of this manual is prohibited. This 61-1
manual is being used during review sessions conducted by PharmacyPrep.
www.pharmacyprep.com Sterile Preparations

6. a nurse practitioner prescribed sitagliptine 50 mg daily for diabetic patient. What is


appropriate?
A. process, dispense and document
B. tell patient to contact to doctor to get a new prescription
c. call nurse and inform this is not in the scope of nurse practice
D. complain of college of nursing
ans. a

Copyright © 2000-2016 TIPS Inc. Unauthorized reproduction of this manual is prohibited. This 61-2
manual is being used during review sessions conducted by PharmacyPrep.
www.pharmacyprep.com Sterile Preparations

PHARMACY PREP
STERILE PREPARATIONS

7) What type of laminar flow hood is used in preparation of cytotoxic (neoplastic)


products?
A-Horizontal laminar airflow hood
B-Vertical Laminar airflow hood
C-Vertical and Horizontal laminar airflow hood
D-None of the above
Ans-B

8) What size of filters is used in water sterilization?


A-0.22 mm B-0.33 mm C-0.11 mm D-0.44 mm E-0.55 mm
Ans-A

9) What sizes of HEPA filters are used in laminar airflow hood that is used for parenteral
preparations?
A) 0.22 mm B-0.33 mm C-0.11 mm D) 0.44 mm E) 0.55 mm
Ans-B

40) What is incorrect about hypodermic needle in sterile


preparations?
A) Avoid touching hub and plunger of needle
B) Avoid touching bevel of needle
C) gauge is diameter of syringe shaft
D) Leur-lock syringes are used in cytotoxic preparation
E) All parenteral preps are have to manipulate in laminar airflow
hood
Ans. C

Tips: gauge is diameter of needle.

Copyright © 2000-2011 TIPS Inc. Unauthorized reproduction of this manual is prohibited. This 44-1
manual is being used during review sessions conducted by PharmacyPrep.
www.pharmacyprep.com Sterile Preparations

Which of the following correct size of HEPA filters are


used in laminar airflow hood in sterile preps?
A) 0.22 microns
B) 0.22 micrometer
C) 0.33 microns
D) 0.33 micrometers
E) none of the above

Copyright © 2000-2011 TIPS Inc. Unauthorized reproduction of this manual is prohibited. This 44-2
manual is being used during review sessions conducted by PharmacyPrep.
PharmacyPrep.Com Pharmaceutical Storage Conditions

PHARMACY PREP
PHARMACEUTICAL STORAGE CONDITIONS

1- You received a new drug in your pharmacy and the manufacture instructions tell you to
keep this drug in a cool place. Which temperature should you keep this drug?
a) <0 C b) 2 to 8 C c) 8 to 15 C d) 15 to 30 C e) >30 C

2-All of the following antibiotics extemporaneous suspensions are stored at room


temperature, EXCEPT
A-Metronidazole B-Clarithromycin C-Azithromycin D-Clindamycin
E-Cotrimoxazole

3) Insulin is stored at?


A-Room temperature
B-Refrigerator
C-Freezer
D-Deep freezer
E-Cool temperature

4) All vaccines should be stored in?


A-Room temperature
B-Refrigerator
C-Freezer
D-Deep freezer
E-Cool temperature

5) What is the optimal temperature for vaccine fridge is?


A. 5 °C
B. 0°C
C. 1 °C
D. 2°C
E. 3°C

6) When should pharmacy staff defrost refrigerator?


A) More than 10 cm ice in the freezer compartment
B) More than 5 cm ice in the freezer compartment
C) More than 1 cm ice in the freezer compartment
D) It should be never defrosted
Ans. C

7. Storage of Dukoral should be:


A. At room temp
B. In the fridge
C. Away from light

Copyright © 2000-2016 TIPS Inc. Unauthorized reproduction of this manual is prohibited. This manual is 63-1
being used during review sessions conducted by PharmacyPrep.
PharmacyPrep.Com Pharmaceutical Storage Conditions

D. In a safe cabinet
Ans (B)
Tips: It is a vaccine.

8. All the drugs and supplies use in the sterile room should be unpack ?
A. After bringing into the sterile room
B. Before brining into the sterile room.
C. Anytime and anywhere can be unpacked
D. Manufacturer should not pack supplies that are used in sterile preps
Ans. B

9. The temp of the fridge in the pharmacy should be recorded at least


A. Once per day
B. Twice per day
C. Three times per day
D. Once every other day
Ans. (B)
Tips: It should be recorded once in the morning and once in the night

PHARMACEUTICAL STORAGE CONDITIONS


ANSWERS:

1- Ans: C

2- Ans: A

3- Ans: B

4- Ans: B

5. Ans: A

Copyright © 2000-2016 TIPS Inc. Unauthorized reproduction of this manual is prohibited. This manual is 63-2
being used during review sessions conducted by PharmacyPrep.
PHARMACY PREP

PHARMACEUTICAL CARE AND DRUG RELATED PROBLEMS

1. What is the most important focus of pharmaceutical care?


A. The pharmacist
B. The patient
C. The prescription
D. The patient chart
E. Pharmacist salary

2. A 60-year-old patient with congestive heart failure who has been stabilized for 3
months on digoxin, furosemide, and potassium chloride is gradually placed on the
following additional medicines. Which of these drugs is most likely to cause a problem if
added to patient treatment?
A. Quinidine
B. Temazepam (restoril)
C. Meperidine hci (demerol)
D. Aspirin
E. Nitroglycerin

3. What is NOT a requirement of pharmaceutical care?


A. Prevention of disease
B. Cure of disease
C. Decrease of symptoms
D. Slowing of disease progression
E. Dispensing the cheaper drug

4. Which of the following does NOT included in drug related problems?


A. Adverse effects of drugs
B. Improper storage conditions
C. Receiving too little drug
D. Receiving inappropriate drug for condition
E. Dispensing error

5. A frequent customer of your pharmacy medical profile includes; metaprolol 50 mg,


metformin 500 mg tid, allopurinol 50 mg, ASA 81 mg daily for the past 3 yrs. Recently
doctor have started atorvastatin 40 mg daily. Which of the following drug may give
stomach upset?
A. Metaprolol
B. Metformin
C. Allopurinol
D. Atorvastatin
E. ASA

Copyright © 2000-2016 TIPS Inc. Unauthorized reproduction of this manual is prohibited. This 64-1
manual is being used during review sessions conducted by PharmacyPrep.
6. A doctor has prescribed salbutamol inhaler prn for stuffy nose 3 months ago.
However, patient experiencing tremor and shaky for past 6 days. What to do?
A. Increase dose of salbutamol
B. Decrease dose of salbutamol
C. Ask patient how often use salbutamol puffs
D. Refer to doctor
E. Refer to emergency

7. A 65 yo COPD patient using ipratropium PRN inhalers and tiotropium once daily.
Complains dry mouth and dry eye. What to do?
A. refer to doctor
B. recommend use frequent sip of water or ice chips C)recommend to use artificial eye
drops
D. refer to emergency
E. B and C

8. A hypertensive patient who has COPD and who is non compliant would be best
treated with which of following beta blockers?
A. Propranolol B. Atenolol C. Esmolol D. Timolol

9. A regular customer of your pharmacy brings a prescription of Advair (salmeterol +


fluticasone) bid. Currently patient profile has salbutamol prn and fluticasone bid. What
is appropriate action to do?
A. Counsel patient on how to use Advair device
B. Counsel patient to space Advair from fluticasone
C. Ask patient if doctor want him to discontinue fluticasone
D. Ask patient to discontinue fluticasone, as the same fluticasone is present in
combination product (Advair)
E. Ask patient if his asthma is uncontrolled

10. A doctor has prescribed oxycontin for prn 7days after appendectomy surgery. Which
of the following appropriate products to recommend?
A. mineral oil B. psyllium C. sennakot/stool softener D. stool softener only
E. Magnesium antacids

11. A pregnant women diagnosed UTI, and she allergic to penicillin, what is the drug of
choice?
A. Nitrofurontoin B. Fosphomycin C.Cephalosporin's D.Tetracycline E. Ciprofloxacin

12. A patient is using tetracycline for the treatment of acne. What is the best
counseling?
A. take with glass of water B. take with glass of milk C. must take empty stomach D.
take with glass orange juice E. avoid taking with antacids

Copyright © 2000-2016 TIPS Inc. Unauthorized reproduction of this manual is prohibited. This 64-2
manual is being used during review sessions conducted by PharmacyPrep.
13. A patient is medication shows ramipril, nadolol, ASA and rosuvastatin. What are the
adverse effect may be related to ramipril?
A. bradycardia, bronchospasm, cardiac decompensation
B. skin rash, and proteinuria
C. postural hypotension, fever, positive Coombs' test
D. First dose syncope, postural hypotension, and palpitation

14. A patient is treated with hydrochlorothiazide 50 mg daily for uncomplicated


hypertension should be monitored regularly for altered plasma levels of?
I) increased levels of uric acid
II) increase levels of glucose
III) decreased level of potassium
A. I only B. III only C. I and II only D. II and III only E. All are correct

15. A patient had ileostomy surgery, which of the following dosage form is not used?
A. immediate release
B. sustain release
C. oral dosage form
D. intra venous
E. sublingual

Copyright © 2000-2016 TIPS Inc. Unauthorized reproduction of this manual is prohibited. This 64-3
manual is being used during review sessions conducted by PharmacyPrep.
PHARMACEUTICAL CARE AND DRUG RELATED PROBLEMS
ANSWERS

1. Ans: B

2. Ans: A
Tips: Although digoxin and quinidine may be used together, it is well documented that
administering quinidine to a patient previously stabilized on digoxin will cause serum
digoxin levels to rise an average 2 to 2.5 fold. The mechanism of this interaction may
involve both a displacement of digoxin from tissue-binding sites and a reduction in renal
clearance of digoxin. Even though the significance of this interaction remains
controversial, many clinicians suggest reducing the dose of digoxin by 50% when adding
quinidine. In any case, the patient should be monitored carefully for signs of digoxin
toxicity.

3. Ans: B

4. Ans: E

5. Ans: D

6. Ans: C

7. Ans: E

8. Ans: B
Tips: Cardio selective BBs "EMAA". Esmolol is iv infusion. Acebutolol and Atenolol are
long acting once daily. Timolol is long acting but not cardio selective

9. Ans: C

10. Ans: C
Tips: stimulants laxative like senna, bisacodyl are effective to treat constipation caused
by opioids.

11. Ans: A

12. Ans: C

13. Ans: B

14. Ans: E
Tips: Thiazide cause "hyperGLUC"
15. Ans: C

Copyright © 2000-2016 TIPS Inc. Unauthorized reproduction of this manual is prohibited. This 64-4
manual is being used during review sessions conducted by PharmacyPrep.
www.pharmacyprep.com Adverse Drug Reactions

PHARMACY PREP

ADVERSE DRUG REACTIONS

1. Which of the following drugs give severe rebound hypertension?


A. prazocin B. propranolol C. clonidine D. captopril E. enalapril

2. All of the drug cause orthostatic hypotension, except:


A. propranolol B. prazocin C. captopril D. doxazocin E. enalapril

3. Neuroleptic malignant syndrome side effects is not associated with?


A. haloperidol B. Levodopa C. clozapine D. Chlorpromazine
E. Risperidone

4. Nitroglycerine patch comes off during shower, what is suitable action?


A. refer to the physician
B. put another patch
C. reapply the same patch
D. use glue and reapply the same patch
E. None of the above

5. Which of the following drug should not be taken if it is not effective with first dose?
A. Nitro glycerine spray
B. Nitro-glycerine SL
C. Beta blockers
D. Nifedipine
E. Sumatriptan

6. Drugs that gives bronchospasm?


A. Salbutamol B. acebutolol C. Salmeterol D. Formoterol
E. Ipratropium

7. Drugs that alter TSH levels?


A. Lithium
B. Amiodarone
C. Levothyroxin
D. L-thyroxin
E. All of the above

8. All of the following drugs can cause weight gain except?


A. Fluoxetine
B. Paroxetine
C. Risperidone
65-1
Copyright © 2000 - 2016 TIPS Inc. Unauthorized reproduction of this manual is prohibited. This manual is
being used during review sessions conducted by PharmacyPrep.
www.pharmacyprep.com Adverse Drug Reactions

D. Quetiapine
E. topiramate

9. Which of the following drugs priapism side effects, patient should report to doctor?
A. Trazodone
B. Sildenafil
C. fluoxetine
D. Alprostadil
E. All of the above

10. All of the following drugs can give pulmonary fibrosis, except?
A. Bromocriptine
B. Cisapride
C. Amiodarone
D. Ropinirole
E. Methotrexate

11. Headache is side effects of?


A. Amlodipine B. Sumatriptan C. Acetaminophen D. Sildenafil
E. Fluoxetine

12. Venous pooling is associated with?


A. Statins B. Triptans C. Sodium nitroprusside D. Verapamil
E. Beta blockers

13. Black stool and tongue can cause by?


A. Loperamide
B. Bismuth sub-salicylates
C. Ciprofloxacin
D. Tetracycline
E. Erythromycin

14. The rare but serious side effect of clindamycin is?


A. Constipation
B. Diarrhea
C. Stomach upset
D. Nausea and vomiting
E. Bloating

15. The most common side effect of erythromycin?


A. Constipation
B. Diarrhea
C. GI discomfort
D. Nausea and vomiting
65-2
Copyright © 2000 - 2016 TIPS Inc. Unauthorized reproduction of this manual is prohibited. This manual is
being used during review sessions conducted by PharmacyPrep.
www.pharmacyprep.com Adverse Drug Reactions

E. Bloating

16. Phenytoin is associated with?


A. Gingival hyperplasia
B. Diarrhea
C. GI irritation
D. Nausea and vomiting
E. Bloating

17. Neutropenia is side effect of?


A. Clopidogrel
B. ASA
C. Ticlopidine
D. Warfarin
E. LMWH

18. Angioedema is associated with?


A. ACE I
B. ARBs
C. CCBs
D. A and B
E. Beta blockers

19. Lactic acidosis is associated with?


A. metformin
B. Chlorpropamide
C. Gliclazide
D. Roziglitazone
E. Acarbose

20. Torse de pointes is associated with?


A. Amiodarone
B. quinidine
C. Procainamide
D. all of the above

21. What is most common side effects of SSRIs?


A. Nausea and vomiting
B. Sexual dysfunction
C. Sedation
D. GI upset
E. weight gain

22. Bupropion side effects include?


65-3
Copyright © 2000 - 2016 TIPS Inc. Unauthorized reproduction of this manual is prohibited. This manual is
being used during review sessions conducted by PharmacyPrep.
www.pharmacyprep.com Adverse Drug Reactions

A. bulimia nervosa
B. anorexia nervosa
C. sexual dysfunction
D. weight gain
E. A and B

23. A 50 year old female currently she is on atorvastatin, and atenolol and Tylenol # 1 for
arthritic pains. She visit pharmacy complaining of diarrhea, which of the following medication
may associated to her problem:
A. Atenolol
B. Codeine in Tylenol
C. Tylenol
D. Atorvastatin
E. All of the above

24. All of the following drugs may require WBC or CBC monitoring, except?
A. clozapine B. ticlopidine C. levothyroxine D. propylthiouracil E. methimazole

25. All of the following drugs give constipation side effect, except?
A. Atropine B. Ipratropium C. Tiotropium D. Scopolamine E. Donepezil

26. A 45 yo person using ramipril 10 mg daily for the treatment of hypertension. However,
hypertension was not controlled, and now doctor have added prazosin 10 mg daily. What is
the pharmacist concern?
A. potassium levels
B. low blood pressure
C. side effects of prazosin
D. drug interactions
E. patient age

27. A 72 yo person on atenolol and ASA 325mg, clopidogrel 75 mg, therapy. Recently
diagnosed with hypothyroidism. Doctor has recommend low dose, levothyroxine 12.5 mcg
daily dose. Why is the low dose given?
I)cardiovascular disease II)age III)drug interactions with atenolol, ASA and clopidogrel
A. I only B. III only C. I and II D. II and III E.I,II, III

28. A customer of your pharmacy using sumatriptan for migraine headache. However, this
person headache is not relieved after using sumatriptan. What is appropriate to do?
A. double the dose of sumatriptan
B. wait for 2 hour and try again sumatriptan
C. add other triptan with sumatriptan
D. Decrease dose and gradually increase the dose
E. Do not use sumatriptan

65-4
Copyright © 2000 - 2016 TIPS Inc. Unauthorized reproduction of this manual is prohibited. This manual is
being used during review sessions conducted by PharmacyPrep.
www.pharmacyprep.com Adverse Drug Reactions

29. A 65 yr old Parkinson's patient is on levodopa/carbidopa therapy. Experiencing nausea and


vomiting. What is incorrect?
A. treat nausea vomiting using dimenhydrinate
B. treat nausea vomiting using metoclopramide
C. Recommend vitamin B 6 supplement
D Recommend Diclectin (vitamin B 6 + doxylamine)
E. Recommend ondansetron a serotonin antagonist

30. A 72 year old regular customer of your pharmacy brings a new prescription of
levothyroxine 12.5 mcg daily. Recently patient is discharged from hospital after MI. Dr
prescribed low dose of levothyroxine because?
I) Myocardial infarction
II) Age 72 yr
III) Regular customer of your pharmacy
A. I only B. III only C. I and II D. II and III E.I,II, III

31. A 55 yo man using warfarin 5 mg daily to treat deep vein thrombosis. Recently doctor
prescribed metoprolol 50 mg bid to treat hypertension. What is the pharmacist concern?
A. high dose of warfarin
B. deep vein thrombosis at age of 55 y
C. Metoporolol is a beta blocker
D. Not a concern
E. INR monitoring is required

32. Which of the following therapy is pharmacist concern?


A. Patient using methotrexate but not receiving folic acid supplements
B. Patient in pregnancy using carbamazepine but not receiving folic acid supplements
C. Patient using isoniazid not receiving vitamin B 6
D. Patient using levodopa and using vitamin B 6 supplements
E. All of the above

33. Which of the following drug pharmacist should counsel to stop taking and contact doctor
immediately if you feel suicidal ideas?
A. Haloperidol
B. Mefloquine
C. Lisinopril
D.Hydrochlorothiazide
E. Isoniazid
Ans. B

34. What agency in Canada issue warning related adverse reactions for drugs in post
marketing surveillance?
A. Health Canada's, Med Effect
B. Institute of Safe Medication Practices Canada

65-5
Copyright © 2000 - 2016 TIPS Inc. Unauthorized reproduction of this manual is prohibited. This manual is
being used during review sessions conducted by PharmacyPrep.
www.pharmacyprep.com Adverse Drug Reactions

C. Canadian Institute of Health Information


D. Canadian Pharmacist Association
ans. A

35. Health professionals, Adverse reactions detected through cases should report to?
A. Canada vigilance program
B.ISMP
C. CIHI
D. C.PhA

36. Which of the following is NOT extra pyramidal symptoms?


A. Tardive dyskinesia B. Rigidity C. Tremor D. Dystonia E. Vivid dreams

37. Which of the following is NOT require to monitor chest X-ray ?


A. Methotrexate b. Amiodarone C. Bleomycin D. Bromocriptine E. Clozapine
Ans. E

38. What is used for treating constipation in children


a. Mineral oil B. Sennkot C. Psyllium D. Polyethylene glycol E. sucralfate
Ans.D

ADVERSE DRUG REACTIONS


ANSWERS:

1. Ans: C
65-6
Copyright © 2000 - 2016 TIPS Inc. Unauthorized reproduction of this manual is prohibited. This manual is
being used during review sessions conducted by PharmacyPrep.
www.pharmacyprep.com Adverse Drug Reactions

2. Ans: A

3. Ans: B

4. Ans: C

5. Ans: E

6. Ans: B

7. Ans: E

8. Ans: E

9. Ans: B

10. Ans: D

11. Ans: A

12. Ans: C

13. Ans: B

14. Ans: B

15. Ans: C

16. Ans: A

17. Ans: C

18. Ans: D

19. Ans: A

20. Ans: D

21. Ans: A

22 Ans:. E

23. Ans: D
65-7
Copyright © 2000 - 2016 TIPS Inc. Unauthorized reproduction of this manual is prohibited. This manual is
being used during review sessions conducted by PharmacyPrep.
www.pharmacyprep.com Adverse Drug Reactions

24. Ans: C

25. Ans: E
Tips: all anticholinergic drug gives constipation side effect. However donepezil is
anticholinesterase (agonist) drug.

26. Ans: B

27. Ans: C
Tips: low dose levothyroxine is used in seniors and cardiovascular disease patient. However
no interactions with atenolol, ASA and clopidogrel

28. Ans: E
Tips: do not use triptans if no relief with first dose.

29. Ans: B

30. Ans: C

31. Ans: C

65-8
Copyright © 2000 - 2016 TIPS Inc. Unauthorized reproduction of this manual is prohibited. This manual is
being used during review sessions conducted by PharmacyPrep.
www.pharmacyprep.com Adverse Drug Reactions

65-9
Copyright © 2000 - 2016 TIPS Inc. Unauthorized reproduction of this manual is prohibited. This manual is
being used during review sessions conducted by PharmacyPrep.
PharmacyPrep.Com Drug Interactions

PHARMACY PREP

DRUG & DRUG INTERACTIONS

1. A 55 year old male patient receiving warfarin for deep vein thrombosis, he get the
prescription of carbamazepine; which of the following drug interaction possible:
I-Carbamazepine decrease metabolism of warfarin
II-Carbamazepine is has not interaction with warfarin
III-Carbamazepine increase metabolism of warfarin
A. I only B. III only C. I and II only D. II and III only E. All of the above

2. A 20 year old female patient currently on fluoxetine for depressive disorder, she get
the prescription of Tylenol # 3 for sever headache. Which of the following drug
interaction are possible:
A. Fluoxetine metabolism increases
B. Codeine metabolism increases
C. Fluoxetine metabolism decrease
D. Codeine metabolism decrease
E. No change and there is no drug interactions

3. A 40-year old female is regular customer of your pharmacy, her current medical
history include lovastatin. She brings prescription for her recent condition, Which of the
following medication is safe for her medical profile:
A. Erythromycin
B. Ketoconazole
C. Grapefruit juice
D. Niacin
E. Hydrochlorothiazide

4. Enzyme complex not working as a drug-metabolizing enzyme may include:


A. Cytocrome P-450
B. Cytocrome P450 3A4
C. Alcohol Dehidrogenase
D. UDP Glucoronyl Transferase
E. Cytocrome oxidase.

5. What is incorrect about tetracycline antibiotics:


I-Take with water and milk
II-Take with food
III- Must take empty stomach
A. I only B. III only C. I and II only D. II and III only E. All of the above

6. Tetracyclin binds with?

Copyright © 2000-2016 TIPS Inc. Unauthorized reproduction of this manual is prohibited. This 66-1
manual is being used during review sessions conducted by PharmacyPrep.
PharmacyPrep.Com Drug Interactions

A. Mono valent ions


B. Bi valent ions
C. Tri valent ions
D. B and C only
E. All of the above

7. Ciprofloxacin should be taken with:


A. Avoid taking with full glass of milk
B. Take with or after food
C. Take empty stomach
D. Take with or without food
E. A and D

8) A patient using calcium supplements 500 mg tid. Now gets the prescription of
ciprofloxacin 500 mg bid for 3 days. What is appropriate method of administration?
A) Take Ca supplements and ciprofloxacin together morning and evening.
B) Take Ca supplements before 2 hour and after 4 hours of ciprofloxacin.
C) Stop taking Ca supplements for 3 days and continues ciprofloxacin
D) Stop taking ciprofloxacin and continue Ca supplement daily.
E) Call the doctor, change to levofloxacin.

9. Which of the following statement is incorrect about alcohol and drug interactions?
A. Alcohol and Metronidazole can give disulfiram reactions
B. Alcohol and metformin can give lactic acidosis
C. alcohol and chlorpropamide an antidiabetic drug can give lactic acidosis
D. alcohol and benzodiazepine can give sedation

10. A patient using Sildenafil 50mg should not take nitrates, because?
A. It can cause hypotension
B. It can cause hypertension
C. It can cause angina
D. It can cause congestive heart failure
E. None of the above

11. All of the following agent can cause urine discoloration, EXCEPT;
A. Metronidazole
B. Rifampin
C. Sulfasalazine
D. pyrivinium pamoate
E. tetracycline

12. Iron supplement should be separated 2 to 4 hrs from all, except?


A. Thyroxin B. Tetracycline C. Cholestyramine D. Atorvastatin E. None
13. What type of mechanism for drug interactions of warfarin and antibiotics?

Copyright © 2000-2016 TIPS Inc. Unauthorized reproduction of this manual is prohibited. This 66-2
manual is being used during review sessions conducted by PharmacyPrep.
PharmacyPrep.Com Drug Interactions

A. Altering GI flora
B. altering prothrombin time
C. increasing bleeding time
D. Increasing warfarin metabolism

14. A patient has migraine 3 to 4 times a months. Currently also diagnosed with benign
prostatic hyperplasia. Doctor has prescribed prazosin. Medical profile also includes
constipation. What drug is recommendation are suitable for migraine prophylaxis?
A. Amitriptyline
B.Verapamil
C. Propranolol
D. Sumatriptan
E. Nortriptyline

15. What over the counter drugs should be avoided with MAOIs?
A. Pseudoephedrine
B. Xylometazoline
C. Dextromethorphan
D. Food containing Tyramine
E. All of the above

16. Calcium supplement should separated 2 to 4 hrs from all, except?


A. Ciprofloxacin
B. Tetracycline
C. Alendronate
D. Thyroxin
E. Acetyl salicylic acid

17. All of the following are pharmacodynamic reactions, except?


A. Release of 5-ASA from sulfasalazine
B. Diuretic and insulin actions
C. Sedative with alcohol gives sedation
D. None of the above

18. A 60-year-old women-taking calcium supplement 500 mg tid. Recently doctor


prescribed ciprofloxacin 500 mg tid for 3 days. What is appropriate recommendation?
A. Take calcium supplement with 2 hours apart from ciprofloxacin
B. Skip calcium supplement during ciprofloxacin therapy
C. Skip ciprofloxacin and just take calcium supplements
D. Recommend other antibiotics such as ofloxacin
E. Calcium and ciprofloxacin can be combined

19. Tetracycline binds with?

Copyright © 2000-2016 TIPS Inc. Unauthorized reproduction of this manual is prohibited. This 66-3
manual is being used during review sessions conducted by PharmacyPrep.
PharmacyPrep.Com Drug Interactions

I-antacids
II-bi valent ions
III-tri valent ions
A. I only B. III only C. I and II only D. II and III only E. All of the above

20. A patient using atorvastatin 80 mg daily, also wants to use daily 1 glass of grapefruit
juice, what is pharmacist concern?
A. grapefruit juice induce CYP3A4
B. grapefruit juice inhibit CYP3A4
C. grapefruit juice is substrate for CYP3A4
D. grapefruit juice is essential for statin metabolism
E. grapefruit juice may decrease cholesterol

21. A patient using sildenafil 50 mg, should be cautious taking high dose of take the
following antihypertensive drugs
A. Amlodipine B. Propranolol C. Hydrochlorothiazide D. Prazosin
E. Ramipril

22. Which of the following is incorrect?


A. Insulin plus physical exercise increase risk of hypoglycemia
B. Insulin plus alcohol intake increase risk of hypoglycemia
C. Insulin plus glyburide increase risk of hypoglycemia
D. Insulin plus alcohol increase risk of lactic acidosis
E. Insulin overdose can cause sweating, confusion and palpitation

23. A 60 year old women taking calcium supplement bid. Recently doctor prescribed
alendronate 70 mg/wk. What is appropriate recommendation?
A. Take together calcium supplements and alendronate 70 mg once weekly
B. Do not take calcium supplements during the therapy of alendronate
C. Separate calcium supplements 2 to 4 hours from the dose of alendronate
D. Do not use alendronate while taking calcium supplement therapy

24. A 56 yo female using levothyroxine 25 mcg daily and metformin 500 bid. She comes
to buy Calcium supplements to prevent osteoporosis. What is true statement?
A. levothyroxine, calcium supplement should be separated for 4 hours so take
levothyroxine empty stomach first thing in the morning.
B. avoid taking calcium supplements because she is using levothyroxine
C. Separate Ca supplement 4 hour from levothyroxine
D. take Ca supplement in morning and take levothyroxine in evening
E. Never take levothyroxine, metformin and calcium supplement together

25. A 56 yo female using metformin 500 bid, and ciprofloxacin 500 mg tid for 3days. She
comes to buy calcium supplements 1500 mg tid to prevent osteoporosis. What is true
statement?

Copyright © 2000-2016 TIPS Inc. Unauthorized reproduction of this manual is prohibited. This 66-4
manual is being used during review sessions conducted by PharmacyPrep.
PharmacyPrep.Com Drug Interactions

A. recommend to separate Ca supplement 4 hour after ciprofloxacin


B. recommend to take ciprofloxacin tid, metformin bid, and Ca supplement tid as doctor
prescribe
C. Recommend to start Ca supplement after she finish taking Ciprofloxacin
D. Never combine ciprofloxacin and Ca supplements
E. All of the above

26. A customer of your pharmacy enquiring that can she use herbal therapy like
grapefruit juice to treat her borderline high cholesterol, She using atorvastatin for
cholesterol therapy. what is appropriate to do?
A. talk to your doctor
B. definitely she can use grapefruit juice
C. She should start taking statins and avoid grapefruit juice
D. She should never take grapefruit juice for high cholesterol
E. Give her some information to read about high cholesterol therapy

27. A customer of your pharmacy using allopurinol for the past 10 yrs for hyperurecimia
and currently diagnosed for cancer. A doctor want to initiate azathioprine
chemotherapy. What is appropriate to do?
A. decrease dose of azathioprine
B. decrease dose of allopurinol
C. avoid allopurinol during chemotherapy
D. avoid azathioprine chemotherapy
E. do not change, any medications

28. A customer of your pharmacy profile shows nitroglycerin SL spray prn. He brings
prescription of sildenafil 50 mg prn. What to do?
A. Do not dispense sildenafil 50 mg because it can cause severe hypotension with
nitroglycerin SL
B. Dispense low dose sildenafil 25 mg
C. Dispense sildenafil 50 mg and counsel customer not to combine with nitroglycerin
D. dispense sildenafil 50 mg and counsel not to use nitroglycerin SL in case of chest pain
E. Talk to doctor and inform doctor about patient profile nitroglycerin SL

29. A customer of your pharmacy wants to use St. Johns wart to improve mood.
Currently he is on paroxetine for major depression. What is your opinion?
A. ok to use B. do not use St. Johns wart during paroxetine therapy C. refer to doctor D.
give him some literature of general health E. none of the above

30. Which of the following is the least drug interaction with levothyroxine?
A. Iron supplements
B. Calcium supplements
C. Magnesium supplements
D. Aluminum supplements

Copyright © 2000-2016 TIPS Inc. Unauthorized reproduction of this manual is prohibited. This 66-5
manual is being used during review sessions conducted by PharmacyPrep.
PharmacyPrep.Com Drug Interactions

E. Vitamin D supplements

31. A regular customer, takes a glass of grapefruit juice daily. If medication profile have
amiodarone, atorvastatin, hydrochlorothiazide, amlodipine and ASA 81 mg daily. Which
of the following drug is the least concern?
A. atorvastatin
B. amlodipine
C. hydrochlorothiazide
D. amiodarone

32. Warfarin and carbamazepine drug interactions are categorized as?


A. metabolism inhibitors
B. Pharmacodynamic interaction
C. enzyme inducer
D. Pharmacokinetic interaction
E. Receptor & receptor interactions

33. A regular customer of your pharmacy. Brings a new prescription of sildenafil 50 mg.
Which of the following drugs in patient profile is the least concern?
A. Nitroglycerin SL spray
B. Nitroglycerin SL tablets
C. Nitroglycerin transdermal patch
D. Sodium nitroprusside
E. Warfarin 5 mg tablets

34. If the above patient profile have nitrates, what is the pharmacist concern?
A. priapism
B. double vision
C. headache
D. hypotension
E. mood changes

35. A patient medical profile include, metformin, gliclazide, rosuvastatin, and ASA 81
mg. Occasionally patient uses omega 3, 6,9 and multivitamin supplements. Patient is
chronic alcoholic, renal disease, and liver diseases. Recently patient discharged from
hospital after congestive heart failure. What is the least concern of pharmacist with
respect to metformin?
A. chronic alcoholism
B. renal disease
C. disulfuram like reactions
D. congestive heart failure
E. lactic acidosis

Copyright © 2000-2016 TIPS Inc. Unauthorized reproduction of this manual is prohibited. This 66-6
manual is being used during review sessions conducted by PharmacyPrep.
PharmacyPrep.Com Drug Interactions

36. A patient lifestyle include high proteins diet. Which of the following is the most
concern for the pharmacist?
A. theophylline
B. levodopa
C. renal disease
D. orlistat
E. liver diseases

37. All of the following drugs decrease efficacy of oral contraceptives, except?
A. Rifampin
B. Amoxicillin
C. Phenytoin
D. Gabapentin
E. Carbamazepine
Ans. d
Tips: antibiotics and antiepileptic drugs likely decrease efficacy of oral contraceptive
pills.

38.All of the following drug can be taken on an empty stomach, except


a. levothyroxine
B. Alendronate
C. Omeprazole
D. Metformin
E. Zafirlukast
Ans. D

Copyright © 2000-2016 TIPS Inc. Unauthorized reproduction of this manual is prohibited. This 66-7
manual is being used during review sessions conducted by PharmacyPrep.
PharmacyPrep.Com Drug Interactions

DRUG & DRUG INTERACTIONS


ANSWERS:

1.Ans: B
Tips: Carbamazepine is an inducer of CYP2C9, and the same enzyme is substrate of
warfarin. Therefore Carbamazepine increase metabolism of warfarin

2. Ans: D
Tips: Fluoxetine inhibits CYP 2D6 and this same enzyme is a substrate for codeine and
therefore decrease codeine metabolism

3. Ans: E

4. Ans: C

5. Ans: C

6. Ans: D
Tips: Tetracycline's bind with bi and tri valent ions such as calcium and aluminum ions.

7. Ans: E

8. Ans: C

9. Ans: C

10. Ans: A

11. Ans: E

12. Ans: D

13. Ans: A

14. Ans: C

15. Ans: E

16. Ans: E

17. Ans: A

Copyright © 2000-2016 TIPS Inc. Unauthorized reproduction of this manual is prohibited. This 66-8
manual is being used during review sessions conducted by PharmacyPrep.
PharmacyPrep.Com Drug Interactions

18. Ans: B

19. Ans: E

20. Ans: B
Tips: Grape fruit juice is potent inhibitor of the most common drug metabolizing enzyme
CYP3A4

21. Ans: D

22. Ans: D

23. Ans: C

24. Ans: A

25. Ans: C

26. Ans: C

27. Ans: A
Tips: azathioprine is metabolized by xanthine oxidase, thus reduce the dose of
azathioprine to 2/3 or 3/4

28. Ans: E

29. Ans: B

30. Ans: E

31. Ans: C

32. Ans: C
Tips: carbamazepine induce CYP 2C9 and 1A2, these enzymes are substrate of warfarin

33. Ans: E

34. Ans: D

35. Ans: C

36. Ans: C

Copyright © 2000-2016 TIPS Inc. Unauthorized reproduction of this manual is prohibited. This 66-9
manual is being used during review sessions conducted by PharmacyPrep.
PharmacyPrep.Com Drug Interactions

Copyright © 2000-2016 TIPS Inc. Unauthorized reproduction of this manual is prohibited. This 66-10
manual is being used during review sessions conducted by PharmacyPrep.
www.Pharmacyprep.com Clinical Biochemistry

PHARMACY PREP
CLINICAL BIOCHEMISTRY

1. Thyroid Replacement therapy for hypothyroidism should be monitored by:


A. Free T 4
B. Sensitive TSH assay
C. Free thyroxine assay
D. Total thyroxin
E. Serum TSH assay

2. Serum TSH is elevated in?


A. Hyperthyroidism
B. Hypothyroidism
C. Hyperparathyroidism
D. Hypoparathyroidism
E. Pheochromacytoma

3. Which of the following plasma proteins has the GREATEST ability to bind acidic drugs
A. Albumin
B. Fibrinogen
C. Hemoglobin
D. Alpha glycoproteins
E. Beta lipoprotein

4. Which of the following is not the correct match:


A. Hypoparathyroidism hypocalcimia
B. Hypoparathyroidism hyperphosphetemia
C. Hyperparathyroidism hypercalcemia
D. Hyperparathyroidism causes deficiency phosphates
E. Hyperparathyroidism causes deficiency of calcium

5. Cockgraft-gault equation is used to estimate?


A. Creatinine clearance.
B. Blood Urea Nitrogen
C. Transminases
D. Urine analysis
E. Blood coagulation

6. Which of the following is NOT decreased in elderly?


A. Creatinine clearance
B. Serum creatinine
C. Blood urea nitrogen
D. Albumin

Copyright © 2000-2016 TIPS Inc. Unauthorized reproduction of this manual is prohibited. This manual is
being used during review sessions conducted by PharmacyPrep 67-1
www.Pharmacyprep.com Clinical Biochemistry

E. Adipose tissue

7. Which of the following is NOT decreased in elderly?


A.fat content
B. lean body mass
C. kidney function
D. Glomerular filtration
E. Serum creatinine

8. What are the following lab test increase indicate myopathies?


A. Creatinin kinase (CK-MM)
B. Creatinin kinase (CK-BB)
C. Creatinin kinase (CK-MB)
D. ADH, AST, ALT & ALP
E. ALP & ALT

9. A patient using warfarin should be monitored for?


A. Creatinine kinase
B. Prothrombin time
C. Partial thromboplastin time (aPTT)
D-Liver Function Test (LFT)
E. Renal Function Test (RFT)

10) Which of the following laboratory investigations help to determine, myocardial


infarction?
A) Troponin-I and CK-MB
B) CK-MB, Troponin C and ECG
C) ECG, Troponin I, CK-MM
D) Troponin I, CK-MB, ECG
E) Troponin C, CK-MB, ECG

11. What are the following lab test increase indicate myocardial infarction?
A. Creatinine kinase B. Troponin T C. Troponin I D. Troponin C E. ALP

12. A Patient with DVT was prescribed dalteparin. Which of the following lab test need
to monitor?
A. INR B. aPTT C. PT D. Vitamin K levels E. None of the above

13. Which of the following is used test calcium levels?


A. Parathyroid hormone B. Calcitonin C. Thyroid hormone D. Sun exposure

14. A patient creatinine clearance is 300 ml/hr, it is a?


A. chronic renal disease
B. normal renal function

Copyright © 2000-2016 TIPS Inc. Unauthorized reproduction of this manual is prohibited. This manual is
being used during review sessions conducted by PharmacyPrep 67-2
www.Pharmacyprep.com Clinical Biochemistry

C. excessive renal function


D. severe renal disease
E. Acute renal disease

15. What is the best urine sample for the urine analysis?
A. end of the sample
B. sample from midstream
C. sample from end of stream
D. Sample form beginning of stream
E. any one of them is ok

16) A patient creatinine clearance 300 ml/hour. This patient is categorized as?
A) normal CrCl
B) Chronic renal disease
C) Severe renal disease
D) hepatic disease
E. None of the above

17. Anticancer drugs in chemotherapy often cause the following side effects?
A. Neutropenia
B. Thrombocytopenia
C. Leukopenia
D. decrease in basophils
E. Increased monocyte

18. Which of the following test is the most sensitive to liver cirrhosis?
A. bilirubin B. AST C. ALT D. ALP E. CK

19. Which of the following is the most sensitive test to detect cholestasis jaundice?
A. bilirubin B. AST C. ALT D. ALP E. CK

20. Which of the following is the most sensitive to detect acetaminophen


hepatotoxicity?
A. bilirubin B. AST C. ALT D. ALP E. CK

21. An osteoarthritis patient using NSAIDS to treat joint pain. Approaches to your
pharmacy ask if any pain reliever change dark brown stool color or "coffee ground".
What is appropriate?
A) some NSAIDS can change stool color to dark brown
B) No, NSAIDS change stool color however if you notice dark brown of coffee ground
stool color you should contact doctor immediately
C) all NSAIDS give dark stool color
D) color change is quite normal and not a concern.
E) Tell patient to avoid taking coffee

Copyright © 2000-2016 TIPS Inc. Unauthorized reproduction of this manual is prohibited. This manual is
being used during review sessions conducted by PharmacyPrep 67-3
www.Pharmacyprep.com Clinical Biochemistry

22) Which of the following is the major ketone increase in blood diabetic ketoacidosis?
A) Acetone
B) Beta hydroxy butyric acid
C) Ethyl and methyl ketone
D) Long change ketones
E) Acetyl acetic acid

23. All of the following drugs can increase CK-MM, except?


A. Rosuvastatin B. Benzfibrate C. Atorvastatin plus niacin D. Simvastatin E) Ramipril

24. A laboratory test indicated increased number of neutrophil. This may be due to?
A. Viral infections B. Inflammation C. Bacterial infection in sinusitis
D) Seasonal flu E. Parasites infections

25. Decreased CD 4 count indicates?


A. Viral infections B. Inflammation C. Bacterial infection D. COPD E. Parasites
infections

25. A patient on vancomycin IV to treat MRSA. Which of the following lab test should be
monitored?
A. Liver function test
B. Complete blood count
C. Renal function test
D. Creatinine kinase
E. Chest X-ray

26) A patient is diagnosed with chronic bronchitis and pneumonia in COPD. Which of the
following is increased?
A. Basophills B. Neutrophil C. Lymphocytes D. WBCs E.RBCs

27. A patient laboratory shows increase in Eosinophills. This can be indicator of?
A. COPD B. Infection C. Asthma D. pneumonia E. ulcer

28. A 55 yo patient using warfarin to treat deep vein thrombosis. Which of the following
factors increase risk of bleeding?
A. Using vitamin K supplements
B. Using ASA 650 mg for pain
C. Using dark green vegetables such as broccoli, avacado
D. Using supplements containing vitamin K
E. Using iron supplements

29. A patient serum thyroid function test (Serum TSH) shows, 10 mU/L. Doctor should
initiate therapy of?

Copyright © 2000-2016 TIPS Inc. Unauthorized reproduction of this manual is prohibited. This manual is
being used during review sessions conducted by PharmacyPrep 67-4
www.Pharmacyprep.com Clinical Biochemistry

A. methimazole
B. Propylthiouracil
C. Levothyroxine
D. Radioactive iodide
E. Lugol's solution

30) Which of the following test are used determine anemia?


A) MCV
B) Hematocrit
C) Serum ferritin
D) MCHC
E) TIBC

31. What is lab test are related to diagnosis of lupus like syndrome?
A. Increase ESR
B. Increase C-reactive proteins
C. Antinuclear antibody test
D. Associated with myalgia, arthralgia and fever
E. All of the above

32. Which of the following is NOT a lab investigation for rheumatoid arthritis?
A. Erythrocyte sedimentation rate (ESR)
B. C-reactive protein (cRP)
C. Rheumatic factor
D. Arthralgia
E. Bone Mineral Density

33. KP is a 50 year old man using atorvastatin 40 mg daily. Get muscle pain. Which of the
following lab test increase?
A) ALT B) AST C) CK-MB D) CK-MM E) Troponin I

34. KG is a 50 yo patient on rosuvastatin 20 mg daily. Recently complain muscle pain.


What is correct?
A) Pharmacist refers patient to doctor to check CK-mm
B) pharmacist refers patient to emergency
C) pharmacist tell patient stop rosuvastatin and refer to dr
D) pharmacist tell patient to reduce the dose of rosuvastatin
E) Pharmacist should change rosuvstatin to simvastatin

35. MP is 49 year old male a type 2 diabetic patient. Currently patient has uncontrolled
blood glucose. Which of the following are NOT related to diabetes or and glucometry
tests?
A. Glucose tolerance test
B. Fasting blood glucose test

Copyright © 2000-2016 TIPS Inc. Unauthorized reproduction of this manual is prohibited. This manual is
being used during review sessions conducted by PharmacyPrep 67-5
www.Pharmacyprep.com Clinical Biochemistry

C. HbA1c test
D. Urine ketone test
E. Serum Ferritin

36. A 70 year old patient is using heparin to prevent stroke. Doctor suspect patient has
hemorrhage. Doctor should consider monitoring?
A) INR B) PT C) aPTT D) heparin assay E) no lab test

37. Acetaminophen overdose associated with hepatotoxicity. This can increase the liver
transaminases. Which of the following are liver transaminases?
A. ALP, AST and ALT
B. ALP and AST
C. AST and ALT
D. ALT and ALP
E. Bilirubin

38. Which of the following liver enzyme are indicator or liver cirrhosis?
A. AST/ALT B. Serum albumin C. Prothrombin time D. Bilirubin
E. Alkaline. Phosphate

39. A patient presents with dark urine and muscle pain. Which of the following drugs
may be patient is on?
A) metronidazole B) Rifampicin C) bismuth D) lithium E) Atorvastatin

40.What lab test is used to determine liver cirrhosis?


A. AST/ALT
B. serum albumin
C. prothrombin time
D.Bilirubin
E. Globulin
Ans. D
Tips. direct bilirubin, and urine bilirubin significant increased and AST/ALT minimally
increased in hepatocelluar jaundice, obstructive jaundice and liver cirrhosis.

41. what lab test is for viral hepatitis patterns?


A. AST/ALT
B. ALP
Ans. A

42. ESR is nonspecific test that is often used as screening tests for patients
with unexplained fever, arthritis and muscle symptoms. The ESR elevation
indicates, all of the following, except?
A. RA

Copyright © 2000-2016 TIPS Inc. Unauthorized reproduction of this manual is prohibited. This manual is
being used during review sessions conducted by PharmacyPrep 67-6
www.Pharmacyprep.com Clinical Biochemistry

B. Lupus like syndrome


C. polymyalgia rheumatic
D. giant cell arthritis
E. osteoporosis
Ans.

43. A Patient with DVT was prescribed dalteparin. Which of the following
lab test need to monitor?
A. INR
B. aPTT
C. PT
D. Vitamin K levels
E. None of the above
ans.e

Copyright © 2000-2016 TIPS Inc. Unauthorized reproduction of this manual is prohibited. This manual is
being used during review sessions conducted by PharmacyPrep 67-7
www.Pharmacyprep.com Clinical Biochemistry

CLINICAL BIOCHEMISTRY
ANSWERS:

1. Ans: E

2. Ans: B

3.Ans: A

4. Ans: E

5. Ans: A
Tips: Cockgraft-gault equation is used to estimate creatinine clearance.

6. Ans: E

7. Ans: A

8. Ans: A
Tips: CK-MM indicator of skeletal muscle, CK-BB is for brain and CK-MB for cardio.

9.Ans: B

10) Ans. D

11. Ans: C

12. Ans: E

13. Ans: A

14. Ans: D

15.Ans: B

16) Ans. C
Tips: CrCl <50 ml/min is chronic renal disease

17.Ans: C

18. Ans: B

19. Ans: D

Copyright © 2000-2016 TIPS Inc. Unauthorized reproduction of this manual is prohibited. This manual is
being used during review sessions conducted by PharmacyPrep 67-8
www.Pharmacyprep.com Clinical Biochemistry

20. Ans: B

21. Ans: B

22) Ans: B

23. Ans:E

24. Ans: C

25. Ans: C
Tips: vancomycin renal toxicity thus patient monitored for renal function tests.

25. Ans: D

26) Ans: B
Tips: chronic bronchitis is bacterial infection in COPD, so increase neutrophils.
WBCs Normal Indicator
Neutrophils 55% to 75% Bacterial infection
Lymphocytes 20% to 40% Viral infection
Monocytes 0% to 7% Tuberculosis
Eosinophils 0% to 5% Parasites infection
Basophils 0% to 1% Inflammation, Allergies, Asthma,

27. Ans: C
Tips: Asthma and allergies increase esinophils and basophils.

28. Ans: B

29. Ans: C

30) Ans: C

31. Ans: E

32. Ans:E

33.Ans. D

34. Ans. C

35. Ans. E

Copyright © 2000-2016 TIPS Inc. Unauthorized reproduction of this manual is prohibited. This manual is
being used during review sessions conducted by PharmacyPrep 67-9
www.Pharmacyprep.com Clinical Biochemistry

36. Ans. C

37. Ans. C
Tips: AST and ALT are liver transaminase. Whereas ALP is alkaline phosphotase enzyme.

38. Ans. D
Tips. direct bilirubin, and urine bilirubin and alkaline phosphate significant increased and
AST/ALT minimally increased in hepatocelluar jaundice, obstructive jaundice and liver
cirrhosis.

39. Ans. E
Tips. this is symptoms of myopathies associated with statins.

Copyright © 2000-2016 TIPS Inc. Unauthorized reproduction of this manual is prohibited. This manual is
being used during review sessions conducted by PharmacyPrep 67-10
PharmacyPrep.Com Therapeutic Drug Monitoring

PHARMACY PREP
THERAPEUTIC DRUG MONITORING

1-Which antipsychotic drugs monitored for WBC or CBC?


A-Olanzapine
B-Clozapine
C-Risperidone
D-Haloperidol
E-Quetiapine

2) What is NOT monitored for patient using risperidone?


A) WBC
B) diabetes
C) blood pressure
D) Parkinson’s symptoms
E) Weight gain

3) Patient using statins should be monitored for?


A-LFT and CK-MM
B-LFT only
C-CK-MM only
D-Renal function test
E-LDL only

4) CK-MM indicates?
A-Creatinin kinase in skeletal muscles
B-Creatinin kinase in brain
C-Creatinin kinase in Heart
D-Creatinin kinase in liver
E-Creatinin kinase in renal elimination

5) A patient taking acetaminophen, what liver enzyme may be elevated?


A-ALT B-AST C-LFT D-ALP E-LDH

6) A patient taking atorvastatin, what liver enzymes may be elevated?


A-ALT and AST
B-LDH and ALT
C-LDH only
D-ALT only
E-AST only

7) Patient using warfarin should be monitored for?


A-Prothrombin time (PT)
B-INR

Copyright © 2000-2016 TIPS Inc. Unauthorized reproduction of this manual is prohibited. This 68-1
manual is being used during review sessions conducted by PharmacyPrep.
PharmacyPrep.Com Therapeutic Drug Monitoring

C-aPTT
D-A and B
E-All of the above

8) Hypothyroidism patient should be monitored


A-Decrease TSH
B-Elevated TSH
C-Elevated FT 3
D-Decreased FT 4
E-TBG

9) A 50-year-old male a regular customer of your pharmacy, his current medications


include: Hydrochlorthiazide, Furosemide 50 mg, and Digoxin 0.125 mcg
Which of the following he should be monitored?
I-Monitor K levels
II-Serum creatinin
III-Blood pressure
a) I only
b) III only
c) I and II only
d) II and III only
e) All are correct

26) Which drugs patient may complain sandy and gritty feeling in eye? The pharmacist
recommends patent to consult doctor get eye exam
I) amiodarone
II) hydroxychloroquine
III) Quinidine
a) I only
b) III only
c) I and II only
d) II and III only
e) All are correct

11) A 52 yo women using Synthroid 75 mcg daily dose. Recently her blood test done it
showed her serum TSH are less than 0.4 mU/L. What is the appropriate action to do?
A) Increase dose of Synthroid to 100 mcg
B) Double the dose Synthroid
C) Do not change the dose of Synthroid
D) Decrease dose of Synthroid
E)Recommend to take empty stomach

Copyright © 2000-2016 TIPS Inc. Unauthorized reproduction of this manual is prohibited. This 68-2
manual is being used during review sessions conducted by PharmacyPrep.
PharmacyPrep.Com Therapeutic Drug Monitoring

12) A 68 yo male using warfarin 4 mg daily and currently his INR is 2 to 3. Recently he
cut his finger slightly with nail clipper. and had slightly bruise on his face. What is
appropriate action do?
A) Refer him to emergency
B) Ask him to discontinue warfarin and contact doctor immediately
C)give him first aid like band aid his finger nail.
D)Recommend vitamin K supplements
E)Recommend to increase dose of warfarin

13) A 45 yo man using rosuvastatin 20 mg daily and metformin 500 mg bid, gliclazide SL
30 mg daily. A doctor recommends monitoring of lab test for metformin?
I)Renal Function Test
II) Creatinine clearance
III) Liver Function Test
A) I only B) III only C)I and II D)II and III E)I, II, III

14) A 45 yo man using rosuvastatin 20 mg daily and metformin 500 mg bid, gliclazide SL
30 mg daily. A doctor recommends regular monitoring of lab test for rosuvastatin?
A)CK-mm B)LFT C)Renal function test D) B.P E)ECG

15) Procainamide is type of?


A)ester type of local anesthetic
B)amide type of local anesthetic
C)it is gas general anesthetic
D)It is antiarrhythmic drug
E)It is potassium channel blocker

16) A 65 yo man using digoxin, hydrochlorothiazide, furosemide and ramipril for


congestive heart failure for the past one year. Which of the following laboratory test
require monitoring?
A)Na+ B)Cl C)K+ D)Ca2+ E)Mg2+

17)A 55 yo customer of your pharmacy. Currently on hydrochlorothiazide 75 mg daily,


metformin 500 mg tid, gliclazide 30 mg daily, and allopurinol 100 mg. Which of the
following is not monitored?
A)uric acid levels B)glucose test C)Blood pressure D)ECG E)CrCl

18) Which of the following diuretic may require Ca2+ monitoring?


A)hydrochlorothiazide B)spironolactone C)furosemide D)Acetazolamide E)Mannitol

19) A 45 yo man using rosuvastatin 20 mg daily and metformin 500 mg bid, glicalizide SL
30 mg daily. A doctor recommends regular monitoring of lab test for rosuvastatin?
A) LFT B) CK-mm C) CrCl D)glucose E)Renal function test

Copyright © 2000-2016 TIPS Inc. Unauthorized reproduction of this manual is prohibited. This 68-3
manual is being used during review sessions conducted by PharmacyPrep.
PharmacyPrep.Com Therapeutic Drug Monitoring

20) Which of the following lithium serum levels gives lithium toxic symptoms?
A) > 1.5 mEq/L
B) < 1.5 mEq/L
C) > 2.0 mEq/L
D) > 3.0 mEq/L
E) > 4.0 mEq/L

21) A 45 yo man recently diagnosed with serum TSH 10 mU/L. However normal range of
serum TSH is 0.5 to 5 mU/L. This patient should be initiated?
A) hyperthyroidism therapy
B) hypothyroidisms therapy
C) Methimazole
D) Propylthiouracil
E) Radioactive therapy

22) A patient is using furosemide to treat congestive heart failure. Which of the
following electrolytes loss is NOT monitored?
A) Potassium
B) Calcium
C) Magnesium
D) Sodium
E) Aluminum

23) Which of the following does NOT required monitoring, serum TSH?
A) amiodarone
B) Prednisone
C) Levothyroxine
D) methyl prednisone
E) infliximab

24) A 70 yo patient is using hydrochlorothiazide 50 mg daily to treat hypertension.


Which of the following is NOT monitored?
A) uric acid
B) blood glucose
C) cholesterol (lipids)
D) hypocalcemia
E) hypokalemia

25)A 29 yo patient using lithium carbonate to treat manic depression. What is NOT monitored?
A) serum TSH
B) CBC
C) blood glucose levels
D) serum level of lithium
E) eye exam

Copyright © 2000-2016 TIPS Inc. Unauthorized reproduction of this manual is prohibited. This 68-4
manual is being used during review sessions conducted by PharmacyPrep.
PharmacyPrep.Com Therapeutic Drug Monitoring

THERAPEUTIC DRUG MONITORING


ANSWERS:

1) Ans: B
Tips: Clozapine associated with side effect of agranulocytosis, thus it should be
monitored for WBC or CBC.

2) Ans: A

3) Ans: A

4) Ans: A

5) Ans: A

6) Ans: A

7) Ans: D

8 Ans:) B

9) Ans: C

10) Ans: E
Tips: Amiodarone, hydroxychloroquine and quinidien may conreal deposits.

11) Ans: D
Tips: TSH are less than 0.4 mU/L is indicator of overdose of Synthroid, thus decrease
dose.

12) Ans: C

13) Ans: E

14) Ans: B

15) Ans: B

16) Ans: C
Tips: hydrochlorothiazide and furosemide can cause hypokalemia, this can lead to
digitalis toxicity.

17) Ans: D

Copyright © 2000-2016 TIPS Inc. Unauthorized reproduction of this manual is prohibited. This 68-5
manual is being used during review sessions conducted by PharmacyPrep.
PharmacyPrep.Com Therapeutic Drug Monitoring

18) Ans: C
Tips: furosemide can cause hypocalcemia (loop loose calcium)

19) Ans: A

20) Ans: C

21) Ans: B

22) Ans: E

23) Ans: E

24) Ans: D
Tips: thiazides cause hypercalcemia or hyperGLUC. Whereas loop diuretics cause
hypocalcemia

25) Ans: E

Copyright © 2000-2016 TIPS Inc. Unauthorized reproduction of this manual is prohibited. This 68-6
manual is being used during review sessions conducted by PharmacyPrep.
www.pharmacyprep.com OTC medications dermatological conditions

PHARMACY PREP
OTC DRUGS FOR DERMATOLOGICAL CONDITIONS

1-Incorrect statement about poison ivy treatment?


A-Hydrocortisone 0.5%
B-Calamine lotion
C-Antihistamine
D-Antibiotics
E-Oatmeal cream

2-Incorrect pharmacotherapy for psoriasis treatment?


A-Hydrocortisone 1%
B-Coal tar
C-Tazarotene
D- Antibiotics
E-UVA light + coal tar

3) What is incorrect about degree of burns classification?


A) 1st degree is superficial burns, that may cause by sunburn
B) 2nd degree is superficial and partial thickness burns that may cause by hot water, oil spills
and splashes
C) 3rd degree is the severe burns that can give lathery skin.
D) If it burn put burn part in cold water
E) If it burn put burn part in warm water

4-Incorrect statement about sunburn?


A-peak time of sunburn is 10:00 am to 4:00pm
B-High altitude has low sunburn
C-cloud do not protect sunburn
D-beach area may have more sunburn
E-Apply sunscreen before get into sun

5-What is incorrect about sunburn treatment?


A-Use cool baths or wet compresses with tap water or saline
B-Take plenty of fluids
C-Topical vitamin E applied two minutes after UV exposure may decrease erythema and
edema
D-Acetaminophen can relieve pain
E-Avoid NSAIDs for sunburn pain

6) What is incorrect about head lice treatment?


A-Permethrin 1% cream rinse apply to hair and scalp (1/2 to 1 bottle) and leave it on for 10
min then rinse.
B-Permethrin may repeat after 7 days if live head lice is observed.

Copyright © 2000-2016 TIPS Inc. Unauthorized reproduction of this manual is prohibited. This manual is 69-1
being used during review sessions conducted by PharmacyPrep.
www.pharmacyprep.com OTC medications dermatological conditions

C-May temporarily exacerbate itching and pruritus in head.


D-May gives burning/stinging, tingling numbness in scalp
E-After permethrin if it gives itching and pruritus refer to doctor

7) What is incorrect about head lice?


A-All close contact need not to treated if one person has head lice
B-Head lice can fly and transmit to other family member
C-Long hair or short hair both can have head lice
D-Hygiene is a not criteria for head lice
E-Nits can hatch head lice in 7 to 10 days

8-Contact dermatitis and itching is managed by:


I-oatmeal cream
II-calamine lotion or zinc oxide
III-local anesthetics (procaine)
A-I only
B-III only
C-I and II only
D-II and III only
E-All of the above

9) What is the drug of choice to treat complicated diaper rash (diaper rash more than 3 days):
A. barrier bases (zinc oxide, petrolatum)
B. antifungal agents clotrimazole 1% cream, miconazole and Nystatin
C. Anti-inflammatory agents (0.5% hydrocortisone)
D. Alcohol wipes
E. Changing diaper

10-What is incorrect in treatment of diaper rash?


A-Keep diaper area dry
B-clean diaper area with alcohol swabs
C-Apply petrolatum on diaper area
D-Apply 0.5% hydrocortisone

11-What does sun protection factor (SPF) 15 mean?


A-if it takes 10 minutes to get sunburn, in SPF 15, it would take 15x10 min
B-If it takes 10 minutes to get sunburn in SPF 15, it would take 15x100 min
C- If it takes 10 minutes to get sunburn in SPF 15, it would take 150x100 min
D- If it takes 10 minutes to get sunburn in SPF 15, it would take 150x1000 min
E-If it takes 10 minutes to get sunburn in SPF 15, it would take 1500x100 min

12) All of the following are used for dandruff, except?


A-Ketoconazole
B-Coal tar

Copyright © 2000-2016 TIPS Inc. Unauthorized reproduction of this manual is prohibited. This manual is 69-2
being used during review sessions conducted by PharmacyPrep.
www.pharmacyprep.com OTC medications dermatological conditions

C-Zinc pyrethione
D-Hydrocortisone
E-Selenium sulfide

13-Which of the following is NOT a pharmacotherapy for acne?


A-Oral contraceptives
B-Corticosteroids
C-Tetracyclin
D-Retinoids
E-A and B

14-Which of the following is pharmacotherapy for acne in pregnancy?


A-Oral contraceptives
B-Erythromycin
C-Tetracycline
D-Retinoids
E-Minocycline

15) A 20 year old women treated with topical treatment for acne, however, it not treated,
which of the following is recommended?
A-Benzoyl peroxide
B-Tretinoin
C-Isotretinoin
D-Salicylic acid
E-None

16) Which of the following skin condition have symptoms of red scaly patches?
A) Psoriasis B) Dermatitis C) Acne D) Rosacea E) Urticaria

17) Characteristic of Rosacea symptoms is/are?


A. Pustules B. Nodules C. Pimples D. Erythema E. Red scaly patches

18) Which of the following used to treat acne vulgaris?


I) retinol
II) cis/trans retinoic acid
III) Vitamin E
A) I only B) III only C) I and II only D) II and III E)all

19) What is NOT a symptoms of acne?


A) White head (comedon)
B) Black head (comedon)
C) Pimple
D) Postule
E) Red scaly patches

Copyright © 2000-2016 TIPS Inc. Unauthorized reproduction of this manual is prohibited. This manual is 69-3
being used during review sessions conducted by PharmacyPrep.
www.pharmacyprep.com OTC medications dermatological conditions

20. What is appropriate socks for athletes foot and diabetic foot problems?
A. 100% cotton socks
B. Nylon socks
C. Synthetic fiber
D. Synthetic socks
E. Cotton (60% less) blend with synthetic fiber, nylon, acrylic or rayon

21. What is the drug of choice for head lice in child with seizures?
A. permethrin 1%
B. Lindane 1%
C. Sulfur 5%
D. Isopropyl mersteate
E. permethrin 5%
Ans. a

22. What is the drug of choice to treat mild acne with inflammation?
A. benzoyl peroxide
B. topical retinoids
C. Tetracycline
D. Clindamycin topical
E. Erythromycin topical
Ans. b

23.What is not related to acne?


A. popping white heads with hands cause pimples, infection, scarring.
B. small dark spots on skin are blackheads or open comedone
C. Pink or silvery scaly patches
D. acne forms popules, pustules
E. acne forms cysts and nodules
Ans. c
Tips. white head, pimples, popules, pustules, cysts and nodules are forms of acne. Pink and
silvery patches are psoriasis.

24. What is incorrect related to rosacea acne?


A. Erythema is present rosacea acne
B. Papulopustular lesions and eye involvement with or without
C. Drug of choice is metronidazole cream
D. Telangiectasia or facial edema or rhinophyma
E. Caused by Corynebacterium parvum
Ans. e

Copyright © 2000-2016 TIPS Inc. Unauthorized reproduction of this manual is prohibited. This manual is 69-4
being used during review sessions conducted by PharmacyPrep.
www.pharmacyprep.com OTC medications dermatological conditions

25. What is appropriate socks for athletes foot and diabetic foot are recommended?
A. 100% cotton socks
B. Nylon socks
C. Synthetic fiber
D. Synthetic socks
E. Cotton (60% less) blend with synthetic fiber, nylon, acrylic or rayon
Ans. e

Copyright © 2000-2016 TIPS Inc. Unauthorized reproduction of this manual is prohibited. This manual is 69-5
being used during review sessions conducted by PharmacyPrep.
www.pharmacyprep.com OTC medications dermatological conditions

OTC DRUGS FOR DERMATOLOGICAL CONDITIONS


ANSWERS:

1- Ans: D

2- Ans: D

3) Ans: E
BURNS CLASSIFICATION
st nd
1 degree 2 degree 3rd degree
superficial burns, that may superficial and partial severe burns that can give
cause by sunburn thickness burns that may lathery skin.
cause by hot water, oil spills
and splashes

4- Ans: B

5- Ans: E

Head lice
6- Ans: E

7) Ans: B

8- Ans: C
Tips: Astringents such as calamine lotion, aluminum acetate, zinc oxide
9. Ans: C
Tips: The drug of choice to treat complicated diaper rash is clotrimazole 1% cream.
Uncomplicated diaper rash Complicated diaper rash
Limited rash Confluent tomato red plaques, white scaly border.
ZnO, petrolatum, hydrocortisone Clotrimazole, miconazole, nystatin and ZnO,
hydrocortisone

10- Ans: B

11- Ans: A
Tips: snow, high altitude, and beach, mountains can give more sun burn.

12) Ans: D

13) Ans: B

Copyright © 2000-2016 TIPS Inc. Unauthorized reproduction of this manual is prohibited. This manual is 69-6
being used during review sessions conducted by PharmacyPrep.
www.pharmacyprep.com OTC medications dermatological conditions

14- Ans: B

15) Ans: C

16) Ans: A

17) Ans: D

18) Ans: C

19) Ans: E Tips: Red scaly patches are symptoms of psoriasis.

20. Ans. E

Copyright © 2000-2016 TIPS Inc. Unauthorized reproduction of this manual is prohibited. This manual is 69-7
being used during review sessions conducted by PharmacyPrep.
PHARMACY PREP
OTC AND PRESCRIPTION DRUGS FOR OPHTHALMIC, EAR AND MOUTH
DISORDERS

1. What is an optimal range of pH for ophthalmic products


A. 2-3 B. 4-5 C. 5-6 D. 6-8 E. 8-10

2. Opthalmic agents contraindicated in glaucoma patients include which of the following


substances:
A. antioxidants B. decongestants C. emollients D. antipruritics
E. All of the above

3. Which of the following lenses can be worn continuously for 6 days:


A. Soft lenses
B. Extended wear soft lenses
C. Frequent replacement soft lenses
D. Hard lenses
E. Soft and hard lenses

4. Image formed by lens received by:


A. iris B. retina C. sclera D. conjunctiva E. pupil

5. What is correct statement about treatment in eruption cysts in infants


A. In general cysts rupture spontaneously
B. Use teeters to rupture cysts
C. Manually cysts can be removed
D. None of the above

6. Which of the following treatment is used to treat cold sores?


I-Antihistamine
II-Zinc oxide (ZnO)
III-Benzocaine 7.5% gel
A. I only B. III only C. I and II only D. II and III only E. All of the above

7. What is treatment of cold sores in chicken pox?


I-Calamine lotion
II-Oatmeal bath (micellar colloidal bath)
III-antibiotics
A. I only B. III only C. I and II only D. II and III only E. All of the above

8. Which of the following is the incorrect action of sympathomimetics?


I- Increase of IOP
II- Increase the drainage of H 2 O humor from the eye
III- Decrease the H 2 O humor in the eye
A. I only B. III only C. I and II only D. II and III only E. All of the above

Copyright © 2000-2016 TIPS Inc. Unauthorized reproduction of this manual is prohibited. This 70-1
manual is being used during review sessions conducted by PharmacyPrep.
9. Which of the following ophthalmic conditions required referral to doctor?
A. Red eye B. Pain in eye C. Blurred vision D. Dry eye E. B and C

10) Which of the following ophthalmic conditions require referral to doctor? except
A) Red eye or pink eye
B) Pain in eyes
C) Blurred vision
D) Dry eye in diabetic patient
E) Blepharitis
Ans. A

11) A customer of your pharmacy using metformin 500 mg bid and glyburide qd.
Complains to have gritty and sandy feeling in eye. What is appropriate to do?
A) Refer to doctor because of diabetic condition
B) Refer to doctor because sandy and gritty feeling in eye is conjunctivitis
C) Refer to doctor because glyburide side effect is dry eye
D) Refer to doctor because metformin side effect is dry eye
E) Refer to doctor because these are side effects of anticholinergic drugs

12. Which of the following can be used for earwax removal?


A) carbamide peroxide
B) soft mineral oil
C) Burrow's solution (aluminum acetate)
D) All of the above
Ans. D

13. Otitis externa (swimmers ear), is treated by?


A. amoxicillin high dose
B. amoxicillin + clavulanate
C. Aluminum acetate (Burrow's solution)
D. Cephalosporin's
E. All of the above

14. A 60 yo customer of your pharmacy medical profile include; metformin, glyburide,


amiodarone, rosuvastatin, ASA 81 mg. Recently doctor prescribed methrotrexate to
treat psoriasis. Patient reports sandy and gritty eyes. Pharmacist direct patient to see
doctor. Which of the following drug concern prompted pharmacist refer to doctor?
A. methotrexate
B. metformin
C. Glyburide
D. amiodarone
E. rosuvastatin

Copyright © 2000-2016 TIPS Inc. Unauthorized reproduction of this manual is prohibited. This 70-2
manual is being used during review sessions conducted by PharmacyPrep.
15. Which of the following eye condition can be treated by non prescription therapy?
A. Eye pain
B. Vision changes
C. Blepharitis
D. Red eye
E. Foreign object in eye
Ans. D

16. What is incorrect about ophthalmic lens?


A. Soft lens are made of Hydroxyethyl methacrylate (HEMA)
B. Surfactant in lens solution prevent infection
C. Protein cleaners in lens solution remove protein deposits
D. Wetting agents in lens solution produce lubricant effect
E. Hard lens can wear for more than 24 hours
Ans. E

17. Which of the following condition require referral to doctor?


A. Ear pain especially on movement
B. Too much ear wax
C. Fullness in ear
D. Significant debris and fluid discharge from ear canal
E. All of the above
Ans. e

OTC AND PRESCRIPTION DRUGS FOR OPHTHALMIC, EAR


AND MOUTH DISORDERS
ANSWERS:

1.Ans: D

2. Ans: B

3. Ans: B

4. Ans: B

5. Ans: A

6. Ans: D

7. Ans: C
Tips: Calamine lotion (ZnO+FeO) and oatmeal bath and oral antihistamine can be helpful
in treatment of cold sores.

Copyright © 2000-2016 TIPS Inc. Unauthorized reproduction of this manual is prohibited. This 70-3
manual is being used during review sessions conducted by PharmacyPrep.
8. Ans: D
Tips: Sympathetic agonist (sympathomimetics) increases IOP. Open angle glaucoma
(OAG) is due to increase in IOP and increase in H 2 O humor of eye or could be due
decrease in drainage of H 2 O humor in the eye. For the treatment of OAG sympathetic
blockers such as Timolol are effectively used to decrease IOP and H 2 O humor.

9. Ans: E

10. Ans: D

11. Ans: A
Tips: Diabetic retinopathy symptoms require referral to doctor

12. Ans: C

13. Ans: C

14. Ans: D
Tips: amiodarone gives ocular deposits SEs.

Copyright © 2000-2016 TIPS Inc. Unauthorized reproduction of this manual is prohibited. This 70-4
manual is being used during review sessions conducted by PharmacyPrep.
Copyright © 2000-2016 TIPS Inc. Unauthorized reproduction of this manual is prohibited. This 70-5
manual is being used during review sessions conducted by PharmacyPrep.
PharmacyPrep.Com OTC Antihistamine's, Decongestants and Antitussives

PHARMACY PREP
OTC ANTIHISTAMINE, DECONGESTANTS AND ANTITUSSIVES

1) Which of the following is NOT an environmental measure to prevent allergic rhinitis:


A-Avoidance of window or attic fans
B-Avoid outdoor activities during pollen season
C-Shower or bath following outdoor activity to avoid contamination of bedding
D-Use air conditioning
E-Open windows over night to get fresh air.

2-Sympathomimetics decongestant acts as?


A-Vasoconstrictor
B-Vasodilator
C-Congestion
D-Anti-inflammatory actions
E-None of the above

3-Sympathomimetics decongestant can cause, all of the following complications, except?


A- Open angle glaucoma
B- hypertension
C- diabetics
D-Can cause rebound congestion in prolong use
E-Can cause vasodilatation

4) All of the following drugs gives bronchodilators, except


A-short acting beta2 agonist
B-long acting beta2 agonist
C-Propranolol
D-Salbutamol
E-Salmeterol

5-Opioids may be used as all the following agents, EXCEPT:


a) Antitussive b) Analgesic c) Antidiarrheal d) Anti-inflammatory
e) Pre-anesthetic

6-Alpha 1 agonist, phenylephrine has all of the following side effect EXCEPT:
A-vasoconstriction B-Headache C) orthostatic hypotension D-Arrhythmias
E-Mydriasis

7- Fexofenadine HCI (Allegra 12 Hour and Allegra 24 Hour) is classified as:


A-Histamine H 2 -Receptor Antagonist
B- Histamine H 1 -Receptor Antagonist
C-Proton Pump inhibitor (PPI)
D-Serotonin reuptake inhibitor (SSRI)

Copyright © 2000-2016 TIPS Inc. Unauthorized reproduction of this manual is prohibited. This manual is 71-1
being used during review sessions conducted by PharmacyPrep.
PharmacyPrep.Com OTC Antihistamine's, Decongestants and Antitussives

E-5-HT 1 Receptor Agonist

8-A mother comes to your pharmacy with concern of her child 2 yr old having fever and
pharyngitis, currently using amoxicillin. What is appropriate recommendation?
A-recommend ASA
B-recommend oral ibuprofen
C-Recommend acetaminophen suppositories
D-Recommend cold sponges
E-do not recommend any medications

9) A customer of your pharmacy travelling to London, Ontario, wants to buy some


medications for nausea and vomiting in motion sickness. What is appropriate
recommendation?
A) Dimenhydrinate B)Dextromethorphan C) Scopolamine D)Ginger root E) Meclazine

10) A customer of your pharmacy using MAO Inhibitors for major depression. Now looking for
nasal congestion medication. What is recommended?
A) pseudoephedrine B) ephedrine C) codeine D)d extromethorphan E) saline nasal drops

11. A regular customer, purchased dextromethorphan 2 times in last 20 days. Pharmacist talk
to patient and checked patient profile and referred to doctor. Which of the following
medication in patient profile prompted pharmacist?
A) Glyburide B) Metformin C) Rosuvastatin D) Ramipril E) All of the above

12. A 50 year old school bus driver brings prescription of alprazolam and clonazepam to treat
anxiety. He is frequent customer of pharmacy to purchase diphenhydramine and
dimenhydrinate. Pharmacist informed to doctor patient use of benzodiazepines and
antihistamine. Dr reported to employer of patient. What ethical principle were applied?
A) Nonmaleficence
B) Beneficence
C) Autonomy
D) Veracity
E) Justice

13. Which of the following is the most commonly abused over the counter medication?
A. Dextromethorphan
B. Pseudoephedrine
C. Tylenol #1
D. Diphenhydramine
E. Marijuana
Ans. a

Copyright © 2000-2016 TIPS Inc. Unauthorized reproduction of this manual is prohibited. This manual is 71-2
being used during review sessions conducted by PharmacyPrep.
PharmacyPrep.Com OTC Antihistamine's, Decongestants and Antitussives

14. A doctor prescribed a sedative antihistamine to a taxi driver. What is appropriate


pharmacist action?
A. Dispense drug and counsel patient not to drive after taking this medication.
B. Call doctor to change to non sedative antihistamine
C. Change to non sedative antihistamine and dispense to patient.
D. counsel patient to not to take alcohol with sedatives
E. Give option to patient to decide to take and not to drive

Copyright © 2000-2016 TIPS Inc. Unauthorized reproduction of this manual is prohibited. This manual is 71-3
being used during review sessions conducted by PharmacyPrep.
PharmacyPrep.Com OTC Antihistamine's, Decongestants and Antitussives

OTC ANTIHISTAMINE, DECONGESTANTS AND ANTITUSSIVES


ANSWERS

1) Ans: E

2- Ans: A

3- Ans: E

4) Ans: C

5- Ans: D

6- Ans: C
Tips: Hypertension is common side effect phenylephrine.

7- Ans: B

8- Ans: B

9) Ans: A

10) Ans: E
Tips: with MAO I for nasal congestion avoid using pseudoephedrine, ephedrine can cause
hypertension crisis For cough avoid dextromethorphan cause serotonin syndrome

11. Ans: D
Tips: ACE Inhibitors may induce cough. This can be managed by changing therapy to ARBs.

12. Ans: A

Copyright © 2000-2016 TIPS Inc. Unauthorized reproduction of this manual is prohibited. This manual is 71-4
being used during review sessions conducted by PharmacyPrep.
PharmacyPrep.Com OTC Antihistamine's, Decongestants and Antitussives

Copyright © 2000-2016 TIPS Inc. Unauthorized reproduction of this manual is prohibited. This manual is 71-5
being used during review sessions conducted by PharmacyPrep.
PharmacyPrep.Com OTC drugs for Nausea, Vomiting, Constipation..

PHARMACY PREP
OTC DRUGS FOR NAUSEA, VOMITING, CONSTIPATION, DIARRHEA,
HEMORRHOIDS AND HELMENTHS

1) A 60 yo customer of your pharmacy using oxycontin daily for severe pain after fracture.
What is the treatment best option for constipation?
I) Sennakot II) Docusate sodium III) high fiber diet
a) I only b) III only c) I and II only d) II and III only e) I, II, III

2- Psyllium (Metamucil) is a bulk-forming laxative agent. Patients using should get the
following instructions:
I- Avoid psyllium (Metamucil) in pregnancy
III- It should mix the dose with a glass of cool water or other fluid and drink it down quickly.
II-This should be followed with more fluids.
a) I only b) III only c) I and II only d) II and III only e) I, II, III

3)Antacids are normally used to equilibrate the stomach pH and protect the stomach from the
effect of acids. Which of the following antacids has a cathartic side effect?
a) Calcium carbonate
b) Dihydroxyaluminum sodium carbonate
c) Magnesium hydroxide
d) Calcium sulfate
e) Aluminum silicate

4- Which antihelmenthic drug is used to treat pin worm?


A-Pyremethamine
B-Pyridoxin
C-Streptomycin
D-Pyrivenium pamoate
E-Metronidazole

5- Lactulose is an osmotic laxative agent that may be used to treat:


I- Constipation
II- Portal-systemic encephalopathy
III- Renal tubular necrosis
a) I only b) III only c) I and II only d) II and III only e) All are correct

6) A mother of 6 month old child approaches to your pharmacy. Her child has diarrhea. She is
also breast feeding her child, what is appropriate recommendation?
A) recommend ORS and ask to continue breast feeding
B) Recommend ORS and ask to stop breast feeding
C) Recommend ORS and refer to doctor

Copyright © 2000-2016 TIPS Inc. Unauthorized reproduction of this manual is prohibited. This manual is 72-1
being used during review sessions conducted by PharmacyPrep.
PharmacyPrep.Com OTC drugs for Nausea, Vomiting, Constipation..

D) Refer to emergency
E) Do nothing, it will be alright

7) Drug of choice for constipation induced by codeine?


A-Psyllium B-Lactulose C-Sodium docusate D-Senna E-Mineral oil

8) All of the following are self care measure should be recommended in hemorrhoids, except?
A. Increase high fibre diet B) Increase fluid intake C) Avoid food on the run
D) Regularize toilet habits e)Prolong stay on toilet seat

9) Drug of choice for constipation in pregnancy?


A-Psyllium B-Lactulose C-Sodium docusate D-Senna E-Mineral oil

10) What is incorrect recommendation to prevent traveler’s diarrhea?


A-Avoid ice cubes B-Take fresh salad C-Take cooked food D-Used boiled or
bottled water E-Avoid seafood

11)A mother comes to pharmacy and has concern about a 6 months baby have diarrhea. She
is also breast-feeding child, what is appropriate recommendation?
A-recommend ORS and ask to continue breast feeding
B-Recommend ORS and ask to stop breast feeding
C-Recommend ORS and refer to doctor
D-Refer to emergency
E-Do nothing, it will be alright

12) A cancer patient experiencing severe nausea and vomiting, should be recommended, all
except?
A-Avoid large meals B-Avoid fat-oily meals C-Avoid taking small and frequent meals
D-Take divided meals E-Avoid heavy meals

13) Drug of choice for constipation induced by codeine?


A-Psyllium B-Lactulose C-Sodium docusate D-Senna E-Mineral oil

14) All of the following are self care measure should be recommended in hemorrhoids,
except?
a)Increase high fiber diet b)Increase fluid intake C)Avoid food on the run
D)Regularize toilet habits e) Prolong stay on toilet seat

15) Drug of choice for constipation in pregnancy?


A-Psyllium
B-Lactulose
C-Sodium docusate
D-Senna
E-Mineral oil

Copyright © 2000-2016 TIPS Inc. Unauthorized reproduction of this manual is prohibited. This manual is 72-2
being used during review sessions conducted by PharmacyPrep.
PharmacyPrep.Com OTC drugs for Nausea, Vomiting, Constipation..

16) What is incorrect recommendation to prevent travelers diarrhea?


A-Avoid ice cubes B-Take fresh salad C-Take cooked food D-Used boiled or bottled
water E-Avoid sea food

17) Which of the following is the correct statement about nausea and vomiting in pregnancy
A-initial therapy is Diclectin B-initial therapy is dimenhydrinate and self care for nausea and
vomiting recommended C)Refer to emergency D)None of the above

18) A customer of your pharmacy, want to buy something for hard stool. What is good
recommendation
A) Senna B) Bran C) Psyllium D) Docusate E) Enema

19) A pregnant women comes to your pharmacy, and have nausea and vomiting. What is
appropriate to do initial therapeutic approach in first 3 days of nausea and vomiting?
A-Recommend dimenhydrinate to treat mild N&V and self care
B-Refer to doctor for Diclectin
C-Recommend self care measure to minimize nausea vomiting
D-Refer to emergency
E-Recommend ginger root tablets

20) Which of the following drug gives diarrhea?


A-Atropine B-Ipratropium C-Scopolamine D-Pilocarpine E-Trihexphenidyl

21. Bisacodyl mechanism of action (Dulcolax)


a. Stimulant laxative
b. Lubricant
c. Bulk forming
d. Stool softener
e. Stimulant laxative and stool softener

22) Which of the following microorganism have can cause traveler diarrhea in customer
travelling to south east Asia?
I) E. coli II) Shigella sp III) Compylobacter jejuni
a) I only b) III only c) I and II only d) II and III only e) I, II, III

23) A patient is buying Tylenol # 1, 200 tablets. Which of the following is pharmacist concern?
I) Constipation II) Liver toxicity III) Addiction
a) I only b) III only c) I and II only d) II and III only e) I, II, III

Copyright © 2000-2016 TIPS Inc. Unauthorized reproduction of this manual is prohibited. This manual is 72-3
being used during review sessions conducted by PharmacyPrep.
PharmacyPrep.Com OTC drugs for Nausea, Vomiting, Constipation..

24) A patient with terminal metastatic breast cancer gets the prescription of oxycontin and
Psyllium. Later after few days her daughter comes to pick a prescription of hydromorphone
and lactulose. What is pharmacist concern?
A) addiction potential B) False prescription C) Contraindication D) constipation E) Overdose

25) 1. What suggestions and advice should be given to the patient with pinworm infection?
I. Treatment of pin worms should be taken for all members of the family
II. Must not share under garments
III. Must not share dishes
a) I only b) III only c) I and II only d) II and III only e) All of the above

26. What is the drug of choice to treat chemotherapy induced nausea and vomiting?
A. Ondansetron B. Dexamethasone C. Lorazepam D. Metoclopramide E.
Dimenhydrinate
Ans. b

27. What is the drug of choice to treat constipation in children?


A. Castol oil
B. Senna
C. Biscodyl
D.Glycerine suppositories
Ans. d

Copyright © 2000-2016 TIPS Inc. Unauthorized reproduction of this manual is prohibited. This manual is 72-4
being used during review sessions conducted by PharmacyPrep.
PharmacyPrep.Com OTC drugs for Nausea, Vomiting, Constipation..

OTC DRUGS FOR NAUSEA, VOMITING, CONSTIPATION, DIARRHEA,


HEMORRHOIDS AND HELMENTHS
ANSWERS
1. Ans: E
2- Ans: D
Tips: Psyllium is safe to use in pregnancy

3- Ans: C

4- Ans: D

5- Ans: C
Tips: Portal – systemic encephalopathy. Osmotic laxative like lactulose, decrease blood
ammonia levels and as result this effect on mental status in encephalopathy patients.

6) Ans: A ( it was D before)

7) Ans: D

8) Ans: E

9) Ans: A

10) Ans: B

11) Ans: A

12) Ans: C

13) Ans: D

14) Ans: E

15) Ans: A

Copyright © 2000-2016 TIPS Inc. Unauthorized reproduction of this manual is prohibited. This manual is 72-5
being used during review sessions conducted by PharmacyPrep.
PharmacyPrep.Com OTC drugs for Nausea, Vomiting, Constipation..

16) Ans: B

17) Ans: B

18) Ans: D

19) Ans: A

20) Ans: D

21. Ans: A

22) Ans: E

23) Ans: C

24) Ans: B

25) Ans: E
Tips: spreads by oral-fecal infestation.

Copyright © 2000-2016 TIPS Inc. Unauthorized reproduction of this manual is prohibited. This manual is 72-6
being used during review sessions conducted by PharmacyPrep.
PharmacyPrep.Com OTC drugs for headache, sports injuries, pressure ulcers, and back pain

PHARMACY PREP

OTC DRUGS FOR HEADACHE, SPORTS INJURIES, PRESSURE ULCERS, AND LOW
BACK PAIN

1. Correct statement/s about ASA may include


I. Low dose are used in heart problem
II. Less potent than naproxen as an analgesic
III. More potent than acetaminophen as analgesic
a) I only b) III only c) I and II only d) II and III only e) All of the above

2-What is antidote of acetyl salicylic acid (ASA)?


A-N-acetylcysteine B-Flumazenil C-Naloxone D-Naltrexone E-None of the above

3-Anti-inflammatory action of ASA is due to?


A-Irreversible inhibition of platelet aggregation
B-Prevention of prostaglandin synthesis
C-Inhibition of COX-I receptors
D-Inhibition of Cox-1 and Cox-II receptors
E-Inhibition of Cox-II receptors

4. Dyspnea means
a) Painful muscle spasms b) Pain in the heart c)Pain in extremities d)Painful breathing
e) Painful menstruation

5. Over dosage of acetaminophen in patients would affect


I) Liver functions ii) Prothrombin time iii) Renal functions
a) I only b) III only c) I and II only d) II and III only e) All of the above

6-Which of the following is not a side effect of an opioids analgesic for example morphine:
a) C.V depression
b) Constipation
c) Sedation
d) CNS depression
e) Dilatation of eye pupil

7-A sports person has ankle sprain while playing ice hockey, what is inappropriate
recommendation?
A-Rest to injured part
B-Apply ice
C-Warm compress
D-Elevate injured area
E-Recommend OTC NSAIDs

Copyright © 2000-2016 TIPS Inc. Unauthorized reproduction of this manual is prohibited. This manual is 73-1
being used during review sessions conducted by PharmacyPrep
PharmacyPrep.Com OTC drugs for headache, sports injuries, pressure ulcers, and back pain

8) Mechanism of action of sumatriptan is?


A) 5HT 1b/d agonist
B) 5HT 1b/d antagonist
C) 5HT 3 antagonist
D) 5HT 3 agonist
E) 5HT 2 agonist

9) What is treatment for tension headache?


A) Triptans B-Ergot alkaloids C-Propranolol D-Acetyl salicylic acid
E) Acetaminophen

10) All of the following can trigger migraine, except?


A) Smell
B) Food like chocolates
C) Light
D) Emotional stress
E) Physical activity

11) A 45 yo man diagnosed with benign prostatic hyperplasia, and migraine prophylaxis.
Which of the following is the appropriate therapy for migraine prophylaxis?
A) Propranolol
B) Nortriptyline
C) Amitriptyline
D) Sumatriptan
E) Zolmitriptan

12) A customer of your pharmacy using sumatriptan for migraine attacks. What is correct
statement?
A)after taking sumatriptan migraine headache does not relieve than double the dose of
sumatriptan
B)after taking sumatriptan migraine headache does not relieve than decrease the dose of
sumatriptan
C)after taking sumatriptan migraine headache does not relieve than do not use sumatriptan
D. If sumatriptan is not effective in relieving headache then try other triptan
E. None of the above

13) A 12 month old child swallowed acetaminophen 10 ml syrup. A panic mother contact
pharmacy. What is the initial symptoms of acetaminophen overdose?
A) Nausea B) Vomiting C) Stomach upset D) Diarrhea E) Hepatotoxicity

14.Nociceptive pain are caused by?


A. Psychological factors
B. Injury to body tissue

Copyright © 2000-2016 TIPS Inc. Unauthorized reproduction of this manual is prohibited. This manual is 73-2
being used during review sessions conducted by PharmacyPrep
PharmacyPrep.Com OTC drugs for headache, sports injuries, pressure ulcers, and back pain

C. Damage to nerve
D. Diabetes
E. Post herpes infection
Ans. B

15. A patient with renal disease and has low back pain. What is the treatment option?
A. Diclofenac topical
B. Capsaicin
C. Oral NSAIDs
D. Methadone
E. Codeine
Ans. A

16. All of the following can be used to manage low back pain, except?
A. Tramadol
B. Acetaminophen
C. Diclofenac
D. Colchicine
E. Tramacet
Ans. D

17. A 12 yo boy got severe injury in his ankle during a soccer game. All of the following are
appropriate, except?
A. use acetaminophen for pain
B. use ibuprofen pain
C. Apply cold compress
D. Apply warm compress
E. Rest injured area
Ans. D

18. Fingolimod is?


A. spingosine-1-phosphate receptor agonist
B. Adhesion molecule
C. TNF alpha inhibitor
D. 5-alpha reductase inhibitor
E. Anti-CD52 monoclonal antibody
ans. A

Copyright © 2000-2016 TIPS Inc. Unauthorized reproduction of this manual is prohibited. This manual is 73-3
being used during review sessions conducted by PharmacyPrep
PharmacyPrep.Com OTC drugs for headache, sports injuries, pressure ulcers, and back pain

OTC DRUGS FOR HEADACHE, SPORTS INJURIES, PRESSURE ULCERS, AND LOW
BACK PAIN
ANSWERS:

1. Ans: C

2- Ans: E

3- Ans: B

4. Ans: D

5. Ans: C

6- Ans: E

7- Ans: C
Tips: R-I-C-E is recommended approach for sports injuries.
R= rest to injured part, I = apply ice, C = cold compress, E = elevated injured part.

8) Ans: A

9) Ans: E

10) Ans: D

11) Ans: A

12) Ans: C

13) Ans: B

Copyright © 2000-2016 TIPS Inc. Unauthorized reproduction of this manual is prohibited. This manual is 73-4
being used during review sessions conducted by PharmacyPrep
PharmacyPrep.Com OTC drugs for headache, sports injuries, pressure ulcers, and back pain

Copyright © 2000-2016 TIPS Inc. Unauthorized reproduction of this manual is prohibited. This manual is 73-5
being used during review sessions conducted by PharmacyPrep
www.PharmacyPrep.Com Asthma and COPD

PHARMACY PREP
ASTHMA AND COPD

1) Theophylline serum levels are monitored in neonates and infants because


A) They are poorly absorbed in systemic circulations
B) Drug has narrow therapeutic index
C) Slowest rate of clearance
D) Fastest rate of clearance
E) It is not used in neonates and infants

2) A patient is suffering from COPD and asthma. He is on theophylline, ranitidine and


now doctor prescribed ciprofloxacin which of the following is correct?
A) ranitidine have drug interaction with theophylline
B)Ciprofloxacin have drug interaction with theophylline
C)Ranitidine and ciprofloxacin both have drug interactions with theophylline
D) None of the above

3) Which of the following is NOT a side effect of theophylline?


A-Insomnia
B-Sedation
C-Appetite suppressant
D-None of the above

4) A 3 yr child was newly diagnosed with asthma. All of the following questions are
important to ask his mother EXCEPT:
A) Any cardiovascular problems?
B) Does other family member have asthma?
C) How often symptoms come?
D) Old asthma medication he used before?
E) How many times wake up in night due to symptoms

5) A 45 yo patient with emphysema and pain, and is not controlled, the doctor switched
him to morphine and his daughter came to pharmacy, you will call the doctor because:
a) You will ask to change the medication
b) To tell the dose of medication because its too much
c) To ask for written prescription of morphine
d) Her concern about constipation

6. What is true about Ipratropium?


I. Reversible competitive of anticholinergic
II. Synergistic with Theophylline in asthma
III. Quaternary ammonium compound
a. I only
b. III only
Copyright © 2000-2016 TIPS Inc. Unauthorized reproduction of this manual is strictly prohibited and 74-1
it is illegal to reproduce without permission. This manual is being used during review sessions
conducted by PharmacyPrep.
www.PharmacyPrep.Com Asthma and COPD

c. I and III only


d. II and III only
e. I, II, and III only

7) Which of the following is incorrect statement of leukotriene receptors inhibitors?


A) used to control the dose of steroids
B) Indicated to treat congestion in children over 2 yr
C) Treatment of choice for beta blockers and Aspirin induced asthma
D) Patient used leukotriene receptors inhibitors require increase requirement of
steroids.
E) Montelukast is used in children over 2 yr age

8) A 40 yr old asthma patient brings a new prescription of montelukast bid. Currently


medication profile include salbutamol prn, Symbicort (budesonide and formoterol) 1
puff bid. What is the action to do?
A) ask patient to stop using Symbicort
B) ask patient to stop using salbutamol prn
C) dispense montelukast and counsel how to use it
D) Contact doctor to change Symbicort to Advair
E) Contact doctor to change to Zafirlukast

9) A COPD patient brings a prescription of doxycycline daily to treat pneumonia. Which


of the following is the correct statement?
A) ask patient if he is taken flu vaccine
B) ask patient if his close contacts have taken flu vaccine
C) ask patient if his close contacts have taken flu and pneumonia vaccine
D) ask patient if he had taken flu and pneumonia vaccine
E) ask patient if he has taken pneumonia vaccine only

10. What is inappropriate for acute asthma?


A) beta2 agonist inhaler B) iv hydrocortisone C) iv epinephrine D) inhaler
anticholinergic

11. A 55 yo male admitted in emergence with asthma exacerbations. What is the


preferred drug of choice?
A) corticosteroids B) salbutamol nebulizer C) antihistamine D) theophylline

12. Which of the following asthma device at home helps patient to determine when to
go emergence?
A) Peak expiratory flow rate (PEFR)
B) Forced expiratory volume in one second (FEV1)
C) Forced vital capacity (FVC)
D) Total lung capacity

Copyright © 2000-2016 TIPS Inc. Unauthorized reproduction of this manual is strictly prohibited and 74-2
it is illegal to reproduce without permission. This manual is being used during review sessions
conducted by PharmacyPrep.
www.PharmacyPrep.Com Asthma and COPD

13. Which of the following is NOT a side effect of oral prednisone?


A) weight gain
B) acute pancreatitis
C) Hypertension
D) viral infections
E) Steven Johnson syndrome

14) The following medications are used in asthma emergencies:


A. Salbutamol
B. Ipratropium
C. Salmeterol
D. All of the above

15) Asthma acute exacerbation can be triggered by all of the following EXCEPT:
A. warm and dry weather
B. stressful emotional time
C. bacterial and viral pneumonia
D. withdrawal of asthma medications

16) Emphysema is characterized by:


A. Permanent enlargement of small sacs in lungs
B. Symptoms are scanty sputum
C. Shortness of breath.
D. All of the above

17) Which of the following inhaler require rinsing mouth after use.
A. Servent (salmetrol)
B. Ventolin (albuterol)
C. Atrovent (ipratropium)
D. Pulmicort (budesonide)

18) In emergency treatment of asthma, the preferred first line treatment is:
A. theophylline
B. salbutamol
C. cromolyn sodium
D. Zafirlukast

19. Nosocomial infection is


A. Nasal infection
B. Mixed bacterial infection
C. Mixed viral infection
D. Infections acquired in a hospital or other health care settings

Copyright © 2000-2016 TIPS Inc. Unauthorized reproduction of this manual is strictly prohibited and 74-3
it is illegal to reproduce without permission. This manual is being used during review sessions
conducted by PharmacyPrep.
www.PharmacyPrep.Com Asthma and COPD

20. Drugs that Acts as long acting B 2 adrenergic agonist:


A. Isoetharine
B. Metaproterenol
C. Terbutaline
D. Salmeterol

21) Which drug is not use in asthma?


A. B2 adrenergic antagonist
B. Methylxanthine
C. Leukotriene antagonist
D. Corticosteroid

22) All of the following are side effects of Salbutamol except:


A. Tremors
B. Palpitation
C. Restlessness
D) Oral thrush

23) Therapeutic use of Pulmicort:


A. Prevent Nocturnal asthma
B. Prevent exercise induced asthma (EIA)
C. Indicated in COPD and Emphysema
D. All of the above

24) Which drug decreases effect of Theophylline?


A. Ciprofloxacin
B. Allopurinol
C. Pulmonary edema
D. Carbamazepine

25) Drug of choice for Aspirin induced asthma:


A. Salbutamol
B. Zafirlukast
C. Montelukast
D. Fluticasone

26) Which anticholinergic drugs is used in asthma?


A. Ipratropium bromide
B. Scopolamine
C. Pilocarpine
D. Atropine

Copyright © 2000-2016 TIPS Inc. Unauthorized reproduction of this manual is strictly prohibited and 74-4
it is illegal to reproduce without permission. This manual is being used during review sessions
conducted by PharmacyPrep.
www.PharmacyPrep.Com Asthma and COPD

27) Fluticasone drugs are use to:


A. Control inflammation in asthma and allergic rhinitis.
B. Control asthma only
C. Treat Aspirin induced asthma
D. Relaxes the muscles surrounding the bronchioles.

28. Combivent available dosage forms is/are:


A. Nebulizers
B. MDI
C. A & B
D. Capsule

29. Risk factors that causes COPD except:


A. Second hand smoke
B. Family history
C. Asthma
D. Age

30. High risk factor for COPD


A. Asthma
B. Smoking
C. Second hand smoke
D. Air pollution

31. Function as mast cell inhibitor:


A. Cromolyn sodium
B. Budesonide
C. Zafirlukast
D. Beclomethasone

32) Onset of action for short acting drugs is?


A. 10-15 minutes
B. 10-20 seconds
C. 10-15 seconds
D. 10-20 minutes

33) What to do to reduce corticosteroids side effects?


A. Stop corticosteroid
B. Rinse mouth and gargle after using the drug
C. Chew a gum
D. Brush your teeth every after using the drug

Copyright © 2000-2016 TIPS Inc. Unauthorized reproduction of this manual is strictly prohibited and 74-5
it is illegal to reproduce without permission. This manual is being used during review sessions
conducted by PharmacyPrep.
www.PharmacyPrep.Com Asthma and COPD

34) Drugs are not affected by catecholamine O-methyl transferase (COMT) enzymes
except:
A. Salmeterol
B. Salbutamol
C. Metoproterenol
D. Terbutaline

35) Function as mast cell stabilizers:


A. Zafirlukast
B. Ipratropium
C. Prednisone
D. Nedocromil

36) Rx: Beclomethasone (Vanceril) inhaler: dispense # 1


Sig: ii puffs inhaled bid. In filling the prescription, the technician should:
A. Inform the patient that this is a corticosteroid that has many serious side effects.
B. Include the following specific directions on the label: “Inhale 2 puffs into each nostril
3x a day”
C. Provide the patient the package insert included with the inhaler
D. Tell the patient to use this inhaler only when he really needs it.

37) All of the following drugs can be indicated for the treatment of asthma EXCEPT:
A. Ipratropium
B. Metoprolol
C. Albuterol
D. Zafirlukast

38) A 45 yo patient with emphysema and pain, and is not controlled, the doctor switched
him to morphine and his daughter came to pharmacy, you will call the doctor because:
a) You will ask to change the medication
b) To tell the dose of medication because its too much
c) To ask for written prescription of morphine
d) Her concern about constipation

39) What is true about Ipratropium?


I. Reversible competitive of anticholinergic
II. Synergistic with Theophylline in asthma
III. Quaternary ammonium compound
a) I only
b) III only
c) I and III only
d) II and III only
e) I, II, III

Copyright © 2000-2016 TIPS Inc. Unauthorized reproduction of this manual is strictly prohibited and 74-6
it is illegal to reproduce without permission. This manual is being used during review sessions
conducted by PharmacyPrep.
www.PharmacyPrep.Com Asthma and COPD

40) Which of the following is incorrect statement of leukotriene receptors inhibitors?


A) used to control the dose of steroids
B) Indicated to treat congestion in children over 2 yr
C) Treatment of choice for beta blockers and Aspirin induced asthma
D) Patient used leukotriene receptors inhibitors require increase requirement of steroids.
E) Montelukast is used in children over 2 yr age

41) A 40 yr old asthma patient brings a new prescription of montelukast bid. Currently
medication profile include salbutamol prn, Symbicort (budesonide and formoterol) 1
puff bid. What is the action to do?
A) ask patient to stop using Symbicort
B) ask patient to stop using salbutamol prn
C) dispense montelukast and counsel how to use it
D) Contact doctor to change Symbicort to Advair
E) Contact doctor to change to Zafirlukast

42) A COPD patient brings a prescription of doxycycline daily to treat pneumonia.


Which of the following is the correct statement?
A) ask patient if he is taken flu vaccine
B) ask patient if his close contacts have taken flu vaccine
C) ask patient if his close contacts have taken flu and pneumonia vaccine
D) ask patient if he had taken flu and pneumonia vaccine
E) ask patient if he has taken pneumonia vaccine only

43) What is inappropriate for acute asthma?


A) beta2 agonist inhaler B) iv hydrocortisone C) iv epinephrine D) inhaler
anticholinergic

44) A 55 yo male admitted in emergence with asthma exacerbations. What is the


preferred drug of choice?
A) corticosteroids B) salbutamol nebulizer C) antihistamine D) theophylline

45) Which of the following asthma device at home helps patient to determine when to go
emergence?
A) Peak expiratory flow rate (PEFR)
B) Forced expiratory volume in one second (FEV1)
C) Forced vital capacity (FVC)
D) Total lung capacity

46) Which of the following is NOT a side effect of oral prednisone?


A) weight gain
B) acute pancreatitis
C) Hypertension
D) viral infections
E) Steven Johnson syndrome

Copyright © 2000-2016 TIPS Inc. Unauthorized reproduction of this manual is strictly prohibited and 74-7
it is illegal to reproduce without permission. This manual is being used during review sessions
conducted by PharmacyPrep.
www.PharmacyPrep.Com Asthma and COPD

47) What is true about ipratropium?


I) It is reversible competitive inhibitor of acetylcholine
II) this give synergistic effect to theophylline in asthma
III) Quaternary amine salts
a) I only
b) III only
c) I and III only
d) II and III only
e) I, II, III

48) The volume of air that passes in and holds and out of the lungs in normal breathing
is called?
A. Total lung capacity
B. Tidal volume
C. Breathing capacity
D. Peak flow
E. Vital capacity

49) Wheezing results from?


A. obstruction of airways due to narrowing
B. irreversible obstruction of airways due to permanent enlargment of alveoli
C. obstruction of airways due to particles
D. onstruction of airways due to infection

50) Orthopnea is?


A. Shortness of breath when a person lie down but relieved when sit.
B. Shortness of breath when a person lie sit but relieved when lie down
C. Shortness of breath when a person standing and relieved when sit
D. Shortness of breath when a person sit and relieved when stand

51.Which of the following is diagnostic test NOT related asthma?


A. spirometer
B. bronchoprovocation challenge test
C. peak flow meter
D. alpha1 antiprotease levels
E. methanacholine or histamine test

52. What is the drug of choice for excercise induced asthma?


A. Salbutamol B. ICS C. Fluticasone D. Salmeterol
Ans. A

53.What drug is used if asthma is not controlled by salbutamol use alone?


A. Prednisone
Copyright © 2000-2016 TIPS Inc. Unauthorized reproduction of this manual is strictly prohibited and 74-8
it is illegal to reproduce without permission. This manual is being used during review sessions
conducted by PharmacyPrep.
www.PharmacyPrep.Com Asthma and COPD

B. Fluticasone inhaler
C. hydrocortisone injection
D. Long acting beta agonist
E. Tiotropium
Ans. b

54. What is the drug of choice for asthma in hospital emergency?


A. fluticasone
B. oral prednisone
C. hydrocortisone inj
D. topical steroid
E. oxygen
Ans. c

55. Which of the following is used for asthma attacks or exacerbations?


I. Salbutamol
II. Ipratropium
III. Theophylline
A. I only B. III only C. I and II D. II and III E. I,II,III
Ans.C

56. Dysphonia is side effect of?


A. ipratropium
B. Prednisone oral
C. Hydrocortisone injection
D. Formeterol
E. Fluticasone inhaler
ans. e

57. When do you use omalizumab?


A. Allergens induced asthma
B. excercise induced asthma
C. Drug induced asthma
D. Asthma maintenance therapy
E. All of the above
Ans. A

58. In COPD which of the following is least likely used?


A. Tiotropium
B. Salmeterol
C. Formeterol
D. Fluticasone inhaler
E.oral prednisone
Copyright © 2000-2016 TIPS Inc. Unauthorized reproduction of this manual is strictly prohibited and 74-9
it is illegal to reproduce without permission. This manual is being used during review sessions
conducted by PharmacyPrep.
www.PharmacyPrep.Com Asthma and COPD

Ans. d

59. A copd patient prescribed theopylline. Which of the following least likely toxic
symptoms if theophylline serum levels are over 10-20 mcg/ml?
A.vomiting B. tachycardia C. sweating D. anxiety & restlness E.sedation
Ans. E

60. Which of the following least likely used in COPD or emphysema?


A. antibiotics B. salbutamol C. ipratropium D. theophylline E. ICS
Ans. E

Copyright © 2000-2016 TIPS Inc. Unauthorized reproduction of this manual is strictly prohibited and 74-10
it is illegal to reproduce without permission. This manual is being used during review sessions
conducted by PharmacyPrep.
www.PharmacyPrep.Com Asthma and COPD

ASTHMA AND COPD


ANSWERS:

1)Ans: C
Tips: neonates and infants have slowest rate of clearance

2) Ans: C
Tips: Theophylline is substrate of CYP1A2, and ciprofloxacin is the inhibitor of CYP1A2

3) Ans: B

4) Ans: A

5) Ans: D

6. Ans: E
Tips: Theophylline is added id doses not response to anticholinergic)

7) Ans: D

8) Ans: C

9) Ans: D

10. Ans: C

11. Ans: B
Tips: In an emergence situation, the most rapidly acting agent is used first. The choice of
route of administration depends on severity of attack.

12. Ans: A

13. Ans: E

14) Ans-A

15) Ans-A

16) Ans: D

17) Ans-D

18) Ans-B

Copyright © 2000-2016 TIPS Inc. Unauthorized reproduction of this manual is strictly prohibited and 74-11
it is illegal to reproduce without permission. This manual is being used during review sessions
conducted by PharmacyPrep.
www.PharmacyPrep.Com Asthma and COPD

19. Ans: D

20. Ans: D

21) Ans: A

22)Ans: D

23)Ans: D

24) Ans: D

25) Ans: C ( it was B before)

26) Ans: A

27) Ans: A

28. Ans: C

29. Ans: D

30. Ans: B

31. Ans: A

32) Ans: D

33) Ans: B

34) Ans: A
Tips: Short acting is not affected by COMT enzyme because this is activated by non
COMT enzyme.

35) Ans: D

36) Ans: C

37) Ans: B
Tips: Asthma is characterized by increased responsiveness of trachea and bronchi to
various stimuli and narrowing of airways. It is associated with shortness of breath, chest
tightness, wheezing and coughing. The Beta-2 receptors stimulation generally dilates the
bronchial smooth muscles and helps in controlling asthma.
Copyright © 2000-2016 TIPS Inc. Unauthorized reproduction of this manual is strictly prohibited and 74-12
it is illegal to reproduce without permission. This manual is being used during review sessions
conducted by PharmacyPrep.
www.PharmacyPrep.Com Asthma and COPD

38) Ans: D

39) Ans: E
Tips: Theophylline is added id doses not response to anticholinergic)

40) Ans: D

41) Ans: C

42) Ans: D

43) Ans: C

44) Ans: B
Tips: In an emergence situation, the most rapidly acting agent is used first. The choice of
route of administration depends on severity of attack.

45) Ans: A

46) Ans: E

47) Ans: E

48)Ans. B

49) Ans. A

50) Ans. A

51) Ans. D (no answer was given)

Copyright © 2000-2016 TIPS Inc. Unauthorized reproduction of this manual is strictly prohibited and 74-13
it is illegal to reproduce without permission. This manual is being used during review sessions
conducted by PharmacyPrep.
PharmacyPrep.Com Smoking Cessation

PHARMACY PREP
SMOKING CESSATION

1-What is incorrect statement about smoking cessation?


A-Person using nicotine replacement therapy, should stop smoking same time as start NRT
B-Person using bupropion can set quit date in 1 to 2 week of after start of treatment
C-A person using nicotine patches can smoke but reduce number of cigarettes
D-A person with hypertension, body weight less than 45 Kg should start with low dose of NRT
i.e. step 2 patch
E-Patches can be applied on while taking shower

2) Nicotine patches are stored in?


A-Refrigerator B-Room temperature C-Cold temperature
D-Freezer E-Any temperature

3) What is incorrect about nicotine patches?


A. Stored in room temperature
B. Avoid sun light exposure
C. Hygroscopic, thus open from pack when you ready to use
D. Does not need to protect from light
E. Can be applied on arms and thighs

4) What is incorrect about bupropion?


A-Reduce the seizure threshold
B- Avoid in bulimia nervosa
C-Avoid in anorexia nervosa
D-Avoid in sexual dysfunction
E-It has low weight gain side effects

5-Which of the following is least likely caused by bupropion?


A-anorexia
B-weight gain and sexual dysfunction
C- se re res
D-hypertension
E-gingivitis

6) All of the following can be used in smoking cessation, EXCEPT:


A) Fluoxetine
B. bupropion
C. Nicotine transdermal patch
D. Nicotine inhaler
E. Nicotine gum

Copyright © 2000-2016 TIPS Inc. Unauthorized reproduction of this manual is prohibited. This manual is 75-1
being used during review sessions conducted by PharmacyPrep
PharmacyPrep.Com Smoking Cessation

7. What is the first line therapy for smoking cessation?


a. nicotine replacement therapy
B. bupropion
C. varenicline
D. Clonidine
E. atomoxetine
Ans. A

Copyright © 2000-2016 TIPS Inc. Unauthorized reproduction of this manual is prohibited. This manual is 75-2
being used during review sessions conducted by PharmacyPrep
PharmacyPrep.Com Smoking Cessation

SMOKING CESSATION
ANSWERS:

1) Ans: C

2) Ans: B

3) Ans: D

4) Ans: D
Tips: bupropion has least sexual dysfunction among antidepressants.

5) Ans: B

6) Ans: A

Copyright © 2000-2016 TIPS Inc. Unauthorized reproduction of this manual is prohibited. This manual is 75-3
being used during review sessions conducted by PharmacyPrep
PharmacyPrep.Com Sleeping Disorders

PHARMACY PREP

SLEEPING DISORDERS

1) What is incorrect recommendations for persons complaining insomnia?


A-make a regular sleep schedule for 7 days/wk
B-Over the counter diphenhydramine for 3 days may be helpful
C-It is essential to exercise before bedtime
D-Avoid heavy meals before bedtime
E-Avoid long day nap can give good sleep in night

2-What is incorrect about zopiclone?


A-A non benzodiazepine act on benzodiazepine receptor
B-Associated with sever hangover effect
C-It indicated for short time insomnia
D-It is short acting
E-It has low dependency

3-All of the following are long acting benzodiazepines except?


A-Diazepam
B-Clonazepam
C-Flurazepam
D-Temazepam
E-Chlorozepate
Tips: TEM are short acting Temazepam Estazolam, and Midazolam

4) Zopiclone is?
A-Non-benzodiazepine act on BZ 1 receptors
B-Benzodiazepine act on BZ 1 receptors
C-Non-benzodiazepine act on BZ 2 receptors
D-Non-benzodiazepine act on BZ 1 and BZ 2 receptors
E-Benzodiazepine act on BZ 2 receptors

5) A 75 yo senior is receiving treatment of anxiety, paroxetine daily, clonazepam 5 mg bid,


oxazepam 10 mg tid. Which of the following is the important to monitor?
A) Insomnia
B) Anxiety
C) Disorientation and falls
D) Confusion
E) Depression

6) Which of the following drug has NOT taste change side effects?
A) zopiclone B) captopril C) metformin D) tetracycline E)metronidazole

Copyright © 2000-2016 TIPS Inc. Unauthorized reproduction of this manual is prohibited. This manual is 76-1
being used during review sessions conducted by PharmacyPrep
PharmacyPrep.Com Sleeping Disorders

7) Which of the following drug has faster rebound insomnia side effects?
A) Triazolam B) Diazepam C) Oxazepam D) lorazepam E) Alprazolam

8) What benzodiazepines are preferable to treat the type of insomnia that is associated with
difficulty in falling in sleep?
A) Short acting B) Intermediate acting C) Long acting D) intermediate and long acting E)
short and long acting

9) MK is 50 year patient comes to pick up prescription of lorazepam 0.5 mg tab. Which of the
following statement is relevant?
A. Taking alcohol concomitant with lorazepam can cause severe sedation thereby avoid.
B) Take benzodiazepine with food
C) avoid taking with grapefruit juice
D) All of the above

Copyright © 2000-2016 TIPS Inc. Unauthorized reproduction of this manual is prohibited. This manual is 76-2
being used during review sessions conducted by PharmacyPrep
PharmacyPrep.Com Sleeping Disorders

SLEEPING DISORDERS
ANSWERS

1) Ans: C
Tips: avoid exercise at least 2 to 3 hours before bedtime

2) Ans: B

3) Ans: D

4) Ans: A

5) Ans: C
Tips: Lethargy = fatigue; Hangover = disorientation; Sedation = sleepy; Somnolence = sleepy

6) Ans: D

7) Ans: A

8) Ans: A

9) Ans: A

Copyright © 2000-2016 TIPS Inc. Unauthorized reproduction of this manual is prohibited. This manual is 76-3
being used during review sessions conducted by PharmacyPrep
PharmacyPrep.Com Eating disorders and weight loss treatments

PHARMACY PREP
EATING DISORDER AND WEIGHT LOSS TREATMENTS

1. What is incorrect about anorexia nervosa?


I-It is characterized by deliberate loss of weight
II-refusal to maintain normal body weight, fear of weight gain and amenorrhea
III-Binge eating is common in anorexia nervosa
A-I only B-III only C-I and II only D-II and III only E-All of the above

2-You notice Ms A’s, a young girl of normal weight, in the pharmacy purchasing laxatives. You
take the time to talk to her about her laxative use, and find that she has been binge eating
and using laxatives about three times per week for the last six months.
Based on this information, which of the following statements is most likely to be correct?
A. She may have bulimia nervosa of the non-purging type since she is not self-inducing
vomiting
B. She may have bulimia nervosa of purging type since she uses laxatives
C. She does not have bulimia nervosa, since she is of normal weight
D. She does not have bulimia nervosa, since her bingeing episodes are not frequent enough

3-What is overweight body mass index (BMI)


A. 18.5 to 24.9
B. 25.0 to 29.9
C. >30
D. <30
E. All of the above

4-What is obese body mass index (BMI)


A. 18.5 to 24.9
B. 25.0 to 29.9
C. >30
D. <30
E. All of the above

5-All of the following drugs are used to treat weight loss except?
A-Meridia B) Orlistat C-Bupropion D-Topiramate E-Mirtazapine

6-Which of the following antidepressant should be avoided in bulimia an anorexia nervosa?


A-Venlafaxine B-Fluoxetine C-Bupropion D-Mirtazapine
E-Paroxetine

7) What is body mass index (BMI)


A) Weight in kg/height in meter2

Copyright © 2000-2016 TIPS Inc. Unauthorized reproduction of this manual is prohibited. This manual is 77-1
being used during review sessions conducted by PharmacyPrep
PharmacyPrep.Com Eating disorders and weight loss treatments

B)Weight in kg/height in meters


C) height in meter2 / weight in kg
D)Weight in grams/ height in m
E) none of the above

8-What is normal body mass index (BMI)


A- 18.5 to 24.9
B- 25.0 to 29.9
C- >30
D- <30
E- All of the above

Copyright © 2000-2016 TIPS Inc. Unauthorized reproduction of this manual is prohibited. This manual is 77-2
being used during review sessions conducted by PharmacyPrep
PharmacyPrep.Com Eating disorders and weight loss treatments

EATING DISORDERS
ANSWERS

1.Ans: B

2- Ans: B

3- Ans: B
Tips: BMI = weight/height2 = kg/m2

4- Ans: C
Tips: = kg/m2

5- Ans: E

6- Ans: C

7) Ans: A
Tips: BMI = weight/height2 = kg/m2

8- Ans: A

Copyright © 2000-2016 TIPS Inc. Unauthorized reproduction of this manual is prohibited. This manual is 77-3
being used during review sessions conducted by PharmacyPrep
PharmacyPrep.Com Eating disorders and weight loss treatments

Copyright © 2000-2016 TIPS Inc. Unauthorized reproduction of this manual is prohibited. This manual is 77-4
being used during review sessions conducted by PharmacyPrep
Pharmacyprep.com

PHARMACY PREP

GASTROINTESTINAL DISORDERS

1) Correct statements regarding the characteristics of hernias may include:


I-It is a protrusion of a loop of an organ or tissue normally through the GI area
II-Hernia hiatus is a herniation of stomach through the esophageal hiatus of diaphragm
III-Strangulated hernia is an incarcerated hernia that is so tightly constricted as to
compromise the blood supply of the contents of the hernia sac.
a) I only
b) III only
c) I and II only
d) II and III only
e) All are correct

2- Misoprostol is used as mucosal protectants of GI and may be classified as:


a) PGE 1 analog
b) H 2 inhibitor
c) Proton pump inhibitor
d) NSAID
e) Antimuscarinic

3- Drugs are mainly metabolized in the GIT by many different reactions. The most
common drug reaction in the GIT may include:
a) Oxidation
b) Reduction
c) Hydrolysis
d) Dealkylation
e) Ionization

4. Acetyl salicylic acid (ASA) is one of the most used worldwide analgesics. Which of the
following reactions metabolizes it in the gastrointestinal tract and liver?
a) Oxidation
b) Hydrolysis
c) Reduction
d) Dealkylation
e) Deamination

5- Accumulation of fluids in the peritoneal cavity is normally known as:


a) Pannus
b) Starvation
c) Edema
d) Ascitis
e) Cholecystitis

Copyright © 2000-2016 TIPS Inc. Unauthorized reproduction of this manual is prohibited. This 78-1
manual is being used during review sessions conducted by PharmacyPrep.
Pharmacyprep.com

6- Examples of the used of Karaya gum in ostomy patients may include:


a) Decrease odor from colostomy
b) Act as germicidal agent
c) Liquidify the stomal effluent
d) Protect skin from stomal effluent
e) Soften the stomal effluent

7- All of the following are examples of gastric acid stimulators, EXCEPT:


a) Gastrin
b) Acetylcholine
c) Somatotropin
d) G-Cell
e) None of the above

8- Gastroesophageal reflux (GERD) is mainly characterized by:


a) Diaphragm wall inflammation
b) Diaphragm perforation
c) Reflux of the stomach content into the esophagus
d) Hyperacidity of stomach
e) All are correct

9) One of the most common conditions associated with acute hemorrhagic gastritis may
include:
a) Chronic alcoholism
b) H. pyloric bacterial infection
c) Carcinoma of stomach
d) Carcinoma of colon
e) Traveler’s diarrhea

10) Antacids are normally used to equilibrate the stomach pH and protect the stomach
from the effect of acids. Which of the following antacids has a cathartic side effect?
a) Calcium carbonate
b) Dihydroxyaluminum sodium carbonate
c) Magnesium hydroxide
d) Calcium sulfate
e) Aluminum silicate

11- Elongated proliferations in the small intestine are called:


I- Villi
II- Fontanelle
III- Hernias
a) I only
b) III only

Copyright © 2000-2016 TIPS Inc. Unauthorized reproduction of this manual is prohibited. This 78-2
manual is being used during review sessions conducted by PharmacyPrep.
Pharmacyprep.com

c) I and II only
d) II and III only
e) All are correct

12- Metoclopramide is a parasympathetic agent that may have which of the following
characteristics?
I- Cholinomimetic agent that stimulates motility of the stomach
II- Used in treatment of diabetes gastro paralysis and GERD
III- Used centrally as antiemetic during cancer chemotherapy
a) I only
b) III only
c) I and II only
d) II and III only
e) All are correct

13- All of the following are correct statements regarding bile, EXCEPT:
a) It is secreted by the liver
b) It is poured into the small intestine via the bile ducts
c) It is constituted of bile acids, cholesterol, phospholipid, bilirrubin and electrolytes
d) Bile is acid due to its bicarbonate content
e) It is also responsible for the alkalinization of intestinal content

14) Proton Pump Inhibitors are irreversible direct inhibitors of K pump. Correct
statements regarding these drugs may include:
I- Used in gastric and duodenal ulcers
II- Omeprazole and lansoprazole are examples of proton pump inhibitors
III- They decrease stomach acid secretion
a) I only
b) III only
c) I and II only
d) II and III only
e) All are correct

15- Zollinger-Ellison disease is a grave stomach disease. Drug of choice in the treatment
of Zollinger-Ellison diseases may include:
a) Ranitidine
b) Famotidine
c) Loratidine
d) Omeprazol
e) Antacids

16- Correct statements regarding the administration of antacids may include:


I- Taken 1hr before meals
II- Taken 3hrs after meals

Copyright © 2000-2016 TIPS Inc. Unauthorized reproduction of this manual is prohibited. This 78-3
manual is being used during review sessions conducted by PharmacyPrep.
Pharmacyprep.com

III- Taken 2hrs before bedtime


a) I only
b) III only
c) I and II only
d) II and III only
e) All are correct

17- All of the following enzymes are found in saliva, EXCEPT:


a) Amylase
b) Ptyalin
c) Gastrin
d) Lysoenzyme
e) All are correct

18- Which of the following is/are examples of salivary glands?


I- Parotid
II- Submandibular
III- Sublingual
a) I only
b) III only
c) I and II only
d) II and III only
e) All are correct

19- The digestive process is done in many different steps. The first step of digestion may
be taken in:
a) Mouth
b) Esophagus
c) Stomach
d) Small intestine
e) Large intestine

20- Wrong statement regarding stomach secretions may include:


a) Mucus is present in stomach as lubricant and protect surface from acid
b) Acids breakdown food, kill bacteria and convert pepsin to pepsinogen
c) Pepsinogen is also called “chief cell”
d) Gastrin is the one that inhibit acid secretion
e) The intrinsic factor in stomach is secreted from parietal cell

21- The small intestine is classified in three parts, the anatomical order division of small
intestine is known as:
a) Jejunum, duodenum and ileum
b) Ileum, jejunum and duodenum
c) Duodenun, jejunun and ileum

Copyright © 2000-2016 TIPS Inc. Unauthorized reproduction of this manual is prohibited. This 78-4
manual is being used during review sessions conducted by PharmacyPrep.
Pharmacyprep.com

d) Duodenun, ileum and jejunun


e) Jejunun, ileum and duodenun

22- Conditions that may be the main cause of duodenal ulcers include:
a) High motility of stomach
b) Bacterial migration
c) Squirting of acid stomach content into the duodenal wall
d) Reflux of stomach content into the esophagus
e) Hyperacidity of stomach

23- The pancreas is constituted of endocrine and exocrine glands. What is the function
of endocrine gland of pancreas?
a) Production of enzymes
b) Production of insulin
c) Production of proteins
d) Production of glycogen
e) Production of bile

24- Colon is the last part of the digestive system and may be anatomical divided in:
a) Ascending colon
b) Transverse colon
c) Descending colon
d) Sigmoid colon and rectum
e) All are correct

25- H 2 is involved in gastric acid and pepsin secretion. All of the following are examples
of H 2 inhibitors, EXCEPT:
a) Ranitidine
b) Nizatidine
c) Cimetidine
d) Omeprazole
e) Famotidine

26- H 2 inhibitors are used in the treatment of gastric hyper secretion and GI ulcers. One
of the most potent H 2 inhibitors may include:
a) Famotidine
b) Cimetidine
c) Ranitidine
d) Nizatidine
e) Promethazine

27- Example(s) of conditions that may be treated with H 2 inhibitors may include:
a) Gastric hyper secretion
b) Duodenal ulcers

Copyright © 2000-2016 TIPS Inc. Unauthorized reproduction of this manual is prohibited. This 78-5
manual is being used during review sessions conducted by PharmacyPrep.
Pharmacyprep.com

c) Zollinger-Ellison syndrome
d) Pain associated with GERD
e) All are correct

28- Proton pump inhibitors used in gastrointestinal complications are irreversible


inhibitors of:
a) Na+ pump
b) Na+/K+ pump
c) K+ pump
d) Cl- pump
e) Mg2+/K+ pump

29- The most common side effect during treatment with proton pump inhibitors may
include:
I- Headache
II- Diarrhea
III- GIT pain
a) I only
b) III only
c) I and II only
d) II and III only
e) All are correct

30- Ulcers are complications of gastrointestinal tract. The risk of having ulcers is bigger
in people having which of the following conditions:
a) Smokers
b) Alcoholics
c) NSAID long-term treatment
d) Corticosteroids long-term treatment
e) All are correct

31- Examples of effective treatment of ulcers may include:


I- Antacids
II- H 2 inibitors
III- Proton pump inhibitors
a) I only
b) III only
c) I and II only
d) II and III only
e) All are correct

32- Sulcrafate is used in gastrointestinal complications and may act as:


a) Antacid
b) H2 inibitor

Copyright © 2000-2016 TIPS Inc. Unauthorized reproduction of this manual is prohibited. This 78-6
manual is being used during review sessions conducted by PharmacyPrep.
Pharmacyprep.com

c) Mucosal protectant
d) Antisecretory prostaglandin analog
e) Proton pump inhibitor

33- Antacids are commonly used to neutralize gastric acid pH. Example of best antacids
constitution may include which of the following combinations?
a) Magnesium hydroxide only
b) Aluminum hydroxide only
c) Calcium carbonate only
d) Magnesium and aluminum hydroxide
e) Calcium and aluminum hydroxide

34- Antacids neutralize the gastrointestinal acid pH by which of the following


mechanism of action?
a) Reduce the concentration and total load of acid in the gastric content by increasing
gastric pH.
b) Inhibit (H+, K+)-ATPase enzyme system at the secretory surface of the gastric parietal
cell
c) Inhibit H 2 neurotransmitters
d) Inhibit gastrin secretion
e) Inhibit the conversion of pepsin to pepsinogen

35- Diarrhea is a common side affect seen with which of the following antacids?
a) Aluminun containing antacid
b) Magnesium containing antacid
c) Calcium containing antacid
d) Magnesium and aluminum containing antacid
e) Calcium and aluminum containing antacid

36- Constipation and/or diarrhea may be side effects of antacids therapy. Which of the
following antacids may have constipation as side effect?
a) Aluminun containing antacid
b) Magnesium containing antacid
c) Calcium and magnesium containing antacid
d) Magnesium and aluminum containing antacid
e) Calcium and aluminum containing antacid

37- Correct statements regarding the advantages of taken antacids mixtures of


aluminum and magnesium may include:
I- Provides more sustained action
II- Permit a lower dose of each compound
III- The constipation effect of aluminum may counter the diarrhea caused by magnesium
a) I only
b) III only

Copyright © 2000-2016 TIPS Inc. Unauthorized reproduction of this manual is prohibited. This 78-7
manual is being used during review sessions conducted by PharmacyPrep.
Pharmacyprep.com

c) I and II only
d) II and III only
e) All are correct

38- Laxative is normally contra-indicated during pregnancy. The only agent with laxative
effects that may be used during pregnancy include:
a) Senna
b) Magnesium hydroxide
c) Psyllium
d) Docusate sodium
e) Bisacodyl

39- Examples of stimulant laxative may include all of the following, EXCEPT:
a) Senna
b) Cascara
c) Bisacodyl
d) Lactulose
e) Castor oil

40- Common or regular diarrhea may best be treated by the administration of:
I- Loperamide
II- Aluminum hydroxide
III- Bismuth subsalicylate
a) I only
b) III only
c) I and II only
d) II and III only
e) All are correct

41- Antidiarrheal agent that appears to be bactericidal to H. pylori being useful in the
treatment of duodenal ulcers and gastric ulcers associated with H. pylori:
a) Bismuth subsalicylate
b) Loperamide
c) Docusate sodium
d) Psyllium
e) Milk of magnesium

42- Drug(s) used for the short-term treatment of gastro esophageal reflux and peptic
ulcer diseases may include which of the following?
I- H 2 receptors antagonists
II- Proton pump inhibitors
III- Corticosteroids
a) I only
b) III only

Copyright © 2000-2016 TIPS Inc. Unauthorized reproduction of this manual is prohibited. This 78-8
manual is being used during review sessions conducted by PharmacyPrep.
Pharmacyprep.com

c) I and II only
d) II and III only
e) All are correct

43- One of the most frequent side effect seen with SULCRAFATE therapy may include:
a) Diarrhea
b) Constipation
c) Edema
d) Cramps
e) Vomiting

44- Metoclopramide can act as effective agent in the management of many different
conditions. Some characteristics of metoclopramide include:
I- It increases the rate of gastric emptying
II- It has both peripheral and central effect
III- It can induce Parkinson’s disease
a) I only
b) III only
c) I and II only
d) II and III only
e) All are correct

45- Drug of choice in treating NSAIDs-induced ulceration may include:


a) Ranitidine
b) Omeprazole
c) Misoprostol
d) Sulcrafate
e) Aluminium hydroxide

46- A 5-Aminosalicylate active metabolite drug used in the treatment of Crohn’s disease:
a) Lansoprazole
b) Famotidine
c) Misoprostol
d) Sulfasalazine
e) Omeprazole

47- Active metabolite of sulphasalazine may include:


a) Acid salicylic acid
b) Salicilyc acid
c) Salicylates
d) Aminosalicylate
e) 5-Aminosalicylate

Copyright © 2000-2016 TIPS Inc. Unauthorized reproduction of this manual is prohibited. This 78-9
manual is being used during review sessions conducted by PharmacyPrep.
Pharmacyprep.com

48- Sulfasalazine, a 5-Aminosalicylate active metabolite drug suffer its metabolism


mainly where?
a) Colon
b) Stomach
c) Small intestine
d) Duodenum
e) Esophagus

49) Lipase inhibitor used to treat obesity may include:


a) Diethylpropiom
b) Orlistat
c) Benzphetamine
d) Mazindol
e) Phenylpropanolamine

50- Which of the following drugs are available in a transdermal form for the prevention
of motion sickness?
a) Metoclopramide
b) Granisetron
c) Ondasetron
d) Scopolamine
e) Diphenhydramine

51- Simvastatin used in hypercholesteramia is characterized by which of the following


mechanism of action?
a) Inhibiting xanthine oxidase
b) Inhibiting HMG-CoA reductase
c) Inhibit acetylcholinesterase
d) Acting as a bile sequestrant
e) Interfering with fat absorption from GI

52- Lactulose is an osmotic laxative agent that may be used to treat:


I- Constipation
II- Portal-systemic encephalopathy
III- Renal tubular necrosis
a) I only
b) III only
c) I and II only
d) II and III only
e) I,II,III

53- The primary function of simethicone used in antacid products is to act as:
a) Suspending agent
b) Adsorbent

Copyright © 2000-2016 TIPS Inc. Unauthorized reproduction of this manual is prohibited. This 78-10
manual is being used during review sessions conducted by PharmacyPrep.
Pharmacyprep.com

c) Buffer
d) Anti-flatulent
e) Flavoring agent

54) Chronic alcoholism and smoking is risk factors of?


I) GERD II) Gastritis III) Crohn's disease
A) I only B) III only C) I and II D) II and III E) I,II,III

56) What is correct about sucralfate?


A) taken after meals B) taken empty stomach C) Taken with antacids is more effective
D) It prevents heart burn E) It is taken with proton pump inhibitors

57) Which of the following is the drug of choice to treat Crohn's disease
A) Methotrexate B) azathioprine C) NSAIDS D. Infliximab E. 5ASA

58) A 34 yo male diagnosed with chronic gastritis and peptic ulcer disease. What
organism are possible cause?
A) E. coli
B) Helicobacter pilori
C) Giardia lamblia
D) Compylobacter jejuni

59) What drugs when administered with antacids, decrease the therapeutic efficacy?
I) Proton pump inhibitors (PPIs)
II) H 2 RA
III) Sucralfate
A) I only B) III only C) I and II D) II and III E) I, II, III

60) A customer walks into your pharmacy and wants something for fast relief of heart
burn symptoms?
A) ranitidine 150 mg
B) cimetidine
C) Mg antacids
D) Omeprazole
E) sucralfate

61.Ulcer is the ulceration of mucosa and underlying structuresof the Upper GI caused
by?
a. increased acidic gastric secretion
b. decrease mucus production
c. H.pylori infection
D. all of the above
ans. d

Copyright © 2000-2016 TIPS Inc. Unauthorized reproduction of this manual is prohibited. This 78-11
manual is being used during review sessions conducted by PharmacyPrep.
Pharmacyprep.com

62. What is the drug of choice to treat inflammatory bowel disease?


A. budosenide
B. 5-ASA
C. budosenide inj.
D. Methotrexate
E. Infliximab
Ans. B

GASTROINTESTINAL DISORDERS
ANSWERS

1- Ans: E
Comments: Hernias is a protrusion of a loop of an organ or tissue normally through the
GI area. Hernia hiatus is a herniation of stomach through the esophageal hiatus of
diaphragm. Strangulated hernia is an incarcerated hernia that is so tightly constricted as
to compromise the blood supply of the contents of the hernia sac.

2- Ans: A
Comments: Misoprostol is a prostaglandin analogue that increases bicarbonate and
mucin release and reduces acid secretion.

3- Ans: C
Comments: The most common drug reaction in the GIT is hydrolysis because most of
the drugs have an ester and/or amide group attached to their molecular formula and/or
react with water contests in GIT.

4- Ans: B
Comments: Salicylate compounds are largely hydrolyzed in the GIT, liver and blood to
salicylate, which is further metabolized primarily in the liver.

5- Ans: D
Comments: Ascitis is an effusion and accumulation of serous fluid in the abdominal
cavity; called also abdominal or peritoneal dropsy.

6- Ans: D

Copyright © 2000-2016 TIPS Inc. Unauthorized reproduction of this manual is prohibited. This 78-12
manual is being used during review sessions conducted by PharmacyPrep.
Pharmacyprep.com

Comments: Karaya gum is used as skin adhesives and protective skin barriers in the
fitting and care of ostomy, colostomy appliances and in other conditions involving an
artificial stoma

7- Ans: C
Comments: Somatotropin is a hormone growth with no effect on gastric acid secretion.

8- Ans: C
Comments: Gastro-esophageal Reflux disease is a chronic, pathologic, potentially life-
threatening disease manifested by reflux of the stomach and duodenal contents into the
esophagus, which is principally characterized by heartburn and regurgitation

9- Ans: A
Comments: Acute hemorrhagic gastritis is an erosive inflammation of the stomach with
bleeding normally seen with chronic alcoholism.

10- Ans: C
Comments: Magnesium salts used in antacids cause diarrhea. It also may cause systemic
effect as hypermagnesemia.

11- Ans: A
Comments: Villi and microvilli are found in the small intestine and are formed of
capillaries that works in the absorption of digestive food to rich the venous and
capillaries drainage of the gut.

12- Ans: E
Comments: Metoclopramide can be classified as dopaminergic blocking agent,
gastrointestinal emptying adjunct, peristaltic stimulant and antiemetic agent. It can be
used in cancer chemotherapy avoiding nausea and vomiting as well in diabetes
gastroparesis.

13- Ans: D
Comments: Bile is a fluid secreted from the liver and poured into the small intestine via
the bile ducts. It constituted of conjugated bile acids, cholesterol, phospholipid, bilirubin
diglucuronide and electrolytes. Bile is also responsible for the alkalinization in the
intestinal content due to its bicarbonate content.

14- Ans: E
Comments: Proton-pump inhibitors are the ultimate mediator of gastric acid secretion
inhibition; they are specific inhibitors of H+, K+ ATPase enzyme system at the secretory
surface of the gastric parietal cells. The most widely used proton-pump inhibitors are
omeprazole and lansoprazole.

15- Ans: D

Copyright © 2000-2016 TIPS Inc. Unauthorized reproduction of this manual is prohibited. This 78-13
manual is being used during review sessions conducted by PharmacyPrep.
Pharmacyprep.com

Comments: Omeprazole is a proton-pump inhibitor is the drug of choice in treating


Zollinger-Ellison disease and reflux disease.

16- Ans: E
Comments: Antacids should be taken 1 hour before meals, 3 hours after meals and at
bedtime

17- Ans: C
Comments: Gastrin, also known as G-cell is present in the stomach and stimulates acid
secretion.

18- Ans: E
Comments: Salivary glands are composed of 3 pairs of glands: Parotid, submandibular
and sublingual gland.

19- Ans: A
Comments: The first step of digestion is done in the mouth by the salivary enzymes,
amylase, ptyalin and lysozyme.

20- Ans: D
Comments: Gastrin is responsible for the stimulation of gastric acid secretion

21- Ans: C
Comments: The small intestine is anatomically divided in Duodenum, Jejunum and
ileum.

22- Ans: C
Comments: Duodenal ulcers also known as peptic ulcers are mainly characterized by
squirting of acid stomach into the duodenal wall.

23- Ans: B
Comments: The pancreas has two different glands. 1-Exocrine, responsible for the
production of enzymes which go to the duodenum via the pancreatic duct and 2-
Endocrine gland, responsible for the production of insulin, the blood sugar regulator.

24- Ans: E
Comments: The colon is anatomically divided into: Ascending colon, transverse colon,
descending colon, sigmoid colon and rectum.

25- Ans: D
Comments: Omeprazole is a proton-pump inhibitor.

Copyright © 2000-2016 TIPS Inc. Unauthorized reproduction of this manual is prohibited. This 78-14
manual is being used during review sessions conducted by PharmacyPrep.
Pharmacyprep.com

26- Ans: A
Comments: Famotidine is the most potent H 2 blocker. It reduces the gastric acid in 94%
up to 10 hours.

27- Ans: E
Comments: H2 blockers can be used to treat gastric hyper secretion, duodenal ulcers,
Zollinger-Ellison disease and reduce pain associated with gastroesophageal reflux
disease.

28- Ans: C
Comments: Proton-pump inhibitors are irreversible direct inhibitor of potassium pump.

29- Ans: E
Comments: The most frequent side effect seen in the treatment with proton-pump
inhibitors are: Abdominal pain or colic, diarrhea or constipation, unusual tiredness,
muscle pain, headaches, dizziness and skin rashes

30- Ans: E
Comments: Long-term treatment with NSAIDs and corticosteroids definitely increase the
risk of ulcers disease because these drugs cause many different GI side effects as pain,
cramps, acidity of stomach and other GI motility effects. Ulcers are also increased in
alcoholism and in smokers due to irritation and destruction of gastric mucosa. Other
agents that may increase the incidence of ulcers are coffee, orange juice, fatty and spice
food and emotional stress.

31- Ans: E
Comments: The basic pharmacological choices in the treatment of ulcers include H2-
antagonists, proton-pump inhibitors, misoprostol and sucralfate. Ulcers associated with
H. pylori can be treated by triple therapy with omeprazole, clarithromycin and
amoxicillin.

32- Ans: C
Comments: Sucralfate acts as protective coating of the gastric mucosa, it is particularly
used in ulcerated areas.

33- Ans: D
Comments: The mixtures of aluminum and magnesium are the most widely used
combination of antacids, because they compensate the side effects of each other.

34- Ans: A
Comments: Antacids act neutralizing gastric acid, which increases the pH of refluxed
gastric contents and lower esophageal sphincter pressure.

Copyright © 2000-2016 TIPS Inc. Unauthorized reproduction of this manual is prohibited. This 78-15
manual is being used during review sessions conducted by PharmacyPrep.
Pharmacyprep.com

35- Ans: B
Comments: Diarrhea is the most common side effect seen with the administration of
magnesium hydroxide antacid. Aluminum and calcium carbonate cause constipation.

36- Ans: E (was A before)


Comments: Constipation is the most common side effect seen with the administration of
aluminum containing antacid. Magnesium is an antacid with cathartic side effect.

37- Ans: E
Comments: Magnesium and aluminum mixtures are commonly used together because
they provide more even sustained action than used as single agent and permit lower
dose of each compound. The constipation effect of aluminum may counter the diarrhea
caused by magnesium.

38- Ans: C
Comments: Therapeutically safe in pregnancy, psyllium is a bulk-forming agent used as
laxative, it contains plant matter that absorbs water and softens the stool.

39- Ans: D
Comments: Lactulose is an osmotic laxative.

40- Ans: E
Comments: Diarrhea caused by infections, toxins and drugs are mostly treated with
specific agents, however the regular diarrhea normally caused for stress or stomach
indisposition is best treated with loperamide, bismuth subsalicylate or aluminum
hydroxide.

41- Ans: A
Comments: Duodenal and gastric ulcers are often caused by H. pylori. The treatment
objective is eradication of the bacterium with combination of antibiotics and h2-
antagonists, however bismuth subsalicylate is also used because it appears to be
bactericidal to H. pylori in high doses concentration.

42- Ans: C
Comments: Corticosteroids are neither used in the treatment of GERD nor in the
treatment of peptic ulcers. They normally induce or make worsen the ulcer condition.

43- Ans: B
Comments: Sucralfate is used as protective coating of the gastro intestinal mucosa and
have constipation as its main side effect.

44- Ans: E

Copyright © 2000-2016 TIPS Inc. Unauthorized reproduction of this manual is prohibited. This 78-16
manual is being used during review sessions conducted by PharmacyPrep.
Pharmacyprep.com

Comments: Metoclopramide can be used as gastrointestinal emptying adjunct,


peristaltic stimulant, antiemetic agent and dopaminergic blocking agent, which may
induce Parkinson’s disease due to the blockage of dopamine.

45- Ans: C
Comments: Misoprostol is a prostaglandin analogue that increases bicarbonate and
mucin release and reduces acid secretion. It is used to treat NSAID-induced ulceration.

46- Ans: D
Comments: Sulfasalazine is an 5-Aminosalicylate (5-ASA) active metabolite agent used as
the drug of choice in the treatment of crohn’s disease.

47- Ans: E
Comments: The pharmacologic action of sulfasalazine is seen through its active
metabolite 5-Aminosalicylate (5-ASA).

48- Ans: A
Comments: Sulfasalazine is metabolized in the colon by resident bacteria into 5-ASA and
sulfa pyridine. Sulfapyridine is absorbed while the 5-ASA remains in the colon.

49- Ans: B
Comments: Orlistat is a lipase inhibitor being used to treat obesity, it binds to
pancreatic and gastric lipase and inactivate the enzyme reducing the dietary absorption
of fat by about 30%.

50- Ans: D
Comments: Scopolamine is the only available transdermal drug used to treat motion
sickness.

51- Ans: B
Comments: Simvastatin is a statin (HMG-CoA reductase inhibitor) used as
antihyperlipidemic agent.

52- Ans: C
Comments: Lactulose is an osmotic laxative that can be used to treat constipation and
portal systemic encephalopathy as well.

53- Ans: D
Tips: Simethicone is mainly used in antacids preparations as antiflatulant agent (anti-
gas).

54) Ans: E

Copyright © 2000-2016 TIPS Inc. Unauthorized reproduction of this manual is prohibited. This 78-17
manual is being used during review sessions conducted by PharmacyPrep.
Pharmacyprep.com

56) Ans: B
Tips: Sucralfate is taken empty stomach to have better action. Because it make barrier
on to ulcer and prevent acid contact. Avoid taking with food, antacids, H 2 RA, and PPIs.

57) Ans: E

58) Ans: B

59) Ans: E

60) Ans: E

BIBLIOGRAPHIC REFERENCE

1- COMPREHENSIVE PHARMACY REVIEW – Lippincott William & Wilkins – Fourth edition

2- CPS-COMPENDIUM OF PHARMACEUTICALS AND SPECIALITIES - Canadian Pharmacist


Association – 2001 edition.

3- MEDICAL DICTIONARY – Dorland’s illustrated – 27th edition.

4- PHARMACY PREP – Lectures series & study guide for Evaluating Examination-TIPS -
2003/2004

5- THERAPUTIC CHOICES – Canadian Pharmacist Association -Third edition

6- USP DI – Drug Information for the Health Care Professional–15th edition – Volume I.

A patient brings prescription for ciprofloxacin 500 mg bid for 3 days. Currently patient
calcium carbonate 500 mg tid. What is appropriate recommendation.
a) Separate 2 hr before and 4 hr after calcium supplement from ciprofloxacin dose
b) Skip using calcium supplements for 3 days till finish of ciprofloxacin
C) For 3 days combination of ciprofloxacin and calcium supplements does not affect
therapy
D) None of the above

Copyright © 2000-2016 TIPS Inc. Unauthorized reproduction of this manual is prohibited. This 78-18
manual is being used during review sessions conducted by PharmacyPrep.
Pharmacyprep.com Diabetes Mellitus

PHARMACY PREP
DIABETES
1)What is incorrect about insulin's?
A-insulin Lispro is rapid acting
B-regular insulin's is available as iv and sc
C-NPH is intermediate acting insulin is available as iv and sc
D-Glargine is long acting insulin should not physically mix with other insulin
E-All insulin preps should be stored in refrigerator

2. What is/are NOT symptoms of hypoglycemia


A-sweating B-hunger C-polyuria D-fainting E-fatigue

3-Hypoglycemia symptoms, except?


A-palpitation
B-tremor or shaking
C-fails to think
D-sweating
E-Polyphagea

4-Which of the following drug associated hypoglycemia treated with glucose rather than
sucrose:
I-Metformin
II-Insulin
III-Acarbose
A-I only
B-III only
C-I and II only
D-II and III only
E.I,II,III

5-A 17-year-old type 1 diabetic patient is currently on diabetic medications. All of the
following antidiabetic drugs are not used in type I diabetes EXCEPT:
A-Chlorpropamide
B-Glyburide
C-Repaglinide
D-Nateglinide
E-Pioglitazone

6-Which of the following antidiabetic drug is increased risk of edema and congestive
heart failure
A-Metformin
B-Chlorpropamide
C-Glyburide
D-Rosiglitazone

Copyright © 2000-2016 TIPS Inc. Unauthorized reproduction of this manual is prohibited. This 79-1
manual is being used during review sessions conducted by PharmacyPrep.
Pharmacyprep.com Diabetes Mellitus

E-All of the above


7) Diabetic patient with hypercholesterolemia has high cholesterol, high LDL, normal TG.
What would be the drug of choice?
a) Lovastatin
b) Colestipol
c) Gemfibrozil
d) Niacin
e) Metformin

8)Which of the following antidiabetic drug effect on lipid levels (increase HDL, LDL and
decrease TG).
A-Metformin B-chlorpropamide C-Pioglitazone D-Repaglinide
E-Insulin

9) Which of the following is the intestinal lipase inhibitor:


A-Acarbose B-Metformin C-Pioglitazone D-Orlistat E-Repaglinide

10-Chlorpropamide is not used with ethanol because


A-It can induce hypoglycemia
B-Hypoglycemia occur
C-Alcohol intolerance occur
D-Disulfiram like reaction
E-All of the above

11-Which antidiabetic agent decrease mortality rate?


A-Metformin b-Chlorpropamide C-Acarbose D-Rosiglitazone
E- Mitiglinide

12-What is incorrect statement about metformin?


A) May cause lactic acidosis a rare side effect
B-Decrease gluconeogenesis
C-Increase glucose uptake
D-Avoid in sulfa allergy
E-May cause anorexia

13-What is correct about sulfonylureas?


A-All sulfonylureas have sulfa allergy
B-Gliclazide is long acting 2nd generation drug used single daily dose
C-Glyburide is long acting 2nd generation drug used single daily dose
D-Chlorpropamide is 1st generation drug has short half life.
E-A and B are correct statements

14-Which of the following antidiabetic drug may be taken regardless of meals:


A-Metformin

Copyright © 2000-2016 TIPS Inc. Unauthorized reproduction of this manual is prohibited. This 79-2
manual is being used during review sessions conducted by PharmacyPrep.
Pharmacyprep.com Diabetes Mellitus

B-Chlorpropamide
C-Glyburide
D-Rosiglitazone
E-Acarbose

15) An obese patient with type II diabetic, a drug of choice is?


A-Metformin B-Chlorpropamide C-Glyburide D-Insulin

16) A doctor prescribed metformin 1000 mg tid as initial treatment, what is anticipated
most common side effect?
A-Diarrhea B-lactic acidosis C-liver failure D-Congestive heart
failure E-Renal failure

17) The risk of lactic acidosis, associated with metformin, is enhanced by?
I-Congestive heart failure II-Renal disease
III-Alcohol intake
A-I only
B-III only
C-I and II only
D-II and III only
E-I, II, III

18) A obese patient has CHF, renal disease and drinks alcohol. Recently diagnosed with
type II DM, what is drug of choice?
A-Metformin
B-Chlorpropamide
C-Rosiglitazone
D-Acarbose
E-Glyburide

19) HbA1c is test shows blood sugar levels for?


A-the past 3 months
B- the past 6 months
C- the past 3 years
D- the past 3 days
E- the past 3 weeks

20) Sitagliptin (Januvia) act by?


A) Inhibiting DPP4 enhances incretin like GLP hormones
B) Inhibiting insulin
C) Inhibiting glucagon
D) Increasing incretin levels
E) Analog of incretin

Copyright © 2000-2016 TIPS Inc. Unauthorized reproduction of this manual is prohibited. This 79-3
manual is being used during review sessions conducted by PharmacyPrep.
Pharmacyprep.com Diabetes Mellitus

21) What antidiabetic drugs should skip if you skip meals?


A-Metformin B-Chlorpropamide C-Rosiglitazone D-Acarbose
E-Glyburide

22) Oral antidiabetic drugs that does not give hypoglycemia's?


A-Chlorpropamide and metformin
B-Acarbose and metformin
C-insulin
D-rosiglitazone
E. Glyburide and metformin

23) A type II DM patient using Acarbose, experiencing hypoglycemia should be treated


with?
A-A candy
B-Sucrose
C-Juice with sugar
D-Glucose
E. Insulin

24) In which of the following cases insulin should not be started?


A- high potassium levels
B- low potassium levels
C- high sodium levels
D- low sodium levels
E-urine ketone <160 mg/dL

25) A patient with hyperglycemic in the evening and hypoglycemic in the morning and
currently on regular insulin and NPH insulin, he should receive:
I) Increase NPH insulin in the morning
II) Decrease NPH insulin in the evening
III) Increase regular insulin levels in the morning
A. I only
B. III only
C. I and II only
D. II and III only
E) I, II, III

26) A patient is in emergence with diabetic ketoacidosis. Which of the following insulin
is used to treat diabetic ketoacidosis (DKA) ?
A) Rapid lispro
B) Regular
C) NPH

Copyright © 2000-2016 TIPS Inc. Unauthorized reproduction of this manual is prohibited. This 79-4
manual is being used during review sessions conducted by PharmacyPrep.
Pharmacyprep.com Diabetes Mellitus

D) Glargine
E) Detemir

27. Diabetic patient have increased risk following infections, except?


A) skin and foot infections
B) Cellulitis
C) Gangrene
D) UTI
E) Pneumonia

28. A type I DM patient on insulin regimen. Mistakenly take double the dose of insulin
at bed time. What symptoms are/is NOT expected?
A) confusion B) sweating C) heart burn D) palpitation
E) Tremors

29. What ketones are formed in blood sample of diabetic ketoacidosis patient?
I) beta hydroxy butyric acid
II) acetone
III) glucose
A) I only B) III only C) I and II D) II and III E) I, II, III

30. A T2DM patient currently using low dose of glyburide, metformin and insulin. What
is the pharmacist concern?
A. Weight gain
B. Hypoglycemia events
C. sulfa allergy
D. alcohol intake
ans. b

31. Glitazones, what is monitored?


a. hypoglycemia
B. weight gain
C. liver function test
D. renal function test
E. lung function test
ans. C

Copyright © 2000-2016 TIPS Inc. Unauthorized reproduction of this manual is prohibited. This 79-5
manual is being used during review sessions conducted by PharmacyPrep.
Pharmacyprep.com Diabetes Mellitus

DIABETES
ANSWERS:

1) Ans: C

2. Ans: C
Tips; sweating, hunger, dizziness, fainting, fatigue are symptoms of hypoglycemia.
Polyurea, polydipsea, polyphagea, neuropathic pains, and blurred vision are symptoms
hyperglycemia.

3- Ans: E

4- Ans: B
Tips: Acarbose impair the digestion of sucrose. Hypoglycemia in-patient taking acarbose
should be treated with glucose rather than sucrose.

5- Ans: E

6- Ans: D
Tips. Health Canada warning on rosiglitazone regarding increased risk of edema and
congestive heart failure.

7) Ans: A

8) Ans: C

9) Ans: D

Copyright © 2000-2016 TIPS Inc. Unauthorized reproduction of this manual is prohibited. This 79-6
manual is being used during review sessions conducted by PharmacyPrep.
Pharmacyprep.com Diabetes Mellitus

Tips: Orlistat (Xenical) act as intestinal lipase inhibitor and prevents absorption of fat
soluble vitamins ADEK.

10- Ans: D

11- Ans: A

12- Ans: D
Tips: Metformin is a biguanide chemical class of medication , it does not have sulfa
group, and thus it can be used in sulfa allergy patients.

13- Ans: E

14- Ans: D
Tips: Rosiglitazone can be taken with or without meals, where as metformin.
Chlorpropamide, glyburide taken with food, and acarbose is taken with first bite of
meals.

15) Ans: A

16) Ans: A

17) Ans: E

18) Ans: D
Tips Metformin is contraindicated in renal, hepatic, diseases. Chlorpropamide dose
adjustment required in renal impairment and rosiglitazone is contraindicated in
congestive heart failure.

19) Ans: A

20) Ans: A

21) Ans: D

22) Ans: B

23) Ans: D

24) Ans: B

25) Ans: C

26) Ans: B

Copyright © 2000-2016 TIPS Inc. Unauthorized reproduction of this manual is prohibited. This 79-7
manual is being used during review sessions conducted by PharmacyPrep.
Pharmacyprep.com Diabetes Mellitus

27. Ans: E

28. Ans: C

29. Ans: C (was D before)

Copyright © 2000-2016 TIPS Inc. Unauthorized reproduction of this manual is prohibited. This 79-8
manual is being used during review sessions conducted by PharmacyPrep.
Pharmacyprep.com Diabetes Mellitus

Copyright © 2000-2016 TIPS Inc. Unauthorized reproduction of this manual is prohibited. This 79-9
manual is being used during review sessions conducted by PharmacyPrep.
Pharmacyprep.com

PHARMACY PREP

THYROID DISORDERS

1-Select incorrect combinations in the following pairs:


A-Antithyroid drugs thionamides (PTU, methimazole)
B-Antithyroid drugs Lugol's solution
C-Lugol’s solution 5%I +10% KI
D-Hypothyroidism Thyroxin (T 4 ) and liothyrosine (T 3 )
E-Hypothyroidism Graves disease
F-Hypothyroidism Hoshimoto thyroiditis

2-All of the following are the symptoms of hypothyroidism EXCEPT:


A-Weight gain B-Fatigue C-Impaired memory D-Constipation
E-Heat intolerance

3-Conversion of T 4 to T 3 is catalyzed by:


A-Tetraiodinase enzyme
B-Deiodinase enzyme
C-Triiodinase enzyme
D-Hydrogenase enzyme
E-Iodinase enzyme

4) Which of the following laboratory monitoring indicated hypothyroidism:


I) Elevated TSH levels
II) No change in TSH levels
III) Decreased TSH levels
a) I only
b) III only
c) I and II only
d) II and III only
e) I, II, III

5-All of the following are the symptoms of hyperthyroidism, EXCEPT:


A-weight gain B-heat intolerance C-palpitation D-diarrhea
E-anxiety

6-Which of the following condition is hyperthyroidism


I-Hoshimoto thyroiditis
II-Myxedema
III-Graves disease
a) I only
b) III only
c) I and II only

Copyright © 2000-2016 TIPS Inc. Unauthorized reproduction of this manual is prohibited. This 80-1
manual is being used during review sessions conducted by PharmacyPrep.
Pharmacyprep.com

d) II and III only


e) I, II, III

7-Lugol solution contain:


A-Iodine
B-KI 10%+Iodine 5%
C-KI
D-Radioactive iodine
E-Sodium iodide

8) A patient using Lugol's solution should be counseled:


A) Lugol's solution may cause stains
B)Use topical only
C) Used to treat hypothyroidism patients
D) Lugol's solution is tincture iodine
E) None of the above

9-Drug of choice in pregnancy for hyperthyroidism


A-Lugol's solution
B-Propylthiouracil
C-Methimazole
D-Levothyroxine
E-Liothyronine

10-Patient receiving levothyroxine (L-T4) (Synthroid) should monitor levels of:


A-INR B-T4 C-TSH D-PT E-fT3

11- A woman with Grave’s disease. Doctor prescribed methimazole. All of the following
statements are true, EXCEPT:
a) This drug inhibits thyroid hormone
b) Onset of action is after 2 weeks synthesis
c) Do not stop the drug if there is a rash
d) Call the doctor if there are symptoms of infection

12- Instructions for Synthroid (levothyroxin) tablet.


A) Should not be taken with Iron supplements
B) Should not be taken within 4 hours of Ibuprofen
C) Should not be cut
D) Can be taken with or without food
E) The drug of choice for hyperthyroidism in pregnancy
Ans. A

13- Symptoms of overdose of levotyroxin all, EXCEPT:

Copyright © 2000-2016 TIPS Inc. Unauthorized reproduction of this manual is prohibited. This 80-2
manual is being used during review sessions conducted by PharmacyPrep.
Pharmacyprep.com

a) Cold tolerance
b) Tachycardia
c) Diarrhea
d) Weight loss

14-What is the most common side effects of propylthiouracil?


A-weight loss
B-weight gain
C-agranulocytosis
D-platelet aggregation
E-diarrhea

15-Hypothyroidism symptoms except?


A-diarrhea B-weight gain C-bradycardia D-hair loss
E-puffy face

16-Elevated TSH indicate?


A-Hyperthyroidism B-Hypothyroidism C-Euthyroidism
D-Hyperglycemia E-Graves disease

17-Hashimoto disease is?


A-Hyperthyroidism B-Hypothyroidism C-Euthyroidism
D-Hyperglycemia E-Graves disease

18-Absorption of levothyroxine is reduced by?


A-Al and Ca antacids
B-Calcium supplements
C-Iron supplements
D-Excessive Soya fiber
E-All of the above

19-What is effect of cotrimoxazole on warfarin


A-decrease INR by inhibiting GI flora
B-Increase INR by inhibiting GI flora
C-does not alter INR
D-Increase the risk of thromboembolism

20) A 55 yo women using levothyroxine 75 mcg to treat hypothyroidism. which of the


following lab test are used to monitor?
I) Serum TSH II) FT4 III)TT3
A)I only B) III only C) I and II D) II and III E) I, II, III

21) A 55 yo women using levothyroxine 75 mcg to treat hypothyroidism. She is


experiencing sweating, heat sensitive and diarrhea. What is the best action to do?

Copyright © 2000-2016 TIPS Inc. Unauthorized reproduction of this manual is prohibited. This 80-3
manual is being used during review sessions conducted by PharmacyPrep.
Pharmacyprep.com

A)Increase dose of levothyroxine


B)Decrease dose of levothyroxine
C)Change treatment to other hormone
D)Double the dose of levothyroxine

22) What is true about levothyroxine?


A) take in the morning with breakfast
B) take in evening with supper
C) take in evening empty stomach
D) take in the morning empty stomach
E) take with full glass of milk

23) All of the following drugs are used to treat thyrotoxicosis, (Graves disease) except?
A) propylthiouracil
B) methimazole
C) Lugol's solution
D) Propranolol
E) Levothyroxine

24) All of the following are correct in hypothyroidism, except?


A) Total T 4 decrease
B) Free T 4 decrease
C) Total T 3 decrease
D) Serum TSH decrease
E) Free Thyroxin index decrease

25) Which of the following hormone is secreted from hypothalamus?


A) Thyroid stimulating hormone
B) Luteinizing hormone
C) Follicular stimulating hormone
D) Prolactin
E) Oxytosin

26) Which of the following stimulate calcitonin production from thyroid gland?
A) Hypocalcemia
B) Hypercalcemia
C) Hypokalemia
D) Hyperkalemia
E) Hypothyroidism

27) A customer of your pharmacy uses insulin to treat diabetes. she is active and
decided to jogging 5 km daily. What is correct suggestion?
A) Take insulin injection before exercise
B) Take insulin injection after exercise

Copyright © 2000-2016 TIPS Inc. Unauthorized reproduction of this manual is prohibited. This 80-4
manual is being used during review sessions conducted by PharmacyPrep.
Pharmacyprep.com

C) Take sandwich before exercise


D) Take sandwich after exercise
E) Stop taking insulin injections

28) Which of the following is NOT a hypothyroidism symptoms?


A) hypertension B) weight gain C) sensitivity to cold D) diarrhea E) dry skin

29) What is incorrect about hypothyroidism?


A) Patient using thyroid hormone therapy
B) Serum TSH is elevated more than 5 mU/L
C) TT 3 is increased
D) Patient is tiered and fatigue
E) Patient have often dry skin

30) A patient is on insulin therapy. Which of the following is NOT correct about
hypoglycemia?
A) Patient may be sweating
B) Patient blood glucose in lower than 4 mmol/L (fasting)
C) Patient have palpitation
D) Patient have confusion
E) Patient should increase insulin dose

31) An asthma patient excessive dose of prednisone. Which of the following can cause?
A) Graves disease
B) Hoshimoto disease
C) Addison disease
D) Cushing syndrome
E) Weight loss

32) A 49 yo women is using treatment of Synthroid 75 mcg daily for hypothyroidism. She
complains palpitation, weight loss and sensitivity to heat. What is appropriate?
A) refer doctor to increase dose of Synthroid 100 mcg
B) refer to doctor to decrease dose of Synthroid
C) hypothyroid symptoms so should refer to doctor
D) She may have some new problem so refer to doctor
E) None

33. The same above patient current laboratory test indicate?


I) Increase serum TSH
II) Increase thyroxine (T 4 )
III) Increase triiodothyronin (T 3 )

Copyright © 2000-2016 TIPS Inc. Unauthorized reproduction of this manual is prohibited. This 80-5
manual is being used during review sessions conducted by PharmacyPrep.
Pharmacyprep.com

A) I only B) III only C) I and II D) II and III E) I, II, III

34. Graves disease is?


A-Hyperthyroidism B-Hypothyroidism C-Euthyroidism D-Hyperglycemia
E-Hashimoto disease

35. Which of the following is the least likely decrease absorption of levothyroxine
A) Al antacids
B) Mg antacids
C) Diary products
D) Iron supplements
E) Low fiber diet

36. If you get prescription of thyroxin120 mcg daily as initial dose for senior patient.
What symptoms may expected?
A) hyperthyroidism
B) hypothyroidism
C) Parkinson's disease
D) Schizophrenic symptoms
E) Serotonin syndrome

THYROID DISORDERS
ANSWERS

Copyright © 2000-2016 TIPS Inc. Unauthorized reproduction of this manual is prohibited. This 80-6
manual is being used during review sessions conducted by PharmacyPrep.
Pharmacyprep.com

1- Ans: E
Tips: Graves disease is hyperthyroidism

2- Ans: E

3- Ans: B

4- Ans: A

5- Ans: A
Tips: in hyperthyroidism, there is weight loss

6- Ans: B

7- Ans: B

8- Ans: A
Tips: Lugol's solution is used as oral drops, Lugol's solution is indicated in
hyperthyroidism patients.

9- Ans: B

10- Ans: C

11- Ans: C

12- Ans: A
Synthroid should not be taken with aluminum hydroxide, cholestyramine resins,
colestipol, ferrous sulfate, sod. Polystyrene sulfonate, soybean flour (infant formula)
due to decreased absorption of levothyroxine from GI tract. May be given to children by
crushing the tab. and suspending them in a small amount of water, breast milk or non-
soybean based formula. Do not store the suspension for any period of time.

13- Ans: A

14- Ans: C

15- Ans: A

16- Ans: B
Tips: elevated serum TSH > 5 mU/L indicate hypothyroidism.

17- Ans: B

Copyright © 2000-2016 TIPS Inc. Unauthorized reproduction of this manual is prohibited. This 80-7
manual is being used during review sessions conducted by PharmacyPrep.
Pharmacyprep.com

18- Ans: E

19- Ans: B

20) Ans: E
Tips: increase serum TSH and decrease Free T4 and TT3

21) Ans: B

22) Ans: D

23) Ans: E

24) Ans: D
Tips: FT4, TT4, TT3 and FTI decrease in hypothyroidism, only serum TSH increase. For
hyperthyroidism, exactly opposite changes.

25) Ans: E

26) Ans: B

27) Ans: C

28) Ans: D

29) Ans: C

30) Ans: E

31) Ans: D

32. Ans: B
Tips: patient experiencing overdose or hyperthyroidism symptoms, may be due
overdose of levothyroxine. So refer to doctor to decrease dose of levothyroxine

33. Ans: D
Tips: In hyperthyroidism increase T4 and T3.

34. Ans: A

35. Ans: E

Copyright © 2000-2016 TIPS Inc. Unauthorized reproduction of this manual is prohibited. This 80-8
manual is being used during review sessions conducted by PharmacyPrep.
Pharmacyprep.com

Tips: Food and formula containing large amount of soybean, fiber, or iron should not be
used for administration of levothyroxine. However low fiber food does not interact with
thyroxin.

36. Ans: A

Copyright © 2000-2016 TIPS Inc. Unauthorized reproduction of this manual is prohibited. This 80-9
manual is being used during review sessions conducted by PharmacyPrep.
Pharmacyprep.com

Copyright © 2000-2016 TIPS Inc. Unauthorized reproduction of this manual is prohibited. This 80-10
manual is being used during review sessions conducted by PharmacyPrep.
PHARMACY PEP

CONTRACEPTION

1-Contraceptive methods that protect STDs such as, gonorrhea, syphilis, chlamydia, and AIDS?
A-Oral contraceptive B-Condoms C-Contraceptive sponges D-Vaginal cream
E-Plan B

2-A 18 year old women walks into pharmacy, wants to buy emergency contraception Plan-B.
Pharmacist on duty believes do not allow dispensing contraceptive, and refuse to give oral
contraceptives. What ethical principle is violated?
A-beneficence
B-Non maleficence
C-Autonomy
D-veracity
E-Justice

3) What is the most common side effects plan B?


A-abdominal pain
B-Constipation
C-Nausea and vomiting
D-vaginal bleeding
E-Deep vein thrombosis

4)If one contraceptive pill missed, recommend?


A-take two pills next day
B-take 3 pills next day
C-take one pill next day
D-use alternative method of protection for next week
E-Refer to doctor

5) A customer of your pharmacy she asking for your opinion to choose a contraceptive
method, that she does not want to use daily and convenience and also near future she has
plan to start family. What is the good contraceptive you recommend?
A)OCP B) Condoms C)Evra path D)Nuvaring E) contraceptive injections

6) Which of the following questions is appropriate to ask a customer who wants to buy plan
B?
I)when was the last period II) are you pregnant? III)how long ago the unprotected intercourse
A) I only B) III only C) I and II only D) II and III only E)I, II, III

Copyright © 2000-2016 TIPS Inc. Unauthorized reproduction of this manual is prohibited. This manual is 81-1
being used during review sessions conducted by PharmacyPrep.
7) Which of the following contraceptive methods prevents gonorrhea and chlamydea
infections?
A) Transdermal contraceptives B) Intrauterine device C) Vaginal cream D) Subdermal
progesterone implants E) None of the above

8) Many women who take OCs are poorly informed about the proper use of these
medications. Which of the following is the first effective methods of providing information?
A) The patient first should be given package insert of OCs
B) Auxiliary labels on OCs
C) Verbal information describing the way medication works
D) Patient should be told that this do not protect STIs
E) Patient using these products should have knowledge of this product from internet

9) What is correct about transdermal contraceptive Evra patch?


I) The patch should be applied to the abdomen, buttocks at the beginning of menstrual cycle
II) The patch should be applied to the upper torso, and upper arm at the beginning of
menstrual cycle
III) The patch should be applied to the chest, and behind the ear at the beginning of menstrual
cycle
A) I only B) III only C) I and II D) II and III E) I, II, III

10. What is contraceptive is the drug of choice in breast feeding women?


A. COC
B. emergency contraceptive
C. progestin only
D. IUDs
ans. C

11.A female client is using combined oral contraceptive pills. She forgot to take one pill on her
second week of 21 contraceptive pills. She had Sunday start. What is appropriate
recommendation?
a. Take two pills next day
b. Stop taking COC till Sunday and start a new pack from Sunday
c. Take two pills on Sunday
D. Stop taking COC till Sunday and take two pills on Sunday
E. Stop using contraceptives and see doctor
ans. a

Copyright © 2000-2016 TIPS Inc. Unauthorized reproduction of this manual is prohibited. This manual is 81-2
being used during review sessions conducted by PharmacyPrep.
12.Which of the following the side effect of emergency contraception?
a. breast tenderness
B. stomach upset
C. Deep vein thrombosis
D. menstrual bleeding
E. none of the above
Ans. b

CONTRACEPTION
ANSWERS:
1- Ans: B

2- Ans: C

3) Ans: C

4) Ans: A

5) Ans: D

6) Ans: E

Copyright © 2000-2016 TIPS Inc. Unauthorized reproduction of this manual is prohibited. This manual is 81-3
being used during review sessions conducted by PharmacyPrep.
7) Ans: E
Tips: STI are only prevented by condoms contraceptive methods

8) Ans: A

9) Ans: C ( no answer given before)

Copyright © 2000-2016 TIPS Inc. Unauthorized reproduction of this manual is prohibited. This manual is 81-4
being used during review sessions conducted by PharmacyPrep.
Copyright © 2000-2016 TIPS Inc. Unauthorized reproduction of this manual is prohibited. This manual is 81-5
being used during review sessions conducted by PharmacyPrep.
Pharmacyprep.com

PHARMACY PREP

GENITOURINARY CONDITIONS

1) Toxic shock syndrome is caused by:


A-E. coli
B-S. aureus
C-Chlamydia
D-Gonorrhea
E-Non gonococci

2-Toxic shock syndrome may have risk with the use of:
I-Tampons use
II-Diaphragm contraceptive
III-Condoms as contraceptive
A-I only
B-III only
C-I and II only
D-II and III only
E.I, II, III

3-Finastride is alpha reductase inhibitor, indicated in:


I-Benign prostate hyperplasia
II-Alopecia
III-Increase sexual function
a) I only
b) III only
c) I and II only
d) II and III only
e) E.I, II, III

4-All of the following are the benign prostate hyperplasia, (BPH) symptoms, EXCEPT:
A-Nocturia
B-Frequent urination
C-Jet urination
D-Urinary retention
E-Post void dripping

5-Correct statement about pre menstrual symptoms (PMS) include:


I-All women have PMS
II-Luteal phase of cycle have PMS
III-vitamins, and estrogens deficiency may cause PMS
a) I only
b) III only

Copyright © 2000-2016 TIPS Inc. Unauthorized reproduction of this manual is prohibited. This 82-1
manual is being used during review sessions conducted by PharmacyPrep.
Pharmacyprep.com

c) I and II only
d) II and III only
e) I, II, III

9. Urinary incontinence is NOT aggravated by:


A-Hydrochlorthiazide
B-Furosemide
C-Diphenhydramine
D-Acetazolamide
E-None of the above

10-Patient profile includes sildenafil 50 mg and brings a prescription of erythromycin for


5 days upper respiratory infections, what is appropriate advice?
A-call doctor to decrease dose of sildenafil 25 mg while using erythromycin
B-Stop using sildenafil while using erythromycin
C-Call doctor to change other antibiotic
D-all of the above

11) A 50 year old lady will begin to take Hormone Replacement Therapy. All of the
following questions will help achieved appropriateness of her medication EXCEPT:
A) Did she have any children?
B) When did you last monitor your lipids?
C) Hysterectomy
D) When is the last menstruation period?

12. A patient is being treated with UTI, what is important to treat?


I. Dysuria and burning
II. Vaginal candidiasis
III. Fever and flare ups
a. I only
b. III only
c. I and III only
d. II and III only
e. I, II, III

13) If a patient is not treated for asymptomatic sexually transmitted infections. What is
correct?
A) symptoms are obvious after few days
B) can cause pelvic pain and dysmenorrhea
C) can cause pelvic inflammatory disease and infertility
D) can cause endometrial cancer
E) can results in cervical cancer

Copyright © 2000-2016 TIPS Inc. Unauthorized reproduction of this manual is prohibited. This 82-2
manual is being used during review sessions conducted by PharmacyPrep.
Pharmacyprep.com

14) What is correct about premenstrual symptoms?


I) all women have premenstrual symptoms
II) pre menstrual symptoms occur during luteal phase
III) a natural product prime rose oil is used
A) I only
B) III only
C) I and II only
D) II and III only
E) I, II III

15) What is incorrect about toxic shock syndrome?


A) caused by infections of S. aureus
B) can cause by tampon use
C) Can cause by condom use
D) can cause by candida infections
E) Can cause cervical cap contraceptives

16) Which of following condition avoid using sildenafil?


A) patients experienced priapism
B) patient have reported visual disturbances
C) patient using nitrates
D) patient have diplopia
E) children or women with hypertension

17) A customer of your pharmacy presents with symptoms of vaginal discharge, yellow
and fishy odor.
A) Recommend over the counter antifungal drugs
B) Refer to doctor is appropriate because this bacterial infection
C) Recommend self care
D) None of the above

18) What is the mechanism action of finasteride?


A-Blockade of intestinal lipase
B-Blockade the conversion of estrogen to dihydroestrogen
C-Blockade of testosterone to dihydrotestosterone
D-Blockade of sebaceous glands
E-Blockade of hair growth

19) What period of menstrual cycle pre-menstrual symptoms (PMS) occur?


A-Follicular phase
B-Ovulation phase
C-Immediately after ovulation
D-Luteal phase
E-During menstruation

Copyright © 2000-2016 TIPS Inc. Unauthorized reproduction of this manual is prohibited. This 82-3
manual is being used during review sessions conducted by PharmacyPrep.
Pharmacyprep.com

20) Which of the following methods of contraception can prevent sexually transmitted
diseases
A-oral contraceptives
B-reservoir type contraceptive
C-Intrauterine devices
D-condoms
E-contraceptive sponges

21. What is not menopause symptom?


A. Nigh sweating
B. Hot flushes
C. Mood changes
D. vasomotor symptoms
E. Fever
ans. E

23.Estrogen vaginal suppositories are used as?


A. contraception
B. to treat acne
C. to treat endometriosis
D. to treat vasomotor symptoms
E. To treat sexual dysfunction
Ans. d
Tips: vasomotor symptoms are hot flushes, night sweating, vaginal dryness can be
treated estrogen vaginal suppositories.

24. Which of the following is NOT associated with toxic shock syndrome?
A. Tampon B. Cervical cap C. contraceptive sponges D. condoms E. IUDs
Ans. D

25. Premenstrual symptoms occurs during?


A. Ovulation phase
B. Follicular phase
C. Luteal phase
D. menstruation
E. post menopause.
Ans.C

Copyright © 2000-2016 TIPS Inc. Unauthorized reproduction of this manual is prohibited. This 82-4
manual is being used during review sessions conducted by PharmacyPrep.
Pharmacyprep.com

26. What is the drug of choice to treat patient with vaginal pruritus, curdy vaginal
discharge, no odor and no color discharge?
A. Metronidazole vaginal cream
B. Metronidazole oral
C. Clotrimazole
D. Amoxicillin
E. Cotrimoxazole
Ans. C

Copyright © 2000-2016 TIPS Inc. Unauthorized reproduction of this manual is prohibited. This 82-5
manual is being used during review sessions conducted by PharmacyPrep.
Pharmacyprep.com

CORRECT ANSWERS
Genitourinary Conditions
1- Ans: B

2- Ans: C

3- Ans: C

4- Ans: C

5- Ans: D

9. Ans: C
Tips: diphenhydramine belongs to the ethanolamine class of antihistamines gives
anticholinergic side effects such as urinary retention, and induce sedation.
Hydrochlorothiazide, furosemide and acetazolamide are diuretics increase urinary
frequency.

10- Ans: A

11) Ans: A

12. Ans: E

13) Ans: C

14) Ans: D

15) Ans: C

16) Ans: C

17) Ans: B
Tips: Color discharge and fishy odor is indicator of bacterial infection, thus refer to
doctor.

18) Ans: C (no answer was given from 18-20)

19) Ans: D

20) Ans: D

Copyright © 2000-2016 TIPS Inc. Unauthorized reproduction of this manual is prohibited. This 82-6
manual is being used during review sessions conducted by PharmacyPrep.
Pharmacyprep.com

Copyright © 2000-2016 TIPS Inc. Unauthorized reproduction of this manual is prohibited. This 82-7
manual is being used during review sessions conducted by PharmacyPrep.
Pharmacyprep.com

PHARMACY PREP.

BONES AND JOINT COMPLICATIONS

1. What is not a screening test for rheumatoid arthritis?


A. ESR
B. CRP
C. Rheumatoid factor
D. BMD
E. joint pain
Ans. d

2. Infliximab is biological response modifier is product of?


A. monoclonal antibody
B. humanize MAB
C. Murine MAB
D. Chimeric MAB
E. All of the above
Ans. d

3. If patient is not treated for hyperuricemia. What happens?


A. Tophaceous gout (chronic gout).
B. Severe hyperuricemia
C. Acute gout attacks
D. Osteoarthritis
E. Osteoporosis
Ans. A

4. Febuxostat mechanism of action?


A. antihyperuricemic agent
B. Uricosuric agent
C. Xanthine oxidase inhibitor
D. Uric acid synthesis inhibitor
E. Used for acute gout
Ans. c

Copyright © 2000-2016 TIPS Inc. Unauthorized reproduction of this manual is prohibited. This 83-1
manual is being used during review sessions conducted by PharmacyPrep.
Pharmacyprep.com

Copyright © 2000-2016 TIPS Inc. Unauthorized reproduction of this manual is prohibited. This 83-2
manual is being used during review sessions conducted by PharmacyPrep.
PharmacyPrep.Com Osteoporosis

PHARMACY PREP
OSTEOPOROSIS

1) These two components in bone are responsible for the hardness and pliability of bone:
a) osteoclasts & collagen
b) mineralized salts & osteocytes
c) mineralized salts & collagen
d) collagen & elastic fibres
e) collagen & mesenchyme

2) A fracture in the shaft of a long bone would be a break in the:


a) epiphysis
b) metaphysis
c) diaphysis
d) epiphyseal plate
e) mesenchyme

3) Yellow marrow consists of:


a) osteoprogenitor cells
b) blood cell progenitor cells
c) hyaline cartilage
d) adipose
e) spongy bone

4) Chondroblasts produce:
a) basement membranes
b) bone matrix
c) cartilage matrix
d) mesothelium
e) endothelium

5) These structures are at the center of compact bone lamellae and carry blood vessels along
the bone length:
a) Haversian canals
b) canaliculi
c) perforating canals
d) osteocytes
e) lacunae

6) The cell type that is responsible for maintaining bone matrix once it has formed is:
a) osteoclasts
b) chondrocytes
c) osteocytes

Copyright © 2000-2016 TIPS Inc. Unauthorized reproduction of this manual is prohibited. This manual is 84-1
being used during review sessions conducted by PharmacyPrep.
PharmacyPrep.Com Osteoporosis

d) fibroblasts
e) osteoblasts

7) Soft connective tissue membranes between the cranial bones at birth are:
a) an indication of microcephaly
b) frontal sinuses
c) epiphyseal plates
d) cribriform plates
e) fontanelles

8) Endochondral and intramembranous are two mechanisms of:


a) bone remodeling
b) embryonic skeletal ossification
c) controlling blood calcium levels
d) cartilage synthesis

9) The two pairs of bones that make up the hard palate are the right and left:
a) zygomatic and temporal
b) palatine and maxillae
c) maxillae and zygomatic
d) maxillae and mandible

10) The two bones that make up the posterior nasal septum are the:
a) nasal and lachrymal
b) inferior nasal choncae and vomer
c) vomer and ethmoid
d) ethmoid and sphenoid

11) Which of the following is the most serious or life-threatening:


a) deviated nasal septum
b) sinusitis
c) damaged cribriform plate
d) damaged or cleft palate
e) ruptured bursae

12)All of the following are the risk factors for osteoporosis, except?
A-Race
B-Family history
C-Obesity
D-Smoking
E-Excessive coffee intake

13- Importance of Vitamin D in osteoporosis:


I- Increase the absorption of calcium from the small intestine

Copyright © 2000-2016 TIPS Inc. Unauthorized reproduction of this manual is prohibited. This manual is 84-2
being used during review sessions conducted by PharmacyPrep.
PharmacyPrep.Com Osteoporosis

II- The best vitamin D analog to treat osteoporosis is calcitriol


III- Antacids containing aluminum and magnesium may interfere with proper activity of
vitamin D.
a) I only
b) III only
c) I and II only
d) II and III only
e) I, II, III

14. All of the following are side effect of a systemic corticosteroids prednisone: EXCEPT:
A-Weight gain
B-Glucose intolerance
C-Osteoporosis
D-Cataract
E-reduce infection

15) Osteoporosis caused by the deficiency of the following:


I. Inadequate intake of vitamin D
II. Inadequate intake of calcium
III. Estrogen deficiency
a) I only
b) III only
c) I and II only
d) II and III only
e) I, II, III

16-What calcium supplement has the most elemental calcium?


A-Calcium citrate B-natural calcium C-Calcium carbonate D-dairy
products E-Calcium gluconate

17) The type of calcium supplement produced from oyster’s shells is?
A-Calcium citrate
B-natural calcium
C-Calcium carbonate
D-dairy calcium
E-Calcium gluconate

18-What is calcium supplement daily dose in adults?


A-1 g B- 500 mg C-800 mg D-100 mg E- 2 g

19-What is incorrect about alendronate?


A-Take first thing in the morning empty stomach with 1 full glass of water
B-Contraindicated renal diseases
C-70 mg/wk dose have equivalent effect as daily 10 mg dose

Copyright © 2000-2016 TIPS Inc. Unauthorized reproduction of this manual is prohibited. This manual is 84-3
being used during review sessions conducted by PharmacyPrep.
PharmacyPrep.Com Osteoporosis

D-Monitor hypercalcemia before initiating treatment


E-Do not lie down for 30 min after oral dose

20-A patient using alendronate 70 mg/wk and calcium supplement, what is incorrect?
A-Avoid calcium supplement while using alendronate therapy
B-It is important to take calcium supplement while using bisphosphonates therapy
C-separate alendronate and calcium supplement at least 2 hour before and 4 after.
D-It is also essential to take vitamin D supplements
E-It is recommended to take daily 800 mg IU vitamin D

21- To monitor alendronate safety and efficacy all are needed EXCEPT:
a) Hypercalcemia
b) Bone pain and swelling
c) Ca and Vitamin D intake
d) Bone mineral density
e) Esophagitis

22- What is correct about osteoporosis?


A) Inadequate Ca and vitamin D can cause osteoporosis
B) Alendronate can be used once month
C) Zolendronoic acid injection is used daily
D) Recommend daily vitamin D 2000 IU in over age 65yo
E) Recommend swimming a good exercise to increase bone mineral density

23- Which of the following increase risk of osteoporosis?


A) History of oral contraceptive use
B) Smoking
C) Physical activity like weight bearing exercises like stair climbing, walking, and jogging.
D) Increase in dietary soy intake
E) Increase intake of broccoli

24- A 35 yo women get the prescription of 50,000/wk unit of vitamin D. What to do?
A) talk to doctor and dispense
B) this could extremely high dose and may cause toxicity
C) reduce dose to 500 units and dispense
D) May be prescription error just dispense 500 units
E) It is outrageous to get prescription of 50, 000 units

25- What is not risk factor of osteoporosis?


a)estrogen deficiency B) androgen deficiency C) race D)age E)protein diet

26- Which of the following is the least recommended exercises in osteoporosis?


A) rope jumping B) basket ball playing C) horse riding D) jogging E) Dancing

Copyright © 2000-2016 TIPS Inc. Unauthorized reproduction of this manual is prohibited. This manual is 84-4
being used during review sessions conducted by PharmacyPrep.
PharmacyPrep.Com Osteoporosis

27- Which of the following is not a cranial suture:


a) epiphyseal
b) lambdoidal
c) coronal
d) sagittal
e) squamous

28. Which of the following is least likely risk factor for osteoporosis?
A) Inadequate vitamin D and Ca supplement intake
B) Age
C) family history of osteoporosis
D) Prednisone therapy for over 3 months
E) alcohol intake

29. What is the most likely risk for osteoporosis?


A) smoking
B) alcohol
C) coffee
D) tea
E) none

30. what type of vertebral fracture in osteoporosis?


A. compression fracture (back pain, decrease in height and round shoulder appearance)
B. tendonitis
C. cartilage break down
D. Bone softening
E. closed fracture
Ans. A

31. What is drug is used to treat vertebral and non vertebral fracture?
A. bisphosphonates
B. Raloxafine
C. Calcium supplement
D. Teriperatide
E. Calcitonin
Ans. A

32. Which of the following drug has once year injection


A. bisphosphonates
B. Raloxafine
C. Zoledronic acid
D. Teriperatide
E. Calcitonin
Ans. C

Copyright © 2000-2016 TIPS Inc. Unauthorized reproduction of this manual is prohibited. This manual is 84-5
being used during review sessions conducted by PharmacyPrep.
PharmacyPrep.Com Osteoporosis

33.Denosumab (Prolia) mechanism of action is?


A. Human IgG2 monclonal antibody (RANKL)
B. Pyrophosphate
C. Chimeric antibody
D. Hormone replacement therapy
E. Parathyroid hormone analog
ans. A

OSTEOPOROSIS
ANSWERS:

1) Ans: C

2) Ans: C

3) Ans: D

4) Ans: C

5) Ans: A

Copyright © 2000-2016 TIPS Inc. Unauthorized reproduction of this manual is prohibited. This manual is 84-6
being used during review sessions conducted by PharmacyPrep.
PharmacyPrep.Com Osteoporosis

6) Ans: C

7) Ans: E

8) Ans: B

9) Ans: B

10) Ans: C

11) Ans: C

12) Ans: C
Tips: obesity or overweight is not a risk factor of osteoporosis. Race, family history, smoking,
excessive coffee intake, deficiency of estrogen, and sedentary lifestyle are some of the risk
factors.

13- Ans: E
Tips: Osteoporosis is characterized by the loss of bone mass. Vitamin D supplements have
been used in the treatment of osteoporosis because vitamin D increases the absorption of
calcium that is essential to reduce bone loss. Antacids interact with vitamin D in many ways,
magnesium containing antacids may lead to hypermagnesemia and aluminum containing
antacids have its levels increased leading to aluminum bone toxicity when currently used with
vitamin D.

14. Ans: E
Tips: Corticosteroids increase the risk of infections.

15- Ans: E

16- Ans: C

17- Ans: C

18- Ans: A

19- Ans: D

20- Ans: A

21- Ans: A

22- Ans: A

Copyright © 2000-2016 TIPS Inc. Unauthorized reproduction of this manual is prohibited. This manual is 84-7
being used during review sessions conducted by PharmacyPrep.
PharmacyPrep.Com Osteoporosis

23- Ans: B

24- A

25- Ans: E
Tips: protein diet like dietary are plant derived phytoestrogen present in soy proteins

26- Ans: C

27- Ans: A

28. Ans: E

29. Ans: C

Copyright © 2000-2016 TIPS Inc. Unauthorized reproduction of this manual is prohibited. This manual is 84-8
being used during review sessions conducted by PharmacyPrep.
PharmacyPrep.Com Osteoporosis

Copyright © 2000-2016 TIPS Inc. Unauthorized reproduction of this manual is prohibited. This manual is 84-9
being used during review sessions conducted by PharmacyPrep.
PharmacyPrep.com Hypertension

PHARMACY PREP
HYPERTENSION

1) According to Joint National Committee (JNC-7) standards. In measuring blood pressure of


someone whose systolic blood pressure in the past had 150 mm Hg. Now if you like to check
blood pressure. You would have to inflate the mercury bulb to?
A-180 mm Hg
B-150 mm Hg
C-200 mm Hg
D-120 mm Hg
E-210 mm Hg
Ans. A

2. What is correct about measuring blood pressure?


A. Measure blood pressure twice on one hand after 5 min interval
B. Measure blood once on left and then right hand after 5 min interval.
C. Measure twice on each hand with 5 min interval
D. Measure once on any hand
E. Measure twice on left hand
ans. B

3.. Which of the following main complication is NOT associated with persistent hypertension?
A. Kidney B. Brain C. Retina of eye D. Liver E. diabetes
Ans. d

4. Which of the following is the blood pressure goal for clients that 60 yo and above without
diabetes or renal diseases?
a. less than 120/80
b. less than 130/80
c. less than 140/80
d. less than 140/90
e. less than 150/90
ans. D

Copyright © 2000-2016 TIPS Inc. Unauthorized reproduction of this manual is prohibited. This manual is 85-1
being used during review sessions conducted by PharmacyPrep.
PharmacyPrep.com Hypertension

Copyright © 2000-2016 TIPS Inc. Unauthorized reproduction of this manual is prohibited. This manual is 85-2
being used during review sessions conducted by PharmacyPrep.
PharmacyPrep.Com Coronary Artery Diseases

PHARMACY PREP
CORONARY ARTERY DISEASES

1) Which one of the following factors is most directly attributed to angina attacks?
A. Emotional Stress
B. Heavy smoking
C. Myocardial ischemia
D. Obesity
E. None of the above

2) Purple toe syndrome side effect is produced by:


A-Amiodarone
B-Reynaud phenomenon
C-Warfarin
D-Rifampin
E-Verapamil

3. Which medication CAN NOT BE used for prinzmetal angina (vasospastic angina)?
A. Amlodipine
B. Metoprolol
C. Acetyl salicylic acid
D. Verapamil
E. Diltiazem

4. Which effect of nitroglycerin is most likely responsible for its therapeutic action in angina
pectoris?
a. Dilation of coronary artery
b. Relaxation of peripheral vascular smooth muscle
c. Decrease myocardial after load
d. A and B
e. A and C

5. Nitroglycerin dilates the coronary arteries in angina pectoris by


a) Decreasing the heart rate reflex
b) Increasing the metabolic work of the myocardium
c) Direct action on smooth muscle in the vessel walls
d) Increasing the effective refractory period in the atrium
e) Blocking beta-adrenergic receptors

6. Which one of the following conditions is not associated with verapamil?


A. Inhibition of calcium ion influx
B. Negative inotropic effect
C. Coronary vasodilatation effect
D. Peripheral vasodilatation effect

Copyright © 2000-2016 TIPS Inc. Unauthorized reproduction of this manual is prohibited. This manual is 86-1
being used during review sessions conducted by PharmacyPrep.
PharmacyPrep.Com Coronary Artery Diseases

E. None of the above

7-Which of the following anticholesterol medication may increase his blood sugar levels:
A-Atorvastatin
B-Niacin
C-Simvastatin
D-Fluvastatin
E-All of the above

8-Administration of which of the following drugs is most likely to PROLONG clotting time?
A. Morphine
B. Vitamin K
C. A barbiturate
D. Acetaminophen
E. Acetyl salicylic acid

9. Which of the following drugs acts by inhibiting renal reabsorption of sodium?


a) Urea
b) Chlorothiazide
c) Theophylline
d) Digitalis glycosides
e) Procainamide

10) All of the following are low molecular weight heparins (LMWH), except?
A-Enoxaparin
B-Dalteparin
C-Tinzaparin
D- Nadroparin
E-Heparin

11) What is not true about low molecular weight heparins


(LMWH)?
A-It act by inhibiting factor Xa
B-It is long acting than heparin
C-It does not require monitoring
D-It is contraindicated in pregnancy
E-It is used as once daily SC injection

12) Unstable/NSTEMI drug of choice?


A-Anticoagulants
B-LMWH
C-Heparin
D-Alteplase
E-Propranolol

Copyright © 2000-2016 TIPS Inc. Unauthorized reproduction of this manual is prohibited. This manual is 86-2
being used during review sessions conducted by PharmacyPrep.
PharmacyPrep.Com Coronary Artery Diseases

13) What is the drug of choice for ST segment elevated myocardial infarction (STEMI)?
A-Anticoagulants B-LMWH C-Heparin D-Alteplase E-Propranolol

14) What is the drug of choice for stable angina?


A-Anticoagulants
B-LMWH
C-Heparin
D-Alteplase
E-beta blockers

15)Which of the following drug may cause bradycardia?


A-Nifedipine
B-Nitroglycerine
C-Amlodipine
D-Verapamil
E-Clonidine

16) Which of the following drug should be taken with food?


A-Fluvastatin B-Lovastatin C-Simvastatin D-Atorvastatin E-Rosuvastatin

17) What statin can be taken anytime of the day?


A-Fluvastatin B-Lovastatin C-Simvastatin D-Atorvastatin E-Rosuvastatin

18) What statin do not require dosage adjustment in renal disease patient?
A-Fluvastatin B-Lovastatin C-Simvastatin D-Atorvastatin
E-Rosuvastatin

19) Which of the following is NOT used for antiplatelets action?


A-ASA B-Ibuprofen C-Naproxen D-Pyroxicam E-Acetaminophen

20) What antiplatelets drugs is used for prophylactic for angina pectoris?
A-dipyridamol b-metoprolol c-nifedipine D-all of the above

21. Which of the following is NOT associated with plaque formation?


A) Coronary artery disease
B) Angina
C) Stroke
D) Reynaud's phenomenon
E) intermittent claudication

22. Which of the following is recommended to minimize the warfarin dispensing error?
I) dose of warfarin should be indicated in milligrams
II) check the color of tablets

Copyright © 2000-2016 TIPS Inc. Unauthorized reproduction of this manual is prohibited. This manual is 86-3
being used during review sessions conducted by PharmacyPrep.
PharmacyPrep.Com Coronary Artery Diseases

III) Recommend lifestyle modification

23. Exercise induced angina, which is relieved by rest nitroglycerine or both referred as?
A) Prinzmetal angina B) stable angina C) unstable angina
D) Myocardial infarction

24. Myocardial oxygen demand is increased by all the following except?


A)exercise B) Beta blockers C) cold air D) smoking

25. A patient with angina receiving metoprolol and diltiazem should be monitored for?
I) increased heart rate
II) decreased cardiac output
III) decreased heart rate
A-I only B-III only C-I and II only D-II and III only E) I, II, III

26) A 35 year old craftsman is on nitroglycerin pills, which of the following statements should
discuss with him:
A) It causes an elevation of intracellular cGMP
B) It can cause postural hypotension, take medication while sitting.
C) Store medication away from light, and keep in tightly metal capped bottle.
D. B&C
Ans-D
Tips: Nitroglycerin increases synthesis of intracellular cGMP. However, mechanism of action
may not be discussed in counselling.

27) Which of the following is/are the ischemic heart diseases?


A. Myocardial infarction
B. Angina
C. Acute coronary syndrome
D. unstable angina
E. All of the above
Ans. E

28) Aspirin should be used cautiously in patient receiving heparin because aspirin:
A. inhibit vitamin k absorption
B. has antithrombin activity
C. inhibit metabolism of heparin
D. inhibit platelet aggregation
Ans-D
Tips: a drug that increases metabolism of the anticoagulant will lower the steady state plasma
concentration both free and bound form, where as one that displaces anticoagulant will
increase plasma level of free form only, until elimination of drug as again lower it to steady
state levels.

Copyright © 2000-2016 TIPS Inc. Unauthorized reproduction of this manual is prohibited. This manual is 86-4
being used during review sessions conducted by PharmacyPrep.
PharmacyPrep.Com Coronary Artery Diseases

29) Nitroglycerin dilates the coronary arteries in angina pectoris by


A. decreasing the heart rate reflex
B. increasing the metabolic work of the myocardium
C. direct action on smooth muscle in the vessel walls
D. increasing the effective refractory period in the atrium
Answer: C

30) In angina pectoris, the MOST rapid and dependable relief will be provided by
A. oral glyceryl trinitrate
B. oral isosorbide dinitrate
C. oral erythrityl tetranitrate
D. sublingual glyceryl trinitrate
Ans: D

31) Nitroglycerin when taken together with Sildenafil will cause:


A. Hypertension
B. Hypotension
C. Headache
D. Hyperglycemia
Ans: B

32) Aspirin acts as:


A. irreversible antiplatelet drug
B. reversible antiplatelet drug
C. antiarrhythmic
D. anticoagulant
Ans: A

33) What are correct statement about Storage of Nitroglycerin:


A. Amber color glass, hygroscopic
B. Amber glass
C. Plastic container
D. Any container can be use
Ans: A

34) What drug shouldn’t be given to post MI?


A. ASA
B. Ace inhibitors
C. CCB and beta blockers
D. All of the above
Ans: D

35) Which drug causes bradycardia?


A. Diltiazem

Copyright © 2000-2016 TIPS Inc. Unauthorized reproduction of this manual is prohibited. This manual is 86-5
being used during review sessions conducted by PharmacyPrep.
PharmacyPrep.Com Coronary Artery Diseases

B. Nifedipine
C. Nicardipine
D. Felodipine
E. Amlodipine
Ans: A
Tips: Diltiazem and Verapamil side effect is bradycardia

36) This drugs cause reflex tachycardia except:


A. Amlodipine
B. Nifedipine
C. Nicardipine
D. Felodipine
Ans: A
Tips: All dihydropyridine calcium channel blockers cause reflex tachycardia except amlodipine

37) Action of nitrates can be described as?


A. vasodilation
B. vasoconstriction
C. bronchoconstriction
D. Bronchodilatation
Ans: A

38) An imbalance between oxygen supply and oxygen demand in cardiac muscle may produce
a condition known as:
A. CHF
B. Heartburn
C. MI
D. Angina pectoris
Ans: D

39) Drugs may be used in acute myocardial infarction: except


A) Beta blocker
B) ACE inhibitor
C) Calcium channel blockers
D) Nitrates
E) Digoxin
Ans: E

40) What dose does Aspirin acts as antiplatelet?


A. 80 mg
B. 365 mg
C. 500 mg
D. 1 g
Ans: A

Copyright © 2000-2016 TIPS Inc. Unauthorized reproduction of this manual is prohibited. This manual is 86-6
being used during review sessions conducted by PharmacyPrep.
PharmacyPrep.Com Coronary Artery Diseases

41) Drugs used in angina pectoris except:


A. Nitrates
B. CCB
C. Cardiac glycoside
D. Beta blockers
Ans: C

42) What is the most common form of Ischemic heart disease:


A. Myocardial infarction
B. Atrial fibrillation
C. Angina pectoris
D. Ventricular arrhythmia
Ans: C

43. An ambulatory patient, the most fastest acting nitroglycerine dosage form?
A. SL spray B. SL tab C. wafers D. liquid E. suspension
ans. a

44. What is the drug of choice to treat hypertension in angina?


A. beta blockers B. CCBs D. ACEi E. ARBs
Ans. A

45. What is true about echocardiogram (ultrasound)?


A. Determines site and severity of thrombus or wall motion abnormalities.
B. Measure heart rhythm
C. Measure angina severity
D. Measure biological marker
E. Measure exercise tolerance
Ans. A

Copyright © 2000-2016 TIPS Inc. Unauthorized reproduction of this manual is prohibited. This manual is 86-7
being used during review sessions conducted by PharmacyPrep.
PharmacyPrep.Com Coronary Artery Diseases

ANSWERS
Coronary artery diseases

1. C

2- C

3. B
Tips: TC page 275, avoid beta blockers in prinzmetal angina, beta blockers cause
vasoconstriction (vasospasm)

4. D

5. C

6. E
Tips: Verapamil act as coronary vasodilator by inhibiting calcium ion influx in coronary smooth
muscles, and it does negative inotropic effects.

7- B

8- E

9. B

10) E(was D before)

11) D

Copyright © 2000-2016 TIPS Inc. Unauthorized reproduction of this manual is prohibited. This manual is 86-8
being used during review sessions conducted by PharmacyPrep.
PharmacyPrep.Com Coronary Artery Diseases

12. A

13) D

14) E

15) D

16) B
Tips: Lovastatin should be always administered with food to increase Bioavailability, otherwise
it can decrease 30% Bioavailability.

17) D
Tips: Atorvastatin can be taken anytime of the day, because has long half life.

18) D
Tips: The best agent for renal disease patient is atorvastatin, because it has minimal renal
elimination, thus do not require dose adjustments. Simvastatin and lovastatin require dose
adjustment in severe renal diseases however it can be used in minor and moderate renal
diseases

19) E

20) A

21. D

22. C

23. B

24. B

25. B

26) Ans-D
Tips: Nitroglycerin increases synthesis of intracellular cGMP. However, mechanism of action
may not be discussed in counselling.

27) Ans-E

28) Ans-D
Tips: a drug that increases metabolism of the anticoagulant will lower the steady state plasma
concentration both free and bound form, where as one that displaces anticoagulant will

Copyright © 2000-2016 TIPS Inc. Unauthorized reproduction of this manual is prohibited. This manual is 86-9
being used during review sessions conducted by PharmacyPrep.
PharmacyPrep.Com Coronary Artery Diseases

increase plasma level of free form only, until elimination of drug as again lower it to steady
state levels.

29)Ans: C

30) Ans: D

31) Ans: B

32) Ans: A

33) Ans: A

34) Ans: D

35) Ans: A
Tips: Diltiazem and Verapamil side effect is bradycardia

36)Ans: A
Tips: All dihydropyridine calcium channel blockers cause reflex tachycardia except amlodipine

37) Ans: A

38) Ans: D

39) Ans: E

40) Ans: A

41) Ans: C

42) Ans: C

Copyright © 2000-2016 TIPS Inc. Unauthorized reproduction of this manual is prohibited. This manual is 86-10
being used during review sessions conducted by PharmacyPrep.
www.pharmacyprep.com

PHARMACY PREP

STROKE

1. The artery that supply blood to brain from heart?


A. Internal carotid artery
B. Coronary artery
C. Suclavian artery
D. Brachial artery
E. Pulmonary artery
Ans. A

1.A client presents with following symptoms, Which of the following symptoms is the most
pharmacist concern, which require referral to emergency?
A. patient face changes to uneven
B. patient arm drifting down
C. patient speech changes to strange
D. severe unprovoked headache
E. All of the above
Ans. E

2) All of the following are initial symptoms of stroke include, Except?


A-headache B-blurred vision C-paralysis D-seizure E-Chest pain
Ans. E

3) What is the drug of choice to treat transient ischemic attack (TIA)


A. Acetyl salicylic acid B. Ticlopidine C. Warfarin D. Heparin E.Ramipril
Ans. A

4) Which of the following factor that does NOT predispose stroke?


A) Age B) Diabetes C) Ethnicity D) Hypertension E) chronic liver diseases
Ans. E

87-1
Copyright © 2000-2016 TIPS Inc. Unauthorized reproduction of this manual is strictly prohibited and it is
illegal to reproduce without permission. This manual is being used during review sessions conducted by
PharmacyPrep.
PharmacyPrep.com Congestive Heart Failure (CHF)

PHARMACY PREP
CONGESTIVE HEART FAILURE

1. What is the classical symptoms of CHF?


A. Swollen legs
B. Dyspnea, fatigue and weight gain
C. Weakness, nausea and vomiting
D. Chest pain radiating to left arm
E. Weight loss
Ans. b

2. What is the drug of choice to treat congestive heart failure?


A. ACEi
B. CCBs
C. Diuretics
D. Digoxin
E. Nitroglycerine
ans. A

3. Which of the following is least likely used to manage CHF?


A. Beta blockers
B. ACEi
C. Losartan
D. verapamil
E. Carvedilol
ans. d

4. What is the drug of choice for mild hypertension in patient with congestive heart failure?
A. Ramipril B. Hydralazine C. Losartan D. Hydrochlorothiazide E. Furosemide
ans. A

Copyright © 2000-2016 TIPS Inc. Unauthorized reproduction of this manual is prohibited. This manual is 88-1
being used during review sessions conducted by PharmacyPrep.
PharmacyPrep.Com Arrhythmias

PHARMACY PREP
DYSARRHYTHMIAS

1. Embolus (clot) enters into common carotid artery and travels toward the brain due to atrial
fibrillation can increase risk?.
A. angina
B. Cardiogenic stroke
C. myocardial infarction
D. DVT
E. CHF
Ans.b

2. A distinct p-waves is seen in?


A. Atrial fibrillation
B. Atril flutter
C. Ventricular tachycardia
D. Sinus tachycardia
E. all of the above
Ans. A

3. Patient with atrial fibrillation. Which of the following is used to prevent risk of cardiogenic
stroke.
A. ASA B. Warfarin C. ACEi D. Beta blockers E. Calcium channel blockers
Ans. B

4. What is not monitored in patient using amiodarone?


A. Serum TSH
B. Chest x-ray
C. Eye exam
D. Renal function test
E. Liver function test
Ans. d

5.What is the most important to monitor in patient using amiodarone?


A. Serum TSH
B. Chest x-ray
C. Eye exam
D. Renal fucntion test
E. Liver function test
Ans.A

6. What type arrhythmias digoxin is contraindicated?


A. Atrial fibrillation

Copyright © 2000-2016 TIPS Inc. Unauthorized reproduction of this manual is prohibited. This manual is 89-1
being used during review sessions conducted by PharmacyPrep.
PharmacyPrep.Com Arrhythmias

B.Atril flutter
C. Ventricular tachycardia
D. Sinus tachycardia
E. all of the above
Ans. C

8)Atrial fibrillation. Which of following is the characteristic wave?


A. P wave B. QRS wave C. QT wave D. T wave E. U wave
Ans. A

2 . Which of the following dysrrhythmia associated with sudden high fever?


A. Atrialfibrillation
B. Proxysmal
C. Ventricular
D. Atrial
E. Premature
Ans. B

Copyright © 2000-2016 TIPS Inc. Unauthorized reproduction of this manual is prohibited. This manual is 89-2
being used during review sessions conducted by PharmacyPrep.
PharmacyPrep.Com Arrhythmias

Copyright © 2000-2016 TIPS Inc. Unauthorized reproduction of this manual is prohibited. This manual is 89-3
being used during review sessions conducted by PharmacyPrep.
PharmacyPrep.Com Peripheral Vascular Diseases

PHARMACY PREP

PERIPHERAL VASCULAR DISEASES

1) Aneurysm is ?
A) bulging of arteries B) a blood clot c) a moving blot clot d)permanent enlargement of
alveoli E) None of the above

2) Thrombus is?
A) bulging of arteries B) a blood clot c) a moving blot clot d)permanent enlargement of
alveoli E) None of the above

3) Embolus is ?
A) bulging of arteries B) a blood clot c) a moving blot clot d)permanent enlargement of
alveoli E) None of the above

4) Symptoms of Reynaud phenomenon?


a) fingers spasm b) extremities turn to pale, or blue C)often in elderly D)all of the above

5) Symptoms of Intermittent claudication?


a) cold feet, B)pulses are absent, c) exercise induced cramping pain d)all of the above

6) Symptoms of Stroke?
A)headache B)confusion C)dizziness D)blurred vision E)all of the above

7) What are the symptoms of deep vein thrombosis?


A) Leg pain B) swollen legs C) tenderness D) Calf pain E) all of the above

8) What is treatment of deep vein thrombosis?


A) warfarin, b) heparin, c) LMWH D) thrombolytic E)All of the above

9) Drug of choice against the prophylaxis of transient ischemic attack?


A)ASA 81 mg chewable B)ASA 81 mg enteric coated C)Alteplase injection D)Warfarin
E)heparin

10. Which of the following monitoring parameter is essential for patient who currently taking
Low molecular weight heparin:
A-International normalized ratio (INR)
B-Prothrombin time
C-Heparin assay
D-Sensitive TSH
E-provide predictable response, thus no monitoring required

Copyright © 2000-2016 TIPS Inc. Unauthorized reproduction of this manual is prohibited. This manual is 90-1
being used during review sessions conducted by PharmacyPrep.
PharmacyPrep.Com Peripheral Vascular Diseases

11. Which of the following are the monitoring parameters of warfarin therapy?
I Complete blood count (CBC)
II PT (prothrombin time)
III INR (international normalization ratio)
a. I only
b. III only
c. I and II
d. II and III
e. I, II, III

12-Which of the following drugs increase international normalized ratio (INR)?


A-Clotrimazole B-Clindamycin C-Azithromycin D-Penicillin's

13) Which of the of the following can be used in a pregnant women with deep vein
thrombosis ?
A-Captopril B-Warfarin C-Atorvastatin D-Heparin E-Rosuvastatin

14) Which of the following is NOT a low molecular weight heparins (LMWH)?
A-Dalteparin B-Heparin C-Nadoparin D-Enoxaparin E-Tinazeparin

15) What is incorrect about Reynaud's phenomenon?


A- Reynaud's phenomenon in which arteries of fingers become spastic (vasospastic).
B-Reynaud's phenomenon may result from atherosclerosis, connective tissue disease,
ingestion of ergot alkaloids, or frequent use of vibrating tools.
C- Reynaud's phenomenon may result from peripheral artriel plaques formation
D) Reynaud's phenomenon symptoms fingers spasm, extremities turn to pale, or blue.
E- Reynaud's phenomenon non-pharmacological is minimizing cold exposure, and use warm
gloves

16)Which of the following the most important drug is used to prevent peripheral vascular
diseases?
A)Beta blocker B)ACEI C) warfarin D)digoxin E)ASA

17) Which of the following the most important drug is used to treat peripheral vascular
diseases?
A)Beta blocker B)ACEI C) warfarin D)digoxin E)ASA

18) Which of the following drug is least likely used to treat peripheral vascular diseases?
A)Beta blocker B)ACEi C) warfarin D)digoxin E) ASA

19) Which of the following is incorrect statement?


A)Reynaud phenomenon symptoms occurs in limb extremities
B)Calcium channel blockers are the drug of choice
C)Arterial plaques are the risk factor for Raynaud phenomenon

Copyright © 2000-2016 TIPS Inc. Unauthorized reproduction of this manual is prohibited. This manual is 90-2
being used during review sessions conducted by PharmacyPrep.
PharmacyPrep.Com Peripheral Vascular Diseases

D)Reynaud phenomenon can be trigger cold exposure


E)Reynaud phenomenon is may occur in old age

20) Which of the following drugs increase risk of blood clots in peripheral vascular system?
A) Enoxaparin B) Alteplase C) combined oral contraceptive pills D. Warfarin E. NSAIDs

21) Which of the following drugs least likely used in DVT patients?
A) warfarin B) Dabigatran C) LMWH D) heparin E) cardio selective beta blockers

22) All are risk factors of deep vein thrombosis (DVT), except?
A) pregnancy
B) cancer and chemotherapy
C) Progestin only oral contraceptive pills
D) diabetes
E) mobility

23) A 64 yo man weighing 115 kg, and tall 60 inches, presents to his physician after long
international flight complaining of pain, swelling of his right lower leg. The patient had knee
surgery a month before he travelled. His medication profile include, CHF (ejection fraction <
15%), remission from lymphoma, MI. His father, mother, and sister are all diseased from
stroke, pulmonary embolism. Given his profile. Patient most likely suffering from?
A) deep vein thrombosis
B) coronary artery disease
C) congestive heart failure
D) diabetes

24) Which of the following is NOT risk factors of deep vein thrombosis (DVT)?
A) Knee surgery B) over weight C) long flight D) family history E) gender

25. What is the drug of choice to treat DVT in pregnancy?


A. Dalteparin
B. Heparin
C. warfarin
D. Dabigatran
E. apixaban
Ans. A

26. All of the following conditions are associated with atherosclerosis plaque, except?
A. DVT
B. Stable angina
C. MI
D. Stroke
E. Raynaud's phenomenon
Ans. E

Copyright © 2000-2016 TIPS Inc. Unauthorized reproduction of this manual is prohibited. This manual is 90-3
being used during review sessions conducted by PharmacyPrep.
PharmacyPrep.Com Peripheral Vascular Diseases

CORRECT ANSWERS
Peripheral Vascular Diseases

1)Ans: A

2) Ans: B

3) Ans: C

4) Ans: D

5) Ans: D

6) Ans: E

7) Ans: D

8) Ans: E

9) Ans: A

10. Ans: E
Tips: INR and PT is monitored for warfarin, aPTT for heparin, where as for LMWH these tests
are not effective

11. Ans: D

12- Ans: A

13) Ans: D

14) Ans: B

15) Ans: C

16) Ans: C
Tips: Anticoagulants like warfarin is commonly used to treat peripheral embolic disorders.

17) Ans: C

18) Ans: A
Tips: beta blocker cause vascular constriction thus it is contraindicated in peripheral vascular
diseases

Copyright © 2000-2016 TIPS Inc. Unauthorized reproduction of this manual is prohibited. This manual is 90-4
being used during review sessions conducted by PharmacyPrep.
PharmacyPrep.Com Peripheral Vascular Diseases

19) C Ans:

20) Ans: C
Tips. Combined oral contraceptive pills are combination of estrogen and progestin increase
platelet aggregation and thus increase risk of blood clot formation.

21) Ans: E

22) Ans: E
Tips: mobility or physical activity lower the risk of deep vein thrombosis

23) Ans: A

24) Ans: E
Tips: RF for above patient, except gender. other RFs include age > 40 yr, cancer, CHF, MI, and
obesity.

Copyright © 2000-2016 TIPS Inc. Unauthorized reproduction of this manual is prohibited. This manual is 90-5
being used during review sessions conducted by PharmacyPrep.
PharmacyPrep.Com Peripheral Vascular Diseases

Copyright © 2000-2016 TIPS Inc. Unauthorized reproduction of this manual is prohibited. This manual is 90-6
being used during review sessions conducted by PharmacyPrep.
Pharmacy Prep
ANTICOAGULANTS

1-Which of the following clotting factor is NOT inhibited by warfarin?


A-2 B-7 C-9 D-10 E-12

2. A drug is said to reach "steady state" when 5 half-lives have elapsed. In the case of warfarin
(a blood thinner), however, the steady state is dependent not upon the drug's half-life, but
upon the half-life of the clotting factors it inhibits. The clotting factors inhibited by warfarin
have the following half-lives:
Factor II - 50 hours
Factor VII - 6 hours
Factor IX - 24 hours
Factor X - 36 hours
Based on this information, how long will it take a patient to reach maximal anticoagulation?
A- 5 days B-10 days C- 15 days D- 20 days E-48 hours

3-The hematocrit (HCT) measures the


A. Total number of blood cells per volume of blood
B. Weight of red blood cells per volume of blood
C. Number of red blood cells per volume of blood
D. Weight of hemoglobin per volume of blood
E. Percentage of red blood cells per volume of blood

4) A patient is using injections of Dalteparin to treat deep vein thrombosis what is monitored?
A) aPTT B) antifactor Xa C) Prothrombin time D) international normalized ratio E) Heparin
assay

5) Which of the following oral anticoagulant approved to treat atrial fibrillation?


A) heparin B) dalteparin C)Tinzaparin D)Dabigatran E)all of the above

6) What is incorrect statement?


A) LMWH therapy are effective in preventing DVT
B) Heparin therapy require close monitoring than LMWH
C) Heparin is weight based intravenous therapy is used in acute DVT
D) Warfarin produce rapid anticoagulation than LMWH
E) Long term heparin therapy can cause osteoporosis side effect

7) What is incorrect statement?


A) LMWH have longer duration of action than heparin
B) LMWH once daily SC administration
C) LMWH do not require routine monitoring
D) Heparin has rapid anticoagulant effect
E) aPTT reflects changes in extrinsic pathway of the coagulation cascade

Copyright © 2000-2016 TIPS Inc. Unauthorized reproduction of this manual is prohibited. This manual is being 91-1
used during review sessions conducted by PharmacyPrep.
8) Patient is hospitalized. Doctors initiated heparin iv infusion. What is incorrect about heparin
A) heparin dose is based on body weight
B) loading dose is 80 units/kg, and maintenance dose is 15 to 25 units/kg/hr.
C) Heparin is slower onset than LMWH
D) LMWH have longer half life
E) LMWH have predictable response thus not monitored

9) Over dose of heparin is treated by?


I) Stop heparin infusion
II) Protamine sulfate
III) oral vitamin K
A. I only
B. III only
C. I and II
D. II and III
E. I, II, III

10) Patient is receiving warfarin after discharged from hospital. He is receiving cotrimoxazole to
treat UTI. One may expect to see the INR (2 - 3)?
I) Increase II) No change III) Decrease
A. I only
B. III only
C. I and II
D. II and III
E. I, II, III

11. What anticoagulant is used in emergency as iv dosage form?


A) Heparin
B) Low molecular weight heparins (LMWH)
C) Warfarin
D) Dabigatran
E. Apixaban

12. The mechanism of action of Acetylsalicylic acid that results in greater effect as an
antiplatelet drug is because of:
A. Selective COX I inhibitor
B. Selective COX II inhibitor
C. COX I and COX II inhibitor
D. Acetylate cyclooxygenase

13) Which of the following anticoagulant available as iv and sc injections


A. Heparin
B. LMWH
C. Warfarin

Copyright © 2000-2016 TIPS Inc. Unauthorized reproduction of this manual is prohibited. This manual is being 91-2
used during review sessions conducted by PharmacyPrep.
D. Apixaban
E. Dabigatran

14) What should be monitored in heparin treatment?


A. INR (International Normalised Ratio)
B. Vitamin K
C. aPTT
D. Serum levels of Heparin
E. Prothrombin Time

15) Which of the following tests are used for antithrombotic therapy:
A. INR, Prothrombin time and aPTT
B. aPTT and Prothrombin Time
C. Heparin assay and a PTT
D.INR only
E. a PTT only

16) Which of the following is the drug of choice to treat deep vein thrombosis (DVT):
A) Warfarin
B. streptokinase
C. Heparin
D. LMWH
E. Rivaroxaban

17) What anticoagulants is safe in renal disease patients?


A. warfarin
B. dalteparin
C. Heparin
D. dabigatran
E. apixaban

18) In order to measure effectiveness of warfarin therapy, the following monitoring tests are
used:
A. INR, must be 2-3
B. Prothrombin test (PT)
C. aPTT
D. A&B

19) At what dose Aspirin has anticoagulant property


A. 80 mg
B. 325mg
C. 500 mg
D. 800 mg

Copyright © 2000-2016 TIPS Inc. Unauthorized reproduction of this manual is prohibited. This manual is being 91-3
used during review sessions conducted by PharmacyPrep.
20) Effects of warfarin include all except:
A. Decreasing the formation of blood clot
B. Nose bleeds
C. Red-orange urine
D. Ringing in the ears

21) All drugs will decrease INR of warfarin except:


A. Rifampin
B. Barbiturate
C. Green leafy vegetable
D. Metronidazole

22) Warfarin site of action is:


A. In vivo
B. In vitro
C. In vivo and in vitro
D. Neither

23) Which drug is not a low molecular weight heparin (LMWH)?


A. Enoxaparin
B. Nadroparin
C. Heparin
D. Dalteparin

24) What is the safest anticoagulant in pregnancy?


A. Dalteparin
B. Warfarin
C. Heparinoids
D. Heparin
E. Rivaroxaban

25) Antidote for heparin toxicity:


A. Vitamin K
B. Protamine sulphate
C. Physostigmine
D. Atropine

26) All of the following increases INR except:


A. ASA and NSAID
B. Chronic alcohol
C. Cotrimoxazole
D. Acute alcohol

Copyright © 2000-2016 TIPS Inc. Unauthorized reproduction of this manual is prohibited. This manual is being 91-4
used during review sessions conducted by PharmacyPrep.
27) Which of the following vitamins is needed for proper blood clotting?
A. Vitamin A
B. Vitamin D
C. Vitamin E
D. Vitamin K

28) Which of the following OTC drugs is contraindicated for a patient taking warfarin?
A. Diphenhydramine
B. Loratadine
C. Psyllium
D. ASA

29) All of the following drugs should be carefully prescribed with aspirin EXCEPT:
A. Enoxaparin
B. Coumadin
C. Heparin
D. Metoclopramide

30) An overdose of warfarin can be treated by administering:


A. Vitamin K
B. Acetylcysteine
C. MESNA
D. Protamine sulfate
E. Sodium bicarbonate

31. What anticoagulants is safe in renal disease patients?


A. warfarin
B. dalteparin
C. Heparin
D. dabigatran
E. apixaban
Ans. A

32. What is rat poison?


A. Coumarin
B. warfarin
C. Heparin
D. LMWH
E. Dabigatran
Ans. A

33. What is incorrect about anticoagulants?


A. warfarin is preferable because antidote is readily available
B. warfarin is less preferable after surgeries because of increase risk of bleeding
C. warfarin require frequent monitoring

Copyright © 2000-2016 TIPS Inc. Unauthorized reproduction of this manual is prohibited. This manual is being 91-5
used during review sessions conducted by PharmacyPrep.
D. warfarin is not available in liquid or injections
E. warfarin require dose adjustment in renal disease patients.
Ans. e

34. what anticoagulant is safe in pregnancy with DVT?


A. warfarin
B. dalteparin
C. Fondaparinux
D. dabigatran
E. apixaban
Ans. B

Copyright © 2000-2016 TIPS Inc. Unauthorized reproduction of this manual is prohibited. This manual is being 91-6
used during review sessions conducted by PharmacyPrep.
ANTICOAGULANTS
ANSWERS:

1- Ans: E

2. Ans: B
Tips: “maximal” anticoagulation will be reached when all clotting factors are inhibited. This will
take approximately 10 days, since 50hrs (the half-life of the clotting factor taking the longest
time to reach steady) x 5 = 250 hours, which is 10.4 days.

3- Ans: E
Tips: Whole blood treated with anticoagulant is centrifuged in a calibrated hematocrit tube. The
volume ratio of the packed red blood cells to total blood volume is determined. The hematocrit
is normally 39 to 49 for men and 33 to 43 for women. The hematocrit value provides some
indication of both the number and size of the red blood cells present in an individual

4) Ans: B

5) Ans: D
Tips: Dabigatran an oral anticoagulants recently approved to treat atrial fibrillations. It is factor
Xa inhibitor. (competitive direct thrombin inhibitor).

6) Ans: D

7) Ans: E

8) Ans: C
Tips: Heparin iv infusion is faster onset than LMWH.

9) Ans: E

10) Ans: A
Tips: antibiotics potentiate anticoagulant effect : cotrimoxazole, erythromycin, metronidazole,
(other drugs: omeprazole, fluconazole, ketoconazole, cimetidine, amiodarone, quinidine,
vitamin E (large doses), acute phenytoin, thyroxin, acute alcohol also potentiate.
Reduce the clearance of warfarin, displacement from warfarin protein binding, decrease
intestinal flora
Drugs that inhibit anticoagulant effect: carbamazepine, penicillin, rifampin, cholestyramine,
barbiturates, high vitamin K, chronic alcohol use

11. Ans-A

Copyright © 2000-2016 TIPS Inc. Unauthorized reproduction of this manual is prohibited. This manual is being 91-7
used during review sessions conducted by PharmacyPrep.
12. Ans: D
Tips: Aspirin is the only drug that irreversibly inhibits the enzyme. This action permits a longer
duration of action causing platelet aggregation and acetylation of cyclooxygenase.

13) Ans: A

14) Ans-C

15) Ans-A

16) Ans-D
Tips: LMWH is the drug of choice to treat DVT. Alternatively heparin is in emergency. The oral
anticoagulants are used for maintenance.

17) Ans. A

18) Ans-D

19) Ans-A

20) Ans: D
Tips: Ringing in the ear is associated with ASA, salicylates, quinine, and quinidine, but not
warfarin.

21) Ans: D
Tips: Metronidazole increases INR

22) Ans: A
Tips: Warfarin site of action is in vivo and heparin is either in vivo or in vitro

23) Ans: C

24) Ans: D

25) Ans: B

26) Ans: B
Tips: Chronic alcohol decreases INR while acute alcohol increases INR.

27) Ans: D

28) Ans: D

Copyright © 2000-2016 TIPS Inc. Unauthorized reproduction of this manual is prohibited. This manual is being 91-8
used during review sessions conducted by PharmacyPrep.
29) Ans: D
Tips: Aspirin is a blood thinning agent indicated for prevention of heart stroke. It should be
carefully prescribed with other blood thinning agents
because of risks of bleeding. Lovenox (Enoxaparin), Coumadin (Warfarin), Heparin, Plavix
(Clopidrogel), Ticlid (Ticlopidine), Depakene (Valproic acid), Persantine (Dipyridamole), Mandol
(Cefamandole), Cefotan (Cefotetan), Cefobid (Cefoperazone) and Moxam (Moxalactam) are
agents that increase bleeding tendency in patients and should be carefully prescribed with
other blood thinning agents.

30) Ans: A
Tips: The overdose of Coumadin can be treated by administration of Vitamin K1 (Mephyton).

Copyright © 2000-2016 TIPS Inc. Unauthorized reproduction of this manual is prohibited. This manual is being 91-9
used during review sessions conducted by PharmacyPrep.
PharmacyPrep.Com Anxiety Disorders

PHARMACY PREP

ANXIETY DISORDERS

1. What is the drug of choice for panic attacks?


A. amitriptyline B. propranolol C. SSRI D. benzodiazepine
Ans. C
Tips: benzodiazepine for short term use only if needed, then the drug therapy is SSRI and
alternate TCA with cognitive behavior therapy.

Copyright © 2000-2016 TIPS Inc. Unauthorized reproduction of this manual is prohibited. This manual is 92-1
being used during review sessions conducted by PharmacyPrep.
PharmacyPrep.com Depression

PHARMACY PREP
DEPRESSION

1. Duration of therapy for first episode of major depression is?


A. 3 mo B. 6 wks C. 1 year D. 2 to 3 yrs E. Lifelong
Ans. c

2. What antidepressants have sweating side effects?


a. SSRIs B. SNRIs C. TCA D. MAOi E. NDRIs
Ans. A
Tips:
Antidepressants Highs Side effects Lows side effects
Mirtazapine Weight gain Nausea & vomiting
Bupropion Sexual dys. & weight gain
Trazadone Sedation Insomnia
Fluvoxamine Drug interaction
Sertraline Drug interaction.
Fluoxetine Wash out period Weight gain
Moclobamide DIs Sexual dysfunction

3. What is the onset of SSRIs?


A. 1 to 4 days B. 1 to 4 wks C. 1 to 4 months D. 1 to 4 years
Ans. B
Antidepressants Onset Tips
SSRI 1 to 4 wks safe
TCA s
MAOi s
SNRI 1 to 4 wks safe
Lithium s a Early toxic: N&V, D
Late: cerebellar ataxis, mental confusion

4 A patient with major depression, doctor initiated fluoxetine. However patient did not
tolerated and there was no improvement. Doctor initiated phenelzine, a week later. After
patient started therapy for phenelzine, 4 days later the patient presented with fever, high
blood pressure, diarrhea, and confusion. What is the most likely cause?
A) Neuroleptic malignant syndrome
B) Hypertension crisis
C) Serotonin syndrome
D) Serotonin withdrawal syndrome
Ans: C

Copyright © 2000-2016 TIPS Inc. Unauthorized reproduction of this manual is prohibited. This manual is 93-1
being used during review sessions conducted by PharmacyPrep.
PharmacyPrep.com Depression

Tips:
Serotonin symptoms Withdrawal symptoms
Autonomic Diarrhea, Fever, Shivering, Flu like fever, chills, dizziness, light
change in BP, and & N, V headed, N &V, sleep disturbances
Neuromuscular Tremors, seizure, myoclonus, Myalgia, lethargy, sensory
Dysfunction hyper reflexes disturbances
Cognitive Agitation, Confusion, Not present
hypomania

5. What serum level lithium give toxicity symptoms?


A) Lithium serum levels > 2 mEq/L
B) Lithium serum levels < 1.5 mEq/L
C) Lithium serum levels < 1 mEq/L
D) Lithium serum levels <0.5 mEq/L
Ans: A
Tips: Lithium serum levels > 2mEq/L can give lithium toxicity.

Initial Onset SEs Lithium Long term SEs Lithium Toxicity


N&V, GI upset, Dry mouth, Fine hand tremors Coarse hand tremor
Fine hand tremor, sedation Weight gain Muscle twitching
Muscle weakness Hypothyroidism Vomiting, severe drowsiness,
Polydypsia acne, psoriasis confusion, nystagmus, seizure,
Polyuria rash, alopecia Ataxia, cogwheel rigidity,
Nocturia Decreased libido coma, death
Arrhythmia: Non specific T-wave
changes, Nephrotoxicity
Nephrogenic
Monitor: Li, Na, Serum TSH, CBC, weight, CrCl (No LFT)

Copyright © 2000-2016 TIPS Inc. Unauthorized reproduction of this manual is prohibited. This manual is 93-2
being used during review sessions conducted by PharmacyPrep.
PharmacyPrep.Com Antipsychotics

PHARMACY PREP
ANTIPSYCHOTICS

1. Drug of choice to treat negative and positive symptoms?


A. haloperidol
B. Olanzapine
C. Fluoxetine
D. Metoclopramide
E. Chlopromazine
Ans. B

1. A patient receiving haloperidol to treat schizophrenia. Presents with hyperthermia (fever),


sweating, tachycardia, muscle rigidity. Which of the following most likely cause?
A) Neuroleptic malignant syndrome
B) Serotonin syndrome
C) withdrawal symptoms
D) Extra pyramidal symptoms
Ans: A
Tips: NMS develops quickly and symptoms stay for 5 to 10 days. Require referral to
emergence.

Extrapyramidal symptoms (EPS) Neuroleptic malignant symptoms (NMS)


Parkinson's (TRAP) Fever
Akathisia Rigidity
Dystonia tachycardia
Tardive dyskinesia Fluctuation BP
WBC, CK

Copyright © 2000-2016 TIPS Inc. Unauthorized reproduction of this manual is strictly prohibited and it is
illegal to reproduce without permission. This manual is being used during review sessions conducted by
PharmacyPrep.
1
PharmacyPrep,Com Dementia

PHARMACY PREP

DEMENTIA

1) Alzheimer is a progressive disease characterized by relentless and loss of mental function


due to lack of:
A. Thiamine
B. Dopamine
C. Acetylcholine
D. Norepinephrine
E. Epinephrine

2. All of the following are symptoms of Alzheimer’s dementia, EXCEPT:


A. Slurred speech
B. Reduced cognitive functions
C. Fatigue
D. Reduced sexual function
E. Incoherent speech

3) All of the following are symptoms of Alzheimer's dementia, except?


A) loosing items
B) impaired perception of time
C) Hallucinations and dilutions
D) Depression
E) able to execute functions

4. Risk factors of Alzheimer’s dementia


A. High cholesterol
B. Smoking
C. Use of alcohol
D. Obesity
E. All of the above

5) All of the following are risk factors of Alzheimer's dementia, except?


A) Age B) Family history C) race D) coronary artery diseases E) sedentary life style

6) What is incorrect about Alzheimer’s dementia?


A)A e er s e en a cc rs e c
B)the drug of choice against Alzheimer's dementia is donepezil
C)Delirium is acute agitation, lack of judgement and thinking.
D)Amnesia is short loss of memory
E) Old age is NOT a risk factors for Alzheimer's dementia

Copyright © 2000-2016 TIPS Inc. Unauthorized reproduction of this manual is prohibited. This manual is 95-1
being used during review sessions conducted by PharmacyPrep.
PharmacyPrep,Com Dementia

7. What is the drug of choice for the treatment of Alzheimer's dementia ?


A. Galantamine
B. Donepezil
C. Rivastigmine
D. Neostigmine
E. Memantine

8. Which of the following is classified as non selective reversible acetyl cholinesterase


inhibitor?
A. Galantamine
B. Donepezil
C. Rivastigmine
D. Memantine
E. Neostigmine

9) What is/are the common side effects of Donepezil, Rivastigmine, and Galantamine?
A. Confusion
B. Sweating
C. Diarrhea
D. Cholinergic agonist effects
E. All of the above

10) Drug that increase effectiveness of Donepezil:


A. Carbamazepine
B. Erythromycin
C. Phenytoin
D. All of the above

11) What drugs is/are not a treatment for Alzheimer's?


A. Rivastigmine
B. Donepezil
C. Memantine
D. Gingko biloba
E. Galantamine

12) Which herbal product is used to treat Alzheimer and dementia?


A. Gingko biloba
B. Vitamin D
C. St. John Wort
D. Omega 3
E. Licorice

13)What is incorrect statement?


A) donepazil is selective acetylcholinesterase inhibitor

Copyright © 2000-2016 TIPS Inc. Unauthorized reproduction of this manual is prohibited. This manual is 95-2
being used during review sessions conducted by PharmacyPrep.
PharmacyPrep,Com Dementia

B) Rivastigmine is a acetylcholinesterase and butyryl cholinesterase inhibitor


C) Galantamine is acetylcholinesterase inhibitor and also effect on acetyl choline
by binding with nicotinic receptors
D) Memantine is NMDA receptor antagonist
E) None of the above
14.What is the drug of choice to treat levy body dementia?
A. Rivastigmine
B. Donepezil
C. Memantine
D. Gingko biloba
E. Galantamine
Ans. A

Copyright © 2000-2016 TIPS Inc. Unauthorized reproduction of this manual is prohibited. This manual is 95-3
being used during review sessions conducted by PharmacyPrep.
PharmacyPrep,Com Dementia

CORRECT ANSWERS
DEMENTIA

1) Ans: C
Tips: deficiency of acetylcholine and damage of brain cells in older age causes dementia.

2) C

3) E

4) E
Tips: Lifestyle factors like alcohol intake have no evidence worsening Alzheimer's.

5) C

6) E
Tips: old age is a risk factor for Alzheimer's dementia

7) Ans: B

8) Ans: A

9) Ans: E
Tips.Dementia is treated by acetylcholinesterase inhibitors i.e. cholinergic agonist, thus
produce cholinergic agonist side effects " DUMBLESS"

10) Ans: A
Tips: Drugs that increases Donepezil are inhibitors of CYP 2D6 or CYP 3A4. Drugs that
decreases Donepezil are inducers of CYP 2D6 or CYP 3A4.

11) Ans: C

12) Ans: A

13) Ans: E

Copyright © 2000-2016 TIPS Inc. Unauthorized reproduction of this manual is prohibited. This manual is 95-4
being used during review sessions conducted by PharmacyPrep.
PharmacyPrep.com Seizure and antiepileptic drugs

PHARMACY PREP

SEIZURE & ANTIEPILEPTICS

1) What type of seizure symptoms in-patient is NOT unconscious?


A) Simple partial
B) Complex partial
C) Generalized seizures
D) Petit mal seizures
E) Absence seizures

2) Phenytoin is indicated the treatment of all of the following types of seizure EXCEPT:
A-Grand mal
B-Complex partial
C-Simple partial
D-Tonic clonic
E) Absence seizure (petit mal)

3)Which of the following drug is NOT used in absence seizure patient?


A-Phenytoin
B-Valproic acid
C-Diazepam
D-Ethosuximide
E-All of the above

4) Which of the following drug least likely to effect on oral contraceptives:


A-Tetracycline
B-Phenytoin
C-Carbamazepine
D-Gabapentin
E-Phenobarbital

5) Which of the following is the drug of choice in status epilepticus?


A-phenytoin iv
B-Carbamazepine iv
C-Diazepam iv
D-Infliximab
E-Valproic acid

6) Which of the following anticonvulsive drug treatment most likely cause kidney stone
formation.
A-Carbamazepine
Copyright © 2000-2016 TIPS Inc. Unauthorized reproduction of this manual is strictly prohibited and it is
illegal to reproduce without permission. This manual is being used during review sessions conducted by
PharmacyPrep.
96-1
PharmacyPrep.com Seizure and antiepileptic drugs

B-Phenytoin
C-Tiagabine
D-Topiramate
E-Phenobarbital

7)The most common side effects of anti seizure drugs:


I-Appetite and body weight changes
II-GI symptoms
III-Head ache and dizziness
A-I only B-III only C-I and II D-II and III E-I, II, III

8) Which of the following antiepileptic drugs has weight loss side effects:
A-Valproic acid
B-valproate
C-Phenytoin
D-Topiramate
E-Carbamazepine

9) A 25-year-old female patient taking gabapentin for neuropathic pain, which of the following
statement is/are true:
I-She should avoid taking oral contraceptive because, this drug decrease effectiveness of
gabapentin
II-Gabapentin chemical structure is not similar to GABA
III-Gabapentin chemical structure contains GABA structure.
A-I only
B-III only
C-I and II only
D-II and III only
E-All of the above

10) Which of the antiseizure drug has weight loss side effects.
A-Topiramate
B-Carbamazepine
C-Phenytoin
D-Valproic acid
E-Gabapentin

11) A 25-year-old female patient taking gabapentin for neuropathic pain, which of the
following statement is not true:
A-She should avoid taking oral contraceptive because, this drug decrease effectiveness of
gabapentin
B-Gabapentin have advantage of the least drug interactions
Copyright © 2000-2016 TIPS Inc. Unauthorized reproduction of this manual is strictly prohibited and it is
illegal to reproduce without permission. This manual is being used during review sessions conducted by
PharmacyPrep.
96-2
PharmacyPrep.com Seizure and antiepileptic drugs

C-Gabapentin chemical structure contains GABA structure


D-Gabapentin can be used seizures and neuralgia
E-Gabapentin is GABA analog

12) What is incorrect about phenytoin?


A-Phenytoin available as suspension, iv, chewable tablets and capsule.
B-Always recommend chlorhexidine to treat deficiency of folic acid
C-Phenytoin blood levels should be monitored to 10 to 20 mcg/ml
D-Phenytoin have saturated kinetics, thus overdose can give toxicity
E-Chronic use of phenytoin can cause megaloblastic anemia

13) Symptoms of Steven Johnson’s Syndrome, include all except


A-Rash
B-Sores on mucus membrane,
C-Blistering mucus membrane, typically mouth, eyes and vagina.
D- Patchy areas rashes.
E-Butterfly rash

14-What is drug of choice in petit-mal seizure?


A-Diazepam
B-Phenytoin
C-Carbamazepine
D-Valproic acid and ethosuximide
E-Gabapentin

15-Valproic acid is used in treatment of


a) Petit-mal
b) Grand-mal
c) Febrile seizure in children
d) All of the above
e) A and C only

16-Anticonvulsive agent that cause weight loss?


A-Topiramate B-Phenytoin C-Carbamazepine D-Valproic acid E-Gabapentin

17-Phenytoin causes elevation of blood glucose levels by?


A-Increasing insulin degradation
B-Decreasing insulin release
C-Increasing glucose absorption
D-increasing glycogen release

18-Side effects of phenytoin are?


Copyright © 2000-2016 TIPS Inc. Unauthorized reproduction of this manual is strictly prohibited and it is
illegal to reproduce without permission. This manual is being used during review sessions conducted by
PharmacyPrep.
96-3
PharmacyPrep.com Seizure and antiepileptic drugs

A-Gingival hyperplasia
B-Steven Johnson syndrome
C-Hirsutism
D-All of the above
E-A and C

19-Epilepsy cause due to deficiency of?


A-Sodium ion B-GABA C-Serotonin D-Dopamine E-Acetylcholine

20-Early symptoms of epilepsy are?


A-Part of brain is involved
B-Sensory seizure or focal motor seizure
C-Patient is conscious
D-All of the above

21-Which one of the following CNS receptors is directly coupled to an ion channels, and this
do not involve secondary messenger system?
A-Alpha (NE)
B-dopamine D 2 (DA)
C-Acetyl choline (N)
D- serotonin (5HT)
E- None of the above

22-What are antimuscarinic drugs that are used in treatment of antipsychotic induced
Parkinson’s disease,
A-Benztropine
B-Trihexphenidyl
C-Bromocriptine
D-Diphenhydramine
E-Procyclidine

23-What type of local anesthetic is the most effective?


A-The most hydrophilic B-The most lipophylic C-The least hydrophilic
D-The least lipophylic

24-Metabolism of local anesthetic takes place in?


A-Skin B-Tissue C-Liver D-Gut E-Local area

25) All of the following are GABA analogs except?


A-Gabapentin B-Pregabalin C-Vigabatrin D-Beclofen E-Phenytoin

26) Tachycardia is the side effect of?


Copyright © 2000-2016 TIPS Inc. Unauthorized reproduction of this manual is strictly prohibited and it is
illegal to reproduce without permission. This manual is being used during review sessions conducted by
PharmacyPrep.
96-4
PharmacyPrep.com Seizure and antiepileptic drugs

A-Anticholinergic
B-Sympathomimetics
C-Sympathetic blockers
D-Non dihydropyridine type of CCBs
E-All of the above

27) Treatment of epilepsy for?


A-Inhibiting GABA neurotransmitter
B-Stimulating NMDA neurotransmitter (N-methyl D-Aspartate)
C-Decrease firing threshold impulse
D-Stimulating sodium ion channels
E-Stimulating potassium ion channels

28) Valproic acid is used to treat?


I-Petit-mal (absent of seizure) II-Grand-mal (general seizure) III-Febrile seizure in children
A-I only B-III only C-I and II only D-II and III only E-I, II,III

29. When changing from phenytoin syrup to phenytoin capsule, at same dose, change in
effect is due to
I. Change in salt form
II. Change in dose
III. Change in dose elimination
A-I only B-III only C-I and II only D-II and III only E-I, II,III

30) What is drug of choice for petit mal (absence) seizure?


A-Diazepam iv
B-Diazepam oral
C. Ethosuximide and valproic acid
D-Phenytoin
E-Carbamazepine

31) Side effects of phenytoin include?


I-Gingival hyperplasia, ataxia, nystagmus
II-Steven Johnsons syndrome
III-Hirsutism and acne
A-I only B-III only C-I and II only D-II and III only E-I, II,III

32) Epilepsy is resulted from the deficiency of?--> GABA action; Early symptoms of partial
seizures are?
I-Part of the brain
II-Sensory seizure or focal motor seizure
III-Patient is conscious
Copyright © 2000-2016 TIPS Inc. Unauthorized reproduction of this manual is strictly prohibited and it is
illegal to reproduce without permission. This manual is being used during review sessions conducted by
PharmacyPrep.
96-5
PharmacyPrep.com Seizure and antiepileptic drugs

A-I only B-III only C-I and II only D-II and III only E-I, II,III

33)What are the common SEs of antiepileptic drugs?


A) Stevens Johnson Syndrome
B) Kidney stone formation
C)Head ache, dizziness, GI, appetite, weight gain,
D) Weight loss, euphoria

34) A 25 yo women with one 2 yr old child. She has been using antiepileptic drugs for the past
1 year. Which drug least likely decrease efficacy of antiepileptic drugs?
a) Phenytoin B) Carbamazepine C) Valproic acid D) Lamotrigine

35) MP is a 23 year old women. In her medication profile she is valproic acid daily for seizure.
She calls to your pharmacy and ask if she should stop taking her medication because for the
past few weeks she has no seizures? What is the common reason for medication non
compliance in patient using antiepileptic medications?
A) because of drugs taste bad
B) because recently had no seizure
C) because expensive medications
D) because complicated storage conditions

36) Ethosuximide is drug of choice for the following seizures:


A. Absence seizures
B. Tonic clonic
C. status epilepticus
D. A&B

37) Oral contraceptives interact with:


A. Antibiotics
B. Anticonvulsant
C. Antifungals
D. All of the above

38) What drugs should be avoided in absence seizure?


A. Phenytoin and valproic acid
B. Valproic acid and carbamazepine
C. Gabapentin and Phenytoin
D. Topiramate and Ethosuximide
E. Phenytoin and Carbamazepine

39) Drugs that has least likely have drug interaction with oral contraceptives except:
Copyright © 2000-2016 TIPS Inc. Unauthorized reproduction of this manual is strictly prohibited and it is
illegal to reproduce without permission. This manual is being used during review sessions conducted by
PharmacyPrep.
96-6
PharmacyPrep.com Seizure and antiepileptic drugs

A. Gabapentin
B. Ethosuximide
C. Valproic acid
D. Lamotrigine

40) One side effect of Phenytoin is gingival hyperplasia, the best way to prevent it:
A. Use chlorhexidine wash
B. Use dental floss
C. Take Vitamin C
D. Gargle with water

41) Anti epileptic drug that doesn’t cause weight gain?


A. Valproic acid
B. Lamotrigine
C. Topiramate
D. Phenobarbital

42) Phenytoin available in which of the dosage forms?


A. Injection
B. Capsule
C. Chewable tablets
D. All of the above

43) Valproic acid is the drug of choice for


A. Absence seizure
B. Myoclonus
C. Tonic
D. all of the above

44) Drug use as “add on” for patients nearly seizure free:
A. Gabapentin
B. Clobazam
C. Valproic acid
D. Phenytoin

45) Phenytoin metabolism is not inhibited by:


A. Sulfonamide
B. Isoniazid
C. Carbamazepine
D. Chloramphenicol

46) Divalproex is a mixture of:


Copyright © 2000-2016 TIPS Inc. Unauthorized reproduction of this manual is strictly prohibited and it is
illegal to reproduce without permission. This manual is being used during review sessions conducted by
PharmacyPrep.
96-7
PharmacyPrep.com Seizure and antiepileptic drugs

A. Gabapentin
B. Valproic acid
C. Sodium valproate
D. A and C

47) What drug inhibits glutamate and aspartate release and blocks sodium channels and
prevents repetitive firing:
A. Topiramate
B. Lamotrigine
C. Clobazam
D. Gabapentin

48) What antiepileptic is found in hospital emergence carts?


A. Diazepam iv
B. Chlorpromazine
C. Carbamazepine
D. Lithium
E) Phenytoin IV

SEIZURE & ANTIEPILEPTICS


Copyright © 2000-2016 TIPS Inc. Unauthorized reproduction of this manual is strictly prohibited and it is
illegal to reproduce without permission. This manual is being used during review sessions conducted by
PharmacyPrep.
96-8
PharmacyPrep.com Seizure and antiepileptic drugs

ANSWERS:

1) Ans: A

2. Ans: E
Tips: Grand mal Valproic acid, Complex partial Carbamazepine, Simple partial
Carbamazepine, Tonic-clonic Valproic acid, Absence seizure Ethosuximide, and Status
epilepticus Diazepam

3) Ans: A

4) Ans: D

5) Ans: C

6) Ans: D
Tips: topiramate is a derivative of fructose. Indicated as anti seizure drug—partial seizure,
tonic clonic seizures. Has high kidney stone formation side effect. Fibrates also causes gall
bladder stone formation

7) Ans: E
Tips: valproic acid and valproate increase appetite and may results in weight gain,
carbamazepine may cause dizziness and confusion.

8) Ans: D

9) Ans: B

10) Ans: A

11) Ans: A
Tips: Gabapentin do not decrease efficacy of oral contraceptives and can be used along with
oral contraceptives.

12) Ans: B

13) Ans: E
Tips: butterfly rash symptoms occurs in lupus erythromatus

14- Ans: D

15- Ans: D
Copyright © 2000-2016 TIPS Inc. Unauthorized reproduction of this manual is strictly prohibited and it is
illegal to reproduce without permission. This manual is being used during review sessions conducted by
PharmacyPrep.
96-9
PharmacyPrep.com Seizure and antiepileptic drugs

16- Ans: A

17- Ans: B

18- Ans: D

19- Ans: B

20- Ans: D

21- Ans: C
Tips; Ach receptors, in the CNS are present on less than 5% of the neuronal population.

22- Ans: A and B

23- Ans: B

24- Ans: C

25) Ans: E

26) Ans: A

27) Ans: A

28) Ans: C
Tips: valproic acid is the drug of choice for generalized seizures and also can be used for petit
mal seizures.

29. Ans: A
Tips: Phenytoin syrup 100mg- more potent 8% increase in drug content; Phenytoin caps
100mg – free acid; Sodium salt of Phenytoin – Dilantin caps.

30) Ans: C

31) Ans: C

32) Ans: A

33) Ans: C
Copyright © 2000-2016 TIPS Inc. Unauthorized reproduction of this manual is strictly prohibited and it is
illegal to reproduce without permission. This manual is being used during review sessions conducted by
PharmacyPrep.
96-10
PharmacyPrep.com Seizure and antiepileptic drugs

Drugs that cause SJS: SASPAN: Sulphonamides, antiepileptic drugs (CBZ)

34) Ans: C

35)Ans: B
Tips: Evidence shows there are two reasons seizure medication have compliance problems. 1)
because of side effects patient may not take regularly. 2) patient recently had no seizure.
Reference: Patient medication related non compliance. CE TT 2010 June

36) Ans:A

37)Ans: D

38) Ans: E
Tips: Phenytoin and Carbamazepine are not use in absence seizure

39)Ans: B

40)Ans: A

41)Ans: C

42)Ans: D

43)Ans: D

44) Ans: B

45) Ans: C
Tips: Carbamazepine stimulates phenytoin metabolism

46) Ans: D

47) Ans: B

48) Ans: A

Copyright © 2000-2016 TIPS Inc. Unauthorized reproduction of this manual is strictly prohibited and it is
illegal to reproduce without permission. This manual is being used during review sessions conducted by
PharmacyPrep.
96-11
PharmacyPrep.com Seizure and antiepileptic drugs

Copyright © 2000-2016 TIPS Inc. Unauthorized reproduction of this manual is strictly prohibited and it is
illegal to reproduce without permission. This manual is being used during review sessions conducted by
PharmacyPrep.
96-12
PharmacyPrep.Com Parkinson’s Disease

PHARMACY PREP

PARKINSON’S DISEASE

1) Parkinson patients are characterized by;


A. Increased ratio of dopaminergic/cholinergic activity in neostratum
B. most commonly occurs in elderly over 40 years of age patient
C. Decreased ratio of dopaminergic/cholinergic activity in neostratum.
D. All of the above

2-Involuntary movements typically involves mouth, face, limbs and trunk is characterized as:
A-Trigeminal neuralgia
B-Tardive dyskinesia
C-Neuroleptic malignant syndrome
D-Extrapyramidal symptoms
E-Akathisia

3-Which of the following Catecholamine Methyl Transferase (COMT) inhibitor has fatal liver
toxicity:
A-Selegiline
B-Tolcapone
C-Entacapone
D-Pergolide
E-Amantadine

4-A 60-year-old adult who has been treated with haloperidol for 3 weeks. He presents with
muscle stiffness, tremors. This is the most likely what kind of side effect:
A-Acute dystonia
B-Akathisia
C-Tardive dyskinesia
D-Parkinsonism disease
E-Pseudo parkinsonism

5-Which of the following is NOT used in treatment of Extrapyramidal side effects (EPS)?
A-Benztropine
B-Diphenhydramine
C-Trihexyphenidyl
D-Donepezil
E-All of the above

6-Selegiline is:
A-COMT inhibitor
B-MAO-B selective inhibitor
C-Non-selective MAO inhibitor

Copyright © 2000-2016 TIPS Inc. Unauthorized reproduction of this manual is prohibited. This manual is 97-1
being used during review sessions conducted by PharmacyPrep.
PharmacyPrep.Com Parkinson’s Disease

D-Levodopa preparations
E-MAO-A selective inhibitor

7) Which of the following is a COMT inhibitor?


A. Amantadine,
B. Bromocriptine,
C. Levodopa,
D. Selegiline
E. Entacapone

8) Why COMT inhibitors (Tolcapone, entacapone) always should used


with levodopa?
A-COMT inhibitor decrease metabolism of levodopa to 3-O-methyldopa
B-COMT inhibitors have synergistic action with levodopa
C-COMT inhibitors have additive action with levodopa
D-COMT inhibitors should not combine with levodopa
E-None of the above

9)All of the following statement is correct, except?


A-Deficiency of dopamine gives Parkinson’s disease
B-Tardive dyskinesia is NOT a symptom of Parkinson’s disease
C-Selegiline is a selective MAO- type B inhibitor
D-Tolcapone and entacapone are COMT inhibitor
E-Ropinarole and pramipexole are non-selective dopamine agonist

10-Tardive Dyskinesia is?


A-No movement or difficulty in movement
B-Involuntary movement of limbs, trunk, mouth and face
C-Slow movement
D-Rigidity
E-Postural instability

11)Which of the following antiviral drug indicated in Parkinson disease treatment:


A-Pergolide
B-Amantadine
C-Rimantadine
D-Selegiline
E-Entacapone

12) A 75 yo Parkinson's patient is on levodopa/carbidopa (Sinemet) therapy taking morning


and night time. However, recently patient experiencing rigidity and involuntary movement in
the evening before taking night time dose? It is characterized as?
A) fluctuation or on-off effect B) Dyskinesia C) Tardive dyskinesia D) Dystonia E) wearing
off effect

Copyright © 2000-2016 TIPS Inc. Unauthorized reproduction of this manual is prohibited. This manual is 97-2
being used during review sessions conducted by PharmacyPrep.
PharmacyPrep.Com Parkinson’s Disease

13) A doctor has entacapone therapy to a Parkinson's patient. Already is using


levodopa/carbidopa (Sinemet SR).
A) to treat wearing off effect
B) To treat tardive dyskinesia
C) to treat side effect of levodopa
D) to potentiate action of levodopa therapy
E) none of the above

14) A patient is on levodopa/carbidopa therapy for the past 3 yrs. Recently patient is getting
periods of poor movements i.e return of tremor, rigidity or slowness. This conditions is
referred as?
A) Tardive dyskinesia B) Akathisia C) Dystonia D) on-off effect or fluctuations E) Freezing

15) Which of the following drug extends the levodopa effects?


A) bromocriptine B) Selegiline C) Ropinirole D) Pramipexole E) Carbidopa

16) Which of the following is NOT a Parkinson's symptoms?


A) tremors B) rigidity C) jerks D) gait E) Tardive dyskinesia

17) Which of the following drugs have levodopa sparing effect?


A) Bromocriptine
B) amantadine
C) pramipexole
D) Ropinirole
E) All of the above

18) Which of the following is NOT side effect of levodopa?


A) hypotension B) Arrhythmias C) peptic ulcer D) sudden onset of sleep
E) pulmonary disease

19) Levodopa is taken empty stomach, what is correct?


A) take 30 min before meals
B) take 8 hours after meals
C) take 2 hours after meals
D) take 4 hours after meals

20) Hypofunction of nigrostriatal pathway resulting in decrease of dopamine levels and


increase of acetylcholine levels. The deficiency in dopamine would cause:
A. Alzheimer’s disease
B. Parkinson’s disease
C. Schizophrenia
D. Serotoninergic syndrome

21) Which of the following is the COMT inhibitor;

Copyright © 2000-2016 TIPS Inc. Unauthorized reproduction of this manual is prohibited. This manual is 97-3
being used during review sessions conducted by PharmacyPrep.
PharmacyPrep.Com Parkinson’s Disease

A. Tolcapone
B. Entacapone
C. Pergolide
D. A&B

22) Treatment of Parkinson’s disease may be treated with all of the following agents, EXCEPT:
A. Amantadine
B. Selegiline
C. Pergolide
D. Donezepil
E. Pramipexol

23) Pramipexole and Ropinirole are categorized as?


A. Selective D 1 agonist
B. Non selective D 1 and D 2 agonist
C. Non selective D 1 and D 2 antagonist
D. Selective D 2 agonist

24) The following antiviral drug can be used in PD treatment:


A. Levodopa
B. levodopa/carbidopa
C. Pergolide
D. Amantadine

25) What is the most common side effect of levodopa?


A. Tremor
B. Dyskinesia
C. Hallucination
D. Hypotension

26) MP is a 75 year old patient diagnosed with Parkinson's disease. Doctor considering to
prescribe medication. What is the drug of choice for Parkinson disease is?
A. Levodopa
B. Carbidopa
C. A and B
D. Dopamine
E. Selegiline

27) Which of the following drug should be avoided in Parkinson's disease?


A. Oral contraceptive pills
B. Metochlopramide
C. B and D
D. Antipsychotic drug

Copyright © 2000-2016 TIPS Inc. Unauthorized reproduction of this manual is prohibited. This manual is 97-4
being used during review sessions conducted by PharmacyPrep.
PharmacyPrep.Com Parkinson’s Disease

28. What is not a Parkinson disease symptom?


A. Tardive dyskinesia
B. Akathisia
C. Tremor
D. Bradykinesia

29) In treating parkinsonism, there is an increase of dopamine but on the other hand it will
increase also what neurotransmitter?
A. GABA
B. Acetylcholine
C. Norepinephrine
D. Epinephrine

30) Parkinson's disease symptoms characterized by difficulty in initiating movements is:


A. Akinesia
B. Bradykinesia
C. Dyskinesia
D. Anaesthesia

31) Over treatment of anti-Parkinson's drugs can result to:


A. Alzheimer's
B. Psychosis
C. Seizure
D. Respiratory arrest

32) An antinauseating drug that should be avoided in Parkinson patients:


A. Metoclopramide
B. Diclectin
C. A and B
D. Ondansetron

33) Which is not true about Parkinson drug?


A. Levodopa does penetrate brain
B. Carbidopa does not penetrate brain
C. Dopamine does not penetrate the brain
D. All of the above

34) How do you manage “wearing off”?


A. Switch to a different dopamine agonist
B. Stop the drug right away
C. Decrease dose of dopamine agonist
D. None of the above

Copyright © 2000-2016 TIPS Inc. Unauthorized reproduction of this manual is prohibited. This manual is 97-5
being used during review sessions conducted by PharmacyPrep.
PharmacyPrep.Com Parkinson’s Disease

35) Nausea and vomiting are adverse effects of Levodopa, how do you manage this?
A. Take levodopa with food
B. Take levodopa without food
C. Take it on empty stomach
D. Do not crash or chew

36) Which of the following is the suitable combination of drugs are initiated after as drug of
choice for Parkinson disease is?
A. Levodopa and carbidopa
B. Carbidopa and Dopamine
C. Levodopa and Tolcapone
D. Dopamine and Entacapone

37) Which of the following antiviral drug indicated in Parkinson disease treatment:
A. Valacyclovir
B. Amantadine
C. Acyclovir
D. selegiline
E. Interferon's

38) Parkinson’s disease is caused by deficiency of?


A-Dopamine in brain B-Acetylcholine C-GABA in CNS D-Serotonin in brain
E-Histamines

Copyright © 2000-2016 TIPS Inc. Unauthorized reproduction of this manual is prohibited. This manual is 97-6
being used during review sessions conducted by PharmacyPrep.
PharmacyPrep.Com Parkinson’s Disease

Parkinson's Disease
ANSWERS

1- Ans: A
Tips: That is why Dopamine agonist & anticholinergics are effective

2- Ans: B
Tips: dyskinesia = slow movement, tardive dyskinesia = involuntary movement of mouth, face,
limbs and trunk. Akinesia = difficulty in movement.

3- Ans: B

4- Ans: E
Tips: Pseudo parkinsonism (TRAP) may be side effects of antipsychotics. Pseudo parkinsonism
may disappear after discontinuation of medication.

5- Ans: D
Tips: anticholinergic agents such as benztropine, diphenhydramine, and trihexyphenidyl are
use in treatment of EPS. Donepezil is acetyl cholinesterase inhibitor is not indicated in EPS
treatment.

6- Ans: B
Tips: MAO-B selective inhibitors; treatment for Parkinson’s disease is: BALSAC =
Bromocriptine, Amantadine, Levodopa, Selegiline, Antimuscarinic (benztropine), and COMT
inhibitors

7) Ans: E

8) Ans: A

9) Ans: E
Tips: Ropinirole and pramipexole are selective D 2 agonist

10- Ans: B

11) Ans: B

12) Ans: E

13) Ans: A

14) Ans: D
Tips: End-of-dose wearing off gives on-off effect. This phenomenon is related to the increasing
loss neuronal storage capability of dopamine as well as short half life of dopa.

Copyright © 2000-2016 TIPS Inc. Unauthorized reproduction of this manual is prohibited. This manual is 97-7
being used during review sessions conducted by PharmacyPrep.
PharmacyPrep.Com Parkinson’s Disease

15) Ans: B

16) Ans: D

17) Ans: E

18) D

19)Ans: A

20) Ans:B

21) Ans: D
Tips: COMT (Catecholamine O-Methyl transferase), the drugs names ends with “capone”.
COMT are indicated in Parkinson's disease

22) Ans: D
Tips: Donepezil is a drug for Alzheimers

23)Ans: D

24) Ans: D

25) Ans: B

26) Ans: C

27)Ans: C

28. Ans: A

29) Ans: B

30) Ans: A

31)Ans: B

32)Ans: A

33) Ans: D

34)Ans: A
Tips: To manage “wearing off”:Administer levodopa more frequently

Copyright © 2000-2016 TIPS Inc. Unauthorized reproduction of this manual is prohibited. This manual is 97-8
being used during review sessions conducted by PharmacyPrep.
PharmacyPrep.Com Parkinson’s Disease

Switch to a different dopamine agonist


Add or increase dose of a dopamine agonist
Switch to Sinemet CR

35) Ans: A
Tips: Take levodopa with food and increase dose of levodopa or dopamine agonist gradually
over several weeks

36)Ans: C
Tips: Sinemet is the combination of levodopa and carbidopa

37) Ans: B

38) Ans: A

Copyright © 2000-2016 TIPS Inc. Unauthorized reproduction of this manual is prohibited. This manual is 97-9
being used during review sessions conducted by PharmacyPrep.
PharmacyPrep.Com Parkinson’s Disease

Copyright © 2000-2016 TIPS Inc. Unauthorized reproduction of this manual is prohibited. This manual is 97-10
being used during review sessions conducted by PharmacyPrep.
Pharmacyprep.com Antimicrobial agents

PHARMACY PREP
ANTI-INFECTIVE AGENTS – ANTIBIOTICS

1. What symptoms distinguish complicated and uncomplicated UTI?


A. Blood in urine (turbid urine)
B.fever
C. pain, fever and chills
D. frequency of urination
E. urgency of urination
Ans.A

2.What is empiric treatment of urinary tract infection, patient has allergy to penicillin?
A. cotrimoxazole 3 d
B. Fluroquinolone 3d
C. Amoxicillin 3 d
D. Cephalexin 7 d
E. Aminoglycoside 14 d
Ans. A

Copyright © 2000-2016 TIPS Inc. Unauthorized reproduction of this manual is prohibited. This 98-1
manual is being used during review sessions conducted by PharmacyPrep.
PharmacyPrep.Com Anticancer drugs and chemotherapy

PHARMACY PREP

ANTICANCER DRUGS AND CHEMOTHERAPY

1. What is the drug of choice to treat chemo induced nausea and vomiting
A. dexamethsone B. Ondansetron C. Metoclopramide D. dimenhydrinate
Ans. a
2) Which of the following chemotherapeutic agent is categorized as low emitogenic?
A) Cisplatin B) Cyclophosphamide C) Iphosphomide D) Vincristine
Ans. D

3) Hemorrhagic cystitis results from irritation of the lining of the bladder by acrolein, a
metabolite of ifosfamide and cyclophosphamide. Acrolein can be inactivated by?
A-Mesna
B-Doxarazoxane
C-Chlropropamide
D-Erythropoeitins
E-None of the above
Ans. A

4) The drug of choice to treat delayed nausea and vomiting?


A-Ondansetron
B-Metoclopramide
C-Dexamethasone
D-Dimenhydrinate
E-Scopolamine

5) Cardio protective agent is?


A-Mesna B-Dexorasoxane C-Acroline D-None of the above
Ans. B

6. Chemotherapy induced nausea and vomiting is mediated through number of receptors,


however which of the following are the most important in CINV?
A. LTC4 and LTD4
B. M1 and 5HT3 receptors
C. H1 and 5HT3 receptors
D. 5HT3 and NK-1 receptors
E.5HT1b/1d and 5HT2
Ans. D
Tips. The CINV is commonly associated with 5HT3 and neurokinin-1 receptors.
Metoclopramide and Prochlorpromazine are dopamine antagonist is used for management of
low emitogenic CINV.

Copyright © 2000-2016 TIPS Inc. Unauthorized reproduction of this manual is prohibited. This manual is 99-1
being used during review sessions conducted by PharmacyPrep.
Pharmacy Prep
Natural Products and Pharmacognosy
1) A customer of your pharmacy wants to buy ginkgo biloba to enhance memory. What to do?
A) give gingko biloba
B) do not recommend gingko
C) refer to doctor
D) refer to emergency
E) say there is no clinical evidence

2) A customer of your pharmacy wants to buy ginkgo biloba to enhance memory. Patient
currently on warfarin therapy . What to do?
A) give gingko biloba
B) do not recommend gingko because, has reports of drug interactions
C) refer to doctor
D) refer to emergency
E) say there is no clinical evidence

3) A customer of your pharmacy wants to buy garlic pills to treat borderline high cholesterol.
What to do?
A) give garlic pills
B) do not recommend garlic pills
C) refer to doctor
D) refer to emergency
E) say there is no clinical evidence

4. Which of the following herbal products can increase blood pressure?


A. Garlic
B. St. John wart
C. Licorice
D. Valerian
E. Feverfew

5. Health Canada approved Natural or herbal products are identified by?


A. Drug identification number
B. Product identification number
C. Natural Product number
D. Natural Health Products
E. Prescription number

Copyright © 2000-2016 TIPS Inc. Unauthorized reproduction of this manual is prohibited. This manual is being 100-1
used during review sessions conducted by PharmacyPrep.
ANSWERS

Natural Products and Pharmacognosy

1) Ans: A
Tips: As long as patient is not medication and/or have no medical conditions that do not
interfere with natural products if this helps to alleviate their problems. I will give it to patient.

2) Ans: B

3) Ans: C

4. Ans: C

5. Ans. C

Copyright © 2000-2016 TIPS Inc. Unauthorized reproduction of this manual is prohibited. This manual is being 100-2
used during review sessions conducted by PharmacyPrep.

Anda mungkin juga menyukai